You are on page 1of 244

Problem Books in Mathematics

Hayk Sedrakyan · Nairi Sedrakyan

Algebraic
Inequalities
Problem Books in Mathematics

Series Editor:
Peter Winkler
Department of Mathematics
Dartmouth College
Hanover, NH 03755
USA

More information about this series at http://www.springer.com/series/714


Hayk Sedrakyan Nairi Sedrakyan

Algebraic Inequalities

123
Hayk Sedrakyan Nairi Sedrakyan
University Pierre and Marie Curie Yerevan, Armenia
Paris, France

ISSN 0941-3502 ISSN 2197-8506 (electronic)


Problem Books in Mathematics
ISBN 978-3-319-77835-8 ISBN 978-3-319-77836-5 (eBook)
https://doi.org/10.1007/978-3-319-77836-5
Library of Congress Control Number: 2018934928

Mathematics Subject Classification (2010): 97U40, 00A07, 26D05

© Springer International Publishing AG, part of Springer Nature 2018


This work is subject to copyright. All rights are reserved by the Publisher, whether the whole or part
of the material is concerned, specifically the rights of translation, reprinting, reuse of illustrations,
recitation, broadcasting, reproduction on microfilms or in any other physical way, and transmission
or information storage and retrieval, electronic adaptation, computer software, or by similar or dissimilar
methodology now known or hereafter developed.
The use of general descriptive names, registered names, trademarks, service marks, etc. in this
publication does not imply, even in the absence of a specific statement, that such names are exempt from
the relevant protective laws and regulations and therefore free for general use.
The publisher, the authors and the editors are safe to assume that the advice and information in this
book are believed to be true and accurate at the date of publication. Neither the publisher nor the
authors or the editors give a warranty, express or implied, with respect to the material contained herein or
for any errors or omissions that may have been made. The publisher remains neutral with regard to
jurisdictional claims in published maps and institutional affiliations.

Printed on acid-free paper

This Springer imprint is published by the registered company Springer International Publishing AG
part of Springer Nature
The registered company address is: Gewerbestrasse 11, 6330 Cham, Switzerland
To Margarita,
a wonderful wife and a loving mother
To Ani,
a wonderful daughter and a loving sister
Preface

In mathematics one often deals with inequalities. This book is designed to teach the
reader new and classical techniques for proving algebraic inequalities. Moreover,
each chapter of the book provides a technique for proving a certain type of
inequality.
The book includes techniques of using the relationship between the arithmetic,
geometric, harmonic, and quadratic means, the principle of mathematical induction,
the change of variable(s) method, techniques using the Cauchy–Bunyakovsky–
Schwarz inequality, Jensen’s inequality, and Chebyshev’s properties of functions,
among others. The main idea behind of the proof techniques discussed in this book
is making the complicated simple, so that even a beginner can understand com-
plicated inequalities, their proofs and applications. This approach makes it possible
not only to prove a large variety of inequalities, but also to solve problems related to
inequalities. To explain each technique of proof, we provide examples and prob-
lems with complete proofs or hints. At the end of each chapter there are problems
for independent study. In Chapter 14 (Miscellaneous Inequalities) are included
inequalities whose proofs employ various techniques not covered in the preceding
chapters. In some cases, the proofs of Chapter 14 use several proof techniques from
the preceding chapters simultaneously. One hundred selected inequalities and their
hints are also provided in the end of Chapter 14, and interested readers are
encouraged to choose and provide any methods of proofs they prefer. In each
chapter we have tried to include inequalities belonging to the same topic and to
present them in order of increasing difficulty, using principles similar to those in
[11]. This allows the reader to try to prove these inequalities step by step and to
refer to the provided proofs only when difficulties arise. We recommend to use the
proofs provided in the book, paying more attention to the choice of the mathe-
matical proof technique.
Most of the inequalities in this book were created by the authors. Nevertheless,
some of the inequalities were proposed in different mathematical olympiads in
different countries or have been published elsewhere (including author-created
inequalities). However, the provided solutions are different from the original ones.
Most such inequalities are included in the books [2, 5, 6, 7, 8, 9], and since the

vii
viii Preface

name of the author of individual inequalities is unknown to us, we cite these books
as the main references. However, for well-known inequalities we have tried to
provide the name of the authors. This book was published in Seoul in Korean [13,
14] and is based on [16], which was later published in Moscow in Russian [15]. The
historical origins provided at the beginning of some chapters are mostly based on
[10] or our personal knowledge.
It was considered appropriate to give the proofs of each chapter at the end of the
same chapter.

Paris, France Hayk Sedrakyan


Yerevan, Armenia Nairi Sedrakyan
Contents

1 Basic Inequalities and Their Applications . . . . . . . . . . . . . . . . . . . . 1


2 Sturm’s Method . . . . . . . . . . . . . . . . . . . . . . . . . . . . . . . . . . . . . . . 9
3 The HM-GM-AM-QM Inequalities . . . . . . . . . . . . . . . . . . . . . . . . . 21
4 The Cauchy–Bunyakovsky–Schwarz Inequality . . . . . . . . . . . . . . . 45
5 Change of Variables Method . . . . . . . . . . . . . . . . . . . . . . . . . . . . . 59
6 Using Symmetry and Homogeneity . . . . . . . . . . . . . . . . . . . . . . . . . 71
7 The Principle of Mathematical Induction . . . . . . . . . . . . . . . . . . . . 81
8 A Useful Inequality . . . . . . . . . . . . . . . . . . . . . . . . . . . . . . . . . . . . . 107
9 Using Derivatives and Integrals . . . . . . . . . . . . . . . . . . . . . . . . . . . 127
10 Using Functions . . . . . . . . . . . . . . . . . . . . . . . . . . . . . . . . . . . . . . . 143
11 Jensen’s Inequality . . . . . . . . . . . . . . . . . . . . . . . . . . . . . . . . . . . . . 155
12 Inequalities of Sequences . . . . . . . . . . . . . . . . . . . . . . . . . . . . . . . . . 177
13 Algebraic Inequalities in Number Theory . . . . . . . . . . . . . . . . . . . . 187
14 Miscellaneous Inequalities . . . . . . . . . . . . . . . . . . . . . . . . . . . . . . . . 197
Appendix—Power Sums Triangle . . . . . . . . . . . . . . . . . . . . . . . . . . . . . . . 239
Bibliography . . . . . . . . . . . . . . . . . . . . . . . . . . . . . . . . . . . . . . . . . . . . . . . . 243

ix
About the Authors

Hayk Sedrakyan is an IMO medal winner, a professor of mathematics in Paris,


France, and a professional math olympiad coach in the greater Boston area,
Massachusetts, USA. He received his doctorate in mathematics at the Université Pierre
et Marie Curie, Paris, France. Hayk is a Doctor of Mathematical Sciences in USA,
France, Armenia. He has been awarded master’s degrees in mathematics from
Germany, Austria, and Armenia and completed part of his doctoral studies in Italy.
Hayk has authored several books on the topic of problem-solving and olympiad-style
mathematics published in USA and South Korea.

Nairi Sedrakyan has long been involved in national and international mathematical
olympiads, having served as an International Mathematical Olympiad (IMO) problem
selection committee member and the president of Armenian Mathematics Olympiads.
He is the author of one of the hardest problems ever proposed in the history of
International Mathematical Olympiads (the fifth problem of the 37th IMO). He has
been the leader of the Armenian IMO Team, jury member of the IMO, jury member
and problem selection committee member of the Zhautykov International Mathematical
Olympiad (ZIMO), jury member and problem selection committee member of the
International Olympiad of Metropolises, and the president of the International
Mathematical Olympiad Tournament of the Towns in Armenia. He is also the author of
a large number of problems proposed in these olympiads and has authored several
books on the topic of problem-solving and olympiad-style mathematics published in
United States, Russia, Armenia, and South Korea. The students of Nairi Sedrakyan
have obtained 20 medals (1 gold medal, 4 silver medals, 15 bronze medals) in IMO.
For his outstanding teaching, Nairi Sedrakyan received the title of best teacher of the
Republic of Armenia and was awarded special recognition by the prime minister.

xi
Overview

This book is designed to teach the reader new and classical mathematical proof
techniques for proving inequalities, in particular, to prove algebraic inequalities.
These proof techniques and methods are applied to prove inequalities of various
types. The main idea behind this book and the proof techniques discussed is making
the complicated simple, so that even a beginner can understand complicated
inequalities, their proofs and applications. The book Algebraic Inequalities is also
devoted to the topic of inequalities and can be considered a continuation of the
book Geometric Inequalities: Methods of proving [12].
It can serve teachers, high-school students, and mathematical competitors.

xiii
Chapter 1
Basic Inequalities and Their Applications

Historical origins. According to [3], mathematical inequalities first were expressed


verbally. Later, the following symbols were introduced:
Less than and greater than: The mathematical symbols < and > first appear
in the book Artis Analyticae Praxis ad Aequationes Algebraicas Resolvendas (The
Analytical Arts Applied to Solving Algebraic Equations) posthumously published
in 1631 and written by the English astronomer and mathematician Thomas Harriot
(c. 1560–1621), who was born in Oxford, England, and died in London, England.
Harriot initially used triangular symbols, but the editor modified them slightly, so that
they resemble the modern less than and greater than symbols. In his book is stated
the following: “The mark of the majority (signum majoritatis) as a > b, signifies a
greater than b and the mark of the minority (signum minoritatis) to a < b signifies
a lesser than b.”
Less than or equal to and greater than or equal to: The double-bar style of the
less than or equal to < and greater than or equal to > symbols was first employed
in 1734 by the French mathematician, geophysicist, and astronomer Pierre Bouger
(1698–1758) born in Le Croisic, France, and died in Paris, France.
In 1670, a similar reduced single-bar notation was employed by the English math-
ematician John Wallis. Wallis used a single horizontal bar above instead of below
the inequality symbols, leading to < and >. Wallis also introduced the symbol for
infinity ∞. His academic advisor was the English mathematician William Oughtred
(1574–1660), who introduced the multiplication symbol × and the abbreviations sin
and cos for the sine and cosine. Oughtred was born in Eton, England, and died in
Albury, England.
Not equal to, not greater than, not less than: These symbols were employed by
the Swiss mathematician, physicist, and astronomer Leonhard Euler (1707–1783),
who was born in Basel, Switzerland, and died in Saint Petersburg, Russian Empire
(now Russia). Euler introduced many modern mathematical notations, for example
the notation for a mathematical function. He is considered one of the greatest math-
ematicians of all time. His doctoral advisor was the prominent Swiss mathematician
Johann Bernoulli (1667–1748), who was born and died in Basel, Switzerland. Euler

© Springer International Publishing AG, part of Springer Nature 2018 1


H. Sedrakyan and N. Sedrakyan, Algebraic Inequalities, Problem Books
in Mathematics, https://doi.org/10.1007/978-3-319-77836-5_1
2 1 Basic Inequalities and Their Applications

was the doctoral advisor of six students, including the well-known Italian math-
ematician Joseph Louis Lagrange (1736–1813). He was born Giuseppe Lodovico
Lagrangia in Turin, Kingdom of Sardinia (now Turin, Italy) and died in Paris, France,
and his son Johann Euler (1734–1800) born and died in Saint Petersburg, Russia.
In this chapter, basic but very useful inequalities are presented. We recommend
that the reader pay special attention to these inequalities, which will lay a foundation
for more challenging inequalities in the forthcoming sections. The main idea behind
the proof techniques and methods presented here is making the complex simple, so that
even a beginner can understand complex inequalities, their proofs and applications.
A background in high-school-level algebra is sufficient for solving the inequalities
presented below.

Problems

Prove the following inequalities (1.1–1.26).


1.1. a 2 + b2 √ ≥ 2ab.
1.2. a +2 b ≥ ab, where a ≥ 0, b ≥ 0.

1.3. ab ≥ 1 +2 1 , where a > 0, b > 0.
 a b
a 2 + b2
1.4. 2
≥ a+b
2
.
1.5. a+b
2
≥ 2
1
+ b1
, where a > 0, b > 0.
a 2 + b2
 a
a+b 2

1.6. 2
≥ 2 .
1.7. a + b > 1 + ab, where b < 1 < a.
1.8. a 2 + b2 > c2 + (a + b − c)2 , where b < c < a.
1.9. (a) 2 ≤ ab + ab , where ab > 0,
(b) ab + ab ≤ −2, where ab < 0.
x1 + ··· +xn
1.10. x1 ≤ n
≤ xn , where x1 ≤ · · · ≤ xn .
x1 + ··· +xn
1.11. x1
y1
≤ y1 + ··· +yn
≤ xn
yn
, where x1
y1
≤ ··· ≤ xn
yn
and yi > 0, i  1, . . . , n.
1
1.12. x1 ≤ (x1 · · · xn ) ≤ xn , where n ≥ 2, 0 ≤ x1 ≤ · · · ≤ xn .
n

1.13. |a1 | + · · · + |an | ≥ |a1 + a2 + · · · + an |.


1.14. a1 + ···n +an ≥ 1 + ···n + 1 , where n ≥ 2, ai > 0, i  1, . . . , n.
 a1 an
√ √
1.15. (a + b) 2 ≥ a b + b a, where a > 0, b > 0.
a+b

1.16. 21 (a + b) + 41 ≥ a +2 b , where a > 0, b > 0.
1.17. a(x + y − a) ≥ x y, where x ≤ a ≤ y.
1.18. x −1 1 + x +1 1 > 2x , where x > 1.
1.19. 1
3k + 1
+ 3k1+ 2 + 3k1+ 3 > 2k1+ 1 + 2k1+ 2 , where k ∈ N.
1.20. ab
(a+b)2
≤ ((1(1−−a)+(1
a)(1 − b)
− b))2
, where 0 < a ≤ 21 , 0 < b ≤ 21 .
Problems 3

1.21. √3k1 + 1 · 2k
2k + 1
+2
< √3k1 + 4 , where k ∈ N.
1.22. 2n−1 ≥ n, where n ∈ N.
1.23. 13 + 23 · 15 + 23 · 45 · 17 + · · · + 23 · 45 · 67 · · · 100 · 1 < 1.
101 103
1−a 1−b
1.24. (a) 1 − b + 1 − a ≤ b + a , where 0 < a, b ≤ 21 ,
a b

n
1 
n n
1 
n
(b) 1 − ai
(1 − ai ) ≤ ai
ai , where 0 < a1 , . . . , an ≤ 21 .
i1 i1 i1 i1
1.25. 1 + 213 + · · · + 1
n 3 < 5
4
, where n ∈ N.
1.26. 1 + a1 + b ≤ 1 − a+b
2
+ ab
3
, where 0 ≤ a ≤ 1, 0 ≤ b ≤ 1.

Proofs

1.1. We have a 2 + b2 − 2ab  (a − b)2 ≥ 0. Note that equality holds if and only
if a  b. √
1.2. The proof is similar to the proof of Problem 1.1. We have a + b − 2 ab 
√ √ 2
a − b ≥ 0. Note that equality holds if and only if a  b.

1.3. Multiplying both sides of the inequality of Problem 1.2 by 2a +ab
b
, we obtain the
required inequality. Note that equality holds if and only if a  b.
1.4. We have 2ab ≤ a 2 + b2 (see Problem 1.1). We obtain the equivalent inequality
a 2 + b2 + 2ab ≤ 2a 2 + 2b2 . Thus, it follows that (a +4b) ≤ a +2 b . The last
2 2 2

a+b  
inequality can be rewritten as   ≤ a 2 + b2 . Therefore, a 2 + b2 ≥ a + b .
2 2 2 2
Note that equality holds if and only if a  b.
1.5. This inequality follows from the inequalities of Problems 1.2 and 1.3. Note
that equality holds if and only if a  b.
1.6. See the proof of Problem 1.4. Note that equality holds if and only if a  b.
1.7. Using that a + b − 1 − ab  (a − 1)(1 − b) and the condition b < 1 < a, it
follows that (a − 1)(1 − b) > 0.
1.8. Let us evaluate the difference between the left-hand side and the right-hand
side of the given inequality:
   
a 2 + b2 − c2 − (a + b − c)2  a 2 − c2 − (a + b − c)2 − b2 
 (a − c)(a + c) − (a − c)(a + 2b − c)  2(a − c)(c − b) > 0,

since according to the assumption, we have b < c < a.


1.9. (a) Multiplying both sides of the inequality by ab, where ab > 0, we deduce
that a 2 + b2 ≥ 2ab.
(b) Dividing both sides of the inequality a 2 +b2 ≥ −2ab by ab, where ab < 0,
we obtain the required inequality.
1.10. We have x1 ≤ x1 , . . . , xn ≤ xn , whence nx1 ≤ x1 + · · · + xn ≤ nxn .
Thus, it follows that x1 ≤ x1 + ···n +xn ≤ xn .
4 1 Basic Inequalities and Their Applications

1.11. From the assumptions of the problem, it follows that yi xy11 ≤ xi ≤ yi xynn , i 
1, . . . , n. Summing up these inequalities, we deduce that
x1 xn
(y1 + · · · + yn ) ≤ x1 + · · · + xn ≤ (y1 + · · · + yn ).
y1 yn

Hence, it follows that xy11 ≤ xy11 ++ ··· +xn


··· +yn
≤ xynn .
1.12. We have that x1 ≤ xi ≤ xn , i  1, . . . , n. Multiplying these inequalities, we
1
obtain x1n ≤ x1 · · · xn ≤ xnn . Thus, it follows that x1 ≤ (x1 · · · xn ) n ≤ xn .
1.13. If a1 + · · · + an ≥ 0, then |a1 + · · · + an |  a1 + · · · + an .
Using a ≤ |a|, we deduce that

|a1 + · · · + an |  a1 + · · · + an ≤ |a 1 | + · · · + |an |.

If a1 + · · · + an < 0, then |a1 + · · · + an |  −a1 − · · · − an . Using the inequality


−a ≤ |a|, it follows that |a1 + · · · + an |  −a1 − · · · − an ≤ |a 1 | + · · · + |an |.
1.14. We have that
⎛ ⎞


1 1 ⎜ a1 a2 an−1 an ⎟ ⎟
(a1 + · · · + an ) + ··· + ⎜ + + ··· + + ⎟ + n,
a1 an ⎝ a2 a1 an an−1 ⎠
  
n(n−1)/2

and using Problem 1.9 (a), it follows that


1 1 n(n − 1)
(a1 + · · · + an ) + ··· + ≥n+2·  n2.
a1 an 2
√ 
1.15. The given inequality is equivalent to the inequality (a + b) ≥ 2 ab · a+b
2
√ √
a+ b

, which can be obtained by multiplying the inequalities a + b ≥ 2 ab
2  √ √
(Problem 1.2) and a +2 b ≥ a +2 b (see Problem 1.6).
1.16. We have
  2
1 1 a+b a+b 1
(a + b) + −  − ≥ 0.
2 4 2 2 2

Therefore, 21 (a + b) + 41 ≥ a +2 b .
1.17. Since a(x + y − a) − x y  ax − x y + a(y − a)  (y − a)(a − x) and y ≥
a ≥ x, it follows that (y − a)(a − x) ≥ 0. Therefore, a(x + y − a) ≥ x y.
1.18. Using the inequality of Problem 1.5, it follows that
Proofs 5

1 1
x −1
+ x +1 2 1 1 2
> , or + > .
2 (x − 1) + (x + 1) x −1 x +1 x

1.19. According to the inequality of Problem 1.18, we have that


1
3k + 1
+ 3k1+ 3  (3k +12)−1 + (3k +12)+1 > 3k2+ 2 . Therefore, 3k1+ 1 + 1
3k + 2
+ 1
3k + 3
>
3
3k + 2
.
Now let us prove that 3k3+ 2 > 2k1+ 1 + 2k1+ 2 .
Indeed, 2k1+ 1 + 2k1+ 2 − 3k3+ 2  (2k + 1)(2k−k+ 2)(3k + 2) < 0.

1.20. The given inequality is equivalent to the following inequality:



2

2 1 1
−1 ≤ −1 −1 .
a +b a b

We have that



2
1 1 2 1 1 1 4 4
−1 −1 − −1  − − − + 
a b a +b ab a b (a + b)2 a + b
1 4 4 a +b (a − b)2 (a − b)2 (a − b)2 (1 − (a + b))
 − 2
+ −  2
− 
ab (a + b) a +b ab ab(a + b) ab(a + b) ab(a + b)2

and 0 < a ≤ 21 , 0 < b ≤ 21 . Then (a − ab(a


b) (1−(a + b)) 2

+ b)2
≥ 0, and therefore,
 2 2  1  1 
−1 ≤ a −1 b −1 .
a+b √
√ inequality is equivalent to the inequality (2k + 1)
1.21. The given 3k + 4 <
(2k + 2) 3k + 1 or to the following inequality: (2k + 1)2 (3k + 4) <
(2k + 2)2 (3k + 1).
The last inequality holds because

(2k + 2)2 (3k + 1) − (2k + 1)2 (3k + 4)  k > 0.

1.22. Since 1 < 2 < 22 < · · · < 2n−1 and the number of positive integers
1, 2, 22 , . . . , 2n−1 is equal to n, it follows that 2n−1 ≥ n.
1.23. Consider a unit line segment and suppose on the first day, we paint 13 of thegiven
segment, the second day 15 of the rest of the segment, on the 51st day, 103
1
of the
rest of the segment. Since every day there remains a part of the given segment,
the sum of the painted parts must be less than 1.
The first day we have painted 13 of the given segment, on the second day 23 · 15 ,
on the 51st day 23 · 45 · · · 100 · 1 . Hence, we deduce that
101 103

1 2 1 2 4 1 2 4 98 100 1
+ · + · · + ··· + · ··· · · < 1.
3 3 5 3 5 7 3 5 99 101 103
1.24 (a) According to the inequality of Problem 1.20, we have that
6 1 Basic Inequalities and Their Applications

((1 − a) + (1 − b))2 (a + b)2 1 − a 1 − b a b


≤ or + ≤ + .
(1 − a)(1 − b) ab 1−b 1−a b a

An alternative proof. Note that 1 − a ≥ a and 1 − b ≥ b, and therefore,


1−a 1−b (1 − a)2 + (1 − b)2 ((1 − a) − (1 − b))2 + 2(1 − a)(1 − b)
+   
1−b 1−a (1 − a)(1 − b) (1 − a)(1 − b)
(a − b)2 (a − b)2 a b
 +2≤ +2 + .
(1 − a)(1 − b) ab b a

Hence, we deduce that 11 − −b


a −b
+ 11 − a
≤ ab + ab .
 1 
n n
(b) Since 1 − ai
(1 − ai ) 
i1 i1

1 − a1 1 − a2 1 − an−1 1 − an
+ + ··· + + +n, using the
1 − a2 1 − a1 1 − an 1 − an−1
  
n(n−1)/2
inequality of Problem 1.24 (a), we obtain that
 

 1 
n n n n
1 a1 a2 an−1 an
(1 − ai ) ≤ + + ··· + + +n  ai .
1 − ai a2 a1 an an−1 a
i1 i1    i1 i i1
n(n−1)/2

1.25. Note that if n ≥ 4, then


1 1 2−1 n − (n − 1)
1+ + ··· + 3  1 + + ··· +
23 n 2 3

n
3

5 1 1 2 1 n−1 5
 − 3
− 2
− 3
− 2
− ··· − 3
< ,
4 2 3 3 4 n 4

since (k +k 1)3 > (k +12)2 , where k ∈ N.


1.26. We have that (1 − a)(1 − b) ≥ 0, and therefore a + b − 1 ≤ ab. Then

1 a+b a+b 1
− 1−  (a + b − 1) ≤ ab.
1+a+b 2 2(1 + a + b) 3

Problems for Independent Study

Prove the following inequalities (1–32).


1. |x − y| < |1 − x y|, where |x| < 1, |y| < 1.
sin x−1
2. sin x−2
+ 21 ≥ 2−sin
3−sin x
x
.
3. bc + ca + ab ≥ a + b2 − 2c , where a > 0, b > 0, c > 0.
a b c 2

4. a1 + b1 − 1c < abc1
, where a 2 + b2 + c2  53 and a > 0, b > 0, c > 0.
   2
5. 3 1 + a 2 + a 4 ≥ 1 + a + a 2 .
Problems for Independent Study 7

6. (ac + bd)2 + (ad − bc)2 ≥ 144, where a + b  4 , c + d  6 .



7. x12 + x22 + · · · + x2n
2
+ na 2√≥ a 2(x1 + x2 + · · · + x2n ).
√ √
a+ b+ c
8. 1
a +b
+ b +1 c + a 1+ c ≤ √ , where a > 0, b > 0, c > 0.
  2 abc  
9. a b − c + b c − a + c a − b < 0, where 0 < a < b < c.
3 2 2 3 2 2 3 2 2

10. a 3 b + b3 c + c3 a ≥ a 2 b2 + b2 c2 + a 2 c2 , where a ≥ b ≥ c > 0.


11. xy + yz + x +y z ≤ (x x+zz) , where 0 < x ≤ y ≤ z.
2

   
 √ √ √
12. 1 + a + 1 + a + a 2 + · · · + 1 + a + · · · + a n < na,
where n ≥ 2, a ≥ 2, n ∈ N.
13. (a) tan2 (α − β) ≤ (n − 4n
1)2
, where tan α  n tan β , n > 0 ,
(b) 1 + cos(α − β) ≥ cos α + cos β, where 0 ≤ α ≤ π2 , 0 ≤ β ≤ π2 .
14. [5x] ≥ [x] + [2x] 2
+ [3x]
3
+ [4x]
4
+ [5x]
5
,
where [a] is the integer part of the real number a.
15. (n!)2 ≥ n n , where n ∈ N.
16. x 6 + x 5 + 4x 4 − 12x 3 + 4x 2 + x + 1 ≥ 0.
17. log2 α ≥ log β log γ , where α > 1, β > 1, γ > 1, α 2 ≥ βγ .
18. log4 5 + log5 6 + log6 7 + log7 8 > 4, 4.
19. 13 + · · · + 3·5···(2n
n
+ 1)
< 21 , where n ∈ N.
3 3
20. 223 −+ 11 · · · nn3 −+ 11 < 23 , where n ≥ 2, n ∈ N.
21. 1 · 1! + · · · + n ·n! < (n + 1)! , where n ∈ N.
22. 1 + 212 1 + 312 . . . 1 + n12 < 2, where n ≥ 2, n ∈ N.
    
23. 1 − p12 1 − p12 . . . 1 − p12 > 21 , where 1 < p1 < p2 < · · · < pn , pi ∈
1 2 n

N , i  1, . . . , n.
24. 21 − 13 + 41 − 15 + · · · − 999 1 1
+ 1000 < 25 .
25. (sin x + 2 cos 2x)(2 sin 2x − cos x) < 4, 5.
26. (a) 1 +a a+ +b b ≤ 1 +a a + 1 +b b , where a ≥ 0, b ≥ 0.
 
(b) 2 +a a+ +b b ≥ 21 1 +a a + 1 +b b , where a ≥ 0, b ≥ 0.
1 1 1 1
a1 + bi an + bi n
(c) n ≤ a1 + b1
a1 + b1 + 2
+ ... + an + bn
an + bn + 2
+ 1
+ 1
1
+ ... + 1 1 < 2n, where
a1 bi + 2 an + bi n + 2
1
i 1 , . . . , i n is some permutation of the numbers 1, . . . , n, ai , bi > 0, i 
1, . . . , n.

n
a1 + 2a2 + ··· + iai 
n
27. i 2 ≤ 2 ai , where ai ≥ 0, i  1, . . . , n.
i1 i1
28. a1 + b1 + 1c ≤ 42
41
, where a1 + b1 + 1c < 1 , a, b, c ∈ N .
4x y
29. y + z + x + z + x + y > 2, where x, y, z > 0.
z

30. 1 < a + ab + d + a + bb + c + b + cc + d + a + dc + d < 2, where a > 0, b > 0, c > 0, d > 0.


31. a + b > c + d, where a, b, c, d ≥ 21 and a 2 + b > c2 + d, a + b2 > c + d 2 .
Hint. If a+b ≤ c+d, then a ≤ c or b ≤ d. If a ≤ c, then b−d > (c−a)(c+a) ≥
c − a. √
32. (b − a)(9 − a 2 ) + (c − a)(9 − b2 ) + (c − b)(9 − c2 ) ≤ 24 2, where 0 ≤ a ≤
b ≤ c ≤ 3.
8 1 Basic Inequalities and Their Applications

Hint. We have that

(b − a)(9 − a 2 ) + (c − a)(9 − b2 ) + (c − b)(9 − c2 ) ≤ 9b + c(9 − b2 ) + (c − b)(9 − c2 ) 


1 3 3
 18c − c3 + bc(c − b) ≤ 18c − c3 + c  18c − c3 .
4 4

33. If 0 < a, b, c < 1, then one of the numbers (1 − a)b, (1 − b)c, (1 − c)a is
not greater than 41 .
34. Let a > 0, b > 0, c > 0, and a + b + c  1. Prove that
  
(a) a + 41 (b − c)2 + b + 41 (c − a)2 + c + 41 (b − a)2 ≤ 2,
 √ √ √
(b) a + 41 (b − c)2 + b + c ≤ 3.

Hint. (a) x + 41 (y − z)2 ≤ x + y+z 2
if x, y, z > 0 and x + y + z  1.
35. Find the smallest possible value of the following expression:
a4 4 2 2

b4
+ ab4 − ab2 − ab2 + ab + ab , where a > 0, b > 0.
Chapter 2
Sturm’s Method

Historical origins. Sturm’s method was proposed by the Prussian mathematician


Friedrich Otto Rudolf Sturm, born 6 January 1841 in Breslau, Prussia (now Wrocław,
Poland), died 12 April 1919 in Breslau, Germany. Sturm obtained his doctorate
in mathematics from the University of Breslau (now University of Wrocław) in
1863 under the supervision of the well-known Prussian mathematician Heinrich
Eduard Schröter. Sturm was the advisor of 19 doctoral students, including the well-
known Prussian mathematician Otto Toeplitz, born 1 August 1881 in Breslau, Prussia,
died 15 February 1940 in Jerusalem, Mandatory Palestine (now Israel). Toeplitz
studied mathematics at the University of Breslau, where under Sturm’s supervision
he obtained his doctorate in 1905.
Besides its various applications, Sturm’s method provides an opportunity to prove
a large number of different inequalities under certain conditions.

Example 2.1 Prove that if the product of positive numbers x1 , . . . , xn (n ≥ 2) is


equal to 1, then x1 + · · · + xn ≥ n.

Proof If x1  · · ·  xn  1, then x1 + · · · + xn  n.

Suppose that among the considered numbers there are at least two different num-
bers. Then among the numbers x1 , . . . , xn there are two numbers such that one of
them is greater than 1 and the other one is less than 1 (Problem 1.12). Without loss of
generality one can assume that those numbers are x1 and x2 , and that x1 < 1 < x2 .
Note that x1 + x2 > 1 + x1 x2 (Problem 1.7).
If one substitutes the given numbers by numbers 1, x1 x2 , x3 , . . . , xn , then their
product is again equal to 1, and the sum satisfies 1+ x1 x2 + x3 +· · ·+ xn < x1 +· · ·+ xn .
Doing the same with the numbers 1, x1 x2 , x3 , . . . , xn , in a similar way we obtain
a new sequence such that two numbers in it are equal to 1. Doing the same at most
n − 1 times, we obtain a sequence such that n − 1 numbers in it are equal to 1, and
the nth number is equal to x1 · · · xn .
On the other hand, x1 · · · xn  1.

© Springer International Publishing AG, part of Springer Nature 2018 9


H. Sedrakyan and N. Sedrakyan, Algebraic Inequalities, Problem Books
in Mathematics, https://doi.org/10.1007/978-3-319-77836-5_2
10 2 Sturm’s Method

Hence, we obtain that n < x1 + x2 + · · · + xn .


From the proof, it follows that equality holds if and only if x1  · · ·  xn  1.

Example 2.2 Prove that if the sum of the numbers x1 , . . . , xn (n ≥ 2) is equal to 1,


then x12 + · · · + xn2 ≥ n1 .

Proof If x1  · · ·  xn  n1 , then x12 + · · · + xn2  n1 .

Suppose that among the considered numbers there are at least two different num-
bers.
Then among the numbers x1 , . . . , xn there are two numbers such that one of them
is greater than n1 and the other one is less than n1 (Problem 1.10). Without loss of
generality, one can assume that those numbers are x1 and x2 , and that x1 < n1 and
x2 > n1 . Therefore, substituting x1 by n1 , and x2 by x1 + x2 − n1 , we obtain a new
sequence of numbers n1 , x1 + x2 − n1 , x3 , . . . , xn such that their sum is again equal to 1.
On the other
 hand,
2  we have that 2
x12 + x22 > n1 + x1 + x2 − n1 (see Problem 1.8), whence
 2  
1 1 2
x12 + ··· + xn2 > + x1 + x2 − + x32 + · · · + xn2 .
n n

Repeating these steps a finite number of times, we obtain a sequence such that
its all terms are equal to n1 , and the sum of their squares is less than the sum of the
 2  2
squares of the numbers x1 , . . . , xn , that is, x12 + · · · + xn2 > n1 + · · · + n1  n1 .
From the proof, it follows that equality holds if and only if x1  · · ·  xn  n1 .

Problems

Prove the following inequalities (2.1–2.6).


2.1 AM-GM (arithmetic mean–geometric mean inequality):
x1 + ··· + xn √
n
≥ n x1 · · · xn , where n ≥ 2, xi > 0, i  1, . . . , n.
2.2 QM-AM (quadratic mean–geometric mean inequality or RMS/root mean square
x12 + ··· + xn2
inequality): n
≥ x1 + ···n + xn .
(1 − x1 ) ··· (1 − xn )
2.3 x1 ··· xn
≥ (n − 1)n , where n ≥ 2, xi > 0, i  1, . . . , n, and
x1 + · · · + xn  1.
2.4 1
1+x1
+ · · · + 1+x 1
n
≥ 1+ √n xn1 ... xn , where n ≥ 2, x1 ≥ 1, . . . , xn ≥ 1.
2.5 abc + bcd + cda + dab ≤ 27 1
+ 176
27
abcd, where a ≥ 0, b ≥ 0, c ≥ 0, d ≥ 0,
and a + b + c + d  1.
2.6 0 ≤ x y + yz + zx − 2x yz ≤ 7
27
, where x ≥ 0, y ≥ 0, z ≥ 0 and x + y + z  1.
Problems 11

2.7 Among all triangles with no angle greater than 75º inscribed in a given circle,
find the triangle such that its perimeter is
(a) the greatest,
(b) the smallest.

2.8 (a) Schur’s inequality: Prove that if for some numbers α and β one has the
inequality [α f (a) + β f (b) ≤ f (αa + βb)]α f (a) + β f (b) ≥ f (αa + βb),
where α ≥ 0, β ≥ 0, α + β  1, a, b are any numbers belonging to
D( f )  I,1 and x1 , . . . , xn , y1 , . . . , yn ∈ I such that y1 ≥ . . . ≥ yn ,y1 ≤
x1 , y1 + y2 ≤ x1 + x2 , . . . , y1 + · · · + yn−1 ≤ x1 + · · · + xn−1 , y1 + · · · + yn 
x1 + · · · + xn , then f (y1 ) + f (y2 ) +· · · + f (yn ) ≤ f (x1 )+ f (x2 ) +· · · + f (xn )

[ f (y1 ) + f (y2 ) + · · · + f (yn ) ≥ f (x1 ) + f (x2 ) + · · · + f (xn )].

(b) Popoviciu’s inequality: Prove that if for numbers α and β one has the
inequality α f (a) + β f (b) ≥ f (αa + βb), where α, β ≥ 0, α + β  1,
and a, b are any numbers belonging to the interval I , then for all numbers
x, y, z from the interval I , one has the inequality
x + y + z x + y y +z z + x 
f (x) + f (y) + f (z) + 3 f ≥2f +2f +2f .
3 2 2 2
2.9 Suppose that for numbers x1 , . . . , x1997 , the following conditions hold:

(a) − √13 ≤ xi ≤ 3, i  1, . . . , 1997,

(b) x1 + · · · + x1997  −318 3.

Find the greatest possible value of the expression x112 + · · · + x1997


12
.
n−1
2.10 Prove that cos α1 cos α2 · · · cos αn (tan α1 + · · · + tan αn ) ≤ (n−1) , where
2
n−2
n 2
π
n ≥ 2 and 0 ≤ αi < 2 , i  1, 2, . . . , n.

n
2.11. Prove that xik (1 − xi ) ≤ ak , where k ≥ 2, k ∈ N, and ak  max(x k (1 −
i1 [0;1]
x) + (1 − x)k x), xi ≥ 0, i  1, . . . , n, x1 + . . . + xn  1, n ≥ 2.
2.12. Prove the following inequalities:

(a) 2(n − 1)(x1 x2 + x1 x3 + · · · + x1 xn + x2 x3 + · · · + x2 xn + · · · + xn−1 xn ) −


n n−1 x1 x2 · · · xn ≤ n − 2, where n ≥ 2,x1 ≥ 0, . . . , xn ≥ 0 and x1 + x2 +
· · · + xn  1,

(b) x1 + x2 n+ ··· + xn − n x1 x2 · · · xn ≤
√ √ 2 √ √ 2 √ √ 2 √ √ 2
≤ ( 1
x − x2 ) +( x1 − x3 ) + ··· + ( x1 − xn ) + ··· + ( xn−1 − xn )
n
, where n ≥ 2,
x1 ≥ 0, . . . , xn ≥ 0.

1f is any real-valued function, D is the domain of function f , D(f ) is denoted by I.


12 2 Sturm’s Method

(c) For n  4, the following inequality


 is called Turkevici’s inequality:
(n − 1)(x1 + x2 + · · · + xn ) + n x1 x2 · · · xn2 ≥ (x1 + x2 + · · · + xn )2 , where
2 2 2 n 2 2

n ≥ 2, x1 ≥ 0, . . . , xn ≥ 0.

Proofs

2.1 Consider the numbers √n x1x1··· xn , . . . , √n x1xn··· xn , and note that their product is equal
to 1. Then according to Example 2.1, we have that √n x1x1··· xn + · · · + √n x1xn··· xn ≥ n,
or
x1 + · · · + xn √
≥ n x1 · · · xn . (2.1)
n
Note that in (2.1), the equality holds if x1  · · ·  xn .
Inequality (2.1) is known as Cauchy’s inequality.
2.2 Consider the numbers x1 + x···1 + xn , . . . , x1 + x···n + xn , and note that their sum is equal to
 2  2
1. According to Example 2.2, we have that x1 + x···1 + xn +· · ·+ x1 + x···n + xn ≥ n1 ,
or

x12 + x22 + · · · + xn2  x1 + x2 + · · · + xn 2


≥ . (2.2)
n n
This ends the proof.
Note that in (2.2), equality holds if and only if x1  · · ·  xn .
2.3 Since x1 + · · · + xn  1, it follows by Problem 1.10 that there are two numbers
such that one of them is not greater than n1 , and the other one is not less than n1 .
Without loss of generality, one can assume that x1 ≤ n1 ≤ x2 .
Let us substitute x1 by n1 , x2 by x1 + x2 − n1 . Then we obtain numbers n1 , x1 +
x2 − n1 , x3 , . . . , xn such that
(1 − x1 ) ··· (1 − xn )
≥ (1 − n1 )(1 − x1 − x2 + n1 ) ··· (1−xn ) , since (1 − x1 )(1 − x2 ) ≥
n ( x 1 + x 2 − n ) ··· x n
x1 ··· xn 1 1 x1 x2
(1 − n )(1 − x1 − x2 + n ) (Problem 1.17) and (1 − x1 )(1 − x2 )  1 + 1 − (x1 + x2 ) .
1 1

n (x1 + x2 − n )
1 1 x1 x2 x1 x2
Repeating these steps a finite number of times, we obtain n numbers equal to
1
n
. For these numbers, the left-hand side of the inequality is equal to (n − 1)n ,
and it is not greater than (1−xx11) ··· (1−xn )
··· xn
.

2.4 Let us set x1 · · · xn  m. According to Problem 1.12, without loss of
n

generality one can assume that x1 ≤ m ≤ x2 , and therefore, for numbers


m, x1mx2 , x3 , . . . , xn , one has
1 1 1 1 1
+ ··· + ≥ + x1 x2 + · · · + , since
1 + x1 1 + xn 1+m 1+ m 1 + xn
Proofs 13

1
1 + x1
+ 1 +1x2 ≥ 1
1+m
+ 1+
1
x1 x2 (Problem 1.17) and 1
1 + x1
+ 1
1 + x2
 1 +
m
1 − x1 x2
1 + x1 + x2 + x1 x2
.
After a finite number of steps, we deduce that 1
1 + x1
+ ··· + 1
1 + xn

1 1
+ ··· +  √n n .
1

+ m 1 + m 1 + x1 ··· xn
n
2.5 If a  b  c  d  14 , then one has equality.
Let a < 41 < b. Consider the following two cases.

(a) If c + d − 176 cd < 0, then



27
  3
A  ab c + d − 176 27
cd + cd(a + b) ≤ cd(a + b) ≤ c + d+(a
3
+ b)
 1
27
,
using Problem 2.1.
(b) If c + d − 176
27
cd ≥ 0, then note that
     
1 1 176 1 1
A≤ a+b− c+d − cd + cd + a+b− .
4 4 27 4 4

Hence, we must prove the inequality for numbers a1  41 , b1  a + b − 41 ,

c1  c, d1  d.

In a similar way, either one can prove the inequality for numbers
a1 , b1 , c1 , d1 , or it will be sufficient to prove the inequality for the case
that among a1 , b1 , c1 , d1 , two numbers are equal to 41 . Continuing in this
way, we obtain that it is sufficient to prove the inequality for numbers
, , , . Note that in this case, the inequality obviously holds.
1 1 1 1
4 4 4 4

2.6 Let x ≥ y ≥ z. Then y ≤ 21 , and therefore,


    
1 1 1 1
0 ≤ y(x + z) + x z(1 − 2y) ≤ y + x+z− + x +z− (1 − 2y).
3 3 3 3

Therefore, if one substitutes the numbers x, y, z by the numbers 13 , y, x +z− 13 in


the expression x y + yz + x z − 2x yz, then its value does not decrease. Continuing
in a similar way, one can substitute the numbers 13 , y, x + z − 13 by the numbers
, , . Hence, we deduce that x y + yz + x z − 2x yz ≤ 19 + 19 + 19 − 27
1 1 1
3 3 3
2
 27
7
.
2.7 According to the law of sines, we have that p  a + b + c 
2R(sin α + sin β + sin γ ).

(a) Let us prove that the greatest perimeter is that of an equilateral triangle.
If a triangle is not equilateral, then without loss of generality one can
assume that α > π3 > β.
Now let us prove that
14 2 Sturm’s Method

π  π
sin α + sin β + sin(α + β) < sin + sin α + β − + sin(α + β).
3 3
We have that
π  π α − π/3 α + π/3
sin α + sin β − sin − sin α + β −  2 sin cos −
3 3 2 2
α − π/3 α + 2β − π/3
− 2 sin · cos 
2 2
 
α − π/3 2π/3 − 2β α+β
 2 sin −2 sin sin 
2 4 2
α − π/3 β − π/3 α+β
 4 sin sin sin < 0.
2 2 2

Therefore, the perimeter of a triangle with angles α, β, γ is less than the


perimeter of a triangle with angles π3 , α + β − π3 , γ . If γ  π3 , then we
obtain that the perimeter of a triangle with angles π3 , α + β − π3 , γ is less
than the perimeter of a triangle with angles π3 , π3 , π3 .
(b) Let us prove that the smallest perimeter is that of a triangle with angles
◦ ◦ ◦
75 , 75 , 30 . It is sufficient to prove that the perimeter of a triangle with
angles α, β, γ is not less than the perimeter of a triangle with angles
◦ ◦
75 , α + β − 75 , γ . 
◦ ◦
Indeed, sin 75 + sin α + β − 75 ≤ sin α + sin β, since
◦  ◦  ◦   ◦ 
sin α + sin β − sin 75 − sin α + β − 75  sin α − sin 75 + sin β − sin α + β − 75 
◦ ◦
α − 75 α + 75
 2 sin cos −
2 2
◦ ◦
α − 75 α + 2β − 75
− 2 sin cos 
2 2
◦ ◦
α − 75 β − 75 α+β
 4 sin sin sin ≥ 0, and
2 2 2
◦ ◦ ◦
0 < α ≤ 75 , 0 < β ≤ 75 .

2.8 (a) If xi  yi , i  1, 2, . . . , n, then f (x1 ) + · · · + f (xn )  f (y1 ) + · · · + f (yn ).


Suppose that for some i, one has xi  yi . From the assumptions, it follows
that if m is the smallest number such that xm  ym , then ym < xm . Let j be
the greatest number such that y j < x j .
Let k be the smallest number greater than j such that xk < yk . Note that there
exists such a number k, for otherwise, we would have the following inequalities:

x1 + · · · + x j−1 + x j > y1 + · · · + y j ,

x j+1 ≥ y j+1 ,

··· ··· ···

xn ≥ yn .
Proofs 15

Summing these inequalities leads us to the contradiction x1 + · · · + xn >


y1 + · · · + yn .
Hence, it follows  that x j > y j ≥ yk > xk .
δ
Let δ  min x j − y j , yk − xk and λ  1 − x j −x k
.
Consider the numbers x1 , . . . , x j−1 , λx j + (1 − λ)xk 
x ∗j , x j+1 , . . . , xk−1 , λxk + (1 − λ)x j  xk∗ , xk+1 , . . . , xn , and let us verify
that for these numbers, the assumptions of the problem hold.
We have x1 + · · · + xi ≥ y1 + · · · + yi , i  1, . . . , j − 1.
The inequality y1 + · · · + y j ≤ x1 + · · · + xj−1 + λx j + (1 − λ)xk holds, since
λx j + (1 − λ)xk  x j − δ ≥ x j − x j − y j  y j .
Since x j+1 ≥ y j+1 , . . . , xk−1 ≥ yk−1 , it follows that x1 + . . . + xi ≥
y1 + . . . + yi , i  j + 1, . . . , k − 1.  
For i ≥ k, we have x1 + · · · + x i  x1 + · · · + x j−1 + λx j + (1 − λ)xk + x j+1 +
· · · + xk−1 + λxk + (1 − λ)x j + · · · + xi ,
and therefore, for the numbers x1 , . . . , x j−1 , x ∗j , x j+1 , . . . , xk−1 , xk∗ , xk+1 , . . . , xn ,
the assumptions of the problem again hold.
On the other hand,
   
f (x1 ) + · · · + f x j−1 + f x j + · · · + f (xk ) + · · · + f (xn ) ≥
       
≥ f (x1 ) + · · · + f x j−1 + f x ∗j + f x j+1 + · · · + f (xk−1 ) + f xk∗ + · · · + f (xn ),
     
since f x ∗j  f λx j + (1 − λ)xk ≤ λ f x j + (1 − λ) f (xk ) and
     
f xk∗   f λxk + (1 − λ)x j ≤ λ f (xk ) + (1 − λ) f x j . Therefore,
   
f x ∗j + f xk∗ ≤ f x j + f (xk ).
On the other hand, δ  x j − y j or δ  yk − xk , whence x ∗j  y j or xk∗  yk .
Therefore, we have substituted the numbers x1 , x2 , . . . , x j , . . . , xn by the num-
bers x1 , . . . , x j−1 , x ∗j , x j+1 , . . . , xk−1 , xk∗ , xk+1 , . . . , xn . Hence, the assumptions
of the problem again hold. On the other hand, the sum f (x1 ) + · · · + f (xn )
did not increase, and the number of indices i satisfying the condition
xi  yi increased by 1. Therefore, after at most n − 1 steps, we obtain that
f (x1 ) + · · · + f (xn ) ≥ f (y1 ) + · · · + f (yn ).
(b) Let x ≥ y ≥ z. Since for the numbers y1  y2  x+y 2
, y3  y4 
z+x
2
, y5  y 6  y+z
2
and x 1  x, x 2  x 3  x 4  x+y+z
3
, x 5  y, x6  z or
x1  x, x2  y, x3  x4  x5  x+y+z 3
, x 6  z, the assumptions of Problem
2.8 (a) hold true, then it follows that
f (x) + f (y) + f (z) + 3 f ( x+y+z
3 )  f (x 1 ) + f (x 2 ) + f (x 3 ) + f (x 4 ) + f (x 5 ) + f (x 6 ) ≥

≥ f (y1 ) + f (y2 ) + f (y3 ) + f (y4 ) + f (y5 ) + f (y6 )  2 f ( x+y y+z z+x


2 ) + 2 f ( 2 ) + 2 f ( 2 ).

2.9 Let us first prove the following lemma.

Lemma If a ≤ b and x > 0, then (a − x)12 + (b + x)12 > a 12 + b12 .


16 2 Sturm’s Method

Note that
   
(a − x)12 + (b + x)12 − a 12 − b12  C12
1
x b11 − a 11 + C12 x b + a 10 + · · · + 2x 12 > 0.
2 2 10


Set yi  3xi i  1, . . . , 1997. We have

−1 ≤ yi ≤ 3, (1)

y1 + · · · + y1997  −954, (2)


y 12 + ··· +y 12
and x112 + · · · + x1997
12
 1 36 1997 .
If any two numbers among the numbers y1 , . . . , y1997 belong to (−1, 3), then
according to the lemma, one can substitute these two numbers by numbers such that
one of them is equal to either −1 or 3. Then conditions (1), (2) hold, and y112 +· · ·+y1997 12

increases.
Therefore, the sum y112 + · · · + y1997
12
is the greatest possible if one substitutes the
numbers y1 , . . . , y1997 by either −1, . . . , −1, 3, . . . , 3, or −1, . . . , −1, 3, . . . , 3, a,
where a ∈ (−1, 3).
Taking into consideration (2), we obtain that only the second case is possible, so
that k  a+2 4
+ 1735, where k is the number of −1’s. Since a+2 4
∈ Z and a ∈ (−1, 3),
we must have a  2.
Therefore, the greatest possible value of x112 + . . . + x1997 12
is equal to
12 12
1736 + 260·3 + 2
36
 189548.
α1 + ··· +αn
2.10 Let n
 ϕ (1). If α1  α2  · · ·  αn  ϕ, then

cos α1 cos α2 · · · cos αn (tan α1 + · · · + tan αn )  cosn ϕ · n · tan ϕ  n · sin ϕ · cos n−1 ϕ 


cos 2 ϕ cos 2 ϕ
 n sin ϕ · (cos ϕ)
2 2 n−1  n (n − 1)n−1 sin2 ϕ · ··· ≤
n−1 n−1
 ⎛ ⎞n
 
 sin2 ϕ + cos

+ · · · + cos
2ϕ n−1
 ⎝ n−1 n−1 ⎠  n (n − 1)n−1 · 1  (n − 1)
2
≤ n (n − 1) n−1
n nn n−2
n 2

If αi  α j for some i and j (i  j), then among those numbers there are two
numbers such that one of them is greater than ϕ and the other one is less than
ϕ (see Problem 1.10).
Let it be the numbers α1 and α2 such that α1 < ϕ < α2 . Thus, substituting α1
by ϕ and α2 by α1 + α2 − ϕ, we obtain a new sequence of numbers ϕ, α1 +
α2 − ϕ, α3 , . . . , αn such that (1) holds. On the other hand, since
Proofs 17

1
cos α1 cos α2  (cos(α1 + α2 ) + cos(α1 − α2 )) <
2
1
< (cos(α1 + α2 ) + cos(2ϕ − (α1 + α2 ))) 
2
 cos ϕ cos(α1 + α2 − ϕ), we have
cos α1 cos α2 · · · cos αn (tan α1 + · · · + tan αn ) 
 sin(α1 + α2 ) cos α3 · · · cos αn + cos α1 cos α2 cos α3 · · · cos αn (tan α3 + · · · + tan αn ) <
< sin(α1 + α2 ) cos α3 · · · cos αn +
+ cos ϕ cos(α1 + α2 − ϕ) cos α3 · · · cos αn (tan α3 + · · · + tan αn ) 
 cos ϕ cos(α1 + α2 − ϕ) cos α3 . . . cos αn (tan ϕ + tan(α1 + α2 − ϕ)+
+ tan α3 + · · · + tan αn ).

Continuing in the same way for the numbers ϕ, α1 + α2 − ϕ, α3 , . . . , αn , we


obtain a new sequence two of whose terms are equal to ϕ. Let us repeat these
steps at most n − 1 times. Then we obtain a sequence n − 1 of whose terms
are equal to ϕ, and the nth term is equal to nϕ − (n − 1)ϕ  ϕ.
Hence, we have obtained
n−1
(n − 1) 2
cos α1 cos α2 · · · cos αn (tan α1 + · · · + tan αn ) < cosn ϕ · n · tan ϕ ≤ n−2
.
n 2

From the proof, it follows that equality holds if and only if α1  α2  · · · 


αn  ϕ, where ϕ  arctan √n−11
.
2.11 Prove that if x ≥ 0, y ≥ 0, x + y ≤ 23 , and k ≥ 2, k ∈ N, then

x k (1 − x) + y k (1 − y) ≤ (x + y)k (1 − x − y). (2.6)

If x + y  0, then (2.6) holds, while if x + y  0, then


 2  2
xk yk x y
· (1 − x) + · (1 − y) ≤ · (1 − x) + · (1 − y) 
(x + y)k (x + y)k x + y x + y
(x + y)2 (1 − x − y) + x y(3(x + y) − 2)
 ≤ 1 − x − y.
(x + y)2

Let xi+1 ≥ xi ≥ 0, i  1, . . . , n − 1, x1 + · · · + xn  1 and n ≥ 3. Then


(n − 2)x1 + (n − 2)x2 ≤ (x3 + · · · + xn ) + (x3 + · · · + xn )  2 − 2x1 − 2x2 , and
therefore,
2 2
x1 + x2 ≤ ≤ .
n 3

Therefore, if we substitute the numbers x1 , . . . , xn by 0, x1 + x2 , x3 , . . . , xn ,


then their sum will be equal to 1. Note that


n
xik (1 − xi ) ≤ (x1 + x2 )k (1 − x1 − x2 ) + x3k (1 − x3 ) + · · · + xnk (1 − xn ).
i1
18 2 Sturm’s Method

Repeating these steps a finite number of times, we end up with the case

n
of n  2, that is, xik (1 − xi ) ≤ x k (1 − x) + (1 − x)k x, and therefore,
i1

n
xik (1 − x i ) ≤ ak .
i1

Remark Note that a1  max(2x(1 − x))  21 , a2  max(x(1 − x))  21 ,


[0;1] [0;1]
1 1
a3  max(x(1 − x)(1 − 2x(1 − x)))  , a4  max(x(1 − x)(1 − 3x(1 − x)))  ,
[0;1] 8 [0;1] 12

5 10 − 14
a5  max(x(1 − x) − 4(x(1 − x))2 + 2(x(1 − x))3 ) 
[0;1] 27

(see Problem 12.12).


2.12 (a) Let x1 ≤ x2 ≤ · · · ≤ xn , n ≥ 3. Consider the following two cases.
If x2 · · · xn−1 ≥ 2(n−1)
n n−1
, then we have that
A  2(n − 1)(x1 x2 + x1 x3 + · · · + x1 xn + x2 x3 + · · · + x2 xn + · · · + xn−1 xn ) − n n−1 x1 x2 · · · xn 
 2(n − 1)((x1 + xn )(x2 + · · · + xn−1 ) + x2 x3 + · · · + x2 xn−1 + · · · + xn−2 xn−1 )+
+ x1 xn (2(n − 1) − n n−1 x2 · · · xn−1 ) ≤ 2(n − 1)(x(1 − x) + x2 x3 + · · · + x2 xn−1 + · · · + xn−2 xn−1 ),

where x  x2 + · · · + xn−1 .
x 2 +···+x 2  x 2
According to Problem 2.2, it follows that 2 n−2 n−1 ≥ n−2 , and therefore,
x2 x3 + · · · + x2 xn−1 + · · · + xn−2 xn−1 ≤ 2(n−2) x , whence
n−3 2

n−3 2 n−1 n−1


A ≤ 2(n − 1)(x(1 − x) + x )  4(n − 2) · x · (1 − x) ≤ n − 2.
2(n − 2) 2(n − 2) 2(n − 2)

If x2 · · · xn−1 < 2(n−1)


n n−1
, then for x1  x2  · · ·  xn  n1 , we have A  n − 2.
Otherwise, if xi  n1 for some value of i, then x1 < n1 < xn .
Substituting x1 by n1 and xn by x1 + xn − n1 , we see that the value of the
expression
A  2(n − 1)((x1 + xn )(x2 + · · · + xn−1 ) + x2 x3 + · · · + x2 xn−1 + · · · + xn−2 xn−1 )+
+ x1 xn (2(n − 1) − n n−1 x2 · · · xn−1 )

increases (see Problem 1.17).


Continuing in a similar way, either one can end the proof of the inequality or
it will be sufficient to prove the inequality for x1  x2  · · ·  xn  n1 .
(b) If y1 , y2 , . . . , yn ≥ 0 and y1 + y2 + · · · + yn > 0, then according
to Problem 2.12 (a), for xi  y1 + y2 y+i ··· + yn , i  1, 2, . . . , n, we obtain that
2(n − 1)qn pnn−2 − n n−1 y1 · y2 · · · yn ≤ (n − 2) pnn , where pn  y1 + y2 + · · · + yn ,
qn  y1 y2 + y1 y3 + · · · + y1 yn + · · · + yn−1 yn . Therefore, for y1 · y2 · · · yn  1
we have

(n − 2) pnn + n n−1
qn ≤ . (1)
2(n − 1) pnn−2
Proofs 19

If x1  0, then we obtain the following inequality:


√ √ 2 √ √ 2 √ √ 2
x2 + · · · + xn x2 − x3 + ··· + x2 − xn + ··· + xn−1 − xn + x2 + · · · + xn
≤ ,
n n

√ √ √ √
or 2 x2 x3 + 2 x2 x4 + · · · + 2 x2 xn + · · · + 2 xn−1 xn ≤ (n − 2)(x2 + · · · + xn ).

The last inequality can be

proved using that 2 ab ≤ a + b(a, b ≥ 0).
x
If xi > 0, let yi  2n√x1 ·x2i ···xn , i  1, 2, . . . , n. Then y1 · y2 · · · yn  1, and
p2 −2q (n−1)( p2 −2q )−2q (n−2) p2 +n
one needs to prove that n n n − 1 ≤ n
n
n n
, or qn ≤ 2(n−1)
n
.
The last inequality holds because by (1), it follows that (see Problem 2.1)
(n−2) pn +n n−1 (n−2) pn2 +n
qn ≤ 2(n−1)n pn−2 ≤ 2(n−1) .
n
(c) For numbers y1  x12 , . . . , yn  xn2 , using the inequality of Problem 2.12
(b), we deduce that
   2
x12 + · · · + xn2 (|x1 | − |x2 |)2 + · · · + (|x1 | − |xn |)2 + · · · + xn−1  − |xn |
− n x12 · . . . · xn2 ≤ ,
n n


or (n − 1)(x12 + · · · + xn2 ) + n n x12 · · · xn2 ≥ (|x1 | + · · · + |xn |)2 ≥ (x1 + · · · + xn )2
(see Problem 1.13).

Problems for Independent Study

Prove the following inequalities (1–4).


1. 1
+ ··· + 1
≤ n x ··· x ,
√ n
where n ≥ 2, 0 < xi ≤ 1, i  1, . . . , n.

1 + x1 1 + xn
 1+ 1 n

a 2 + b2 + c2 + d 2
2. 4
≥ abc + abd +4 acd + bcd , where a > 0, b > 0, c > 0, d > 0.
3

2 2 2
3. 9 ≤ x y + yz + zx ≤ 9 + x4 y z , where x + y + z  x yz and x > 0, y > 0, z > 0.
4. (a) 1 + x1
x1
· · · 1 +xnxn ≥ (n + 1)n , where n ≥ 2, xi > 0, i  1, . . . , n, and
x1 + · · · + xn  1,  n
(b) 1+x1
1−x1
· · · 1−x
1+xn
n
≥ nn −+ 11 , where n ≥ 2, xi > 0, i  1, . . . , n, and
x1 + · · · + xn  1,
(c) (s − (n − 1)x1 ) · · · (s − (n − 1)xn ) ≤ x1 · · · xn , where 0 ≤ xi ≤ n−1
s
, for
i  1, . . . , n, and

x1 + · · · + xn  s.

5. Prove that among all convex n-gons inscribed in a given circle, the greatest area
it that of a regular n-gon.
6. Prove that among all convex n-gons inscribed in a given circle, the greatest
perimeter is that of a regular n-gon.
7. Prove that among all convex polygons inscribed in a given circle, the greatest
sum of the squares of the sides is that of an equilateral triangle.
20 2 Sturm’s Method

8. Let n ≥ 4 and xi > 0, i  1, . . . , n, be real numbers such that x1 +· · ·+ xn  1.


Prove that for all λ (0 < λ < n), one has the inequality
 
1 1 n2 − λ
x1 · · · xn + ··· + − λx1 · · · xn ≤ .
x1 xn nn

9. Prove that for two triangles with angles α, β, γ and α1 , β1 , γ1 , one always has
cos α1 cos β1 cos γ1
+ + ≤ cot α + cot β + cot γ .
sin α sin β sin γ

10. Prove that yia11 · · · yiann ≤ yib11 · · · yibnn , (the summation is over all
(i 1 ,...,i n ) (i 1 ,...,i n )
permutations of the numbers (1, . . . , n)), where y1 > 0, . . . , yn > 0, and a1 ≥
. . . ≥ an , a1 ≤ b1 , a1 + a2 ≤ b1 + b2 , . . . , a1 + · · · + an−1 ≤ b1 + · · · + bn−1 , a1 +
· · · + an  b1 + · · · + bn .
11. Prove that
n(a1 b1 + · · · + an bn ) ≥ (a1 + · · · + an )(b1 + · · · + bn ) if given that from the con-
dition ai < a < a j , it follows that bi ≤ b j , where a  a1 + a2 n+ ··· + an .
12. Let f be an odd function defined and decreasing on (−∞, +∞).
Prove that f (a) f (b) + f (b) f (c) + f (c) f (a) ≤ 0, where a + b + c  0.
13. Suppose that for the numbers a1 , . . . , an (n ≥ 2), the following conditions hold:
(a) a1 ≤ · · · ≤ an ,
(b) a1 + · · · + an  0,
(c) |a1 | + · · · + |an |  S.

Prove that an − a1 ≥ 2Sn


.
14. Let n be a given integer such that n ≥ 2.

(a) Find the smallest constant C such that the inequality


 4
 
xi x j (xi2 + x 2j ) ≤ C xi
1≤i< j≤n 1≤i≤n

holds for all nonnegative numbers x1 , . . . , xn .


(b) Find when the equality holds for the obtained constant C.

15. Prove that

(a) 1
1 + s − x1
+ · · · + 1 + s1− xn ≤ 1, where s  x1 + · · · + xn , x1 · · · xn  1 and
x1 , . . . , xn > 0.
(b) 1
a1n + ··· + an−1
n
+ na1 ··· an
+ a n + ··· + a n 1 + na1 ··· an + ··· + 1
a2n + ··· + ann + na1 ··· an
≤ 1
a1 ··· an
,
1 n−2

where a1 , . . . , an > 0.
16. Prove that 5(a 2 +b2 +c2 ) ≤ 6(a 3 +b3 +c3 )+1, where a, b, c > 0 and a +b+c  1.
Chapter 3
The HM-GM-AM-QM Inequalities

For solving some problems and in order to prove a large number of inequalities, one
often needs to use various means and the relationships among them.
For positive real numbers a and b, the expression 1 +2 1 is called their harmonic
√ a b

mean (HM), ab is called their geometric mean (GM), a +2 b is called their arithmetic

2 2
mean (AM), and a +2 b is called their quadratic mean (QM) or root mean square.
One has the following relationship among the means. In this chapter we consider
the so called the HM-GM-AM-QM inequalities, first for two positive numbers and
afterward more generally for n arbitrary positive numbers.

Lemma (HM-GM-AM-QM inequalities for two positive numbers) Let a and b be


positive real numbers. Then

2 √ a +b a2 + b2
1 1
≤ ab ≤ ≤ . (3.1)
a
+ b 2 2

Moreover, in (3.1) equality holds if and only if a  b.

Proof Proofs of the inequalities (3.1) are given in Chapter 1 (Problems 1.2–1.4).
Let us consider the following examples in order to see how these inequalities can
be applied.

Example 3.1 Prove that a


b
+ b
c
+ c
a
≥ 3, where a > 0, b > 0, c > 0.

Proof Without loss of generality one can assume that c ≥ a, c ≥ b.

Using that
a+b √
≥ ab, (3.2)
2
we deduce that

© Springer International Publishing AG, part of Springer Nature 2018 21


H. Sedrakyan and N. Sedrakyan, Algebraic Inequalities, Problem Books
in Mathematics, https://doi.org/10.1007/978-3-319-77836-5_3
22 3 The HM-GM-AM-QM Inequalities

a b
+ ≥ 2. (3.3)
b a
(c − a)(c − b)
Note that b
c
+ c
a
− b
a
−1 ac
≥ 0, whence
b c b
+ − − 1 ≥ 0. (3.4)
c a a

Summing inequalities (3.3) and (3.4), we obtain ab + bc + ac ≥ 3.


This ends the proof.
  
Example 3.2 Prove that b a+ c + c +b a + a +c b > 2, where a > 0, b > 0, c > 0.

Proof Without loss of generality, one can assume that a ≥ b ≥ c. Thus


       
a b c a b+c b c b+c
+ +  + + + − ≥
b+c c +a a +b b+c a c +a a +b a
     
b c b+c b c b+c
≥ 2+ + − ≥ 2+ + − 
c +a a +b a c +a 2a a
     
1 b c √ 1 b c √
 2+ √ + − b+c ≥ 2+ √ + − b+c 
a 1 + ac 2 a 1 + bc 2
√ √
c √
 2+ √ b + c − 2c ≥ 2.
2a(b + c)

Equality
 holds if a  b+c, b  a  c, which leads to a contradiction. Therefore,
  c
a
b+c
+ c + a + a + b > 2.
b

This ends the proof.



Alternative proof Let us use the inequality b a+ c · 1 ≥ 1 + 2b + c  a +2ab + c ,
  c
a
b
a+c
· 1 ≥ 2
1 + a b+ c
 2b
a+b+c
, a+b
· 1 ≥ 2
1 + a +c b
 2c
a+b+c
. Summing these inequali-
  
ties, we obtain b a+ c + c +b a + a +c b ≥ 2. Equality holds if 1  b a+ c  a b+ c  a +c b ,
  
which leads to a contradiction. Thus, it follows that b a+ c + c +b a + a +c b > 2.
This ends the proof.
Generalization of inequalities (3.1).
Note that (3.1) can be generalized for positive numbers a1 , . . . , an .
√ ··· a12 + ··· + an2
The expressions 1 + ··· + 1 , n a1 · · · an , 1 n n ,
n a + + a
n
are called the har-
a1 an
monic mean (HM), geometric mean (GM), arithmetic mean (AM), and quadratic
mean (QM) or root mean square of the numbers a1 , . . . , an

Lemma (HM-GM-AM-QM inequalities for n arbitrary positive numbers) Let


a1 , . . . , an be positive real numbers. Then
3 The HM-GM-AM-QM Inequalities 23

n √ a1 + · · · + an a12 + · · · + an2
≤ n a1 · · · an ≤ ≤ . (3.5)
1
a1
+ ··· + 1
an
n n

Moreover, in (3.5), inequality holds if and only if a1  · · ·  an .

Proof See Example 3.3, Problems 2.1 and 2.2.



Example 3.3 (HM-GM) Prove that 1
n
+ ··· + 1 ≤ n
a1 · · · an , where n ≥ 2, a1 >
a1 an
0, . . . , an > 0.
x1 + ··· + xn √
Proof Using the inequality n
≥ n
x1 · · · xn for positive numbers xi 
1
+ ··· + 1
1
ai
, i 1, . . . , n (see Problem 2.1), we obtain the following inequality: a1
n
an

n a ··· a .
√ 1
1 n
This ends the proof.

Remark a1 + ···n + an ≥ n a1 · · · an ≥ 1
n
+ ··· + 1 , whence
a1 an


1 1
(a1 + · · · + an ) + ··· + ≥ n2 . (3.6)
a1 an

Let us consider the following examples in order to see how these inequalities can
be applied.

Example 3.4 Solve the following system:


⎧ 
⎪   1

⎨ 1 + x1 + · · · + 1 + x100  100 1 + ,
100


⎪  
⎩ 1 − x + · · · + 1 − x  100 1 − 1 .
1 100
100

Solution. Using the inequality



a1 + · · · + an a12 + · · · + an2
≤ (3.7)
n n
√ √
(see Problem√ 2.2) for the numbers
√ 1 + x1 , . . . , 1 + x100 and
1 − x1 , . . . , 1 − x100 , we deduce that
√ √ 
1 + x1 + · · · + 1 + x100 100 + (x1 + · · · + x100 )

100 100
√ √ 
1 − x1 + · · · + 1 − x100 100 − (x1 + · · · + x100 )
and ≤ . (3.8)
100 100
24 3 The HM-GM-AM-QM Inequalities

From the obtained inequalities and the given system, we deduce that
   
1 x1 + · · · + x100 1 x1 + · · · + x100
1+ ≤ 1+ , and 1− ≤ 1− .
100 100 100 100

Therefore, x1 + · · · + x100  1, and thus it follows that inequality (3.8) is an


equality. √ √
Hence, we obtain that 1 + x1  · · ·  1 + x100 , or x1  · · ·  x100  0.01.
It is not difficult to verify that these numbers satisfy the given system.
The relationship between the arithmetic mean and the geometric mean helps in
solving problems the involve finding maximum and minimum values.

Remark 1 Consider all n-tuples (a1 , . . . , an ) of positive numbers having the same
geometric mean. Prove that among those n-tuples, the minimum arithmetic mean has
the n-tuple with a1  · · ·  an .

Remark 2 Consider all n-tuples (a1 , . . . , an ) of positive numbers having the same
arithmetic mean. Prove that among those n-tuples, the minimum geometric mean has
the n-tuple with a1  · · ·  an .

Example 3.5 Among all triangles with a given area, find the triangle that has the
minimum perimeter.

Proof Let us denote the side lengths of that triangle by a, b, c, and the perimeter
by 2p.
By Heron’s formula and  the inequality of Problem 2.1, we obtain S 
  3 √
p(p − a)(p − b)(p − c) ≤ p (p − a) + (p −3 b) + (p − c)  93 p2 , and therefore,
 √
2p ≥ 2 3 3 · S.
Equality holds if a  b  c. Therefore, among all triangles with a given area, the
one with the minimum perimeter is an equilateral triangle.

Example 3.6 Among the triangles with perimeter equal to 2p, find the triangle with
the maximum area.

Proof Denote the area of that triangle by S, and the side lengths by a, b, c.
√ 
By Heron’s formula, S  p · (p − a)(p − b)(p − c). Let us find the maximum
value of the expression (p − a)(p − b)(p − c). Since (p − a) + (p − b) + (p − c)  p
is a constant, it follows from Remark 2 that the product (p − a)(p − b)(p − c) attains
its maximum value when p − a  p − b  p − c. Therefore, a  b  c, and so
among all triangles with perimeter equal to 2p, the one with maximum area is an
equilateral triangle.
This ends the proof.
Problems 25

Problems

Prove the following inequalities (3.1–3.28).


3.1. (a + b)(b + c)(c + a) ≥ 8 abc, where a > 0, b > 0, c > 0.
3.2. (a + b + c − d )(b + c + d − a)(c + d + a − b)(d + a + b − c) ≤ (a + b)(b +
c)(c + d )(d + a),
where a > 0, b > 0, c > 0, d > 0.
3.3. (a) Schur’s inequality: a3 + b3 + c3 + 3abc ≥ a2 b + ab2 + b2 c + c2 b + a2 c + ac2 ,
where
 a >0, b > 0,  c > 0. 
(b) 1 + b4a +c
1 + c4b +a
1 + a4c+b
> 25, where a > 0, b > 0, c > 0.
− 1)
3.4. log(a
log a
< log a
log(a + 1)
, where a > 1.
3.5. Schur’s inequality (alternative form): abc ≥ (a + b − c)(a + c − b)(b + c − a),
where a > 0, b > 0, c > 0.
3.6. x8 + y8 ≥ 128 1
, if x + y  1.
 
1 2
 2
3.7. a + a + b + 1b ≥ 12.5, if a > 0, b > 0 and a + b  1.
 2  2
≥ ( n ) , if n ≥ 2, x1 > 0, . . . , xn >
2
n2 + 1
3.8. x1 + x11 + · · · + xn + x1n
0 and x1 + · · · + xn  1.
3.9. a4 + b4 + c4ó abc (a + b + c).
3.10. x√2 + y2 ≥ 2 √2(x − y), if√ xy  1. √ √ √
3.11. 6a1 + 1 + 6a2 + 1 + 6a3 + 1 + 6a4 + 1 + 6a5 + 1 ≤ 55, if a1 >
0, . . . , a5 > 0 and √ a1 + ·√ · · + a5  √1.
3.12. 6a + 4b +5c ≥ 5 ab + 3 bc + 7 ca, where a ≥ 0, b ≥ 0, c ≥ 0.
3.13. 2 a4 + b4 + 17 > 16 ab.
3.14. (a) b + c1− a + a −1b + c + a + b1 − c ≥ a1 + 1b + 1c , where a, b, c are the side lengths
of some triangle. √
(b) √2b2 + a2c2 − a2 + √2c2 + b2a2 − b2 + √2a2 + c2b2 − c2 ≥ 3, where a, b, c are the
side lengths of some triangle.
 n+1
3.15. 1n++nb ≥ bn , where n ∈ N, b > 0.
1
 n  n+1
3.16. (a) 1 + 1n < 1 + n +1 1 , where n ∈ N,
 
1 n+1
 
1 n+2
(b) 1 + n > 1 + n+1 , where n ∈ N,
  n−1  n   n
(c) 1 + n − 1 m m
< 1+ n m m
< 1 + m− n
1 m−1
, where m > 1, n > 1, and
m ∈ N, n ∈ N.
 n
3.17. n! < n +2 1 , where n  2, 3, 4, . . . .
1
3.18. (a) n(n + 1) n < n + Sn , where Sn  11 + · · · + 1n , n  2, 3, 4, . . . ,
1
(b) n − Sn > (n − 1)n 1−n , where Sn  11 + · · · + 1n , n  3, 4, . . . .
 
3.19. (qn − 1) qn+1 + 1 ≥ 2nqn (q − 1), where q > 1, n ∈ N.
26 3 The HM-GM-AM-QM Inequalities

3.20. (a) a2 + b2 + c2 + d 2 + ab + ac + ad + bc + bd + cd ≥ 10, where a > 0, b >


0, c > 0, d > 0, and abcd  1.
     abc 3
(b) a − 1 + 1b b − 1 + 1c c − 1 + a1 ≤ 12√+ abc , where a, b, c > 0.
     
(c) a + b − t b + c − t c + a − t ≤ (a + b + c) a1 + 1b + 1c (1 − t)2 +
1 1 1

4 − 3t, where a, b, c, t > 0 and abc  1.


√ √
3.21. n n a1 · · · an −(n − 1) n−1 a1 · · · an−1 ≤ an , where ai > 0, i  1, . . . , n, n 
3, 4, . . ..
√  
n
a1 · · · an + n b1 · · · bn + . . . + n k1 · · · kn
3.22. 
≤ n (a1 + b1 + · · · + k1 )(a2 + b2 + · · · + k2 ) · · · (an + bn + · · · + kn ),
where a1 > 0, . . . , an > 0, b1 > 0, . . . , bn > 0, . . . , k1 > 0, . . . , kn > 0.
√ √
3.23. a1 + a1 a2 + · · · + n a1 · · · an ≤ e(a1 + · · · + an ), where n ≥ 2, a1 ≥
0, . . . , an ≥ 0.
3.24. (a) nak − kan ≤ n − k, where n > k, n, k ∈ N, a > 0.
x2 x3 xn+1
(b) x12 + x22 + · · · + xn n ≥ x1 + · · · + xn , where n ≥ 2, n ∈ N, x1 
3 1
min(x1 , . . . , xn ) > 0.
ax1 −x2 ax2 −x3 axn −x1 n2
3.25. x1 + x2
+ x2 + x3
+ ··· + xn + x1
≥ 
n , where a > 0, xi > 0, i  1, . . . , n.
2 xi
i1

√ √
3.26. x1 + 1 + · · · + p xn + 1 ≤ n + 1, where n ≥ 2, x1 > 0, . . . , xn > 0, x1 + · · · +
p

xn  p, p ∈ N, p ≥ 2.
3.27. (a) (1 + α)n ≥ 1 + n α, where α ≥ −1, n ∈ N (Bernoulli’s inequality).
(b) (1 + α)n > 1 + n α, where α  0, α ≥ −1, n ∈ N, n > 1.

3.28. (a) cos3 t sin t ≤ 3 3
16
,
k
(b) xk (1 − xm ) ≤ k ·m1+ k , where 0 ≤ x ≤ 1, k, m ∈ N,
m

(k + m) m √
y
(c) x
1 − x2
+ 1 − y2
+ z
1 − z2
≥ 3 2 3 , where x, y, z > 0 and x2 + y2 + z 2  1,

(d) 1
1−x
+ 1 −1 y + 1 −1 z ≥ 9 + 23 3 , where x, y, z > 0 and x2 + y2 + z 2  1.

3.29. Find the minimum value of the function f (x)  √


n
1
1+x
+ √n 11− x in [0, 1), where
n ∈ N, n > 1.
3.30. Find the minimum value of the function f (x)  axm + xbn in (0, ∞), where
a > 0, b > 0, m, n ∈ N.
3.31. Find in [a, b] (0 < a < b) a point x0 such that the function f (x) 
(x − a)2 b2 − x2 attains its maximum value in a, b
at x 0 .
3.32. Find the greatest
√ possible value of the product xyz given x > 0, y > 0, z > 0,
and 2x + 3y + π z  1.
3.33. Find the maximum and minimum values of the function y  ax2x+ b , where
a > 0, b > 0.
Problems 27

5 x2 + 6x + 8 + 12
3.34. (a) Find the maximum value of the function y  .
 x+3
3
(x + 1) (x + 3)
2 2 2
(b) Find the maximum value of the function y  3x2 + 4
.

3.35. Suppose the sum of the six edges of a triangular pyramid (tetrahedron) PABC
is equal to S, and  APB   BPC   CPA  900 . Find among such pyramids
one with the greatest volume.
3.36. Solve the system of equations

x + y  2,
xy − z 2  1.

3.37. Solve the system of equations



x + y + z  3,
x2 + y2 + z 2  3.

3.38. Given numbers a, b, c, d , e such that



a + b + c + d + e  8,
a2 + b2 + c2 + d 2 + e2  16,

find the greatest possible value of e.

3.39. Find the minimum value of the expression x1


x2
+ x3
x4
+ x5
x6
if 1 ≤ x1 ≤ x2 ≤ x3 ≤
x4 ≤ x5 ≤ x6 ≤ 1000.

3.40. Solve the equation x4 + y4 + 2  4xy.


xy yz
3.41. Find all integer solutions of the equation z
+ xz
y
+ x
 3.

3.42. Prove that


β β
(a) x1α + · · · + xnα ≥ x1 + · · · + xn , where n ≥ 2, x1 > 0, . . . , xn > 0, α >
β ≥ 0, and x1 · · · xn  1,
β β
(b) x1α + · · · + xnα ≥ x1 + · · · + xn , where n ≥ 2, x1 > 0, . . . , xn > 0, α ≥
(n − 1)|β|, and x1 · · · xn  1,

3.43. Prove that x2 y +y2 z +z 2 x ≤ 4


27
, where x ≥ 0, y ≥ 0, z ≥ 0, and x +y +z  1.

3.44. Prove that

(a) 1+a
+ 11++bc
1 + ab
b
+ 11++cdc + 1+d
1 + da
≥ 4, where a > 0, b > 0, c > 0, d > 0,
and abcd  1,
(b) 11++ab
a
+ 11++bc
b
+ 11++cdc + 1 + da
1+d
≥ 4, where a > 0, b > 0, c > 0, d > 0,
and abcd  1.
28 3 The HM-GM-AM-QM Inequalities

3.45. Prove that 2ST > 3(S + T )(S(bd + df + fb) + T (ac + ce + ea)), where 0 <
a < b < c < d < e < f and a + c + e  S, b + d + f  T .
3.46. Prove

that
√ 3 √
4

a+ ab + abc +
4
abcd
≤ 4
a· a+b
2
· a+b+c
3
· a+b+c+d
4
, where a > 0, b >
0, c > 0, d > 0.
3.47. Prove that a12 + (ab)6 + (abc)4 + (abcd )3 ≤ 1, 43(a12 + b12 + c12 + d 12 ).

Proofs

3.1. Let us use the inequality√(3.2) for positive


√ numbers a√and b, b and c, c and a.
Then we obtain a +2 b ≥ ab, b +2 c ≥ bc, c+a 2
≥ ca, and on multiplying
these inequalities, we deduce that (a + b)(b + c)(c + a) ≥ 8abc.
3.2. Note that three factors on the left-hand side of the given inequality are positive.
If only one factor on the left-hand side is not positive, then the proof is obvious.
Consider the case in which all four factors are positive. In this case, according
to inequality (3.2) for the numbers a + b + c − d and b + c + d − a, a + b + c − d
and d + a + b − c, b + c + d − a and c + d + a − b, c + d + a − b and d + a + b − c,
we obtain the following inequalities:
 (a + b + c − d ) + (b + c + d − a)
(a + b + c − d )(b + c + d − a) ≤  b + c,
2
 
(a + b + c − d )(d + a + b − c) ≤ a + b, (b + c + d − a)(c + d + a − b) ≤ c + d ,

(c + d + a − b)(d + a + b − c) ≤ a + d ,

and on multiplying these inequalities, we obtain the required inequality.


3.3. (a) The given inequality is equivalent to the following inequality:
abc ≥ (a + b − c)(a + c − b)(b + c − a). See Problem 3.5.
(b) The given inequality is equivalent to the following inequality: 7abc + a3 +
b3 + c3 > a2 b + b2 a + c2 a + a2 c + b2 c + c2 b, and the proof follows from
the inequality of Problem 3.3(a).
3.4. The given inequality is equivalent to the following inequality:
log(a − 1) log(a + 1) < log2 a. If log(a − 1) ≤ 0, then log(a − 1) log(a + 1) ≤
0 < log2 a.
Otherwise,
 if log(a − 1) > 0, then using inequality (3.2),  we deduce
 that
2 log(a − 1) log(a + 1) ≤ log(a − 1) + log(a + 1)  log a2 − 1 < log a2 .
This inequality is equivalent to the inequality log(a − 1) log(a + 1) < log2 a.
Alternative proof. The given inequality is equivalent to the following inequal-
ity:
− 1) log a log a + 1
1 − log(a
log a
log a
> 1 − log(a + 1)
, or loga −a 1 > log(a +a 1) . This proof follows from the
inequalities a −a 1 > a +a 1 > 1 and a + 1 > a > 1.
Proofs 29

3.5. Since a > 0, b > 0, c > 0, at least two factors on the right-hand side of
the inequality are positive. If only one factor is not positive, then the proof is
obvious. Consider the case in which all three factors on the right-hand side are
positive.

Since x +2 y ≥ xy for nonnegative numbers, it follows that
 (a + b − c) + (a + c − b) 
(a + b − c)(a + c − b) ≤  a, (a + b − c)(b + c − a) ≤ b,
2

(a + c − b)(b + c − a) ≤ c.

On multiplying these inequalities, we obtain the required inequality.

3.6. We have

2
4  x + y 8
x8 + y8 x4 + y4 x2 + y2 1
≥ ≥ ≥  .
2 2 2 2 128

3.7. Since a + b  1, we deduce from (3.2) that 1


ab
≥ 4. Hence, according to
inequality (3.7), we obtain

 2  2  2  2
2
a + a1 + b + 1b a+ 1
+b+ 1 1
1 + ab 1+4 25
≥ a b
 ≥  .
2 2 2 2 4

3.8. Since x1 + · · · + xn  1, we deduce from (3.6) that x11 + · · · + x1n ≥ n2 . Using


inequality (3.7), we obtain


 2
1 2 1 2 x1 + x11 + · · · + xn + x1n
x1 + + · · · + xn + ≥n 
x1 xn n
 2
2  2
1 + x11 + · · · + x1n 1 + n2 1 + n2
n ≥n  .
n n n

3.9. According to inequality (3.2), we have a4 +b4 ≥ 2a2 b2 , b4 +c4 ≥ 2b2 c2 , c4 +


a4 ≥ 2c2 a2 .
Summing these inequalities, we obtain

a4 + b4 + c4 ≥ a2 b2 + b2 c2 + c2 a2 . (3.9)

According to inequality (3.2), we have

a2 b2 + b2 c2 ≥ 2ab2 c, b2 c2 + c2 a2 ≥ 2abc2 , c2 a2 + a2 b2 ≥ 2a2 bc.

Summing these inequalities, we deduce that

a2 b2 + b2 c2 + c2 a2 ≥ abc(a + b + c). (3.10)


30 3 The HM-GM-AM-QM Inequalities

From inequalities (3.9) and (3.10) it follows that a4 + b4 + c4 ≥ abc(a + b + c).

3.10. Using that x2 + y2  (x − y)2 + 2xy and xy  1, we deduce that x2 + y2 


√ 2 √
(x − y)2 + 2 ≥ 2 2(x − y).

3.11. According to inequality (3.2), for every value of λ, one has the following
inequalities:

(6ai + 1) + λ2 ≥ 2λ 6ai + 1, i  1, . . . , 5.

Summing these inequalities and using that a1 + · · · + a5  1, for λ > 0 we


 n √
+ 5λ2
obtain 11 2λ ≥ 6ai + 1.
i1
 √ 5 √

Taking λ  11 5
, it follows that 55 ≥ 6ai + 1.
i1

3.12. According
√ √to inequality
√ (3.2), we have
5 ab + 7 ac + 3 bc ≤ 5(a2+ b) + 7(a2+ c) +  6a + 4b + 5c. 3(b + c)
2
 
 2 2a + b ≥ 2a b , it follows that 2 a4 + b4 + 17 ≥ 4a2 b2 + 17 >
4 4 2 2
3.13. Since
4 a b + 4 ≥ 16ab, and therefore 2 a4 + b4 + 17 > 16ab.
1 1 1
3.14. (a) Using inequality (3.6) for the numbers b+c−a
and ,
a−b+c b+c−a
1 1
b+c−a + a−b+c
and 1
a+b−c
, a −1b + c and 1
a+b−c
, we obtain that 2

1 1 1
b+c−a + a+b−c + a + b1 − c
1
c
, 2
≥ 1b , a−b+c

Summing these inequalities,
2
1
a
.
we obtain the required inequality.
(b) According to the inequality of Problem 1.2, we have
a b c
 +  +  
2b2 + 2c2 − a2 2c2 + 2a2 − b2 2a2 + 2b2 − c2
 
√ a2 b2 c2
 3  +  +  ≥
3a2 (2b2 + 2c2 − a2 ) 3b2 (2c2 + 2a2 − b2 ) 3c2 (2a2 + 2b2 − c2 )
⎛ ⎞
√ a2 b2 c2 √
≥ 3⎝ 2 2 2 2
+ 2 2 2 2
+ 2 2 2 2
⎠  3,
3a + (2b + 2c − a ) 3b + (2c + 2a − b ) 3c + (2a + 2b − c )
2 2 2


and therefore, √ a
2b2 +2c2 −a2
+ √ b
2c2 +2a2 −b2
c
+ √2a2 +2b2 −c2
≥ 3.
√  n+1
· · + b ≥ (n + 1)
3.15. One has 1 + b + · bn , and thus it follows that 1n++nb ≥
n+1
1
n
bn .

3.16. (a) Using the inequality of


Problem 2.1 for the  numbers
 n
(1 + 1/n), . . . , (1 + 1/n), 1, we obtain ( n ) n + (1 n ) > n+1 1 + 1n ,
1+ 1 +...+ 1+ 1 +1
  
 nn+1  n
or 1 + n +1 1 > 1 + 1n .
Proofs 31

(b) Using the inequality of Problem 2.1 for the numbers


1 1
1− ,...,1 − , 1, we obtain
 n + 1  n + 1
  n+1 
1 n+2
n+1  n+1  n+2
1 − n+2 > 1 − n +1 1 or 1 + 1n > 1 + n +1 1 .
(c) Using the inequality of Problem 2.1 for the numbers
m m   n−1  n
1+ ,...,1 + , 1, we obtain 1 + n −
m m
< 1 + mn m .
 n − 1  n − 1 1

n−1
Using the inequality of Problem 2.1 for the numbers
m m  n   n
1 + , . . . , 1 + , 1, we obtain 1 + mn m < 1 + m −
n
1 m−1
.
 n  n
m−1
√ √ 1+ ··· +n
3.17. Using the inequality of Problem 2.1, we obtain n
n!  n 1 · · · n < n

n+1
2
.
 √
3.18. (a) Sn +n  n+ 11 +· · ·+ 1n  2+ 23 +· · ·+ n +n 1 > n· n 2 · 23 · · · n +n 1  n· n n + 1.

(b) n − Sn  21 + · · · + n −n 1 > (n − 1) · n−1 21 · · · n −n 1  (n − 1)n 1−n .
1

 
3.19. We have qn − 1  (q − 1) qn−1 + · · · + 1 and q > 1. Therefore we obtain
an inequality equivalent to the given inequality:
 n−1  
q + · · · + 1 qn+1 + 1 ≥ 2nqn . (3.11)

According to Problem  2.1 for the casen−1n > 1, we have n+1


qn−1 + · · · + 1 ≥ n · n qn−1 · · · 1  nq 2 and qn+1 + 1 ≥ 2q 2 .
Multiplying these inequalities, we obtain (3.11).

3.20. (a) a2 + b2 + c2 + d 2 + ab + ac + ad + bc + bd + cd ≥ 10 · (abcd )5  10.
10

(b) Since a, b, c > 0, at least two factors on the left-hand side are positive
(see Problem 1.9(a)). If only one factor on the left-hand side is nonposi-
tive, then the proof is obvious. Consider the case in which all three factors
on the left-hand side are positive. In this case, note that





1 1 1 1
3 + 3abc  b a − 1 + +c b−1+ +a c−1+ + bc a − 1 + +
b c a b



1 1
+ ac b − 1 + + ab c − 1 + .
c a

According to Problem 2.1, we deduce that







1 2 1 2 1 2
3 + 3abc ≥ 6 a3 b3 c3 a − 1 + b−1+ c−1+ .
3

b c a
32 3 The HM-GM-AM-QM Inequalities

This is equivalent to the given inequality.


(c) Define a + b + c  A and a1 + 1b + 1c  B. Then one needs to prove that


a b c
t 3 + (AB − A − B)t 2 + + + + A + B − 2AB t + AB + 2 − A − B ≥ 0. (1)
c a b

For t  1, inequality (1) holds, since for t  1 it is equivalent to Problem


3.2(b) for abc  1.
Therefore, for t  1, from (1) it follows that ac + ab + bc ≥ A + B − 3.
√ 
Since t > 0,A  a + b + c ≥ 3 abc  3, B  a1 + 1b + 1c ≥ 3 3 abc 1
 3,
3

and AB + 2 − A − B  (A − 1)(B − 1) + 1 > 0, it follows that


a b c
t 3 + (AB − A − B)t 2 + ( + + + A + B − 2AB)t + AB + 2 − A − B ≥
c a b
≥ t 3 + (AB − A − B)t 2 + (2A + 2B − 2AB − 3)t + AB + 2 − A − B 
 (t − 1)2 (t + AB + 2 − A − B) ≥ 0,
√ √
3.21. an + a1 · · · an−1 + · · · + n−1 a1 · · · an−1 ≥
n−1
  
n−1
 √ √ √
n· an · n−1
a1 · · · an−1 · · · n−1
a1 · · · an−1 , and therefore n n a1 · · · an −
n  
n−1

(n − 1) n−1 a1 · · · an−1 ≤ an .

3.22. a1 a2 an
n
· ··· +
a1 + b1 + · · · + k1 a2 + b2 + · · · + k2 an + bn + · · · + kn

b1 b2 bn
+ n · ··· + ···+
a1 + b1 + · · · + k1 a2 + b2 + · · · + k2 an + bn + . . . + kn

k1 k2 kn
+ n · ··· ≤
a1 + b1 + · · · + k1 a2 + b2 + · · · + k2 an + bn + · · · + kn


1 a1 a2 an
≤ + + ··· + +
n a1 + b1 + · · · + k1 a2 + b2 + · · · + k2 an + bn + · · · + kn


1 b1 b2 bn
+ + + ··· + + ···+
n a1 + b1 + · · · + k1 a2 + b2 + · · · + k2 an + bn + · · · + kn


1 k1 k2 kn
+ + + ... + 
n a1 + b1 + · · · + k1 a2 + b2 + · · · + k2 an + bn + · · · + kn


1 a1 b1 k1
 + + ··· + +
n a1 + b1 + · · · + k1 a1 + b1 + · · · + k1 a1 + b1 + · · · + k1


1 a2 b2 k2
+ + + ··· + + ···+
n a2 + b2 + · · · + k2 a2 + b2 + · · · + k2 a2 + b2 + · · · + k2


1 an bn kn
+ + + ... +  1.
n an + bn + · · · + kn an + bn + · · · + kn an + bn + · · · + kn
Proofs 33

3.23. Note that


1 1 1
a1 ≥ a1 + a1 + · · · + a1 ,
1·2 2·3 n(n + 1)
2 2 2
a2 ≥ a2 + a2 + · · · + a2 ,
2·3 3·4 n(n + 1)
...

an ≥ n(nn+ 1) an , and therefore,


a1 + 2a2 + ··· + kak a1 + 2a2 + ... + nan
a1 + a2 + · · · + an ≥ a21 + a1 2·3
+ 2a2
+ ··· + k(k + 1)
+ ··· + n(n + 1)
.
2 + ··· + kak
1
Let us prove that e a1 + 2a
k(k + 1)
≥ (a1 · · · ak ) k , k  1, . . . , n.
According to Problem 2.1, we have
√ √
a1 + 2a2 + · · · + kak a1 · 2a2 · · · kak
k k
k! 1
≥  (a1 · · · ak ) k .
k(k + 1) k +1 k +1

In order to complete the proof, one needs to prove that



k  k
k!
k +1
> 1e or k! > k +e 1 (see Problem 7.18).
Hence, we deduce that

n
a1 + 2a2 + · · · + kak 
n
1
e(a1 + · · · + an ) ≥ e ≥ (a1 · · · ak ) k .
k(k + 1)
k1 k1

Now let us prove that it is not possible to substitute e in the given inequality
n 1
by a number smaller than e. Indeed, let c(a1 + · · · + an ) ≥ (a1 · · · ak ) k .
k1
Then taking ak  k1 , k  1, . . . , n, it follows that


n
1 
n
1 
n
e
c ≥ 1 > √ (3.12)

k
k1
k
k1 (k!) k
k8
k

(here we have used the inequality from Problem 14.18).


  k
On the other hand, according to Problem 7.17, we have 1 + k2 > 1+
  2 √ 
k k2 + k(k−1) 2
> k, whence k < 1 + k2 , and therefore,
k
2 k


n
e  e
n
√ >   . (3.13)
k· k
k
2
k8 k8 k 1 + k
34 3 The HM-GM-AM-QM Inequalities

From (3.12) and (3.13), it follows that


n n
e  1√ < c 1
, or
k 1 + 2 k
k8 k k1
√2
n 
n 
n
e· 1
− e ·  √ k <c 1
, and therefore,
k 2 k
k8 k8 k 1 + k k1
n n √ 
n
e· 1
k
− e· √2
k k
< c 1
k
, or
k8 k8 k1
n 
n 
7
√ 1√
k k
1
k
1
k
αn · e − 2e · k1
n < c, where αn  k8
n 1− k1
n .
1 1 1
k k k
k1 k1 k1
Passing to the limit, we obtain e ≤ c (see Problems 7.15 and 7.16).
3.24. (a) Note that n − k + kan  1 + ·
· · + 1 + a n + ·
· · + an ≥ n ·
 n−k k

n
1 · · · 1 · an ·
· · an  nak ,
k
whence n − k ≥ nak − kan . 
k k
(b) Note that yxk−1 + (k − 1)y ≥ k k yxk−1 · yk−1  kx, where k ∈ N, k ≥ 2,
whence

xk
≥ kx − (k − 1)y. (1)
yk−1

Using inequality (1), we obtain

x12 x23 xn+1


+ 2 + · · · + nn
x2 x3 x1
≥ (2x1 − x2 ) + (3x2 − 2x3 ) + · · · + (nxn−1 − (n − 1)xn )+
+ ((n + 1)xn − nx1 )  2(x1 + · · · + xn ) − nx1 ≥ x1 + · · · + xn ,

x12 x23 xnn+1


whence x2
+ x32
+ ··· + x1n
≥ x1 + · · · + xn .

ax1 −x2 x2 −x3 xn −x1
ax1 −x2 ax2 −x3 axn −x1
3.25. Note that x1 + x2
+ ax2 + x3 + · · · + axn + x1 ≥ n· n
x1 + x2
· x2 + x3
··· xn +x1

n2

n
n
(x1 + x2 )·(x2 + x3 ) ··· (xn +x1 )
≥ (x1 + x2 )+ ···n +(xn + x1 )  
n .
n 2 xi
i1

3.26. Using the inequality of Problem 2.1 for the numbers xi + 1 and 1, 1, . . . , 1,
  
√  p−1
where i  1, . . . , n, we obtain p xi + 1  p (xi + 1) · 1  · 1 ≤
· ·

√ √ 
p−1
(xi + 1)+(p − 1)·1
 1 + xi
, whence p
x1 + 1 + · · · + p
xn + 1 ≤ 1 + x1
+ ··· +
 p
p p

1 + xpn  n + x1 +···+x
p
n
 n + 1.
Proofs 35

3.27. (a) Since 1+α ≥ 0, it is enough to prove the inequality


for the case 1+nα ≥ 0.
According to Problem 2.1, for n > 1, we obtain n (1 + nα) · 1 · · 1 ≤
 ·
n−1
1 + n α + 1 + ··· + 1
n
 1 + α. Therefore, 1 + nα ≤ (1 + α)n .
(b) See the proof of Problem 3.27(a).
2
cos2 t
 cos2 t

3.28. (a) cos6 t sin2 t  27 · cos3 t
· · 1 − cos2 t ≤

2 4 3 3
2 2
cos t cos t cos t   √
, whence cos3 t · sin t  ≤
2
3 + 3 + 3 +1−cos t
27 4
 27
44
3 3
16
,

and therefore cos3 t sin t ≤ 3 3
16
.
(b) We have
   1
m
  1 1
xk 1 − xm  xkm (1 − xm )m  mxk · · · mxk (k − kxm · · · k − kxm ) m · m ≤
mk ·k
  m+k k
1 mxk + · · · + mxk + (k − kxm ) + · · · + (k − kxm ) m
km ·m
≤ m ·  k
.
m k ·k m+k (m + k) m +1


(c) According to Problem 3.28(b), it follows that x(1 − x2 ) ≤ 2 3
9
, whence

x y z x2 y2 z2
+ +  + +
1 − x2 1 − y2 1 − z 2 x(1 − x2 ) y(1 − y2 ) z(1 − z 2 )

9  2  3 3
≥ √ x +y +z  2 2
.
2 3 2

y
Therefore, x
1 − x2
+ 1 − y2
+ z
1 − z2
≥ 3 3
2
.
(d) We have
1 1 1 1+x 1+y 1+z 1 1 1 x
+ +  + +  + + + +
1−x 1−y 1−z 1 − x2 1 − y2 1 − z 2 1 − x2 1 − y2 1 − z 2 1 − x2
y z 9 x y z √
+ + ≥ + + + ≥ 4.5 + 1.5 3,
1 − y2 1 − z2 1 − x2 + 1 − y2 + 1 − z 2 1 − x2 1 − y2 1 − z 2


(see Problem 3.28b). Therefore, 1
1−x
+ 1
1−y
+ 1
1−z
≥ 4.5 + 1.5 3.

3.29. From inequality (3.2), it follows that 


f (x)  (1 + x)− n + (1 − x)− n ≥ 2 (1 + x)− n (1 − x)− n  2n√1−x
1 1 1 1
2
2
≥ 2,
Since x ∈ [0; 1).
On the other hand, f (0)  2. Thus the minimum value of the function f (x) is
equal to 2.

3.30. Let us represent the function f (x) as


axm
n· +m· b
mxn
f (x)  (m + n) · n
.
m+n
36 3 The HM-GM-AM-QM Inequalities

By Problem 2.1, we
 obtain 
  b m
axm n m+n a n bm
f (x) ≥ (m + n) m+n
n
· mxn
 (m + n) nn mm
, where equality holds
axm
if  b
.
n mxn 
Therefore, x  m+n bn
.
am
 
n m
Answer. min f (x)  f (x0 )  (m + n) m + n nan mb m , where x0  m+n bn
am
.
(0,+∞)

3.31. Let us represent the function f (x) as


f (x)  (x − a)· (x − a)·αβ
α(b − x)· β(b + x)
, where α > 0, β > 0.
Using Problem 2.1, we deduce that

4 4 (x − a) · (x − a) · α(b − x) · β(b + x) ≤ (x − a) + (x − a) + α(b − x) + β(b + x) 
 (2 − α + β)x + (α + β)b − 2a.

Note that the right-hand side does not depend on x if α − β  2, and


equality holds if x − a  α(b − x)  β(b + x). Thus, it follows that
α  x−a
b−x
, β  x−a
b+x
.
Hence, from the equation α − β  2, we deduce

that 2x2 − ax − b2  0. This
2 2
equation has only one positive root, x0  a+ a4 +8b . One can easily prove that
x0 ∈ [a, b]. Therefore,f (x) attains its maximum value in [a, b] at the point
x0 . √
2 2
Answer. x0  a+ a4 +8b .
3.32. Let us represent the
√product xyz as
xyz  2√13π · 2x · 3y · π z, and according to Problem 2.1, for the numbers
√ √  √ 3
2x, 3y, π z, we have xyz  2√13π · 2x · 3y · π z ≤ 2√13π · 2x+ 33y+πz 
√ √
1

54 3π
, where inequality holds if 2x  3y  π z. From 2x + 3y + π z  1,

it follows that x  16 , y  9
3
,z  1

.
Answer. 54√1 3π .

3.33. Note that f (0)  0, and if x < 0, then f (x) < 0. On the other hand, if x > 0,
then f (x) > 0.
Therefore, the function f (x) attains its maximum value in (0, +∞), and its
minimum value in (−∞, 0).
First proof. From inequality (3.2) for the numbers ax2 and b, we have

ax2 + b √
≥ |x| · ab, (3.14)
2

where equality holds if ax2  b.


From inequality (3.14), it follows that x
ax2 +b
≤ √1 .
2 ab
Proofs 37

Therefore, the maximum value of the function f (x) is equal to 2√1ab , and it

attains this value at the point x  ab . Since f (x) is an odd function, its
minimum value is equal to − 2√1ab , which the function f (x) attains at the point

x  − ab .
Alternative proof. In (0, +∞), the function f (x) coincides with the function
g(x)  ax 1+ b . This function attains its maximum value at the point where the
x
function h(x)  ax + bx attains its minimum value.
Since the product ax · bx  ab is constant, the sum ax + bx attains its minimum

value when ax  bx , i.e., when x  ab .
Therefore,in (0, +∞), the function f (x) attains its maximum value at the
point x  ab , and it is equal to 2√1ab .
One can find the minimum value of the function f (x) similarly as in the first
proof.

5 (x + 2)(x + 4) + 12
3.34. (a) We have y  x+3
, and thus if x ≥ −2, then
√ 25x + 50 + x + 4
(25x + 50)(x + 4) + 12 + 12
y x+3
≤ 2
x+3
 13, and y  13
if 25x + 50 
x+ 4, i.e., if x  − 12 . If x ≤ −4, then y < 0.
23

Therefore, the maximum value of the function y is equal to 13.


Answer. 13.
(b) We have
       

3
x2 + 1 x2 + 1 25 x2 + 65 · 5
2 3 5
3
x2 + 1 x2 + 1 25 x2 + 65
y  · ≤
3x2 + 4 2 3x2 + 4
  
5 x2 + 1 + x2 + 1 + 25 x2 + 6
5 12x2 + 16 4 5
  ·   
3 3 3
≤ · 5
 · .
2 3 3x2 + 4 2 15 3x2 + 4 15 2


Note that y  4
· 3 5
, if x2 + 1  25 x2 + 65 , i.e., if x2  13 .
15 2

Therefore, the maximum value of the function y is equal to 4
· 3 5
.
 15 2

Answer. 154
· 3 25 .

3.35. Let
 PA  x,√PB  y, PC  z. Thus, it follows that S  x + y + z + x2 + y2 +
y + z + z + x and VPABC  6 xyz. Since a + b ≥ 2ab, it follows that
2 2 2 2 1 2 2
√ √ √ √  √ √ √ √ √
S ≥ x + y + z + 2 xy + yz + xz ≥ 3 3 xyz + 3 2 3 xy · yz · xz.
 3
Therefore, V ≤ 16 · 3 1 +S √2 , and if x  y  z  3 1 +S √2 , we have
( ) ( )
 3
V  6 · 3 1 + 2 . Then the volume of the pyramid with edge lengths
1 S√
( )
x  y  z  3 1 +S √2 is maximal.
( )
38 3 The HM-GM-AM-QM Inequalities

3.36. From inequality (3.2), it follows that 1 ≥ z 2 + 1, and therefore,


z  0, x  y  1.
2 2 2  2
3.37. Note that the inequality x + y3 + z ≥ x + 3y + z becomes an equality, since
x + y + z  3 and x2 + y2 + z 2  3.
Therefore, x  y  z  1 (see the proof of Problem 2.2).
Answer. (1, 1, 1).

3.38. We have  2  2
a2 + b2 + c2 + d 2 2

4
≥ a + b +4 c + d (Problem 2.2). It follows that 16−e
4
≥ 8−e
4
,
whence 5e − 16e ≤ 0, and therefore, 0 ≤ e ≤ 3.2.
2

If a  b  c  d  1, 2, then e  3.2. Thus, it follows that the maximum


value of e is equal to 3.2.
Answer. 3.2.

3.39. We have
 
x1 x3 x5 1 x2 x4 3 1 x2 x4 1
+ + ≥ + + ≥3 · · 33 ≥ 0.3.
x2 x4 x6 x2 x4 x6 x2 x4 x6 x6

If x1  1, x2  10, x3  10, x4  102 , x5  102 , x6  103 , then xx21 + xx43 +


x5
x6
 0.3.
Therefore, the minimum value of the expression xx21 + xx43 + xx56 is equal to 0.3.
Answer. 0.3.

3.40. From Problem 2.1,  for the numbers x 4 , y4 , 1, 1, it follows that


x + y + 1 + 1 ≥ 4 x · y4 · 1 · 1  4|x| · |y| ≥ 4xy, whence x4 + y4 + 2 ≥ 4xy,
4 4 4 4

and therefore, x4  y4  1.
Answer. (1, 1), (−1, −1).

3.41. We have (xy)2 + (yz)2 + (zx)2  3xyz, whence xyz > 0.


On the other hand, xyz ∈ Z, and hence xyz ≥ 1. From Problem 2.1, it follows

that 3xyz  (xy)2 + (yz)2 + (zx)2 ≥ 3xyz · 3 xyz ≥ 3xyz, and hence xyz  1,
and the inequality becomes an equality. It follows that (xy)2  (yz)2  (zx)2 .
Therefore, x2  y2  z 2  1.
Answer. (1, 1, 1), (1, −1, −1), (−1, 1, −1), (−1, −1, 1).

3.42. (a) Note that for x > 0, α > β ≥ 0, we have xα − xβ ≥ xα−β − 1, since

(xα−β − 1)(xβ − 1) ≥ 0.
Proofs 39

Therefore,
β β
x1α + · · · + xnα − (x1 + · · · + xnβ )  (x1α − x1 ) + · · · + (xnα − xnβ ) ≥

α−β α−β n α−β α−β
≥ (x1 − 1) + · · · + (xnα−β − 1)  x1 + · · · + xnα−β − n ≥ n x1 · · · xn − n  0.

β β
It follows that x1α + · · · + xnα ≥ x1 + · · · + xn .

(b) If β ≥ 0, then from Problem 3.42(a), it follows that x1α + · · · + xnα ≥


β β
x1 + · · · + xn .
If β < 0, then

β 1 1 −β −β −β
x1 + · · · + xnβ  −β
+ ··· + −β
 x2 · · · xn−β + · · · + x1 · · · xn−1 ≤
x1 xn
−β(n−1) −β(n−1) −β(n−1) −β(n−1)
x2 + · · · + xn x + · · · + xn−1
≤ + ··· + 1
n−1 n−1
−β(n−1)
 x1 + · · · + xn−β(n−1) ≤ x1α + · · · + xnα .

β β
Therefore, x1α + · · · + xnα ≥ x1 + · · · + xn .

3.43. Proof. We have


y2 + xz > 27 4
(3 − 3z + z 2 ), z 2 + xy > 27
4
(3 − 3x + x2 ), x2 + yz > 27
4
(3 − 3y + y2 ).
Summing these inequalities, we deduce that 23(x +y +z )+27(xy+yz +zx) >
2 2 2

24, which leads to a contradiction, since 23  23(x + y + z)2 ≥ 23(x2 + y2 +


z 2 ) + 27(xy + yz + zx).
Let z 2 + xy ≤ 27 4
(3 − 3x + x2 ). Then


4 0, 5y + 0, 5y + z 3 4
x2 y + y2 z + z 2 x  x(z 2 + xy) + y2 z ≤ x(3 − 3x + x2 ) + 4  ,
27 3 27

and therefore, x2 y + y2 z + z 2 x ≤ 27
4
.
Alternative proof. Without loss of generality one can assume that
max(x, y, z)  x, in which case x2 y +y2 z +z 2 x ≤ x2 y +xyz +0.5z 2 x +0.5zx2 
0.5x(x + z)(2y + z).
According to Problem 2.1 (AM-GM inequality), we obtain

x2 y + y2 z + z 2 x ≤ 0.5x(x + z)(2y + z)


x + (x + z) + (2y + z) 3 4
≤ 0.5  ,
3 27

and therefore, x2 y + y2 z + z 2 x ≤ 4
27
.
40 3 The HM-GM-AM-QM Inequalities

3.44 (a) Note that


1+a 1+b 1+c 1+d cd + acd ad + adb 1+c 1+d
+ + +  + + + 
1 + ab 1 + bc 1 + cd 1 + da cd + abcd ad + abcd 1 + cd 1 + da

c(1 + ad ) d (1 + ab) cd (1 + ab)
1+ +1+ ≥2+2  4,
1 + cd 1 + da 1 + cd

whence 1+a
1+ab
+ 1+b
1+bc
+ 1+c
1+cd
+ 1+d
1+da
≥ 4.

(b) Note that


1 + ab 1 + bc 1 + cd 1 + da cd + abcd 1 + bc 1 + cd bc + abcd
+ + +  + + + ≥
1+a 1+b 1+c 1+d cd + acd 1+b 1+c bc + bcd
4 4 4b
≥ (1 + cd ) + (1 + bc)  (1 + cd ) +
cd + acd + 1 + c 1 + b + bc + bcd bcd + abcd + b + bc
4 4(b(1 + cd ) + 1 + bc)
+ (1 + bc)   4,
1 + b + bc + bcd 1 + b + bc + bcd

and therefore, 1 + ab
1+a
+ 1 + bc
1+b
+ 1 + cd
1+c
+ 1 + da
1+d
≥ 4.
3.45. Note that (c − b)(c − d ) + (e − f )(e − d ) + (e − f )(c − b) < 0, and therefore,
(bd +df +fb)−(ac+ce+ea) < (c+e)(b+d +f −a−c−e), or τ −σ < s(T −S),
where τ  bd + df + fb, σ  ac + ce + ea, s  c + e.
We have Sτ + T σ  S(τ − σ )+ (S + T )σ < Ss(T − S) + (S + T )(ce + as) ≤
2  
≤ Ss(T − S) + (S + T ) s4 + as  s 2ST − 34 (S + T )s . It fol-
   
lows that 3
(S + T )(Sτ + T σ ) < 3
(S + T )s · 2ST − 43 (S + T )s ≤
 4
  4
1 3
(S + T )s + 2ST − 43 (S + T )s  ST .
2 4 
Therefore, 3(S + T )(S(bd + df + fb) + T (ac + ce + ea)) < 2ST .

3.46. Note that


√ √3
√4
a + ab + abc + abcd
 
4
a· a+b
2 · a+b+c
3 · a+b+c+d
4
 
4 2a 3a 4a 4 2a 3b 4b
 1· · · + 1· · · +
a +b a +b+c a +b+c +d a +b a +b+c a +b+c +d

12 2b 2b 2b 3a 3b 3c 4c 4c 4c
+ 1·1·1· · · · · · · · · +
a +b a +b a +b a +b+c a +b+c a +b+c a +b+c +d a +b+c +d a +b+c +d


4 2b 3c 4d 1 2a 3a 4a
+ 1· · · ≤ 1+ + + +
a +b a +b+c a +b+c +d 4 a +b a +b+c a +b+c +d


1 2a 3b 4b
+ 1+ + + +
4 a +b a +b+c a +b+c +d


1 6b 3a 3b 3c 12c
+ 3+ + + + + +
12 a +b a +b+c a +b+c a +b+c a +b+c +d


1 2b 3c 4d
+ 1+ + +  4.
4 a +b a +b+c a +b+c +d

Therefore,
√ √
3
√4

a + ab + abc + abcd 4 a +b a +b+c a +b+c +d
≤ a· · · .
4 2 3 4
3.47. Without loss of generality one can assume that a ≥ 0, b ≥ 0, c ≥ 0, d ≥ 0.
Proofs 41

Let x, y, z be arbitrary positive numbers. Then according to Problem 2.1


(AM-GM inequality), we obtain
1 1
a12 + (ab)6 + (abc)4 + (abcd )3  a12 + 6 (xa · b)6 + 4 8 (xya · yb · c)4 +
x x y
1 1
+ 3 6 9 (xyza · yzb · zc · d )3 ≤ a12 + 6 (x12 a12 + b12 )+
x y z 2x
1
+ 4 8 (x12 y12 a12 + y12 b12 + c12 )+
3x y
1
+ 3 6 9 (x12 y12 z 12 a12 + y12 z 12 b12 + z 12 c12 + d 12 )  A(a12 + b12 + c12 + d 12 ).
4x y z

We take the numbers x, y, z such that

x6 x8 y4 x9 y6 z 3 1 y4 y6 z 3
1+ + +  + +
2 3 4 2x6 3x4 4x3
1 z3 1
 4 8
+ 3 6   A.
3x y 4x y 4x y6 z 9
3

Therefore, x12  1 − 1
A
, y12  1 − √ 1
2 A(A − 1)
, z 12  1 −
 1
√ , and
3 A( A(A − 1) − 0,5)
3 2

  3
256  2
A 3 A A(A − 1) − 0, 5 − 1  1.
3
(1)
27

Consider the following function:



 3
√ 2
f (t)  27 t 3 t t(t − 1) − 0, 5 − 1 − 1 in [1, 42; 1, 43].
256 3

Note that
  3
256  2
3
f (1, 42)  · 1, 42 3 1, 42 1, 42 · 0, 42 − 0, 5 − 1 − 1 <
27

 3
256 3 256
< · 1, 42 3 1, 42 · 0, 27232 − 1 − 1 < · 1, 42(3 · 0, 4723 − 1)3 − 1 < 0 and
27 27
  3
256  2
3
f (1, 43)  · 1, 43 3 1, 43 1, 43 · 0, 43 − 0, 5 − 1 − 1 >
27
256   3 256
> · 1, 43 3 3 1, 43 · 0, 08 − 1 − 1 > · 1, 43(3 · 0, 48 − 1)3 − 1 > 0.
27 27

Therefore, there exists a number A such that 1, 42 < A < 1, 43 and f (A)  0.
Then (1) holds. It follows that x > 0, y > 0, z > 0 (A > 1.42) and a12 +
(ab)6 +(abc)4 +(abcd )3 ≤ A(a12 +b12 +c12 +d 12 ) ≤ 1, 43(a12 +b12 +c12 +d 12 ).
42 3 The HM-GM-AM-QM Inequalities

Problems for Independent Study

Prove the following inequalities (1–16, 20–26).


p q
1. ab ≤ ap + bq , if p1 + q1  1, a > 0, b > 0, p > 0, q > 0 where p and q are
rational
 nnumbers.
2. 1 + 1n > 2, where n ∈ N.
n
3. (1 + a1 ) · · · (1 + an ) ≤ 1 + 1!S + · · · + Sn! , where n ≥ 2, S  a1 + · · · + an , ai >
 i 1, . . . ,
0, n. 
4. 1 + a1 1 + 1b 1 + 1c ≥ 64, where a > 0, b > 0, c > 0 and a + b + c  1.
√ √
a2n−k + a2n+k ≥ 3a − 1, where n ≥ 2, a > 0, n > k, n, k ∈ N.
n n
5.
6. ana(a−−11) ≥ n + 1 − a 2 , where a > 0, a  1.
n n(n+1)

n+1
√ +√
7. na 1 ≥ (n  + 1)an , where a > 0.  √
√ √ √ √ √ √
8. k + k +1 k + 1 + k + 2 ··· n+ n+1 ≥ n− k n+

k − 1 + 2, where n > k, n, k ∈ N.
9. aa21 + · · · + aan−1
n
+ aan1 ≥ n, where ai > 0, i  1, . . . , n.
10. an+1 + a1 (a2 −a1 )(a3 −a1
2 )···(an+1 −an )
≥ n + 2, where 0 < ak < ak+1 , k  1, . . . , n.
11. 1 + 2 ≤ √1 − x , where 0 ≤ x < 1.
x 1

π
12. sin(2α) < 3α−α 2
3 , where 0 < α < .
 a 4  b 4  c 4  d 4  e 4 2 a b c d e
13. b + c + d + e + a ≥ b + c + d + e + a , where abcde  0.
 1999  b 1999  c 1999  d 1999
14. ab + c + d + a ≥ ab + bc + dc + da , where a > 0, b >
0, c> 0,d >  0.   √
15. (a) a1 a+3 a2 + a2 a+4 a3 + · · · + an−1a1+ an + an a+2 a1 ≥ n 2, where n > 2 and
a1 > 0, . . . , an > 0. √
(b) 1 +xx2 + 1 +yy2 + 1 +z z2 ≤ 3 4 3 , where x2 + y2 + z 2  1.
   n
16. a12 − 1 · · · a12 − 1 ≥ n2 − 1 , where n ≥ 2, a1 > 0, . . . , an > 0,
1 n
and a1 + · · · + an  1.
17. Find the maximum and minimum values of the expression (1 + u)(1 + v)(1 + w)
if 0 < u ≤ 16 7
, 0 < v ≤ 16 7
, 0 < w ≤ 16 7
, and u + v + w  1.
18. Find the maximum value of the expression xp yq if x + y  a, x > 0, y > 0,
and p, q ∈ N .
19. Find the maximum value of the expression a + 2c if for all x, one has ax2 +
bx + c ≤ √1 1− x2 , where |x|< 1.

Hint. Take x  √12 and x  − √12 . Then it follows that a + 2c ≤ 2 2.
√ √
Prove that if a  2, b  0, c  √12 , then 2x2 + √12 ≤ √1−x 1
for |x|< 1.
     2

20. 1 + b 1 + c 1 + a ≥ 2 1 + √3
a b c a+b+c
, where a > 0, b > 0, c > 0.
    abc
Hint. 1 + ab 1 + bc 1 + ac  2 + ab + ab + bc + bc + ac + ac and 13 ab + 13 ab + 13 bc ≥

3 a2
bc
 √3 a .
abc
Problems for Independent Study 43

21. 1 + a1
1 − a1
· 11 −+ aa22 · · · 11 −+ aan+1
n+1
≥ nn+1 , where −1 < a1 , a2 , . . . , an+1 < 1 and a1 + a2 +
· · · + an+1 ≥ n − 1.
Hint. We have that 1+ai ≥ (1−a1 )+ · · · +(1−ai−1 )+(1−ai+1 )+ · · · +(1−an+1 ).
22. (a + b)3 (b + c)3 (c + d )3 (d + a)3 ≥ 16a2 b2 c2 d 2 (a + b + c + d )4 , where a > 0, b >
0, c > 0,d > 0.
Hint. We have (a + b + c + d )2  (a + b)(b + c) + (a + b)(d + a) + (b + c)(c +
d ) + (c + d )(d + a).
 2  2  2  2  2  2
23. 1 + ab + 1 + bc + 1 + ac 1 + ab + 1 + bc + 1 + ac ≥
a+b c+a 2

4 c + a + b ,where a > 0, b > 0, c > 0.
b+c
 2  2  2  2
Hint. We have 1 + ab + 1 + bc + 1 + ac ≥ 13 1 + ab + 1 + bc + 1 + ac .
24. (a2 + bc)3 (b2 + ac)3 (c2 + ab)3 ≥ 64(a3 + b3 )(b3 + c3 )(c3 + a3 )a3 b3 c3 ,
where a > 0, b > 0, c > 0.
Hint. We have (a2 + bc)(b2 + ac)  c(a3 + b3 ) + ab(ab + c2 ).
√ √
25. (a) a + ab + abc ≤ 43 (a + b + c), where a > 0, b > 0, c > 0.
3

√ √ 
(b) a + ab + abc ≤ 3 3 a · a +2 b · a + 3b + c , where a > 0, b > 0, c > 0.
3

5 5 5

26. (ab) 4 + (bc) 4 + (ca) 4 ≤ 9
3
, where a > 0, b > 0, c > 0 and a + b + c  1.
Chapter 4
The Cauchy–Bunyakovsky–Schwarz
Inequality

Historical origins. The Cauchy–Bunyakovsky–Schwarz inequality, also known as


the Cauchy–Schwarz inequality, is one of the most important inequalities in mathe-
matics. The inequality for sums was published in 1821 by the French mathematician
Augustin-Louis Cauchy, born 21 August 1789 in Paris, France, died 23 May 1857
in Sceaux, one of the wealthy suburbs of Paris. Despite the fact that the influen-
tial French mathematician Pierre-Simon Laplace and the Italian–French mathemati-
cian Joseph-Louis Lagrange were Cauchy’s father’s friends and that on Lagrange’s
advice, Cauchy was enrolled in the École Centrale du Panthéon in Paris, France,
Cauchy never obtained a doctorate in mathematics. Besides being a prolific writer,
Cauchy was awarded the title of baron (in return for his services to King Charles X
of France, who asked Cauchy to be tutor to his grandson, the duke of Bordeaux). The
names of Cauchy, Laplace, and Lagrange are among the 72 names inscribed on the
Eiffel Tower. Cauchy was an advisor of 2 doctoral students, including a well-known
Russian mathematician Viktor Bunyakovsky.
The corresponding inequality for integrals was first proved in 1859 by Viktor
Bunyakovsky, born 16 December 1804 in Bar, Russian Empire (now Bar, Ukraine),
died 12 December 1889 in Saint Petersburg, Russian Empire (now Russia). In 1824,
Bunyakovsky received his bachelor’s degree from the University of Paris (the Sor-
bonne), Paris, France, and continuing his studies, he wrote three doctoral dissertations
under Cauchy’s supervision and obtained his doctorate in 1825.
The modern proof of the integral inequality was given in 1888 by the Prussian
mathematician Karl Hermann Amandus Schwarz, born 25 January 1843 in Herms-
dorf, Prussia (now Jerzmanowa, Poland), died 30 November 1921 in Berlin, Ger-
many. He obtained his doctorate from the University of Berlin in 1864 under the
supervision of the well-known mathematicians Ernst Kummer and Karl Weierstrass.
Schwarz was the advisor to 20 doctoral students.

© Springer International Publishing AG, part of Springer Nature 2018 45


H. Sedrakyan and N. Sedrakyan, Algebraic Inequalities, Problem Books
in Mathematics, https://doi.org/10.1007/978-3-319-77836-5_4
46 4 The Cauchy–Bunyakovsky–Schwarz Inequality

There are several generalizations of this inequality, one of which is known as


Hölder’s inequality, named after the German mathematician Otto Ludwig Hölder,
born 22 December in Stuttgart, Germany, died 29 August 1937 in Leipzig, Germany.
In 1877, Hölder began his studies at the University of Berlin, where he was a student
of the well-known mathematicians Leopold Kronecker, Ernst Kummer, and Karl
Weierstrass, and he obtained his doctorate under the supervision of Paul du Bois-
Reymond from the University of Tübingen in 1882. Hölder was the advisor of 58
doctoral students.
In this chapter we study some inequalities whose proofs involve the Cauchy–Bun-
yakovsky–Schwarz inequality. But first, let us prove the simplest case of the
Cauchy–Bunyakovsky–Schwarz inequality for arbitrary numbers a1 , a2 , b1 , b2 .
Consider the vectors a (a1 , a2 ) and b(b 1 , b2 ). 
 
  
It is known that a · b  a1 b1 + a2 b2  | a | · b cos a , b .
Let us estimate the absolute values of the scalar product a · b.  We have
        
       
a | · b · cos a , b  ≤ |
a · b  | a | · b.

On the other hand,


     
   
a | · b  a12 + a22 · b12 + b22 , or
a · b  |a1 b1 + a2 b2 | ≤ |

  
(a1 b1 + a2 b2 )2 ≤ a12 + a22 b12 + b22 . (4.1)

Inequality (4.1) is the simplest formulation of the Cauchy–Bunyakovsky–Schwarz


inequality for arbitrary numbers a1 , a2 , b1 , b2 .
Note that in (4.1), equality holds if and only if a1 b2 − a2 b1  0.
The following generalization of inequality (4.1) for arbitrary numbers
a1 , . . . , an , b1 , . . . , bn , is called the Cauchy–Bunyakovsky–Schwarz inequality:
 2  
a1 + · · · + an2 b12 + · · · + bn2 ≥ (a1 b1 + · · · + an bn )2 . (4.2)

First, let us prove inequality (4.2) for the numbers a1 ≥ 0, . . . , an ≥ 0, b1 ≥


0, . . . , bn ≥ 0.
  
Proof Let xk  a12 + · · · + ak2 b12 + · · · + bk2 , where k  1, . . . , n.

In this case,
  2 
xk+1  a12 + · · · + ak2 + ak+1
2
b1 + · · · + bk2 + bk+1
2

   
 2 2


a1 + · · · + ak + ak+1
2 2 2
b1 + · · · + bk + bk+1 ≥
2 2 2

 2
 
≥ a1 + · · · + ak · b1 + · · · + bk + ak+1 · bk+1  xk + ak+1 bk+1 .
2 2 2 2
4 The Cauchy–Bunyakovsky–Schwarz Inequality 47

Therefore, we obtain xk+1 ≥ xk + ak+1 bk+1 , where k  1, . . . , n − 1.


Summing these inequalities, we deduce that
  
a12 + · · · + an2 b12 + · · · + bn2 ≥ a1 b1 + · · · + an bn , or
 2  
a1 + · · · + an2 b12 + · · · + bn2 ≥ (a1 b1 + · · · + an bn )2 .

Now let us consider the case in which a1 , . . . , an , b1 , . . . , bn are arbitrary real


numbers.
In this case,
 2     
a1 + · · · + an2 b12 + · · · + bn2  |a1 |2 + · · · + |an |2 |b1 |2 + · · · + |bn |2 ≥
≥ (|a1 b1 | + · · · + |an bn |)2 ≥ |a1 b1 + · · · + an bn |2  (a1 b1 + · · · + an bn )2 .

This ends the proof.


Alternative proof.

 2  2  
n
 2
a1 + · · · + an b1 + · · · + bn − (a1 b1 + · · · + an bn ) 
2 2 2
ai b j − bi a j ≥ 0.
i; j1
i≥ j

This ends the proof.


Let us consider the following examples.

Example 4.1 Prove that sin α · sin β + cos α + cos β ≤ 2.

Proof According to the Cauchy–Bunyakovsky–Schwarz inequality, we have


 
sin α · sin β + cos α · 1 + 1 · cos β ≤ sin2 α + cos2 α + 12 · sin2 β + cos2 β + 12  2.

This ends the proof.

Example 4.2 Prove that a 3 + b3 > a 2 + b2 if a > 0, b > 0, and a 2 + b2 > a + b.

Proof According to the Cauchy–Bunyakovsky–Schwarz inequality, we have


         
 3  3 2 3 2 1 2 1 2 3 1 3 1 2  2
a + b3 (a + b)  a 2 + b 2 a2 + b2 ≥ a2 ·a2 +b2 ·b2  a 2 + b2 .

3 3 2 2
Since aa 2 +b
+b2
≥ aa+b
+b
> 1, it follows that a 3 + b3 > a 2 + b2 .
This ends the proof.
48 4 The Cauchy–Bunyakovsky–Schwarz Inequality

Problems

Prove the following inequalities (4.1–4.19, 4.22, 4.23).


4.1. a 2 + b2 + c2 ≥ 14 if a + 2b + 3c ≥ 14.
  
4.2. ab + 1 − a 2 1 − b2 ≤ 1 if |a| ≤ 1, |b| ≤ 1 .
√ √ √
4.3. √ c(a − c) + √c(b − c) ≤ ab if a > c, b > c, c > 0.
4.4. a a 2 + c2 + b b2 + c2 ≤ a 2 + b2 + c2 .
4.5. √1ab + √1bc + √1ca ≤ a1 + b1 + 1c , where a > 0, b > 0, c > 0.
√ √ √  
4.6. a(a + c − b) + b(a + b − c) + c(b + c − a) ≤ a 2 + b2 + c2 (a + b + c),
where a, b, c are  the side lengths  of a triangle.
4.7. (a1 + · · · + an ) a1 + · · · + an ≥ n 2 , where a1 > 0, . . . , an > 0.
1 1

a 2 +···+a 2  2
4.8. 1 n n ≥ a1 +···+a n
n
.
4.9. a1 a2 + a2 a3 + · · · + a9 a10 + a10 a1 ≥ −1 if a12 + · · · + a10 2
 1.
4.10. x + y ≥ x y + x y .
4 4 3 3
 2  3
4.11. |a1 |3 + · · · + |an |3 ≤ a12 + · · · + an2 .
     
3 a 2 + b2 + c2 + x 2 + y 2 + z 2 + 6 a 2 + b2 + c2 x 2 + y 2 + z 2
4.12.
≥ (a + b + c + x + y + z)2 .
4.13. a 2 + b2 + c2 ≥ab + bc + ca.   
4.14. (a1 + · · · + an ) a17 + · · · + an7 ≥ a13 + · · · + an3 a15 + · · · + an5 , where a1 >
0, . . .√, an > 0.√ √
4.15. (a) a + 1 + 2a − 3 + 50 − 3a ≤ 12, where 23 ≤ a ≤ 50 3
,
(b) a + b + c ≤ abc + 2, where a 2 + b2 + c2  2,
(c) 2(a + b + c) − abc ≤ 10, where a 2 + b2 + c2  9,
(d) 1 + abc
≥ 3min(a, b, c), where a 2 + b2 + c2  9.
 k+1  −1
n n  k
n
4.16. ai ai ≥n ai , where k, n ∈ N and a1 > 0, . . . , an > 0.
i1 √ i1 i1
4.17. a+b+c ≥ 3 abc, where a > 0, b > 0, c > 0.
k
3
a +···+a k  k
4.18. 1 n n ≥ a1 +···+a n
, where k, n ∈ N and a1 ≥ 0, . . . , an ≥ 0.
  n 
4.19. 1 + sin α 1 + cos α > 5, where 0 < α < π2 .
1 1

4.20. Find the distance from a point A(x0 , y0 ) to the line defined by the equation
ax + by + c  0 (a 2 + b2  0).
√ 2
4.21. Find the smallest possible value of the expression (u − v)2 + 2 − u 2 − v9

if 0 < u < 2, v > 0. 2  
2
4.22. x12 + x1 +x 2
2
+ · · · + x1 +···+x n
n
≤ 4 x12 + · · · + xn2 .
This inequality is a particular case of Hardy’s inequality
 n    p  n
a1 +···+ak p p
k
≤ p−1 p
· ak , where p > 1, ai ≥ 0, i  1, . . . , n.
k1 k1  
4.23. (a) a11 + a1 +a 2
2
+ . . . + n
a1 +...+an
< 4 1
a1
+ . . . + 1
an
, where a1 > 0, . . . , an > 0,
Problems 49
 
(b) 1
a1
2
+ a1 +a 2
+ · · · + a1 +···+a
n
n
<2 1
a1
+ ··· + 1
an
, where a1 > 0, . . . , an > 0.

Proofs

  
4.1. From (4.2) we have that a 2 + b2 + c2 12 + 22 + 32 ≥ (a + 2b + 3c)2 ≥ 142 ,
and therefore, a 2 + b2 + c2 ≥ 14. 
√ √ √ 2
4.2. We have a · b + 1−a ·2 1−b 2 ≤ a2 + 1 − a2 ·
 √ 2
b2 + 1 − b2  1.
4.3. We have
  √ √ √ √
c(a − c) + c(b − c)  c · a − c + b − c · c ≤

√ 2 √ 2 √ 2 √ 2 √
≤ c + b−c · a − c + c  ab.

4.4. Note that


 
√ √ √ 2 √ 2
a · a 2 + c2 + b2 + c2 · b ≤ a2 + b2 + c2 · a 2 + c2 + b2
 a 2 + b2 + c2 .
  
4.5. √1a · √1b + √1
b
· √1
c
+ √1
c
· √1a ≤ 1
a
+ b1 + 1c b1 + 1
c
+ 1
a
 1
a
+ 1
b
+ 1c .
4.6. Note that
√ √ √
a(a + c − b) + b(a + b − c) + c(b + c − a) 
     
 a(a + c − b) (a + c − b) + b(a + b − c) (a + b − c) + c(b + c − a) (b + c − a)
 
≤ (a(a + c − b) + b(a + b − c) + c(b + c − a)) · (a + c − b) + (a + b − c) + (b + c − a)
 
 a 2 + b2 + c2 (a + b + c).

4.7. We have
2 2  2
√  √ 2  1 1 √ 1 √ 1
2
a1 + · · · + an √ + ··· + √ ≥ a1 · √ + · · · + an · √  n2 .
a1 an a1 an

⎛ ⎞
 2 
4.8. We have a1 + · · · + an2 ⎝12 + ·
· · + 12 ⎠ ≥ (a1 + · · · + an )2 , whence
n
a12 + · · · + an2  a1 + · · · + an 2
≥ .
n n
50 4 The Cauchy–Bunyakovsky–Schwarz Inequality

4.9. We have   2 
|a1 a2 + . . . + a9 a10 + a10 a1 | ≤ a12 + . . . + a92 + a10
2
a2 + . . . + a10
2
+ a12 
1, hence a1 a2 + · · · + a9 a10 + a10 a1 ≥ −1.
4.10. We have
   
x 4 + y 4  x 4 + y 4 · x 4 + y 4 ≥ x 4 + y 4 · 2x 2 y 2

 2  2 
 x 2 + y 2 · (x y)2 + (x y)2 ≥ x 3 y + x y 3 .

4.11. We have
   2  2    
 3  
a1  + · · · + an3   |a1 | · a12 + · · · + |an | · an2 ≤ a12 + · · · + an2 · a14 + · · · + an4
  2  3
≤ a12 + · · · + an2 a12 + · · · + an2  a12 + · · · + an2 .

4.12. Note that the left-hand side of the inequality is equal to


√  2
3 a 2 + b2 + c2 + x 2 + y 2 + z 2  B.
According to the inequality of Problem 4.8, we have
⎛ ⎞2
2   2
a + b + c x + y + z
B ≥ 3⎝ 3 + 3 ⎠  (a + b + c + x + y + z)2 .
3 3

  
4.13. ab + bc + ca ≤ a 2 + b2 + c2 b2 + c2 + a 2  a 2 + b2 + c2 .
4.14. We have
 2 

5 5
  √ 
2 √ 2   5 2
(a1 + · · · + an ) a1 + · · · + an  a1 + · · · + an a1 + · · · + an5

 2
≥ a13 + · · · + an3 .

  
In a similar way, we deduce that a17 + · · · + an7 a13 + · · · + an3 ≥
 5  2
a1 + · · · + an5 .  
Multiplying
 3 these
 inequalities, we obtain (a1 + · · · + an ) a17 + · · · + an7 ≥
a1 + · · · + an3 a15 + · · · + an5 .
4.15. (a) We have
√ √ √
1· α + 1 + 1 · 2α − 3 + 1 · 50 − 3α ≤

  √ 2 √ 2 √ 2
≤ 12 + 12 + 12 α+1 + 2α − 3 + 50 − 3α  12.

(b) We have
 
a · (1 − bc) + (b + c) · 1 ≤ (a 2 + (b + c)2 )((1 − bc)2 + 12 )  4(1 − b2 c2 ) + 2(1 + bc)b2 c2 ≤ 2,
Proofs 51

2 2
since bc ≤ b +c
2
≤ 1.
2 2
(c) Let a  max(a , b2 , c2 ); hence a 2 ≥ 3 and x  bc ≤ b +c
2 2
2
≤ 3, and
therefore,

a · (2 − bc) + (b + c) · 2 ≤ (a 2 + (b + c)2 )((2 − bc)2 + 22 )

 100 + (2x − 7)(x + 2)2 ≤ 10.

(d) Let a ≤ b ≤ c.
If 0 ≤ a ≤ 1, then a(b − a)(c − a) ≥ 0 and 9  a 2 + b2 + c2 ≤
bc + b2 + c2 ≤ (b + c)2 .
Hence 1 + abc ≥ 1 − a 3 + a 2 (b + c) ≥ (1 − a) · 3a + a 2 · 3  3a, whence
1 + abc ≥ 3a.
If a > 1 and c ≥ 2, then 1 + abc ≥ 1 + 2a 2 > 3a, and thus 1 + abc > 3a.
If a > 1 and c < 2, then a 2 + b2 > 5, whence b2 > 2, 5 and 1 + abc ≥
1 + ab2 > 1 + 2, 5a > 0, 5c + 2, 5a ≥ 3a, and therefore, 1 + abc > 3a.
If a < 0 and bc ≤ 3, then 1 > 0 ≥ a(3−bc), and therefore, 1+abc > 3a.
2
If a < 0 and bc > 3, since 9  a 2 + b2 + c2 ≥ a 2 + 2bc, or bc ≤ 9−a 2
,
it follows that

3 − a2 (|a| − 1)2 (|a| + 2)


−a(bc − 3)  |a|(bc − 3) ≤ |a| 1− ≤ 1,
2 2
whence 1 + abc ≥ 3a.

n
4.16. Define aiS  A S . From inequality (4.2), it follows that
i1

n 
 n  n 2
 k+1 2   k−1 2  k+1 k−1
Ak+1 · Ak−1  ai 2 ai 2
≥ ai · ai
2 2
 A2k .
i1 i1 i1

In a similar way, we obtain Ak · Ak−2 ≥ A2k−1 , Ak−1 · Ak−3 ≥ A2k−2 , . . . , A2 ·


A0 ≥ A21 , A1 · A−1 ≥ A20 .
Multiplying these inequalities, we deduce that Ak+1 · A−1 ≥ Ak · A0 , or
n  n 
  
n
−1
k+1
ai ai ≥ aik .
i1 i1 i1

4.17. We have

√ 2 √ 2 √ 2 √ 2 √ 2 √ 2
(a + b + c)(b + c + a)  a + b + c b + c + a ≥
√ √ √ 2
≥ ab + bc + ca ,
√ √ √ 2 √ √ √ 2  √ √ √ 4
4 4 4
ab + bc + ca c+ a+ b ≥ abc + abc + abc ,
√ √ √ 2
(a + b + c)(1 + 1 + 1) ≥ a+ b+ c .

Multiplying these inequalities, we obtain that (a + b + c)3 ≥ 27abc, or


52 4 The Cauchy–Bunyakovsky–Schwarz Inequality

a+b+c √
3
≥ abc.
3

4.18. We have that Ak · Ak−2 ≥ A2k−1 , Ak−1 · Ak−3 ≥ A2k−2 , . . . , A2 · A0 ≥ A21 ,


therefore multiplying these inequalities, we obtain
Ak · A0 ≥ Ak−1 · A1 , Ak−1 · A0 ≥ Ak−2 · A1 , . . . , A2 · A0 ≥ A1 · A1 , A1 · A0 ≥ A0 · A1 .

Multiplying these inequalities, we deduce that


a1k + . . . + ank  a + . . . + a k
1 n
Ak · Ak0 ≥ A0 · Ak1 , or ≥ .
n n


n
Here Ak  aik .
i1
4.19. We have
2  2  2
1 1 1
1 + √
2
1 + √
2
≥ 1+ √ 
sin α cos α sin α cos α
⎛ ⎞2 ⎛ ⎞2
1 1  √ 2
 ⎝1 +  ⎠ ≥ ⎝1 +  ⎠  1 + 2 > 5.
sin 2α 1
2 2

4.20. Let M(x, y) be a point on the line defined by the equation ax + by + c  0.


Let us estimate the minimum value of the distance AM
 
(x − x0 )2 + (y − y0 )2 .   
According to inequality (4.1), we have a 2 + b2 (x − x0 )2 + (y − y0 )2 ≥
0 ) + b(y − y
(a(x − x√ 0 )) ,
2

whence
 a 2 + b2 · (x − x0 )2 + (y − y0 )2 ≥ |ax + by − ax0 − by0 |, or
(x − x0 ) + (y − y0 )2 ≥ |ax√0a+by
2 0 +c|
2 +b2
.
In the last inequality, equality holds if x−x
a
0
 y−y
b
0
.
Solving the following system:

⎨ ax + by + c  0,
x − x0 y − y0
⎩  .
a b

We obtain x  −a axa0 +by 0 +c


2 +b2 + x0 , y  −b axa0 +by 0 +c
2 +b2 + y0 .
|ax0 +by0 +c|
Therefore, the distance from point A to the given line is equal to √
a 2 +b2
.
4.21. We have
 2
9 2 9 9
(u − v)2 + 2 − u2 −  2 − 2 uv + 2 − u 2 + v2 + ≥
v v v

 2 2  2  2
√
9 9 81 √ √ 2
≥ 2 − 2
u 2 + 2 − u2 v2 + + v2 +  v2 + 2 − 2 ≥ 2 · 9 − 2  8.
v v v
Proofs 53

 2
If u  1, v  3, then (u − v)2 + 2 − u 2 − v9  8; hence the minimum
value of the given expression is equal to 8.
4.22. We have

1 1 1 1 1
a12 ≥ a12 1 − √ + a12 √ − √ + · · · + a12 √ − √ ,
2 2 3 n n+1

√ 1 1 √ 1 1 √ 1 1
a22 ≥ a22 2 √ − √ + a22 2 √ − √ + · · · + a22 2 √ − √ ,
2 3 3 4 n n+1
.......... ......... ......... .......

√ 1 1
an2 ≥ an2 n √ − √ .
n n+1

Therefore,

1 √  1 1
a12 + a22 + · · · + an2 ≥ a12 1 − √ + a12 + a22 2 √ − √ + ···+
2 2 3
 
√ √ 1 1
+ a12 + a22 2 + · · · + an2 n √ − √ .
n n+1

Let us prove that


√ √
k +1− k 2 √  1  a 1 + · · · + a k 2
√ √ a1 + · · · + ak2 k ≥ . (4.3)
k· k+1 4 k

According to inequality (4.2), it follows that



√  1
a12 + · · · + ak2 k 1 + · · · + √ ≥ (a1 + · · · + ak )2 .
k
√ √
In order to prove inequality (4.3), it is sufficient to prove that ( √k √k+1 ) k 2 >
4 k+1− k

1 + ··· + √1 .
k
We have
1 1 2 2
1 + √ + ··· + √ ≤ 1 + √ + ··· + √ √ 
2 k 2+1 k+ k−1
√  √ √  √
 1 + 2 2 − 1 + · · · + 2 k − k − 1  2 k − 1.

Let us prove that


√ √ 
4k 2 k+1− k √
√ 4k k
2 k−1< √ √  √ .
k k+1 k + 1 + k2 + k
√ 2 √ √
Since √ 2k √
k+1+ k
<k≤k+ k − 1 < k + k 2 + k − 2 k + 1, it follows
that
54 4 The Cauchy–Bunyakovsky–Schwarz Inequality

2k √ √
√ √ < k + k 2 + k − 2 k + 1, or
k+1+ k
√ √  √ √
2k k + 1 − k < k + k 2 + k − 2 k + 1, or
√  √   √  √
2 k k + 1 + k 2 + k − k + 1 + k 2 + k < 4k k.

Hence we deduce that



√ 4k k
2 k−1< √ .
k + 1 + k2 + k

Now let us prove that on the right-hand side, it is impossible to choose a


multiplier smaller than 4.
 2  2  
Suppose the inequality x12 + x1 +x2
2
+· · ·+ x1 +···+x
n
n
≤ c x12 + x22 + · · · + xn2
holds for all x1 , . . . , xn .
Taking xi  √1i , i  1, . . . , n, we deduce that
2 2
1+ √1
2
1+ √1
2
+ ··· + √1
n 1 1
1+ + ··· + ≤ c 1 + + ··· + .
2 n 2 n

According to the inequality of Problem 14.16, we have


n  2   √ √
1/1 1+···+1/n n
√2 − 2 < c 1 + 1
+ · · · + 1
, whence 4 − 8 < c.
k k 2 n 1+1/2+···+1/n
k1
Passing
to the1 limit,1we obtain
√ +···+ √
lim 4 − 8 11+1 1 +···+n 1 n ≤ lim c. Since
n→∞ 2 n n→∞
1
√ +···+ n √1 n
lim 1 1
 0, we must have 4 ≤ c.
1+ 21 +···+ n1
n→∞
4.23. (a) The proof follows straightforwardly from the proof of part (b).
(b) We have

1 1 1 1 1 1 1 1 1
> 1− 2 + − 2 + ··· + − ,
a1 a1 2 a1 2 2 3 a1 n 2 (n + 1)2
2 2 2
1 2 1 1 2 1 1 2 1 1 1 n2 1 1
> − 2 + − 2 + ··· + − ,..., > − .
a2 a2 2 2 3 a 2 32 4 a2 n 2 (n + 1)2 an an n 2 (n + 1)2

Therefore,

1 1 1 1 1 1 22 1 1
2 + + ··· + >2 1− +2 + − + ···
a1 a2 an a1 22 a1 a2 22 32

1 n2 1 1
+2 + ··· + − .
a1 an n2 (n + 1)2

Let us prove that



1 k2 1 1 k
2 + ··· + − > ,
a1 ak k2 (k + 1)2 a1 + · · · + ak
Proofs 55

or equivalently,

1 k2 k 3 (k + 1)2
(a1 + · · · + ak ) + ··· + > .
a1 ak 2(2k + 1)

We have
⎛  ⎞2

1 k2 1 k2 k 2 (k + 1)2 k 3 (k + 1)2
(a1 + · · · + ak ) + ··· + ≥ ⎝ a1 · + ··· + · ak ⎠  > .
a1 ak a1 ak 4 2(2k + 1)

It follows that
 n
1 1 1 k2 1 1
2 + ··· + > 2 + ··· + −
a1 an k1
a1 ak k2 (k + 1)2

n
k
> .
k1
a1 + · · · + ak

Now let us prove that on the right-hand side, it is  impossible to  choose a multiplier
smaller than 2. Indeed, let a11 + · · · + a1 +···+a
n
n
≤ c 1
a1
+ · · · + 1
an
.
−2
Assume that ak  k, k  1, . . . , n, and thus 2xn+1 ≤ c, where xn  1 + 1
+
  xn 2
· · · + n . Therefore, 2 lim xn − xn ≤ c, or 2 ≤ c. This ends the proof.
1 xn+1 1
n→∞

Problems for Independent Study

Prove the following inequalities (1–16).

1. (sin α1 + · · · + sin αn )2 + (cos α1 + · · · + cos αn )2 ≤ n 2 .



2. a1 +···+a n
≥ n a1 · · · an , where n ≥ 2, a1 > 0, . . . , an > 0.
√ n √ √ √
3. a1 b1 + · · · + an bn ≤ a1 + · · · + an · b1 + · · · + bn , where ai ≥ 0, bi ≥
0, i  1, . . . , n.
 12y2 −2 x2 y12) 2 + 2 (x2 2 y3 − x3 y2 )
4. (x (x1 y3 − x3 y1 )2 ≤
2 2
+
x1 + x2 + x3 y1 + y2 + y3 .
n 2 n n
√   bi
5. ai bi ≤ ai xi xi
, where xi > 0, ai > 0, bi > 0 i 
i1 i1 i1
.n. . , n. n
1, 2
  
n
6. xi yi xi
yi
≥ xi , where xi > 0, yi > 0 i  1, . . . , n.
i1 i1 i1
  
7. ax + by + cz + a 2 + b2 + c2 x 2 + y 2 + z 2 ≥ 23 (a + b + c)(x + y + z).
  
8. ( p1 q1 − p2 q2 − · · · − pn qn )2 ≥ p12 − p22 − · · · − pn2 q12 − q22 − · · · − qn2 ,
if p12 ≥ p22 + · · · + pn2 , q12 ≥ q22 + · · · + qn2 .
56 4 The Cauchy–Bunyakovsky–Schwarz Inequality

   √
9. x 2 + x y + y 2 y 2 + yz + z 2 + y 2 + yz + z 2 z 2 + zx + x 2 +
√ 
+ z 2 + zx + x 2 x 2 + x y + y 2 ≥ (x + y + z)2 .
Hint. We have
 
  √ 2 √ 2
 2  z 2
x + x y + y y + yz + z 
2 2 2 2
y+
x
+
3x
y+ +
3z

2 2 2 2
 x  z  3x z
≥ y+ y+ + .
2 2 4
10. a1 (b1 + a2 ) + a2 (b2 + a3 ) + · · · + an (bn + a1 ) < 1, where n ≥ 3, a1 , . . . , an > 0
and a1 + · · · + a
 n  1, b1 + · · ·
2
+ bn2  1.
 x+y 2  y+z 2  2 √
11. 1 − 2 + 1 − 2 + 1 − z+x 2
≥ 6, where x, y, z ≥ 0, x 2 + y 2 +
z 2  1.  
 a  c
12. + b
+ ≥ 2 abc
1 + (a+b)(b+c)(c+a) , where a, b, c > 0.
 b+c c+a
a+b  √
13. a + (b − c)2 + b + (c − a)2 + c + (a − b)2 ≥ 3, where a, b, c ≥ 0 and
a+ b + c  1.   √
14. a+b
2
− ab+ b+c
2
− bc+ c+a
2
− ca ≥ 2, where a, b, c ≥ 0 and a+b+c  2.
√ √ √ √ √ √
15. 1 − x y 1 − yz+ 1 − yz 1 − zx+ 1 − zx 1 − x y ≥ 2, where x, y, z ≥
0 and x 2 + y 2 + z 2  1.
√ √ √ √
16. x 1 − yz + y 1 − zx + z 1 − x y ≥ 2 3 2 , where x, y, z ≥ 0 and x + y + z  1.
17. Find the distance from apoint A(x0 , y0 , z 0) to the plane defined by the equation
ax + by + cz + d  0 a 2 + b2 + c2  0 .
18. Prove

(a) the following identity:


(a1 c1 + · · · + an cn ) (b1 d1 + · · · + bn dn ) − (a1 d1 + · · · + an dn ) (b1 c1 + · · · + bn cn )

 (ai bk − ak bi ) (ci dk − ck di ).
1≤i<k≤n

(b) (a1 c1 + · · · + an cn )(b1 d1 + · · · + bn dn ) ≥ (a1 d1 + · · · + an dn )(b1 c1 + · · · +


bn cn ),where bi di > 0 (i  1, . . . , n) or bi di < 0 (i  1, . . . , n) and
a1
b1
≤ ab22 ≤ . . . ≤ abnn , dc11 ≤ dc22 ≤ . . . ≤ dcnn .

19. Find the maximum and minimum values of the expression


 
x 2 + y 2 + (x − 2)2 + (y − 1)2
  .
x 2 + (y − 1)2 + (x − 2)2 + y 2
Problems for Independent Study 57

20. Find the minimum value of the expression



1 1 1 1
+ − 1 + − 1 ,
x n an y n bn

where x, y, a, b > 0, x + y  1, a + b  1.
Hint. We have

1 1 1 1 1 b(1 + a + · · · + a n−1 ) 1 a(1 + b + · · · + bn−1 )
+ −1 + −1  + + ≥
x n an y n bn xn an yn bn
⎛  ⎞2 ⎛  n−1 ⎞2
n n 1 + √ab + · · · + √ab
1 (1 + a + · · · + a n−1 )(1 + b + · · · + bn−1 )
⎜ 1 ⎟
≥⎝ √ + ⎠ ≥⎝ √ + √ n−1 ⎠
xy a n−1 bn−1 xy
ab

≥ (2n+1 − 1)2 .
Chapter 5
Change of Variables Method

The change of variables method is a basic technique used to simplify problems in


which the original variable or variables are replaced with other variables in order to
express the given problem in new variables in such a way that the problem becomes
simpler or easier to prove.
In order to prove inequalities, one of the most useful techniques is the change of
variables method. Depending on the problem, one can deal with the single-variable
case or the multiple-variables case. Both cases can be treated using this method.
Let us explain this method by considering the following examples.
(x+y)(1−x y)
Example 5.1 Prove that − 21 ≤ ≤ 21 .
(1+x 2 )(1+y 2 )
Proof Let us perform the following change of variables: x  tan α, y  tan β.
sin(α+β) cos(α+β)
(x+y)(1−x y) α+tan β)(1−tan α tan β) cos α cos β · cos α cos β
It then follows that  (tan 1+tan  
(1+x 2 )(1+y 2 ) ( 2α
)(1+tan2 β ) 1
· 1
cos2 α cos2 β

 sin (α + β) cos (α + β)  2 sin 2 (α + β) , therefore


1
− 21 ≤ 21 sin 2 (α + β) ≤
1
2
.
This completes the proof.
      
Example 5.2 Prove that a 2 1 + b4 + b2 1 + a 4 ≤ 1 + a 4 1 + b4 .

Proof Let us perform


 the following
 change of
 variables:
 a 2  tan α, b2 tan β.
Then tan α 1 + tan β + tan β 1 + tan α ≤ 1 + tan α 1 + tan2 β .
2 2 2

This inequality is equivalent to the following inequality: sin 2α + sin 2β ≤ 2.


This ends the proof.

Example 5.3 Prove that if a > 0, b > 0, c > 0, a +b+c  1, then a 3 +b3 +c3 ≥ 19 .

© Springer International Publishing AG, part of Springer Nature 2018 59


H. Sedrakyan and N. Sedrakyan, Algebraic Inequalities, Problem Books
in Mathematics, https://doi.org/10.1007/978-3-319-77836-5_5
60 5 Change of Variables Method

Proof Let us perform the following change of variables: a  x + 13 , b  y + 13 , c 


z + 13 . Then a 3  x 3 + x 2 + x3 + 27
1
, b3  y 3 + y 2 + 3y + 27
1
, c3  z 3 + z 2 + 3z + 27
1
, and
3 3 3 2 2 2 x+y+z
therefore, a + b + c  x (1 + x) + y (1 + y) + z (1 + z) + 3 + 9  x (1 + x) + 1 2

+y 2 (1 + y) + z 2 (1 + z) + 19 ≥ 19 , as x > − 13 , y > − 13 , z > − 13 .


This ends the proof.
Example 5.4 Prove that if a > 0, b > 0, c > 0, and abc  1, then
1
1+a+b
1
+ 1+b+c 1
+ 1+a+c ≤ 1.
√ √ √
Proof Let us perform the following change of variables: 3 a  x, 3 b  y, 3 c  z.
Then x yz  1 and 1+a+b  x yz+x 3 +y 3 ≤ x yz+x y(x+y)  x+y+z . In a similar way, we
1 x yz x yz z

1
obtain that 1+b+c ≤ x+y+z
x 1
and 1+a+c y
≤ x+y+z . Therefore, we deduce that
1
1+a+b
+ 1+b+c + 1+a+c ≤ x+y+z + x+y+z + x+y+z
1 1 x y z
 1.
This ends the proof.
x22
Example 5.5 Prove that if x1 > 0, . . . , xn > 0 and 1
x1
+···+ 1
xn
 n, then x1 + 2
+
xnn
··· + n
≥1+ 1
2
+ · · · + n1 .
Proof Let us perform the following change of variables: xi  1+ yi , i  1, . . . , n,
where yi > −1. We have (x1 + · · · + xn ) x11 + · · · + x1n ≥ n 2 , and hence y1 + · · · +
yn ≥ 0.
According to Bernoulli’s inequality (Problem 3.27a), we deduce that
x2 xn
x1 + 22 + · · · + nn ≥ 1 + 21 + · · · + n1 + y1 + y2 + · · · + yn ≥ 1 + 21 + · · · + n1 . Therefore,
x22 xnn
x1 + 2
+ ··· + n
≥1+ 1
2
+ · · · + n1 . This ends the proof.
Example 5.6 Prove that if a1 ≥ 1, . . . , an ≥ 1 and n ≥ 2, k ≥ 2, k ∈ N, then
√ √ k √
n−1
k
a1 − 1 + · · · + k an − 1 ≤ (n−1) n−k
k
a1 · · · · · an .
n k

Proof Let us perform the following change of variables: bi  k (n − 1)(ai − 1),
where i  1, 2, . . . , n and b1k < 1, . . . , blk < 1,bl+1 k
≥ 1, . . . , bnk ≥ 1. Then from
Problems
 10.6, 11.14,  it follows  that k   
bk −1 bk −1 b −1 bk −1
1 + 1n · · · · · 1 + ln 1 + l+1n · · · · · 1 + nn ≥
  
b +···+b −l
k k
b +···+b −(n−l)
k k
≥ 1+ 1 n l 1 + l+1 nn 
⎛ k k⎞⎛ k ⎞
b1 +...+bl +1 + · · · + 1 1 + ·
· · + 1 k +bl+1
k k k
+···+bnk


⎜ ⎟⎜ ⎟  b1 +b2 +···+bn k
⎝ n
n−l
⎠⎝
l
n ⎠≥ n
.

bk −1
Note that 1 + i n  (n−1)a i
, i  1, 2, . . . , n, and thus it follows that
 n−1 n  √
n
√ √ k
k
n−1( k a1 −1+···+ k an −1)
n
a 1 · · · · · a n ≥ n
, whence
n−1
(n−1) k k √ √ √
n−k a1 . . . an ≥ k a1 − 1 + · · · + k an − 1.
n k

This ends the proof.


Problems 61

Problems

Prove the following inequalities (5.1–5.5, 5.8–5.12, 5.16–5.20).


√ √
≤ a 2 + b2 + ab
5.1. 4 √ √ + 4 − a · 9 − b ≤ 19, where 0 ≤ a ≤ 2 and 0 ≤ b ≤ 3.
2 2

5.2. n√ m − 1 + m√ n − 1 ≤ mn, where m ≥ 1, n ≥ 1.


5.3. m 2 − n2 +  2mn − n 2 ≥ m, where m > n > 0.
√ √ 
5.4. x > x − 1 + x x − 1 , where x ≥ 1.

5.5. Prove that 1 + √12 + ... + √1n ≥ n n+1 2
, wher e n ∈ N
5.6. Prove that among seven √
arbitrary numbers one can find two numbers x and y
such that 0 ≤ 1+x x−y
y
< 3
3
.
5.7. Prove the inequality of Problem 4.3.
5.8. √1+a|a−b|√ ≤ √1+a|a−c| √ + √1+b|b−c|√ .
2 · 1+b2 2 · 1+c2
√ 2 · 1+c2

5.9. (a) √ Huygens’s inequality: n
(a1 + b1 ) · · · (an + bn ) ≥ n a1 · · · · · an +
n
b1 · · · bn , where ai > 0, bi > 0, i  1, . . . , n,
(b) Milne’s inequality: aa11+b b1
1
+ · · · + aann+b bn
n
≤ (a(a11+···+a
+···+an )(b1 +···+bn )
n )+(b1 +···+bn )
, where ai >
0, bi > 0, i  1, . . . , n.
5.10. (x +x )(y +y8 )−(z +z )2 ≤ x y 1−z 2 + x y 1−z 2 , where x1 > 0, x2 > 0, and x1 y1 −z 12 >
1 2 1 2 1 2 1 1 1 2 2 2
0, x2 y2 − z 22 > 0. √
√ √ √
5.11. (a) a − 1 + b − 1 + c − 1 ≤ √23 abc, where a ≥ 1, b ≥ 1, c ≥ 1,
√ √ √ √ √ √
(b) a − 1 + b − 1 + c − 1 + d − 1 ≤ 3 4 3 abcd, where a ≥ 1, b ≥
1, c ≥ 1, d ≥ 1.
a1k +···+ank  k
5.12. n
≥ a1 +···+a
n
n
, where k ∈ N and a1 > 0, . . . , an > 0.
5.13. Let f (x) be a given differentiable function on [a, a + 4] . Prove that there exists
a point x0 (x0 ∈ [a, a + 4]) such that f  (x0 ) < 1 + f 2 (x0 ) .
5.14. Let f (x) be a function defined in [a, b] such that f (x) > 0, x ∈ (a, b),
and f (a)  f (b)  0, f (x) + f n (x) > 0 for all x ∈ [a, b] . Prove that
b − a > π.   
2
a 2 −b2 2 2
5.15. Prove that if a ≥ 21 , b ≥ 21 , then 2
≥ a +b
2
− a+b
2
.

x1 3+· · ·+xn ≤3 3 , where x1 +· · ·+xn  0 and xi ∈ [−1, 1] , i  1, . . . , n,


n 3 3
5.16. (a)
(b) x + · · · + x  ≤ 2n, where x1 + · · · + xn  0 and xi ∈ [−2.2] , i 
1 n
1, 2, . . . , n. √
y
2 + 1+z 2 ≤ , where x 2 + y 2 + z 2  1.
x z 3 3
5.17. 1+x 2
+ 1+y 4 √
5.18. 1 < √a 2a+b2 + √ b + √ c ≤ 3 2
, where a, b, c
> 0.
√ √ b2 +c2√ c2 +a 2√
√ √ √
2

5.19. 1 − a + 1 − b+ 1 − c+ 1 − d ≥ a + b+ c+ d, where a, b, c, d >
0, a 2 + b2 + c2 + d 2 1. 
√ √ √ 2 √ √ 2 √ √ 2
5.20. a+b+c
3
− 3
abc ≤ max a − b , b − c , c − a , where
a > 0, b > 0, c > 0.
62 5 Change of Variables Method

Proofs
 
5.1. Let a  2 cos α, b  3 cos β, where α, β ∈ 0, π2 . It follows that
 
a 2 + b2 + ab + 4 − a 2 · 9 − b2  4 cos2 α + 9 cos2 β + 6 cos α cos β + 6 sin α sin β 
.
 4 cos2 α + 9 cos2 β + 6 cos (α − β) ≤ 4 + 9 + 6  19.

On the other hand, 6 sin α sin β ≥ 6 sin2 α or 6 sin α sin β ≥ 6 sin2 β.


If 6 sin α sin β ≥ 6 sin2 α, then we have that
4 cos2 α + 9 cos2 β + 6 cos α cos β + 6 sin α sin β ≥ 4 cos2 α + 6 sin2 α +
9 cos2 β + 6 cos α cos β ≥ 4.
On the other hand, if 6 sin α sin β ≥ 6 sin2 β, then
4 cos2 α + 9 cos2 β + 6 cos α cos β + 6 sin α sin β√≥ 6. √
Therefore, in both cases we obtain a 2 + b2+ ab+ 4 − a 2 · 9 − b2 ≥ 4.
5.2. Let m  cos12 α , n  cos12 β , where α, β ∈ 0, π2 . Then
√ √ 1 sin α
· cos 1 sin β
· cos
α+ β
n m−1+m n−1
 cos β  sin 2α+sin 2β
≤ 1.
2 cos α 2
mn 1 2
√ √
cos2 α cos2 β

Therefore, n m − 1 + m n − 1 ≤ mn.
5.3. Let mn  sin α, where α ∈ 0; π2 . It follows that
√ √ √ 
m 2 −n 2 + 2mn−n 2 m cos α+m 2 sin α−sin2 α
m
 m
 cos α + 2 sin α − sin2 α 
 √
 cos α + sin α + sin α (1 − sin α) ≥ cos α + sin α ≥ cos2 α + sin2 α  1.
√ √
Therefore, m 2 − n 2 + 2mn  π− n ≥ m.
2

5.4. Let x  cos2 α , where α ∈ 0, 2 . Then the given inequality can be rewritten
1

as

1 1 1 − cos α 
> tan α + , or 1 > sin α cos α + cos α (1 − cos α).
cos α
2 cos α cos α
(5.1)

From inequality (3.2), it follows that cos α (1 − cos α) ≤ 21 and sin α cos α ≤
1
2
, where equality does not hold simultaneously in both inequalities.
Summing these inequalities, we obtain inequality
 (5.1).
5.5. First, let us prove that √1
k
+ √ 1
n+1−k
≥2 2
n+1
, where k  1, . . . , n.
Let k  (n + 1) sin α, where 0 < α < π2 . It follows that
2
 1   
√ 1 + √ 1  √1
n+1 sin α
+ 1
cos α
≥ √2 2
≥ 2 n+12
.
n+1· sin α n+1· cos α  n+1 sin 2α
  
Since 1 + √1 + · · · + √1n  1
2
√1 + √1 + ··· + 1
2
√1 + √11 , we have 1 +
2
 1 n n

√1 + · · · + √1n ≥ n n+1 2
.
2  
5.6. Let xk  tan αk , where − π2 < αk < π2 , k  1, . . . , 7. Let us divide − π2 , π2
into six equal parts. According to Dirichlet’s principle, at least two among the
Proofs 63

numbers α1 , . . . , α7 belong to the same interval, and therefore, 0 ≤ αi − α j <


π
6
. It follows that

xi − x j   π 3
0≤  tan αi − α j < tan  .
1 + xi x j 6 3
 
5.7. Define a  cosc2 α , b  cosc2 β , where α, β ∈ 0, π2 . After several similar
transformations, we get the inequality sin (α + β) ≤ 1. The last inequality
obviously holds.
5.8. Set a  tan α, b  tan β, c  tan γ . Then the given inequality can be
rewritten as |sin (α − β)| ≤ |sin (α − γ )| + |sin (β − γ )| .
That can be proved in the following way:
|sin (α − β)|  |sin (α − γ ) cos (γ − β) + sin (γ − β) cos (α − γ )| ≤
≤ |sin (α − γ ) cos (γ − β)| + |sin (γ − β) cos (α − γ )| ≤ |sin (α − γ )| +
|sin (γ − β)| , therefore
|sin (α − β)| ≤ |sin (α − γ )| + |sin (β − γ )| .
5.9. (a) Let abii  tan αi , i  1, . . . , n. After several similar transformations,

we obtain 
n
sin2 α1 · · · · · sin2 αn + n cos2 α1 · · · · · cos2 αn ≤ 1. From Problem 2.1,
it follows that
 
n n sin2 α1 + · · · + sin2 αn cos2 α1 + · · · + cos2 αn
sin2 α1 · · · sin2 αn + cos2 α1 · · · cos2 αn ≤ + 
n n
n  
1
 sin2 αi + cos2 αi  1.
n
i1


(b) Let  tan αi , i  1, . . . , n. Hence, we obtain the inequality
ai
bi
b12tan α12
bn2 tan2 αn
+ · · · + bn tan ≤ (b1 tan2 α1 +···+bn tan2 αn )(b1 +···+bn ) , which is
b1 tan2 α1 +b1 2 α +b
n n (b1 tan2 α1 +···+bn tan2 αn )+(b1 +···+bn )
equivalent to   

n 
n

n bi tan2 αi bi
bi sin αi ≤
2 i1

n
bi
i1
, or
i1 cos2 αi
  i1    

n 
n 
n 
n
bi sin αi 2 bi
cos2 αi
bi tan αi ≤ bi . 2
i1  i12  i1 i1
sin αi sin α j 2
The factor bi b j cos 2 α + cos2 α on the left-hand side is not greater than
 2 
j j
sin2 αi
the factor bi b j tan αi + tan2 α j on the right-hand side, that is, cos 2α +
j
sin2 α j
cos2 αi
≤ tan2 αi + tan2 α j . This holds because we have that
 2 
sin αi
tan2 αi + tan2 α j − cos
sin2 α j
 (sin2 αi −sin2 α j )2 ≥ 0.
2 α + cos2 α cos2 αi · cos2 α j
j i
64 5 Change of Variables Method

√ √
5.10. Let z 1  x1 y1 · sin α, z 2  x2 y2 · sin β. Then the given inequality can
be rewritten as
8
A √ √ 2 ≤
x1 y1 cos2 α + x2 y2 cos2 β + x1 y2 cos2 α + x2 y1 cos2 β + x1 y2 sin α − y1 x2 sin β
1 1
≤ + .
x1 y1 cos2 α x2 y2 cos2 β

Note that
8
A≤ . (5.2)
x1 y1 cos2 α + x2 y2 cos2 β + x1 y2 cos2 α + x2 y1 cos2 β

On the other hand,


  
x1 y1 cos2 α + x2 y2 cos2 β + x1 y2 cos2 α + x2 y1 cos2 β x1 y1 cos 1 1
2 α + x y cos2 β 
      2 2
cos α
2
x2 y2 cos β
2 2
β cos α
2
 2 + xx21 yy21 cos 2 β + x y cos2 α
1 1
+ yy21 + yy21 + xx21 cos
cos2 α
+ xx21 cos 2β ≥ 8, and there-
fore, we obtain

8
x1 y1cos2 α + x2 y2 β + x1 y2 cos2 α + x2 y1 cos2 β
cos2
1 1
≤ + .
x1 y1 cos2 α x2 y2 cos2 β (5.3)

From (5.2) and (5.3) follows the required inequality.


5.11. Let a  cos12 α , b  cos12 β , c  cos12 γ , d  cos12 ϕ , where α, β, γ , ϕ ∈
 π
0; 2 .
(a) The given inequality can be rewritten as
tan α + tan β + tan γ ≤ √3 cos α cos
2
β cos γ
, or
cos α cos β cos γ (tan α + tan β + tan γ ) ≤ √23 .
Since cos α cos β cos γ (tan α + tan β + tan γ )  sin (α + β) cos γ +
cos α cos β cos γ   B, it follows from inequality (4.1)
 2  
that B ≤ sin (α + β) + cos2 α cos2 β cos2 γ + sin2 γ 

sin (α + β) + cos α cos β.
2 2 2

Now let us prove sin2 (α + β) + cos2 α cos2 β ≤ 43 .


Indeed,
 2
1
sin2 (α + β) + (cos (α − β) + cos (α + β))  sin2 (α + β) +
2
1 1 1 1
+ cos (α − β) + cos (α − β) cos (α + β) + cos2 (α + β) ≤ sin2 (α + β) + +
2
4 2 4 4
1 1 3 5 1
+ |cos (α + β)| + cos2 (α + β)  − cos2 (α + β) + + |cos (α + β)| 
2 4 4 4 2
 
3 1 2 4 4
− |cos (α + β)| − + ≤
4 3 3 3
Proofs 65

(b) The given inequality can be rewritten as√ M 


cos α cos β cos γ cos ϕ (tan α + tan β + tan γ + tan ϕ) ≤ 4 . 3 3

One can easily prove that


M  sin (α + β) cos γ cos ϕ + sin (γ + ϕ) cos α cos β.
 ϕ 2
According to inequality (3.2), we have that M ≤ sin (α + β) cos γ +cos +
 2 2
cos α+cos β
sin (γ + ϕ) .
2
 
Since cos x+cos y
2
 cos 2 cos x−y
x+y
2
≤ cos x+y
2
, where x, y ∈ 0; π2 , we
must have
M ≤ sin (α + β) cos2 γ +ϕ + sin (γ + ϕ) cos2 α+β 
 2 2 
γ +ϕ γ +ϕ γ +ϕ
 2 cos 2 cos 2 sin 2 cos 2 + sin 2 cos α+β
α+β α+β
2

 2 cos α+β
2
cos γ +ϕ
2
sin α+β+γ
2

≤ 2 cos2 α+β+γ +ϕ
4
sin α+β+γ +ϕ
2

3 α+β+γ +ϕ α+β+γ +ϕ
 4 cos sin .
4 √ 4 √
Since cos3 t sin t ≤ 3 3
16
(see Problem 3.28), we have M ≤ 3 3
4
.

5.12. Let a1 +...+a


n
n
 a, ai  a+xi , i  1, . . . , n. Then from the condition a+xi >
0, it follows that xai > −1. Hence, according to Bernoulli’s inequality, we have
 k
1 + xai ≥ 1 + k xai , i  1, . . . , n, and therefore, (a + xi )k ≥ a k + ka k−1 xi .
Summing these inequalities, we obtain
(a + x1 )k + · · · + (a + xn )k ≥ na k + ka k−1 (x1 + . . . + xn )  na k , since
a k +...+a k  k
x1 +. . .+xn  a1 +. . .+an −na  0. Thus, we deduce that 1 n n ≥ a1 +...+a n
n
.
5.13. Proof by contradiction. Assume that for all x ∈ [a, a + 4] ,
f  (x) ≥ 1 + f 2 (x) .
Let arctan f (x)  g (x) . Then − π2 < g (x) < π2 and f (x)  tan(g(x)).

It follows that f  (x)  cosg2(x)g(x)
≥ 1 + f 2 (x)  cos21g(x) , and hence g  (x) ≥ 1.
Therefore, g (x) − x is a nondecreasing function.
Hence,

g (b) − b ≥ g (a) − a, where b  a + 4 or g (b) − g (a) ≥ b − a. (5.4)

Since
 π π
g (b) , g (a) ∈ − ; , we have g (b) − g (a) < π. (5.5)
2 2
From (5.4)–(5.5) and the condition b − a  4 it follows that π > 4; this leads
to a contradiction.
Therefore, there exists x0 such that x0 ∈ [a, a + 4] and f  (x0 ) < 1 + f 2 (x0 ) .
5.14. Without loss of generality one can assume that a  0 (otherwise, consider the
function f (x − a)). Then we need to prove that b > π.
We proceed with a proof by contradiction argument.
  Assume that  b ≤ π.
Consider the function F (x)  − πb f (x) cos πb x + f  (x) sin πb x in [0, b] .
Note that F (0)  F (b)  0; Hence by Rolle’s theorem, there exists c ∈ (0, b)
such that F  (c)  0.
66 5 Change of Variables Method

On the other hand, we  have


  
F  (c)  πb2 f (c) sin πb c + f  (c) sin πb c 
2

 2       
 πb2 f (c) + f  (c) sin πb c ≥ f (c) + f  (c) sin πb c > 0.
It follows that F  (c) > 0, which leads to a contradiction with F  (c)  0.
Therefore, b > π.
5.15. Without loss of generality one can assume that a ≥ b. Let a  b tan α, where
π
4
≤ α < π2 . Then the given inequality can be rewritten as
− b sin2α+cos α
2
b4 4cos 2α
cos4 α
≥ b √2 cos
1
α  cos α
, or
√ 
b3 cos2 2α ≥ 2 cos3 α 2 − sin α − cos α .
In order to finish the proof, it is sufficient
π π  √ to prove that for α in
, , one has 8 cos 2α ≥ 2 cos α
4 2
1 2 3
2 − sin α − cos α , which is
equivalent to the following inequality: 21 cos2 (α + π/4) sin2 (α + π/4) ≥

2  2 cos3 α (1 − sin (α + π/4)) , or √
1
2
1 − sin2 (α + π/4) sin2 (α + π/4) ≥ 2 2 cos3 α (1 − sin (α + π/4)) .
This inequality holds because for the  difference between its left- and right-


hand sides we have 21 (1 − sin (α + π/4)) sin2 (α + π/4) + sin3 (α + π/4) − 4 2 cos3 α ≥ 0,
      √
since 1 ≥ sin α + π4 and sin2 α + π4 ≥ 2 cos2 α, 1 + sin α + π4 ≥ 2 2 ·
 
cos α π4 ≤ α < π2 .
5.16. (a) Let xi  sin αi , i  1, . . . , n. We have
n    n
3 sin αi − 4 sin3 αi  sin 3αi , and therefore,
i1 i1
n 
n 
n
xi  sin αi  1
3
sin 3αi ≤ n3 .
i1 i1 i1

n
(b) Let xi  2 sin αi , i  1, 2, . . . , n. We have (3 sin αi − 4 sin3 αi ) 
i1

n
sin 3αi , and therefore,
i1 n 
 3    n
x + x 3 + . . . + x 3   2  sin 3αi  ≤ 2 |sin 3αi | ≤ 2n.
1 2 n  
 i1  i1
It follows that x13 + x23 + . . . + xn3  ≤ 2n.
5.17. Without loss of generality one can assume that x, y, z ≥ 0. Indeed, note that
y |x| |y| |z|
2 + 1+z 2 ≤ 1+|x|2 + 1+|y|2 + 1+|z|2 and |x| +|y| +|z|  1.
x z 2 2 2
+ 1+y
1+x 2  π
Let x  tan α, y  tan β, z  tan γ , where α, β, γ ∈ 0, 4 .
We have x y + yz + x z ≤ x 2 + y2 + z2 (Problem 4.13), whence tan α tan β +
tan β tan γ + tan γ tan α ≤ 1.  
tan α+tan β
Hence, tan γ · 1−tan α tan β
≤ 1, or tan(α + β) ≤ tan π2 − γ .
Proofs 67

π π
It follows that α + β ≤ 2
− γ , that is, α + β + γ ≤ 2
. Note that 1+x
x y
2 + 1+y 2 + 1+zz 2

 21 (sin 2α + sin 2β + sin 2γ ) ≤ 21 · 3 sin 2α+2β+2γ
3
≤ 23 sin π3  3 4 3 .
5.18. Note that
√ a
a 2 +b2
+ √b2b+c2 + √c2c+a 2 > a+b
a b
+ b+c c
+ c+a > a
a+b+c
+ b
a+b+c
+ c
a+b+c
 1,
and therefore, √ a +
a 2 +b2
√ b
b2 +c2
+ √c2c+a 2 >
1.
Set a + b  m , b + c  n , c + a  k 2 , where m, n, k > 0.
2 2 2 2 2 2 2 2

We have (m + n)2 > m 2 + n 2 > k 2 , whence m + n > k, and in a similar way,


we obtain n + k > m, m + k > n.
Let an acute triangle with side lengths m, n, k have angles α, β, γ . Then using
the law
√ of sines and the
√ law of cosines,
√ this inequality
√ can be rewritten as
sin α sin 2γ + sin β sin 2α + sin γ sin 2β ≤ 3 sin α sin β sin γ .
Therefore, from inequality (4.2), it follows that
 √ 
sin α sin 2γ + sin β sin 2α + sin γ sin 2β

≤ (sin2 α + sin2 β + sin2 γ )(sin 2α + sin 2β + sin 2γ )

 4 sin α sin β sin γ (2 + 0, 25 cos2 (α − β) − (cos γ − 0, 5 cos(α − β))2

≤ 3 sin α sin β sin γ .

It follows
√ that √ √ √
sin α sin 2γ + sin β sin 2α + sin γ sin 2β ≤ 3 sin α sin β sin γ .
5.19. Let us prove that for an arbitrary number a, where 0 ≤ a ≤ 1, the following
inequality holds:

√ √ 2
1−a ≥ a+ (1 − 4a 2 ).
2
 
Let a  sin2 α, where α ∈ 0, π2 . Then the given inequality can be rewritten
as

2
cos α − sin α ≥ cos 2α(2 − cos 2α). (1)
4

If 0 ≤ α ≤ π4 , then cos α−sin α ≥ 0, cos α+sin α ≤ 2, and 2−cos 2α ≤ 2.
Therefore, inequality (1) holds.
If π4 ≤ α ≤ π2 , then on setting α  π4 + β, the inequality (1) can be rewritten as
cos2 β cos β2 ≥ sin β2 . In order to prove this inequality it is sufficient to prove
that
cos2 β(1 + cos β) ≥ sin β. Since 0 ≤ β ≤ π4 , we have cos2 β(1 + cos β) ≥
√ √
1
2
(1 + 22 ) > 22 ≥ sin β.
It follows that cos2 β(1 + cos β) > sin β. We have
√ √ √ √ √ √ √ √ √
1 − a + 1 − b + 1 − c + 1 − d ≥ a + b + c + d + 22 (1 − 4a 2 +
1 − 4b2 + 1 − 4c2 + 1 − 4d 2 ),
68 5 Change of Variables Method

whence
√ √ √ √ √ √ √ √
1−a+ 1−b+ 1 − c + 1 − d ≥ a + b + c + d.

5.20. Without loss of generality one can assume that a ≥ b ≥ c.


Let a  x 3 , b  y 3 , c  z 3 . Therefore, x ≥ y ≥ z > 0.
√ √ 2
x 3 +y 3 +z 3
Thus, the given inequality can be rewritten as 3
−x yz ≤ x3 − z3 .
We have
x 3 + y3 + z3 1  
− x yz  (x + y + z) (x − z)2 + (x − y)2 + (y − z)2 ≤
3 6
x   √ √ 2  √ 2 √ √ 2  √ 2
≤ (x − z)2 + (x − y + y − z)2  x− z x + xz ≤ x− z x + xz + z 
2
  2 x 3 + y3 + z3   2
 x 3 − z 3 , whence − x yz ≤ x 3 − z3 .
3

This ends the proof.

Problems for Independent Study

Prove the following inequalities (2–27).

1. Given that a 2√ + b2  1. Prove that√ √


(a) |a + b| ≤ 2, (b) |a − b| ≤ 2, (c) |ab| ≤ 21 , (d) |a 2 b + ab2 | ≤ 22 .
   

2. x y − 1 − x 2 1 − y 2  ≤ 1, where |x| ≤ 1, |y| ≤ 1.
√    2
3. 1 − x + 1 − y ≤ 2 1 − x+y
2 2 , where |x| ≤ 1, |y| ≤ 1.
  2
4. x + x1 arccot x > 1, where x > 0.

 1  1 
2
5. cos α+cos β
− 1 ≤ cos α
− 1 cos β
− 1 , where π3 ≤ α < π2 , π3 ≤ β < π
2
.
n(a1 +···+an )
6. a1
1−a1
+ · · · + 1−a
an
n
≥ n−(a 1 +···+an )
, where 0 ≤ a1 < 1, . . . , 0 ≤ an < 1.
7. √1
1+a 2
+ 1+b2 + 1+c2 ≤ 2 , where a, b, c > 0 and a + b + c  abc.
√ 1 √ 1 3

|x−y| |y−z| |x−z|


8. 1+a|x−y|
+ 1+a|y−z| ≥ 1+a|x−z| , where a > 0.
2x (1−x 2 ) 2y (1−y 2 ) 2z (1−z 2 )
9. + + ≤ 1+x x
2 + 1+y 2 + 1+z 2 , where x > 0, y > 0,
y z
z>0
(1+x 2 )2 (1+y 2 )2 (1+z 2 )2
and x y + yz + zx  1. √ √ √ √
√ √  √ 
10. √a1+ a2 + · · · + an ≤ 1 a1 − a2 + 2 a2 − a3 + · · · +
√ √
n an − an+1 , where a1 ≥ . . . ≥ an ≥ an+1  0.
1
11. 1 +···+ 1 − 1 +···+
1
1 ≥ n , where a1 > 0, . . . , an > 0.
1
1+a1 1+an
√ a1 an

12. a + b + c − 2 abc ≥ ab + bc + ca − 2abc, where 0 ≤ a ≤ 1, 0 ≤ b ≤ 1, 0 ≤


c√≤ 1. √ √ √
13. a (1 − b) (1 − c) + b (1 − a) (1 − c) + c (1 − a) (1 − b) ≤ 1 + abc,
where 0 ≤ a ≤ 1, 0 ≤ b ≤ 1, 0 ≤ c ≤ 1.
Problems for Independent Study 69

14. ((x + y)(y +z)(x +z))2 ≥ x yz(2x + y +z)(2y +z +x)(2z +x + y), where x, y, z ≥ 0.
Hint. If x 2 + y 2 + z 2  0, then without loss of generality one can assume that
x + y + z  1.
Let x  tan α tan β, y  tan β tan γ , z  tan α tan γ , where α + β + γ  π2 .
Then one needs to prove that ((1 − x)(1 − y)(1 − z))2 ≥ x yz(1 + x)(1 + y)(1 + z).
15. ab(1−a)(1−b) < 41 , where 0 < a < 1, 0 < b < 1.
(1−ab)2  
Hint. Let a  sin2 α, b  sin2 β, where α, β ∈ 0, π2 .
16. max(a1 , . . . , an ) ≥ 2, where n > 3, a1 + · · · + an ≥ n, a12 + · · · + an2 ≥ n 2 .
Hint. Let ai  2 − bi and bi > 0, i  1, . . . , n. We have that b1 + · · · + bn ≤ n,
b12 + · · · + bn2 − 4(b1 + · · · + bn ) ≥ n(n − 4), and therefore, (b − 4)(b1 + · · · + bn ) ≥
n(n − 4), where b  max(b
  1 , . . . , bn ). 2
(an −an−1 )2 n −an−1 )
√ √ √
17. a1 + 4(n−2) + · · · + an−2 + (a4(n−2) + an−1 + an ≤ n,
where n ≥ 3, a1 , . . . , an ≥ 0, and a1 + a2 + · · · + an  1.
√ √
Hint. an−1  x + t, an  x − t. Then x ≤ √12 , and the left-hand side of the

inequality
 is not greater than (n − 2)(1 − 2x 2 ) + 2x.
18. 2 (x 2 − 1)(y 2 − 1) ≤  2(x − 1)(y − 1) + 1, where 0 ≤ x, y ≤ 1.
19. a + b + c − 3abc ≤ (a 2 + b2 + c2 )3 .
3 3 3
1
20. n−1+x 1
+ · · · + n−1+x1
n
≤ 1, where x1 , . . . , xn > 0 and x1 · · · · · xn  1.
Hint. Let xi  yin , i  1, . . . , n, where yi > 0. Then y1 · · · · · yn  1 and
yn (n−1)y n−1
n − 1 + xi  n − 1 + y1 ·····y
i
n
≥ n − 1 + y n−1 +···+y n−1
i
−yin−1
.
1 n

21. √1−x + √1−y ≥ √ x+y , where 0 ≤ x < 1, 0 ≤ y < 1.


x y x+y
1− 2
√ √
x +···+ x
22. √ x1
1−x1
+ · · · + √ xn
1−xn
≥ 1√n−1 n , where n ≥ 2, n ∈ N, x1 , . . . , xn > 0 and
x1 + . . . + xn  1.
Hint. Take 1 − xi √ai , i  1, . . . , n.
23. √ x 2 + √4xy2 +1 ≤ 22 , where 0 ≤ x, y ≤ 0.5.
4y +1
Hint. Let 2x  tan α, 2y  tan β, where 0 ≤ α, β ≤ π4 .
24. 0 ≤ ab+bc+ca−abc ≤ 2, where a > 0, b > 0, c > 0 and a 2 +b2 +c2 +abc  4.
25. a + b + c ≤ 3, where a > 0, b√> 0, c > 0, and a 2 + b2 + c2 + abc  4.
26. (x − 1)(y − 1)(z − 1) ≤ 6 3 − 10, where x > 0, y > 0, z > 0, and
x + y + z  x yz.  
Hint. Let x  tan α, y  tan β, z  tan γ , where α, β, γ ∈ 0, π2 . Then
β−sin β
α + β + γ  π and β > π4 . Note that (tan α − 1)(tan γ − 1)  2 + cos
cos α cos γ

cos β−sin β
2+ cos π
cos(α+γ − π3 )
.
3
70 5 Change of Variables Method
  
x+y
27. 3
+ 3 y+z
2z 2x
+ 3 z+x ≤ 5(x+y+z)+9 , where x > 0, y > 0, z > 0, and x yz  1.
2y 8
   4
 
 (a+b)
2 −ab+b2 3
Hint. 3 21 a 3 b3 a 3 + b3 ≤ 21 (a + b) ab+ab+ab+a ,
3
4 8
where a > 0, b > 0.
28. Prove that among four arbitrary numbers there are two numbers a and b such
that √ 1+ab √ > 21 .
1+a 2 · 1+b2
29. Given that x + y + z  0 and x 2 + y 2 + z 2  6. Find all possible values of the
expression x 2 y + y 2 z + z 2 x.  √
1− 1−h 2n
30. Let (h n ) be a sequence such that h 1  21 and h n+1  2
, n  1, 2, . . ..
Prove that h 1 + · · · + h n ≤ 1.03.
Chapter 6
Using Symmetry and Homogeneity

In order to prove certain inequalities, one often needs to use the symmetry and
homogeneity of mathematical expressions.

Definition 1 A mathematical expression is called symmetric with respect to the set


of variables x1 , x2 , . . . , xn , n ∈ N, if its value remains unchanged under every per-
mutation of variables x1 , x2 , . . . , xn .

Remark For purposes of simplicity and brevity, if a mathematical expression is sym-


metric with respect to the set of variables x1 , x2 , . . . , xn , then hereinafter we shall
call it symmetric.

Definition 2 A mathematical expression is called homogeneous of degree k with


respect to the set of variables x1 , x2 , . . . , xn , n ∈ N if for every positive λ, on replacing
the variables x1 , x2 , . . . , xn by the variables λx1 , λx2 , . . . , λxn , the value of the given
mathematical expression is multiplied by λk .

Remark For purposes of simplicity and brevity, if a mathematical expression is


homogeneous of degree k with respect to the set of variables x1 , x2 , . . . , xn , then
hereinafter we shall call it homogeneous.

The symmetry and homogeneity of mathematical expressions allow us to deduce


additional conditions for variables. For example, if a mathematical expression is
symmetric, then an additional condition for variables can be assuming without loss
of generality that x1 ≤ x2 ≤ · · · ≤ xn , and if it is homogeneous, then such a condition
can be assuming without loss of generality that x1 ·x2 · · · xn  1 or x1k +x2k +· · ·+xnk  1,
where k ∈ N.
In order to demonstrate how these additional conditions can be obtained, let us
consider the following examples of symmetric or/and homogeneous mathematical
expressions.

© Springer International Publishing AG, part of Springer Nature 2018 71


H. Sedrakyan and N. Sedrakyan, Algebraic Inequalities, Problem Books
in Mathematics, https://doi.org/10.1007/978-3-319-77836-5_6
72 6 Using Symmetry and Homogeneity

Example 6.1 Prove that

x(x − z)2 + y(y − z)2 ≥ (x − z)(y − z)(x + y − z), (6.1)

where x ≥ 0, y ≥ 0, z ≥ 0.

Proof Note that (6.1) can be rewritten as


x3 + y3 + z 3 − x2 y − x2 z − y2 x − y2 z − z 2 x − z 2 y + 3xyz ≥ 0, whence we obtain
a symmetric inequality with respect to the variables x, y, z. Therefore, without loss
of generality, one can assume that x ≥ z ≥ y. It follows that

x(x − z)2 + y(y − z)2 ≥ 0 ≥ (x − z)(y − z)(x + y − z), whence


x(x − z)2 + y(y − z)2 ≥ (x − z)(y − z)(x + y − z).

This ends the proof.

Example 6.2 Hölder’s inequality (particular  case):   


Prove that (a1 b1 c1 + · · · + an bn cn )3 ≤ a13 + · · · + an3 b31 + · · · + b3n c13 + · · · + cn3 ,
where ai > 0, bi > 0, ci > 0, i  1, . . . , n.

Proof Note (that if we substitute the variables a1 , . . . , an by the variables


λa1 , . . . , λan , where λ is an arbitrary positive number, then we obtain an equiva-
lent inequality.
Therefore, we can choose λ such that (λa1 )3 + · · · + (λan )3  1.
Without loss of generality one can assume that a13 + · · · + an3  b31 + · · · + b3n 
c1 + · · · + cn3  1. Hence in order to complete the proof, it is sufficient to prove that
3

a1 b1 c1 + · · · + an bn cn ≤ 1.
a3 +b3 +c3
We have ai bi ci ≤ i 3i i , i  1, . . . , n (Problem 2.1).
Summing these inequalities, we obtain a1 b1 c1 + · · · + an bn cn ≤ 1.
This ends the proof.
 a  b  c
Example 6.3 Prove that b+c + a+c + a+b > 2, where a > 0, b > 0, c > 0.

Proof Without loss of generality one can assume that a + b + c  1.


Note that

a
≥ 2a, (1)
1−a

and the equality in (1) holds if a  21 . It follows that


     
a b c a b c
+ +  + + > 2a + 2b + 2c  2.
b+c a+c a+b 1−a 1−b 1−c

This ends the proof.


Problems 73

Problems

Prove the following inequalities.

6.1. abc ≥ (a + b − c)(b + c − a)(a + c −b), where a >  0, b > 0, c > 0.


6.2. (a1 b1 + · · · + an bn )2 ≤ a12 + · · · + an2 b21 + · · · + b2n .
 n
6.3. (a + b)2 · . . . · (an + bn )2 ≥ an+1 + bn+1 , where a > 0, b > 0.
 β  β β

6.4. a1α + · · · + anα ≤ a1 + · · · + an , where 0 < β < α, a1 > 0, . . . , an >
0.
6.5. Nesbitt’s inequality: b+c a b
+ c+a + c ≥ 23 , where a > 0, b > 0, c > 0.
 a    a+b
6.6. b+c+d
+ a+c+db
+ a+b+d c
+ a+b+c d
> 2, where a > 0, b > 0, c > 0, d >

0. 
6.7. 3 abc+abd +acd +bcd
≤ ab+ac+ad +bc+bd +cd
, where a > 0, b > 0, c > 0, d > 0.
√ 4
√ √ 6
6.8. 2 ab + bc + ac ≤ 3 (b + c)(c + a)(a + b), where a > 0, b > 0, c > 0.
3
 2    
6.9. 8 x3 + y3 + z 3 ≥ 9 x2 + yz y2 + xz z 2 + xy , where x > 0, y > 0, z >
0.
6.10. (a) 4a3 +4b3 +4c3 +15abc ≥ 1, where a ≥ 0, b ≥ 0, c ≥ 0 and a +b+c  1,
 
(b) a3 + b3 + c3 + abcd ≥ min 41 , 19 + 27 d
, where a ≥ 0, b ≥ 0, c ≥ 0 and
a + b + c  1.
  √
a12 +···+an2
6.11. a1 +···+a
n
n
≥ 1
n n
+ 1 − 1n n a1 · · · an , where n ≥ 2, ai > 0, i 
1, . . . , n.
6.12. Turkevici’s inequality:

a4 + b4 + c4 + d 4 + 2abcd ≥ a2 b2 + a2 c2 + a2 d 2 + b2 c2 + b2 d 2 + c2 d 2 ,

where a ≥ 0, b ≥ 0, c ≥ 0, d ≥ 0.
a13 a3
6.13. b1
+ · · · + bnn ≥ 1, where ai > 0, bi > 0, i  1, . . . , n, and (a12 + · · · + an2 )3 
b21 + · · · + b2n .
6.14. ab + bc + ac ≥ a+c + b+a + c+b , where a > 0, b > 0, c > 0.
 n
b+c  a+b
c+a
a1 an
6.15. an +λa1 ···an
+ · · · + an +λan1 ···an ≥ √1+λ
n
, where n ≥ 2, a1 > 0, . . . , an > 0,
1 n

and λ ≥ n2 − 1. 

6.16. ( k 2 − 1)(a1 + · · · + an ) < k 2a1k + · · · + 2n ank , where k ∈ N, k ≥ 2, a1 >
0, . . . , an > 0.
6.17. 3(x2 y +y2 z +z 2 x)(xy2 +yz 2 +zx2 ) ≥ xyz(x +y +z)3 , where x > 0, y > 0, z > 0.
6.18. (x1 + · · · + xn + y1 + · · · + yn )2 ≥ 4n(x1 y1 + · · · + xn yn ), where x1 ≤ · · · ≤ xn ≤
y1 ≤ · · · ≤ yn .
74 6 Using Symmetry and Homogeneity

Proofs

6.1. Without loss of generality one can assume that a ≤ b ≤ c. Note that the given
inequality is equivalent to the following inequality: (b − c)2 (b + c − a) ≥
a(a − b)(c − a).
The last inequality can be proved in the following way: (b − c)2 (b + c − a) ≥
0 ≥ a(a − b)(c − a).
6.2. If a12 + · · · + an2  0 and b21 + · · · + b2n  0, then without loss of generality one
can assume that a12 + · · · + an2  1, b21 + · · · + b2n  1.
a2 +b2 a2 +b2
We have that − i 2 i ≤ ai bi ≤ i 2 i , i  1, . . . , n.
Summing these inequalities we obtain
−1 ≤ a1 b1 + · · · + an bn ≤ 1; hence
  
(a1 b1 + · · · + an bn )2 ≤ 1  a12 + · · · + an2 b21 + · · · + b2n .

If a12 + · · · + an2  0 or b21 + · · · + b2n  0, then the proof is obvious.


6.3. Without loss of generality one can assume that an+1 + bn+1  1.
Therefore, 0 < a < 1, 0 < b < 1 and ai + bi ≥ an+1 + bn+1  1, i 
1, . . . , n.  n
Thus, it follows that (a + b)2 · · · (an + bn )2 ≥ 1  an+1 + bn+1 .
β β
6.4. Without loss of generality one can assume that a1 +· · ·+an  1, and therefore,
β
0 < ai ≤ 1, i  1, . . . , n. Since α > β, we obtain that aiα ≤ ai , i 
1, . . . , n.  β
β β
It follows that a1α + · · · + anα < a1 + · · · + an  1, and hence a1α + · · · + anα ≤
 α
β β
1  a1 + · · · + an .
6.5. Without loss of generality one can assume that a + b + c  1.
Note that if 0 < x < 1, then 1−x x
≥ 9x−1
4
. Therefore,
a b c a b c 9a − 1 9b − 1 9c − 1 3
+ +  + + ≥ + +  .
b+c c+a a+b 1−a 1−b 1−c 4 4 4 2

6.6. Without loss of generality one can assume that a + b + c + d  1.


We have
   
a b c d
+ + +
b+c+d a+c+d a+b+d a+b+c
   
a b c d
 + + + > 2a + 2b + 2c + 2d  2.
1−a 1−b 1−c 1−d

6.7. Without loss of generality one can assume that d  1. From Problems 2.1
and 2.2, it follows that
Proofs 75


 

ab+bc+ac 3
abc + ab + bc + ac
3
3
+ ab + bc + ac
≤  A and
3

4 4


ab + bc + ac + a + b + c ab + bc + ac + 3(ab + bc + ac)
≥  B.
6 6

Let us prove that B ≥ A.


Let ab + bc + ac  3t 2 , where t > 0. Then the inequality B ≥ A is equivalent
to the inequality (t − 1)2 (t + 2) ≥ 0.
6.8. Without loss of generality one can assume that

ab + bc + ac  1. (1)

We need to prove that


8
(b + c)(a + b)(a + c) ≥ √ .
3 3
 
Let a  tan α, b  tan β, c  tan γ , where α, β, γ ∈ 0, π2 . From (1) it
follows that α +β +γ  π2 , and therefore, (b+c)(a +b)(a +c)  cos α cos1 β cos γ .
In order to end the proof, it is sufficient to prove that
√  π
3 3 π
cos α cos β cos γ ≤ , if α, β, γ ∈ 0, and α + β + γ  .
8 2 2
Note that
1 1
cos α cos β cos γ  (cos(α + β) + cos(α − β)) cos γ ≤ (1 + sin γ ) cos γ
2 2
 
1 1
 (1 + sin γ )3 (1 − sin γ )  √ (1 + sin γ )3 (3 − 3 sin γ )
2 2 3
 √
1 1 + sin γ + 1 + sin γ + 1 + sin γ + 3 − 3 sin γ 2 3 3
≤ √  .
2 3 4 8

6.9. Without loss of generality one can assume that

x3 + y3 + z 3  1. (1)

Then we have
   
A  x2 + yz y2 + xz z 2 + xy  2(xyz)2 + xyz + x3 y3 + y3 z 3 + z 3 x3 .

Since x3 + y3 + z 3  1, it follows that xyz ≤ 13 and x3 y3 + y3 z 3 + z 3 x3 ≤


(x3 +y3 +z3 )2  1 , and therefore, A ≤ 2 + 1 + 1  8 .
3 3 9 3 3 9
76 6 Using Symmetry and Homogeneity

6.10. (a) Note that the inequality 4a3 +4b3 +4c3 +15abc ≥ (a + b + c)3 is equivalent
to the inequality a3 + b3 + c3 + 3abc ≥ a2 b + a2 c + b2 a + b2 c + c2 a + c2 b (see
Example 6.1).

(b) If d ≥ 15
4
, then
15 1 3  1
a3 + b3 + c3 + abcd ≥ a3 + b3 + c3 + abc  4a + 4b3 + 4c3 + 15abc ≥ .
4 4 4
 3
If d < 15 , then according to 6.10(a) and the inequality abc ≤ a+b+c 
4   3
1
,
27 
we have 27 a 3
+ b3
+ c 3
− 3 ≥ 15
(1 − 27abc), and therefore,
 4
27 a3 + b3 + c3 − 3 ≥ d (1 − 27abc), or a3 + b3 + c3 + abcd ≥ 19 + 27 d
.

6.11. Without loss of generality one can assume that a1 a2 · · · an  1. We need to


prove that
 
1  2  2
1− a1 + · · · + an2 − √ (n − 1) a12 + · · · + an2 + 2a1 a2 + · · · + 2a1 an−1 + 2a1 an + · · · + 2an−1 an ≥ (n − 1)2 , or
n n
 2
1
(n − 1) √ a12 + · · · + an2 − 1 + 2a1 a2 + · · · + 2a1 an−1 + 2a1 an + · · · + 2an−1 an ≥ n(n − 1).
n

The proof of this inequality follows from Problem 4.1:



Cn2
a1 a2 + a1 a3 + · · · + a1 an−1 + a1 an + · · · + an−1 an ≥ Cn2 (a1 a2 . . . an )n−1  Cn2 .

6.12. Without loss of generality one can assume that a ≥ b ≥ c ≥ d ≥ 0. We have


A  a4 + b4 + c4 + d 4 + 2abcd − a2 b2 − a2 c2 − a2 d 2 − b2 c2 − b2 d 2 − c2 d 2
 (a2 − b2 )(a2 − c2 − d 2 ) + 2bcd (a − b) + (b2 + cd − c2 − d 2 )2 + 2cd (c − d )2
 
 (a − b) a(a2 − d 2 ) − b(c − d )2 + a(a − b)(ab − c2 )

+ (b2 + cd − c2 − d 2 )2 + 2cd (c − d )2 .

Since a − b ≥ 0, a(a2 − d 2 ) ≥ b(a − d )2 ≥ b(c − d )2 and ab ≥ c2 , it follows


A ≥ 0.
that √
6.13. Let 3 bi  ci , i  1, . . . , n. Then we have c16 + · · · + cn6  (a12 + · · · + an2 )3 .
a3 a3
We need to prove that c31 + · · · + c3n ≥ 1.
1 n
Without loss of generality one can assume that a12 + · · · + an2  1, and hence
c16 + · · · + cn6  1.
From inequality (8.4) from Chapter 8, it follows that
a13 an3 a4 a4 (a12 + · · · + an2 )2 1
+ ··· +  13 + · · · + n3 ≥  ≥ 1.
c13 3
cn a1 c1 an cn a1 c13 + · · · + an cn3 a1 c13 + · · · + an cn3

Then from inequality (4.2) from Chapter 4, it follows that 1  (a12 + · · · +


an2 )(c16 + · · · + cn6 ) ≥ (a1 c13 + · · · + an cn3 )2 , whence
a3 a3
1 ≥ a1 c13 + · · · + an cn3 , and hence c31 + · · · + c3n ≥ 1.
1 n
6.14. Without loss of generality one can assume that max(a, b, c)  a. Then a ≥
b ≥ c or a ≥ c ≥ b.
Proofs 77

If a ≥ b ≥ c, then we have
a b c a+c b+a c+b c(a − b) a(b − c) b(a − c)
+ + − − −  + −
b c a b+c c+a a+b b(b + c) c(a + c) a(b + a)
c(a − b) a(b − c) b(a − c)
≥ + −  0,
a(b + a) a(a + b) a(b + a)

whence
a b c a+c b+a c+b
+ + ≥ + + .
b c a b+c c+a a+b
If a ≥ c ≥ b, then we have
a b c a+c b+a c+b c(a − b) a(c − b) b(a − c)
+ + − − −  − −
b c a b+c c+a a+b b(b + c) c(a + c) a(b + a)
c(a − b) a(c − b) b(a − c)
≥ − −  0,
b(b + c) b(b + c) b(b + c)

and therefore,
a b c a+c b+a c+b
+ + ≥ + + .
b c a b+c c+a a+b
6.15. Without loss of generality one can assume that a1 · . . . · an  1. Then
 n the
a1
expression on the left-hand side of the given inequality is equal to an +λ +
 n 1
an
· · · + an +λ .
n
By the Cauchy–Bunyakovsky–Schwarz inequality and inequality (8.4) from
Chapter 8, we have
n 2 n 2
a12 an2
a1n ann
+ ··· +   + ··· +  n
a1n + λ ann + λ n
a12 (a1n + λ) an2 (ann + λ)
n n 2 n n 2
a12 + · · · + an2 a12 + · · · + an2
≥   n ≥ .
n n n  
a12 (a1n + λ) + · · · + an2 (ann + λ) a12 + · · · + an2 a1n + · · · + ann + nλ

In order to complete the proof, it is sufficient to prove that


n
(1 + λ)(b1 + · · · + bn )3 ≥ n2 (b31 + · · · + b3n ) + n3 λ, where bi  ai3 > 0, i 
1, . . . , n, and b1 · . . . · bn  1.
Note that
(b1 + · · · + bn )3  (b21 + · · · + b2n + 2b1 b2 + · · · + 2b1 bn + · · · + 2bn−1 bn )(b1 + · · · + bn )
78 6 Using Symmetry and Homogeneity

  2
≥ (b21 + · · · + b2n + n(n − 1) (b1 · . . . · bn )n−1 n(n−1) )(b1 + · · · + bn )
 (b21 + · · · + b2n + n(n − 1))(b1 + · · · + bn ) ≥ b31 + · · · + b3n + b21 b2 + · · · + b21 bn + · · · + b2n−1 bn
+ n2 (n − 1) ≥ b31 + · · · + b3n + n(n − 1) + n2 (n − 1)  b31 + · · · + b3n + n3 − n,

whence (b1 + · · · + bn )3 ≥ b31 + · · · + b3n + n3 − n. Thus


(1 + λ)(b1 + · · · + bn )3 − n2 (b31 + · · · + b3n ) − n3 λ ≥ (1 + λ)(b31 + · · · + b3n + n3 − n)
− n2 (b31 + · · · + b3n ) − n3 λ ≥ (λ − (n2 − 1))(b31 + · · · + b3n ) + (1 + λ)(n3 − n) − n3 λ
≥ (λ − (n2 − 1))n + (1 + λ)(n3 − n) − n3 λ  0,

and therefore, (1 + λ)(b1 + · · · + bn )3 ≥ n2 (b31 + · · · + b3n ) + n3 λ.


6.16. Without loss of generality one can assume that 2a1k + · · · + 2n ank  1, whence
2i aik < 1.     n n
Therefore, a1 +. . .+an < √
k
1
+. . .+ √
k
1
< √
k
1
+. . .+ √
k
1
+. . .  √
k
1
,
2 2 2 2 2−1
and thus
√ 
( 2 − 1)(a1 + · · · + an ) < 1  k 2a1k + · · · + 2n ank .
k

6.17. According to Example 6.2 from Chapter 6, we have

3(x2 y + y2 z + z 2 x)(xy2 + yz 2 + zx2 ) ≥ xyz(x + y + z)3 ,


  3  3    
3 2 3 2 3 2 3
3(x2 y + y2 z + z 2 x)(xy2 + yz 2 + zx2 )  (13 + 13 + 13 ) x y + y z + z x
 

3
3  3  3 √ 
· zx2 +
3
xy2 +
3
yz 2 ≥ 3 xyz(x + y + z) 3  xyz(x + y + z)3 ,

and therefore, 3(x2 y + y2 z + z 2 x)(xy2 + yz 2 + zx2 ) ≥ xyz(x + y + z)3 .


6.18. Note that if we substitute the numbers x1 , . . . , xn , y1 , . . . , yn by the numbers
x1 + x, . . . , xn + x, y1 + x, . . . , yn + x, where x is an arbitrary number, then we
obtain an equivalent inequality. Hence, we can choose the number x such that

x1 + x + · · · + xn + x + y1 + x + · · · + yn + x  0.

Therefore, without loss of generality one can assume that x1 + · · · + xn + y1 +


· · · + yn  0.
Note that the given inequality is equivalent to the inequality

x1 y1 + · · · + xn yn ≤ 0. (1)

If x1 ≤ · · · ≤ xn ≤ 0 ≤ y1 ≤ · · · ≤ yn , then x1 y1 ≤ 0, . . . , xn yn ≤ 0, and
hence (1) holds.
Proofs 79

In order to complete the proof, it is sufficient to prove (1) if x1 ≤ · · · ≤


xk ≤ 0 ≤ xk+1 ≤ · · · ≤ xn ≤ y1 ≤ · · · ≤ yn (such a number k exists,
for otherwise, we could substitute the numbers x1 , . . . , xn , y1 , . . . , yn by the
numbers −yn , . . . , −y1 , −xn , . . . , −x1 ).
Hence, we have that
x1 y1 + · · · + xn yn  y1 (x1 − xn ) + · · · + yk (xk − xn ) + xk+1 yk+1 + · · · + xn−1 yn−1 + xn (−x1 − . . . − xn
− yk+1 − . . . − yn−1 ) ≤ xn (x1 − xn ) + · · · + xn (xk − xn ) + xk+1 yk+1 + · · · + xn−1 yn−1 + xn (−x1 − . . . − xn
− yk+1 − . . . − yn−1 )  −kxn2 − xn (xk+1 + · · · + xn ) + yk+1 (xk+1 − xn ) + . . . + yn−1 (xn−1 − xn ) ≤ 0,

whence x1 y1 + · · · + xn yn ≤ 0.

Problems for Independent Study

Prove the following inequalities.


1. ln z−y
z−ln y
< ln z−x
z−ln x
< ln y−x
y−ln x
, where 0 < x < y < z.
2. ab bc cd d a ≥ ba cb d c ad , where 0 < a ≤ b ≤ c ≤ d .
x1
3. S−x 1
+ · · · + S−x
xn
n
≥ n−1
n
, where n ≥ 2, S  x1 + · · · + xn , x1 > 0, . . . , xn > 0.
4. a + b + c + 6abc ≥ 4 (a + b + c)3 , where a ≥ 0, b ≥ 0, c ≥ 0.
3 3 3 1

5. a2 (2b + 2c − a) + b2 (2a + 2c − b) + c2 (2a + 2b − c) ≥ 9abc, where a, b, c are


the side lengths of some √ triangle. √

6. (a) n a1 · . . . · an + n b1 · . . . · bn ≤ n (a1 + b1 ) · . . . · (an + bn ), where n ≥
2, ai > 0, bi > 0, i  1, . . . , n,
√ √
(b) √ n
(n + 1)! − n n! ≥ 1, where n ≥ 2, n ∈ N,
(c) n Fn+1 > 1 + √n 1F , where n ≥ 2, n ∈ N, F1  1, F2  2, Fk+2 
n
Fk+1 + Fk ,  k  1, 2,. . .,
 n
(d) n C2n+1 > 2 1 + √n n+1
1
, where n  2, 3, . . .,
n
(e) (1 + a1 ) · . . . · (n + an ) ≥ n 2 , where n ≥ 2, n ∈ N, a1 > 0, . . . , an > 0 and
a1 · . . . · an  1,
 √ √
n a ·...·a + n b ·...·b
(f) n (a(a11−c
+b1 )·...·(an +bn )
1 )·...·(an −cn )
≥ n a ·...·a − n c ·...·c , where n ≥ 2,
√ 1
1
n
n
√1
1
n
n
n ∈ N, bi > 0,

ai > ci > 0, i  1, . . . , n,
√ √
1 −c1 )·...·(an −cn )
n n
Hint. Prove that √n a(a·...·a
1 +b1 )·...·(a
√ n +bn )
≥ 1 ≥ √n a(a1 ·...·a √
n − c1 ·...·cn
.
n + b1 ·...·bn
n n
√ √ √ 1

ab + cd ≤ (a + c + d )(a + c + b), where a > 0, b > 0, c > 0, d >


3 3 3
(g)
0.
Hint. Note that
  
3 √ √ 3 √ √ 3
√ √
a b b + c d d ≤ (a + c)( b + d )2
80 6 Using Symmetry and Homogeneity
 √ 
3
≤ (a + c)(b + d ) + 2(a + c) bd ≤ 3 (a + c)(b + d ) + (a + c)2 + bd .

7. x2 (x2 − 1)2 + y2 (y2 − 1)2 ≥ (x2 − 1)(y2 − 1)(x2 + y2 − 1).


8. x2 (x − 1)2 + y2 (y − 1)2 + z 2 (z − 1)2 ≥ 2xyz(2 − x − y − z).
Hint. Prove that

x2 (x − 1)2 + y2 (y − 1)2 + z 2 (z − 1)2 − 2xyz(2 − x − y − z)  x2 (x − 1)2


+ (y(y − 1) − z(z − 1))2 + 2xyz(x + y − 1)(x + z − 1).

9. (x1 − x2 )(x1 − x3 )(x1 − x4 )(x1 − x5 ) + (x2 − x1 )(x2 − x3 )(x2 − x4 )(x2 − x5 )+


· · · + (x5 − x1 )(x5 − x2 )(x5 − x3 )(x5 − x4 ) ≥ 0.
10. 0 ≤ ab+bc+ca −abc ≤ 2, where a ≥ 0, b ≥ 0, c ≥ 0 and a2 +b2 +c2 +abc  4.
Hint. Note that min(a, b, c) ≤ 1, and without loss of generality one can assume
that (a − 1)(b − 1) ≥ 0.
11. xλ (x − y)(x − z) + yλ (y − z)(y − x) + z λ (z − x)(z − y) ≥ 0, where x, y, z > 0.
Hint. Let x ≥ y ≥ z.
Note that if λ ≥ 0, then we have

xλ (x − y)(x − z) + yλ (y − z)(y − x) + z λ (z − x)(z − y)  (x − y)(xλ (x − z)


− yλ (y − z)) + z λ (z − x)(z − y) ≥ 0.

Otherwise, if λ < 0, then we have

xλ (x − y)(x − z) + yλ (y − z)(y − x) + z λ (z − x)(z − y)  xλ (x − y)(x − z)


+ (y − z)(z λ (x − z) − yλ (x − y)) ≥ 0.
     
a 2 b 2 c 2
≥ √334 , where a > 0, b > 0, c > 0.
3 3 3
12. b+c
+ a+c
+ a+b
13. (a − a + 3)(b − b + 3)(c − c + 3) ≥ (a + b + c)3 , where a > 0, b > 0, c > 0.
5 2 5 2 5 2

Hint. Note that a5 − a2 + 3  a2 (a3 − 1) + 3 ≥ 1 · (a3 − 1) + 3  a3 + 13 + 13 .


14. abc + abd + bcd + acd − abcd ≤ 3, where a > 0, b > 0, c > 0, d > 0, and
a3 + b3 + c3 + d 3 + abcd  5.
 Prove that 
Hint.
3 a3 + b3 + c3 + d 3 (a + b + c + d ) + 12abcd
≥ 5((abc + abd + bcd + acd )(a + b + c + d ) − 4abcd ).
15. 0 ≤ ab+bc+ca−abc ≤ 2, where a ≥ 0, b ≥ 0, c ≥ 0 and a2 +b2 +c2 +abc  4.
 + 1 ≥22(ab + bc 2+ ca), where
16. a2 + b2 + c2 + 2abc
x+y+z  a ≥ 0, b ≥ 0, c ≥ 0.
17. xy+yz+zx ≤ 1 + 48
1
(x − y) + (y − z) + (z − x)2 , where x > 0, y > 0, z > 0,
and xy + yz + zx + xyz  4.
Chapter 7
The Principle of Mathematical Induction

A large number of inequalities that at first glance appear difficult can be easily proved
by a classical mathematical proof technique called the principal of mathematical
induction.
This chapter is devoted to some applications of the principal of mathematical
induction in proving algebraic inequalities. Note that there are different kinds of
mathematical induction. In this chapter we mainly consider applications of the clas-
sical form of the principle of mathematical induction called the (first) principle of
mathematical induction, since this variety is applied most often. Nevertheless, we
also formulate two other relatively important variants of the principle of mathematical
induction.
Let Sn be a statement about an arbitrary positive integer n. The (first) principle of
mathematical induction is used to prove that the statement Sn holds for all positive
integral values of n.
The principle of mathematical induction implies that in order to prove that state-
ment Sn holds for all such values of n, one needs to prove first that statement S1 holds
and then that from the assumption that statement Sk holds for an arbitrary positive
integer k, it follows that statement Sk+1 holds as well.
Putting together these explanations, we deduce the following formulation.
(First) principle of mathematical induction.
Let Sn be a statement about an arbitrary positive integer n.

(i). The basis (first step/base case): S1 is true.


(ii). The inductive step: Whenever Sk is true for k ∈ N, then Sk+1 is also true.
The assumption that Sk holds for some k ∈ N is called the induction hypoth-
esis (inductive hypothesis).
(iii). Conclusion: If (i) and (ii) hold, then Sn is true for all n ∈ N.

Remark Note that sometimes instead of n ∈ N, one can consider n ∈ N0 


{0, 1, 2, . . .}, in which case one needs to verify (the basis) that S0 is true.

© Springer International Publishing AG, part of Springer Nature 2018 81


H. Sedrakyan and N. Sedrakyan, Algebraic Inequalities, Problem Books
in Mathematics, https://doi.org/10.1007/978-3-319-77836-5_7
82 7 The Principle of Mathematical Induction

So, the (first) principle of mathematical induction implies that if the basis and the
inductive step are proved, then Sn is true for all positive integers n.
Let us consider another useful variant of the principle of mathematical induc-
tion called the second principle of mathematical induction. (It is also called “strong
induction,” because a stronger induction hypothesis is used.)
Second principle of mathematical induction.
Let Sn be a statement about an arbitrary positive integer n.
(i). The basis: S1 is true.
(ii). The inductive step: Whenever Si is true for all i ≤ k, k ∈ N, it follows that
Sk+1 is true.
(iii). Conclusion: If (i) and (ii) hold, then Sn is true for all n ∈ N.
Now let us consider the following variant of the principle of mathematical induc-
tion that is used to prove that statement Sn holds not for all positive integers n, but
only for all positive integers greater than or equal to a certain positive integer k.
Different starting point (induction basis other than 1 or 0).
Let m be a given positive integer and let Sn be a statement about an arbitrary
positive integer n, where n ≥ m.
(i). The basis: Sm is true.
(ii). The inductive step: Whenever Si is true for all m ≤ i ≤ k, it follows that
Sk+1 is true.
(iii). Conclusion: If (i) and (ii) hold, then Sn is true for all n ∈ N, n ≥ m.
Below we provide examples of inequalities whose proofs make use of the (first)
principle of mathematical induction.
Example 7.1 Prove that 1
2n+1
+ 1
2n+2
+ ··· + 1
2n+n
≥ 11
30
, where n > 1, n ∈ N.

Proof If n  2, then we have 30


11
≥ 30
11
.
Assume that the given inequality is true for n  k, that is,
1 1 1 11
+ + ··· + ≥ . (7.1)
2k + 1 2k + 2 2k + k 30
Let us prove that the given inequality is true for n  k + 1.
For n  k + 1 we need to prove that
1 1 1 11
+ + ··· + ≥ .
2(k + 1) + 1 2(k + 1) + 2 2(k + 1) + (k + 1) 30

Adding to both sides of inequality (7.1) the expression 1


3k+1
1
+ 3k+2 1
+ 3k+3 − 2k+1
1

1
2k+2
, we obtain
1 1 11 1 1 1 1 1
+ ··· + ≥ + + + − − .
2k + 3 3k + 3 30 3k + 1 3k + 2 3k + 3 2k + 1 2k + 2

Since 1
3k+1
+ 1
3k+2
+ 1
3k+3
− 1
2k+1
− 1
2k+2
> 0 (Problem 1.19), it follows that
7 The Principle of Mathematical Induction 83

1 1 11
+ ··· + ≥ .
2k + 3 3k + 3 30
Therefore, the given inequality is true for every positive integer n.
This ends the proof.

Example 7.2 Prove that


1 1
+ · · · + 2 < 1, (7.2)
22 n
where n  3, 4, . . . .

Proof Note that it is impossible to prove this inequality similarly to how we proved
Example 7.1. In the proof of this inequality and in the proof of some other inequalities,
the method of induction is applied to another inequality (a more general one), from
which follows the validity of the considered inequality.
In order to prove inequality (7.2), let us prove the following inequality:
1 1 1
2
+ ··· + 2 < 1 − . (7.3)
2 n n
We carry out the proof by mathematical induction on n.
For n  3, we have 212 + 312 < 1 − 13 .
Assume that (7.3) is true for n  k (k ≥ 3), and let us prove that it is true for
n  k + 1. For n  k, we have 212 + · · · + k12 < 1 − k1 .
2 . Then we obtain
1
Let us add to both sides of this inequality the expression (k+1)
1
22
+ · · · + k 2 + (k+1)2 < 1 − k + (k+1)2 .
1 1 1 1

Since 1− k1 + (k+1) 2 < 1− k+1 , we deduce that the inequality 22 +· · ·+ k 2 + (k+1)2 <
1 1 1 1 1

1 − k+1
1
coincides with the given inequality for n  k + 1.
Therefore, the given inequality holds for every positive integer n ≥ 3.
This ends the proof.

In some inequalities the principle of mathematical induction is used in the fol-


lowing way:

(a) One first proves that the considered inequality holds for values of the positive
integer n equal to n 1 , . . . , n k , . . . , where n 1 < · · · < n k < · · ·.
(b) From the validity of the considered inequality for arbitrary n  k (k ≥ 2),
follows that it holds for n  k − 1.
x1 ···xn (1−x1 )···(1−xn )
Example 7.3 Ky Fan inequality: Prove that (x1 +···+xn )n
≤ ((1−x1 )+···+(1−xn ))n
, where
n ≥ 2, 0 < x1 ≤ 21 , . . . , 0 < xn ≤ 21 .

Proof Let us first prove the given inequality for n equal to 21 , . . . , 2k , . . . .


(1−x1 )(1−x2 )
For n  2, we have (x x+x1 x2
)2
≤ ((1−x )+(1−x ))2
(Problem 1.20).
1 2 1 2
84 7 The Principle of Mathematical Induction

Assume that the given inequality holds for n  2k (k ∈ N), and let us prove that
it holds for n  2k+1 .
We have
 p p
x1 · · · x2 p x1 · · · x p x p+1 · · · x2 p x1 + · · · + x p x p+1 + · · · + x2 p
 2 p   p ·  p ·  2 p
x1 + · · · + x2 p x1 + · · · + x p x p+1 + · · · + x2 p x1 + · · · + x2 p
⎛   ⎞p
      x1 +···+x p x p+1 +···+x2 p
(1 − x1 ) · · · 1 − x p 1 − x p+1 · · · 1 − x2 p ⎜ p p ⎟
≤    p ·     p · ⎝   ⎠
(1 − x1 ) + · · · + 1 − x p 1 − x p+1 + · · · + 1 − x2 p x1 +···+x p x +···+x2 p 2
+ p+1
p p
 
(1 − x1 ) · · · 1 − x2 p
≤    p     p
(1 − x1 ) + · · · + 1 − x p 1 − x p+1 + · · · + 1 − x2 p
⎛    ⎞p
x +···+x x +···+x  
1 − 1 p p 1 − p+1 p 2 p (1 − x1 ) · · · 1 − x2 p
⎜ ⎟
× ⎝    2 ⎠    2 p ,
1− 1
x +···+x p x +···+x2 p
+ 1 − p+1 (1 − x1 ) + · · · + 1 − x2 p
p p

where p  2k .
x1 ···x2 p (1−x1 )···(1−x2 p )
We obtain ≤ , whence the given inequality holds
(x1 +···+x2 p )2 p ((1−x1 )+···+(1−x2 p ))2 p
for n  2 .
k+1

Now let us prove that if the given inequality holds for m, then it holds true for
m − 1, where m ≥ 3 (m ∈ N).
1 ···x m (1−x1 )···(1−xm )
We have (x1 x+···+x m ≤ .
m) ((1−x1 )+···+(1−xm ))m
x1 +···+xm−1
Taking xm  m−1 , where by Problem 1.10, 0 < xm ≤ 21 , we obtain
 
x1 · · · xm−1 · x 1 +···+x m−1 (1 − x1 ) · · · (1 − xm−1 ) · 1 − x1 +···+x
m−1
m−1
m−1
  ≤   m .
x 1 +···+x m−1 m
x1 + · · · + xm−1 + m−1 (1 − x1 ) + · · · + (1 − xm−1 ) + 1 − x1 +···+x
m−1
m−1

Therefore, we have
x1 · · · xm−1 (1 − x1 ) · · · (1 − xm−1 )
≤ .
(x1 + · · · + xm−1 ) m−1
((1 − x1 ) + · · · + (1 − xm−1 ))m−1

Problems

Prove the following inequalities (7.1–7.4, 7.6, 7.7, 7.10, 7.11, 7.13–7.20, 7.24,
7.25).

7.1. (a) 1
2
· · · 2n−1
2n
≤ √1 ,
3n+1
where n ∈ N.
1
(b) n+1
+ < 25
+ 1
· · · + 2n
, where n ≥ 2, n ∈ N.
1
36 n+2
√ √
7.2. (a) a + a + · · · + a ≤ 1+ 24a+1 , where a ≥ 0,
  
n
Problems 85

 √
(b) 2 3 4 . . . n < 3, where n ≥ 2, n ∈ N.
7.3. x12 + · · · + (2n − 1)xn2 ≤ (x1 + · · · + xn )2 , where x1 ≥ · · · ≥ xn ≥ 0.
7.4. (a) |sin(x1 + · · · + xn )| ≤ |sin x1 | + · · · + |sin xn |,
(b) sin(x1 + · · · + xn ) ≤ sin x1 + · · · + sin xn , where x1 , . . . , xn ∈ [0, π ],
(c) |cos x1 | + |cos x2 | + |cos x3 | + |cos x4 | + |cos x5 | ≥ 1, where x1 + x2 + x3 +
x4 + x5  0.
7.5. Prove
(a) Bellman’s inequality: if a function f (x) is defined in [0, a) (or [0, +∞))
and for arbitrary numbers x ≥ y ≥ z from that interval we have f (x)− f (y)+
f (x) ≥ f (x − y + z) and f (0) ≤ 0, then for all numbers a > x1 ≥ · · · ≥
xn ≥ 0, the following inequality  holds: f (x1 )− f (x2 )+· · ·+(−1)
n−1
f (xn ) ≥
f x1 − x2 + · · · + (−1) xn ,
n−1
 
(b) tan x1 − tan x2 + · · · + (−1)n−1 tan xn ≥ tan x1 − x2 + · · · + (−1)n−1 xn ,
where π2 > x1 ≥ · · · ≥ xn ≥ 0,
 r
(c) a1r − a2r + · · · + (−1)n−1 anr ≥ a1 − a2 + · · · + (−1)n−1 an , where a1 ≥
· · · ≥ an ≥ 0, r ≥ 1.
7.6. (a) (x1 + · · · + x5 )2 ≥ 4(x1 x2 + x2 x3 + x3 x4 + x4 x5 + x5 x1 ),where x1 >
, x5 > 0.
0, . . .
(b) x1 xn2 + x22 + x2 x12 + x32 +· · ·+ xn−1 xn−2
2 2
+ xn2 + xn xn−1 + x12 ≤ 21 (x1 +
· · · + xn )2 , where n ≥ 3 and x1 > 0, . . . , xn > 0.
7.7. 2 (x 1
1
+ · · · + xn )2 ≤ (x1 + · · · + nxn ) · max(x1 , . . . , xn ), where x1 ≥
0, . . . , xn ≥ 0.
7.8. Prove Theorem 11.1 (Chapter 11).
7.9. Prove Theorem 11.2 (Chapter 11).
7.10. (a) a1 + · · · + ann ≤ na1 · . . . · an , where a1 ≥ · · · ≥ an ≥ 1;
(b) a1 + · · · + ann ≥ na1 · . . . · an , where 0 ≤ a1 ≤ · · · ≤ an ≤ 1.
a2 an2
7.11. (a) a21 + · · · + an−1 ≥ 4(an − a1 ), where a1 > 0, . . . , an > 0;
a3 a3 3
(b) b1 1c1 + · · · + bn ncn ≥ (b1 +···+b
(a1 +···+an )
n )(c1 +···+cn )
, where ai > 0, bi > 0, ci > 0, i 
1, . . . , n.
7.12. Prove that
(a) if f (x) is defined in I 1 and is a convex function,2 then
(x2 + x1 )( f (x2 ) − f (x1 )) + · · · + (xn + xn−1 )( f (xn ) − f (xn−1 )) ≥ (xn + x1 )( f (xn ) − f (x1 )),

where√n ≥ 2, √ x1 < √· · · <√ xn , x√ 1, . . . , x


√n ∈ I .
(b) a b + b c + c a ≥ a c + b a + c b, where a ≥ b ≥ c ≥ 0;
x
(c) x1x2 · x2x3 · . . . · xnx1 ≥ x2x1 · x3x2 · . . . · xn n−1 · x1xn , where xn ≥ · · · ≥ x1 >
0, n ≥ 3;
a(c−b)
(d) (c+b)(2a+b+c) b(a−c)
+ (a+c)(2b+a+c) c(b−a)
+ (b+a)(2c+b+a) ≤ 0,
where a ≥ b ≥ c > 0.

1I is defined as the domain of function f , so I = D(f)


2 See Chapter 11.
86 7 The Principle of Mathematical Induction

a1 +···+an−1
7.13. n−1
+ a1 +···+a
n+1
n+1
≥ 2 a1 +···+a
n
n
, where n ≥ 2, ak +a2 k+2 ≥ ak+1 , k 
1, . . . , n − 1.
7.14. (a) a1 +a3 +···+a
n
2n−1
≥ a0 +a2n+1
+···+a2n
, where ak ≥ ak−12+ak+1 , k  1, 2, . . . , 2n −
1, n ∈ N,
a 0 +a 2 +···+a 2n
3 +···+a 2n−1 ≥ , where a > 0, n ∈ N.
n+1
(b) a+a n
7.15. 1 + · · · + n > ln(n + 1), where n ∈ N.
1

7.16. 1 + · · · + n √1 n ≤ 3 − √2n , where n ∈ N.


7.17. (1 + α)n ≥ 1 + nα + n(n−1) α 2 , where α ≥ 0, n ∈ N.
 k+1 k 2
7.18. k! ≥ e , where k ∈ N.
n   √
7.19. sin(2i x) ≤ 1 + 3 n, where n  0, 1, 2, . . . .
2
i0
7.20. cos α + · · · + cosnnα ≥ − 21 , where n ∈ N, 0 ≤ α ≤ π2 .
7.21. Let the numbers a1 , . . . , an (n ≥ 2) be greater than 1 and |ak+1 − ak | < 1
for k  1, . . . , n − 1. Prove that the sum aa21 + aa23 + · · · + aan−1
n
+ aan1 is less than
2n − 1.
7.22. Let √1  x1 ≤ x2 ≤√· · · ≤ xn+1√ . Prove that
(a) xx2 2−x1 + · · · + xxn+1n+1−xn ≤ 4n−3
2
, where n ∈ N,
√ √
(b) xx2 2−x1 + · · · + xxn+1n+1−xn < 1 + 21 + · · · + n12 , where x2 , . . . , xn+1 ∈ N.
7.23. Prove that, if α1 > 0, . . . , αn > 0, β1 > 0, . . . , βn > 0, and α1 + · · · + αn ≤
β1 βn α1 αn
β1 + · · · + βn ≤ π, then cos sin α1
+ · · · + cos
sin αn
≤ cos
sin α1
+ · · · + cos
sin αn
.
7.24. 2(a 2012
+ 1)(b 2012
+ 1)(c 2012
+ 1) ≥ (1 + abc)(a 2011
+ 1)(b 2011
+ 1)(c2011 + 1),
where a > √ 0, b > 0,c > 0.
7.25. (a) b1 ≥ b2 ,
(b) Newton’s inequality: bk2 ≥ bk−1 bk+1 , for k  2, . . . , n − 1, where
n ≥ 2, n ∈ N, a1 > 0, . . . , an > 0, and
1
bk  (a1 · · · ak−1 ak + a1 · · · ak−1 ak+1 + · · · + a1 · · · ak−1 an + · · · + an−k+1 · · · an−1 an ).
Cnk

Proofs

7.1. (a) We proceed by mathematical induction on n.


The given inequality holds for n  1, since 21 ≤ √14 .
Assume that the given inequality holds for n  k, and let us prove that it holds
for n  k + 1.
For n  k, we have 21 · · · 2k−1
2k
≤ √3k+1
1
.
2(k+1)−1
Multiplying both sides of this inequality by 2(k+1)
, we deduce that
1 2k − 1 2k + 1 1 2k + 1
··· · ≤√ · .
2 2k 2k + 2 3k + 1 2k + 2
Proofs 87

Taking into consideration that √1


3k+1
· 2k+2
2k+1
≤ √1
3k+4
(Problem 1.21), we obtain
1
2
··· ·
2k−1
2k

2(k+1)−1
2(k+1)
√ 1
3(k+1)+1
.
Therefore, the given inequality holds for every positive integer n.
(b) In order to prove the given inequality, let us prove the following inequality:
1 1 1 1 25
+ + ··· + + ≤ , where n ≥ 2, n ∈ N. (1)
n+1 n+2 2n 4n + 1 36
We proceed by mathematical induction on n.
Note that inequality (1) holds for n  2, since 13 + 41 + · · · + 19  25
36
.
Assume that inequality (1) holds for n  k, where k ≥ 2, k ∈ N. Let us
prove that inequality (1) holds for n  k + 1.
We have
1 1 1 1 1 1
+ + ··· + +  +
k+2 k+3 2(k + 1) 4(k + 1) + 1 k+1 k+2
 
1 1 1 1 1 1 1 25
+ ··· + + + + − + − ≤
2k 4k + 1 2k + 1 2k + 2 k + 1 4k + 5 4k + 1 36
 
1 1 1 1 1 25 1
+ + − + − ≤ +
2k + 1 2k + 2 k + 1 4k + 5 4k + 1 36 (2k + 1)(2k + 2)
4 25
− ≤ .
(4k + 1)(4k + 5) 36

Then inequality (1) holds for n  k + 1.


Therefore, the given inequality holds for every positive integer n ≥ 2.
7.2. (a) We proceed by induction on n.
√ √
The given inequality holds for n  1, since a ≤ 1+ 24a+1 .

√ √
Assume that the inequality a + a + · · · + a ≤ 1+ 24a+1 holds. We have
  
 k  √ √

√  √ 1 + 4a + 1 1 + 4a + 1
a + a + · · · + a  a + a + · · · + a ≤ a +  ,
    2 2
  
k+1 k

and thus it follows that the given inequality holds for n  k + 1.


Therefore, the inequality holds
for every positive integer n.
 √
(b) We prove the inequality k (k + 1) . . . n < k + 1, where 1 ≤ k ≤ n
and k ∈ N , by induction
√ on p  n − k (n is assumed to be a constant).
For p  0, we have n < n + 1, whose proof is obvious.
Assume that the given inequality holds for p  n − m, and let us prove that
it holds for p  n − m + 1  n − (m − 1).
 √
For p  n −m, we have m (m + 1) . . . n < m +1, and hence we obtain

√ √
(m − 1) m (m + 1) . . . n < (m − 1)(m + 1) < m.
88 7 The Principle of Mathematical Induction

 √
Hence, the inequality k (k + 1) . . . n < k+1 holds for 1 ≤ k ≤ n, k ∈
N. 


For k  2, we have 2 3 4 . . . n < 3, which ends the proof.
7.3. We proceed by induction on n.
For n  1, we have x12 ≤ x12 .
Assume that the inequality x12 + · · · + (2k − 1)xk2 ≤ (x1 + · · · + xk )2 holds.
2
Adding (2k + 1)xk+1 to both sides of this inequality, we obtain the following
inequality:

x12 + · · · + (2k − 1)xk2 + (2k + 1)xk+1


2
≤ (x1 + · · · + xk )2 + (2k + 1)xk+1
2
.
(7.4)

Let us prove that

(x1 + · · · + xk )2 + (2k + 1)xk+1


2
≤ (x1 + · · · + xk+1 )2 , (7.5)

which is equivalent to (2k + 1)xk+1 2


≤ (x1 + · · · + xk+1 )2 − (x1 + · · · + xk )2 
(2(x1 + · · · + xk ) + xk+1 )xk+1 ,or to the following inequality:
0 ≤ xk+1 (2(x1 + · · · + xk ) − 2kxk+1 )  2xk+1 ((x1 − xk+1 ) + · · · + (xk − xk+1 )).

This inequality holds because according to the assumption of the problem,


we have 0 ≤ xk+1 ≤ xi , i  1, . . . , k.
Hence, from (7.4) and (7.5), it follows that x12 +· · ·+(2k−1)xk2 +(2k + 1)xk+1
2

(x1 + · · · + xk+1 ) , which is the given inequality for n  k + 1.
2

Therefore, the given inequality holds for every positive integer n.


7.4. (a) We proceed by induction on n.
For n  2, we have |sin(x1 + x2 )| ≤ |sin x1 | + |sin x2 |.
Indeed, since sin(x1 + x2 )  sin x1 cos x2 + sin x2 cos x1 and |cos α| ≤ 1,
it follows that |sin(x1 + x2 )| ≤ |sin x1 cos x2 | + |sin x2 cos x1 | ≤ |sin x1 | +
|sin x2 |.
Assume that the given inequality holds for n  k, and let us prove that it
holds for n  k + 1.
Indeed, we have
|sin((x1 + · · · + xk ) + xk+1 )| ≤ |sin(x1 + · · · + xk )| + |sin xk+1 | ≤ (|sin x1 | + · · · + |sin xk |) + |sin xk+1 |,

which is the given inequality for n  k + 1.


Therefore, the given inequality holds for every positive integer n.
(b) As for x ∈ [0, π ], we have that sin x ≥ 0, and then from part (a), it follows
that sin x1 + · · · + sin xn ≥ |sin(x1 + · · · + xn )| ≥ sin(x1 + · · · + xn ).
(c) We have
 π   π 
   
|cos x1 | + |cos x2 | + |cos x3 | + |cos x4 | + |cos x5 |  sin − x1  + · · · + sin − x5 ,
2 2
Proofs 89

and then from part (a) it follows that


 π   π    π  π   
5π 
     
sin − x1  + · · · + sin − x5  ≥ sin − x1 + · · · + − x5   sin  1.
2 2 2 2 2 

Therefore, |cos x1 | + |cos x2 | + |cos x3 | + |cos x4 | + |cos x5 | ≥ 1.


7.5. (a) Let us first prove the given inequality for n  1, 3, 5, 7, . . . , 2k + 1, . . ..
For n  1, we have f (x1 ) ≥ f (x1 ).
Assume that the given inequality holds for n  2k − 1 (k ∈ N), and let us
prove that it holds for n  2k + 1.
We have f (x1 ) − f (x2 ) + · · · + f (x2k−1 ) ≥ f (x1 − x2 + · · · + x2k−1 ), and
therefore,
               
f x1 − f x2 + · · · + f x2k−1 − f x2k + f x2k+1 ≥ f x1 − x2 + x3 − · · · + x2k−1 − f x2k + f x2k+1 .

Note that x1 ≥ x  x1 − x2 + x3 − · · · + x2k−1 ≥ y  x2k ≥ z  x2k+1 ≥ 0,


since
x1 − x  (x2 − x3 ) + · · · + (x2k−2 − x2k−1 ) ≥ 0 and x − y  (x1 − x2 ) + · · · + (x2k−1 − x2k ) ≥ 0.

Therefore, f (x) − f (y) + f (x) ≥ f (x − y + z).


Thus

f (x1 ) − f (x2 ) + · · · + f (x2k−1 ) − f (x2k ) + f (x2k+1 )


≥ f (x1 − x2 + · · · + x2k−1 ) − f (x2k ) + f (x2k+1 )
≥ f (x1 − x2 + x3 − · · · + x2k−1 − x2k + x2k+1 ),

from which it follows that the given inequality holds for n  2k + 1.


Hence, the given inequality holds for an arbitrary odd number n.
Now let us prove that the given inequality holds for n  2k. Let x1 ≥ x2 ≥
· · · ≥ x2k .
Let us take x2k+1  0, from which we obtain

f (x1 ) − f (x2 ) + · · · − f (x2k ) + f (0) ≥ f (x1 − x2 + · · · − x2k + 0), or


f (x1 ) − f (x2 ) + · · · − f (x2k ) ≥ f (x1 − x2 + · · · − x2k ), as f (0) ≤ 0.

(b) Let us prove that tan x − tan y + tan z ≥ tan(x − y + z), where π2 > x ≥
y ≥ z ≥ 0.
The last inequality can be rewritten as tan x − tan y ≥ tan(x − y + z) − tan z,
sin(x−y)
or cos x cos y
≥ cos zsin(x−y)
cos(x−y+z)
.
If x  y, then the proof is obvious.
If x  y, then the given inequality is equivalent to the following inequality:
cos z cos(x − y + z) ≥ cos x cos y, which holds because 0 ≤ z ≤ y < π2
and 0 ≤ x − y + z ≤ x < π2 .
Let us consider the function f (x)  tan x. Since f (0)  0 and the assumption
of Problem 7.5(a) holds, it follows that the given inequality holds.
90 7 The Principle of Mathematical Induction

(c) The proof follows from Problem 7.5(a) and Problem 9.34.
7.6. (a) Let us begin by proving the more general inequality

(x1 + · · · + xn )2 ≥ 4(x1 x2 + x2 x3 + · · · + xn−1 xn + xn x1 ), (7.6)

where n ≥ 4, x1 > 0, . . . , xn > 0.


We proceed by induction on n.
If n  4, then the inequality (x1 + x2 + x3 + x4 )2 ≥ 4(x1 x2 + x2 x3 + x3 x4 +
x4 x1 ) holds, since
(x1 + x2 + x3 + x4 )2 − 4(x1 x2 + x2 x3 + x3 x4 + x4 x1 )  (x1 − x2 + x3 − x4 )2 ≥ 0.

Assume that (7.6) holds for n  k, and let us prove that (7.6) holds for
n  k + 1.
Let max(x1 , . . . , xk+1 )  xi . It follows that
(x1 + · · · + (xi−2 + xi−1 ) + xi + · · · + xk+1 )2
≥ 4(x1 x2 + x2 x3 + · · · + xi−3 (xi−2 + xi−1 ) + (xi−2 + xi−1 )xi + xi xi+1
+ · · · + xk xk+1 + xk+1 x1 ) > 4(x1 x2 + x2 x3 + · · · + xi−3 xi−2 + xi−2 xi + xi−1 xi
+ xi xi+1 + · · · + xk xk+1 + xk+1 x1 ) ≥ 4(x1 x2 + x2 x3 + · · · + xi−3 xi−2 + xi−2 xi−1
+ xi−1 xi + xi xi+1 + · · · + xk xk+1 + xk+1 x1 )

(here we assume that x0  xk+1 , x−1  xk ).


(b) If n ≥ 4, then we have

x1 xn2 + x22 + x2 x12 + x32 + · · · + xn−1 xn−2
2 2
+ xn2 + xn xn−1 + x12
≤ x1 (xn + x2 ) + x2 (x1 + x3 ) + · · · + xn−1 (xn−2 + xn ) + xn (xn−1 + x1 )
1
 2(x1 x2 + x2 x3 + xn−1 xn + xn x1 ) ≤ (x1 + · · · + xn )2 .
2
The last inequality holds by to Problem 7.6(a).
If n  3, then without loss of generality one can assume that max(x1 , x2 , x3 ) 
x3 .
Thus, it follows that
 x2   x1 
x1 x32 + x22 + x2 x12 + x32 + x3 x12 + x22 ≤ x1 x3 + + x2 x3 +
2 2
x32 + x12 + x22 1
+  (x1 + x2 + x3 ) .
2
2 2
7.7. We proceed by induction on n.
If n  1, then 21 x12 ≤ x12 . Obviously, the last inequality holds.
Assume that the given inequality holds for n  k, and let us prove that it
holds for n  k + 1.
If n  k, we have 21 (x1 + · · · + xk )2 ≤ (x1 + · · · + kxk )max(x1 , . . . , xk ).
Adding to both sides of this inequality the expression
Proofs 91

x2
xk+1 (x1 + · · · + xk ) + k+1
2
, we obtain on the left-hand side the expres-
sion 2 (x1 + · · · + xk+1 ) , and on the right-hand side the expression
1 2
2
xk+1
(x1 + · · · + kxk )max(x1 , . . . , xk ) + xk+1 (x1 + · · · + xk ) + 2
, which is
not greater than (x1 + · · · + (k + 1)xk+1 )max(x1 , . . . , xk+1 ).
2
xk+1
Indeed, since max(x1 , . . . , xk ) ≤ max(x1 , . . . , xk+1 )  c, 2
≤ xk+1
2

cxk+1 , we have
2
xk+1
(x1 + · · · + kxk )max(x1 , . . . , xk ) + xk+1 (x1 + · · · + xk ) + ≤ (x1 + · · · + kxk )c
2
+ kcxk+1 + cxk+1  (x1 + · · · + (k + 1)xk+1 )c.

Therefore, the given inequality holds for every positive integer n.


7.8. Let us first prove that for arbitrary numbers x1 , . . . , xn belonging to the domain
I, we have
f (x1 ) + · · · + f (xn ) x + ··· + x 
1 n
≥ f . (7.7)
n n
We proceed by induction on n.
If n  2, then inequality (7.7) coincides with the condition of Theorem 11.1.
Assume that for n  k, inequality (7.7) holds, and let us prove that it holds
for n  k + 1.
Let us consider the following expression:
x + ··· + x 
1 k+1
f (x1 ) + · · · + f (xk ) + f (xk+1 ) + (k − 1) f  A.
k+1
Note that
x + ··· + x  x + ··· + x 
1 k+1 1 k+1
A  ( f (x1 ) + · · · + f (xk )) + f (xk+1 ) + f + ··· + f
k+1 k+1
  
k−1
 
x + ··· + x  xk+1 + (k − 1) x1 +···+x k+1
1 k k+1
≥ kf + kf
k k
⎛ x1 +···+xk+1 ⎞
x1 +···+xk x +(k−1)
+ k+1  
⎠  2k f x1 + · · · + xk+1 .
k+1
≥ 2k f ⎝ k k
2 k+1

 x1 +···+xk+1 
Hence, we deduce that f (x1 ) + · · · + f (xk+1 ) + (k − 1) f ≥
  k+1
2k f x1 +···+x
k+1
k+1
,
and therefore,
f (x1 ) + · · · + f (xk+1 ) x + ··· + x 
1 k+1
≥ f .
k+1 k+1
Thus, it follows that inequality (7.7) holds.
Now let us prove the inequality (11.3). Let αi  pi
qi
, i  1, . . . , n, where
pi , qi ∈ N, i  1, . . . , n.
92 7 The Principle of Mathematical Induction

Let us denote the least common multiple of the numbers q1 , . . . , qn by q


Therefore, qpii  lqi , where li ∈ N, i  1, . . . , n.
Let us estimate the left-hand side of the given inequality, α1 f (x1 ) + · · · +
αn f (xn )  l1 f (x1 )+···+l
q
n f (x n )
 B.
Applying inequality (7.7) to the numbers x1 , . . . , x1 , . . . , xn , . . . , xn , note
     
l1 ln
 total numberis equal to q and since α1 + · · · + αn  1, we obtain
that their
B ≥ f l1 x1 +l2 x2q+···+ln xn  f (α1 x1 + · · · + αn xn ).
7.9. We proceed by induction on n.
If n  2, then inequality (11.7) coincides with the condition (11.5) of Theorem
11.2.
Assuming that (11.7) holds for n  k, let us prove that it holds for n  k + 1,
that is, that β1 f (a1 ) + · · · + βk+1 f (ak+1 ) ≥ f (β1 α1 + · · · + βk+1 αk+1 ), where

β1 ≥ 0, . . . , βk+1 ≥ 0 and β1 + · · · + βk+1  1, a1 , . . . , an ∈ I.

We have
β1 f (a1 ) + · · · + βk+1 f (ak+1 )
 
β1 βk
 (β1 + · · · + βk ) f (a1 ) + . . . + f (ak ) + βk+1 f (ak+1 )
β1 + · · · + βk β1 + · · · + βk
 
β1 a1 + · · · + βk ak
≥ (β1 + · · · + βk ) f + βk+1 f (ak+1 )
β1 + · · · + βk
 
β1 a1 + · · · + βk ak
≥ f (β1 + · · · + βk ) + βk+1 ak+1  f (β1 a1 + · · · + βk ak + βk+1 ak+1 ),
β1 + · · · + βk

therefore β1 f (a1 ) + · · · + βk+1 f (ak+1 ) ≥ f (β1 α1 + · · · + βk+1 αk+1 ).


βi
We have used (11.7) for n  k, substituting xi by ai , and αi by β1 +···+β k
, i
1, . . . , k.
Hence inequality (11.7) holds for every positive integer n.
7.10. (a) We proceed by induction on n.
For n  1, the given inequality holds because a1 ≤ a1 .
Assume that the given inequality holds for n  k, and let us prove that it holds
for n  k + 1.
For n  k, we have a1 + · · · + akk ≤ ka1 · · · ak . On the other hand, by to the
assumption of the problem, we have 1 ≤ ak+1 . Multiplying these inequalities,
we obtain a1 + · · · + akk ≤ ka1 · · · ak ak+1 . By to the assumption of the problem,
we have ak+1 k+1
≤ a1 · · · ak+1 . Summing these inequalities, we deduce that

a1 + · · · + akk + ak+1
k+1
≤ ka1 · · · ak+1 + a1 · · · ak+1  (k + 1)a1 · · · ak+1 .

Therefore, the given inequality holds for every positive integer n.


(b) We proceed by induction on n.
For n  1, the given inequality holds because a1 ≥ a1 .
Proofs 93

Assume that the given inequality holds for n  k, and let us prove that it
holds for n  k + 1.
For n  k, we have a1 +· · ·+akk ≥ ka1 · · · ak . On the other hand, by assumption,
we have 1 ≥ ak+1 . Multiplying these two inequalities, we deduce that a1 +
· · · + akk ≥ ka1 · · · ak ak+1 .
By to the assumption of the problem, we have ak+1 k+1
≥ a1 · · · ak+1 .
Summing these inequalities, we obtain a1 + · · · +akk +ak+1 k+1
≥ (k + 1)a1 · · · ak+1 .
Therefore, the given inequality holds for every positive integer n.
7.11. (a) We proceed by induction on n.
a2  
For n  2, we have a21 ≥ 4(a2 − a1 ), a22 − 4a1 a2 + 4a12  (a2 − 2a1 )2 ≥ 0 .
Assume that the given inequality holds for n  k, and let us prove that it
holds for n  k + 1. 2
a ak2
For n  k, we have a21 + · · · + ak−1 ≥ 4(ak − a1 ).
Let us add to both sides of this inequality the expression 4(ak+1 − ak ). Then
a2 ak2
we obtain a21 + · · · + ak−1 + 4(ak+1 − ak ) ≥ 4(ak+1 − a1 ).
It is left to prove that

a22 a2 a2 a2 a2
+ · · · + k + k+1 ≥ 2 + · · · + k + 4(ak+1 − ak ).
a1 ak−1 ak a1 ak−1

Indeed, this inequality is equivalent to the following inequality:


2
ak+1
≥ 4(ak+1 − ak ), or (ak+1 − 2ak )2 ≥ 0.
ak

Therefore, the given inequality holds for every positive integer n.


(b) We proceed by induction on n.
For n  2, we have
 
a13 a23
+ (b1 + b2 )(c1 + c2 )  a13 + a23
b1 c1 b2 c2
   
c2 b2 b1 c1 c1 b1 b2 c2
+ a13 + a13 + a23 + a23 + a23 + a13
c1 b1 b2 c2 c2 b2 b1 c1
 
c2 b2 3 b1 c1 c1 b1 3 b2 c2
≥ a13 + a23 + 3 3 a13 · a13 ·a + 3 3 a23 · a23 ·a  (a1 + a2 )3 ,
c1 b1 2 b2 c2 c2 b2 1 b1 c1

and therefore,

a13 a3 (a1 + a2 )3
+ 2 ≥ .
b1 c1 b2 c2 (b1 + b2 )(c1 + c2 )

Assume that the given inequality holds for n  k, and let us prove that it
holds for n  k + 1. 3
a a3 (a1 +···+ak )3
For n  k, we have b1 1c1 + · · · + bk kck ≥ (b1 +···+b k )(c1 +···+ck )
.
94 7 The Principle of Mathematical Induction

3
ak+1
Let us add to both sides of this inequality the expression bk+1 ck+1
. Then we
obtain

a13 a3 3
ak+1 (a1 + · · · + ak )3 3
ak+1
+ ··· + k + ≥ +
b1 c1 bk ck bk+1 ck+1 (b1 + · · · + bk )(c1 + · · · + ck ) bk+1 ck+1
((a1 + · · · + ak ) + ak+1 )3
≥ ,
((b1 + · · · + bk ) + bk+1 )((c1 + · · · + ck ) + ck+1 )

and hence the given inequality holds for n  k + 1.


Therefore, the given inequality holds for every positive integer n.
7.12. (a) We proceed by induction on n.
For n  2, we have

(x2 + x1 )( f (x2 ) − f (x1 )) ≥ (x2 + x1 )( f (x2 ) − f (x1 )).

For n  k, we have
                 
x2 + x1 f x2 − f x1 + · · · + xk + xk−1 f xk − f xk−1 ≥ xk + x1 f xk − f x1 .

Let us add to both sides of this inequality the expression


(xk+1 + xk )( f (xk+1 ) − f (xk )), from which we obtain

(x2 + x1 )( f (x2 ) − f (x1 )) + · · · + (xk + xk−1 )( f (xk ) − f (xk−1 ))


+ (xk+1 + xk )( f (xk+1 ) − f (xk )) ≥ (xk + x1 )( f (xk ) − f (x1 ))
+ (xk+1 + xk )( f (xk+1 ) − f (xk )).

In order to prove that the given inequality holds for n  k + 1, it is sufficient


to prove
the following inequality:
(xk + x1 )( f (xk ) − f (x1 )) + (xk+1 + xk )( f (xk+1 ) − f (xk )) ≥ (xk+1 + x1 )( f (xk+1 ) − f (x1 )), or
(xk+1 − xk ) f (x1 ) + (xk − x1 ) f (xk+1 ) ≥ (xk+1 − x1 ) f (xk ). (7.8)

Since f (x) is a convex function, it follows that


 
xk+1 − xk xk − x1 xk+1 − xk xk − x1
f (x1 ) + f (xk+1 ) ≥ f x1 + xk+1  f (xk ),
xk+1 − x1 xk+1 − x1 xk+1 − x1 xk+1 − x1

which implies inequality (7.8).


This ends the proof of the given inequality for n  k + 1.
Therefore, the given inequality holds for every positive integer n (n > 1).
(b) Note that one can rewrite inequality (a) of this problem in the following
way:
           
xn f x1 + x1 f x2 + x2 f x3 + · · · + xn−1 f (xn ) ≥ x2 f x1 + x3 f x2 + · · · + xn f xn−1 + x1 f (xn ),


where on taking f (x)  − x, we obtain
Proofs 95

 √   √   √   √   √   √ 
a − c +c − b +b − a ≥b − c +a − b +c − a .

√ √ √ √ √ √
Thus, it follows that a c + c b + b a ≤ b c + a b + c a.
(c) Consider the function f (x)  − ln x. By to inequality (a), we have
       
xn − ln x1 + x1 − ln x2 + · · · + xn−1 (− ln xn ) ≥ x2 − ln x1 + · · · + xn − ln xn−1 + x1 (− ln xn ), or
xn x1 xn−1 x2 x3 xn x1
x1 · x2 · · · xn ≤ x1 · x2 · · · xn−1 · xn .

(d) Consider the function


 
x 1 1 1
f (x)   − .
(a + b + c − x)(a + b + c + x) 2 a+b+c−x a+b+c+x
 
We have f (x)  1
2
1
(a+b+c−x)2
1
+ (a+b+c+x)2 .


If 0 ≤ x ≤ a, then f (x)  (a+b+c−x)3 − (a+b+c+x)
1 1
3 ≥ 0, and thus it follows
that a f (c) + c f (b) + b f (a) ≥ b f (c) + a f (b) + c f (a), or
c b a
a· +c· +b·
(a + b)(a + b + 2c) (a + c)(a + c + 2b) (b + c)(b + c + 2a)
c b a
≥b· +a· +c· .
(a + b)(a + b + 2c) (a + c)(a + c + 2b) (b + c)(b + c + 2a)

7.13. Note that the given inequality is equivalent to the following inequality:
(a1 + · · · + an−1 )n(n + 1) + (a1 + · · · + an−1 )n(n − 1) + (an + an+1 )n(n − 1)
   
≥ 2 n 2 − 1 (a1 + · · · + an−1 ) + 2 n 2 − 1 an ,

or
 
2(a1 + · · · + an−1 ) + n(n − 1)an+1 ≥ n 2 + n − 2 an . (7.9)

We prove this inequality by induction.


For n  2, we obtain the following obvious inequality: 2a1 + 2a3 ≥ 4a2 .
Assume that the given inequality holds for n  k, that is,
 
2(a1 + · · · + ak−1 ) + k(k − 1)ak+1 ≥ k 2 + k − 2 ak , (7.10)

and let us prove that it holds for n  k + 1.


Let us add to both sides of inequality (7.10) the expression 2ak +(k + 1)kak+2 −
k(k − 1)ak+1 , from which we deduce that
     
2 a1 + · · · + ak−1 + ak + (k + 1)kak+2 ≥ k 2 + k ak + k 2 + k ak+2 − k(k − 1)ak+1
      
 k 2 + k ak + ak+2 − k(k − 1)ak+1 ≥ k 2 + k · 2ak+1 − k(k − 1)ak+1  k 2 + 3k ak+1 .

 
Hence, we obtain 2(a1 + · · · + ak ) + (k + 1)kak+2 ≥ k 2 + 3k ak+1 , and thus it
follows that the given inequality holds for n  k + 1.
Therefore, inequality (7.9) holds for every positive integer n.
96 7 The Principle of Mathematical Induction

7.14. (a) Let us rewrite the given inequality as a1 + a3 + · · · + a2n−1 ≥


n+1 (a0
n
+ a2 + · · · + a2n ) and prove it by induction.
For n  1, we obtain the following obvious inequality: a1 ≥ a0 +a
2
2
.
Assume that the given inequality holds for n  k, and let us prove that it
holds for n  k + 1, that is,
k
a1 + a3 + · · · + a2k−1 ≥ (a0 + a2 + · · · + a2k ), or
k+1
k
a1 + a2 + · · · + a2k−1 + a2k+1 ≥ (a0 + a2 + · · · + a2k ) + a2k+1 .
k+1
Let us prove that
k+1 (a0
k
+ a2 + · · · + a2k ) + a2k+1 ≥ k+2 k+1
(a0 + a2 + · · · + a2k + a2k+2 ),
or a2k+1 ≥ (k+1)(k+2) (a0 + a2 + · · · + a2k ) + k+2
1 k+1
a2k+2 .
Since a2k+1 ≥ a2k +a2k+2
2
, it follows that in order to complete the proof of the last
inequality, it is sufficient to prove that a2k +a2k+2
2
≥ (k+1)(k+2)
1
(a0 + · · · + a2k ) +
k+1
a
k+2 2k+2
, or
k(k + 3) k(k + 1)
a2k − a2k+2 ≥ a0 + a2 + · · · + a2k−2 .
2 2

We prove this inequality by induction.


For k  1, we obtainthe obvious inequality 2a2 − a4 ≥ a0 , since a2 ≥
a1 +a3
2
≥ 21 a0 +a
2
2
+ a2 +a
2
4
.
Let k  m, then the following inequality holds:
m(m + 3) m(m + 1)
a2m − a2m+2 ≥ a0 + · · · + a2m−2 .
2 2
Therefore,
m(m + 3) m(m + 1)
a2m − a2m+2 + a2m ≥ a0 + a2 + · · · + a2m−2 + a2m .
2 2
It is sufficient to prove that
m(m + 3) m(m + 1) (m + 1)(m + 4) (m + 1)(m + 2)
a2m − a2m+2 + a2m ≤ a2m+2 − a2m+4 , or
2 2 2 2
(m + 1)(m + 2) (m + 1)(m + 2)
(m + 1)(m + 2)a2m+2 ≥ a2m + a2m+4 .
2 2
Note that this inequality holds.
(b) Let ak  −a k . As −a k ≥ −a 2−a . Then by to inequality (a) of this
k−1 k+1

problem, we have −a−a −···−a ≥ −1−an+1


−···−a 2n
3 2n−1 2
1+a 2 +···+a 2n
n
, and therefore a+a 3 +···+a 2n−1 ≥
n+1
n
.
7.15. We proceed by induction on n.
For n  1, we have 1 > ln 2.
Assume that the given inequality holds for n  k, that is, 1 + 21 + · · · + k1 >
ln(k + 1).
Proofs 97

1
Let us add to both sides of this inequality the expression k+1 . Then 1 + 21 +
· · · + k1 + k+1
1
> ln(k + 1) + k+1
1
.
Now let us prove that ln(k + 1) + k+11
> ln(k + 2), which is equivalent to
 
1 k+2 k + 2 k+1
> ln(k + 2) − ln(k + 1)  ln , or 1 > ln .
k+1 k+1 k+1
 n  n
Since lim 1 + n1  e and the sequence 1 + n1 is monotonically increas-
n→∞
ing (Problem 3.16(a)), it follows that
 k+1  k+1
1 k+2
e > 1+ , or 1 > ln .
k+1 k+1

7.16. We proceed by induction on n.


For n  1, we have 1 ≤ 1.
Assume that the given inequality holds for n  k, that is, 1+ 2√1 2 +· · ·+ k √1 k ≤
3− √2 , and let us add to
k
both sides of the last inequality the expression
1√
(k+1) k+1
. Then we obtain
1 1 1 2 1
1 + √ + ··· + √ + √ ≤3− √ + √ .
2 2 k k (k + 1) k + 1 k (k + 1) k + 1

In order to complete the proof, it is sufficient to prove that 3− √2k + (k+1)1√k+1 ≤


3− √2 .
k+1
Indeed, since 1
k+1
≤ √ 2
k(k+1)+k
, we have
√ √ 
1 2 2 k+1− k 2 2
√ ≤ √ √ √   √  √ − √ .
(k + 1) k + 1 k(k + 1) k + 1 + k k(k + 1) k k+1

Therefore, the given inequality holds for every positive integer n.


7.17. We proceed by induction on n.
For n  1, we have 1 + α ≥ 1 + α.
Assume that the given inequality holds for n  k, and let us prove that it
holds for n  k + 1.
For n  k, we have (1 + α)k ≥ 1 + kα + k(k−1)
2
α2 .
Hence, we deduce that
 
k(k − 1) 2 k(k − 1) 2
(1 + α)k+1 ≥ (1 + α) 1 + kα + α  1 + kα + α + α + kα 2
2 2
k(k − 1) 3 k(k + 1) 2 k(k − 1) 3 k(k + 1) 2
+ α  1 + (k + 1)α + α + α ≥ 1 + (k + 1)α + α .
2 2 2 2

It follows that (1 + α)k+1 ≥ 1 + (k + 1)α + k(k+1)


2
α 2 , and therefore, the given
inequality holds for every positive integer n.
7.18. We proceed by induction on k.
For k  1, we have 1 > 2e .
98 7 The Principle of Mathematical Induction
 m+1 m
Assume that the given inequality holds for k  m, that is, m! > , and
 m+2
e
m+1
let us prove that it holds true for k  m + 1, that is, (m + 1)! > e .
 m
We have m! > m+1 e
, and therefore
 m+1
m+1
(m + 1)! > e . (7.11)
e
 n  n
Since lim 1 + n1  e and the sequence 1 + n1 is monotonically increas-
n→∞
ing (Problem 3.16(a)), it follows that
 m+1
1
e > 1+ . (7.12)
m+1
 m+1
Multiplying inequalities (7.11) and (7.12), we obtain (m + 1)! > m+2 e
.
Therefore, the given inequality holds for every positive integer k.
7.19. Let us first prove that

3 3
2|sin x| + |sin 2x| ≤ . (7.13)
2
  
4
We have 2|sin x|+|sin 2x| ≤ 3 (3 − 3|cos x|)(1 + |cos x|) ≤ 3 46 
√2 3 √ 2

3 3
2
.
We proceed by induction on n.
For n  0, we obtain the following obvious inequality: |sin x| ≤ 1.
Assume that the given√ inequality holds for n ≤ k, that is, |sin x| + |sin 2x| +
· · · + |sin 2n x| ≤ 1 + 23 n, for all x Let us prove that it holds for n  k + 1.
Let us consider √ the following two cases.
   √
(a) If |sin x| ≤ 23 , then |sin x| + |sin 2x| + · · · + sin(2k · 2x) ≤ 23 + 1 +
√ √
3
k  1 + 23 (k + 1).
2 √ √
(b) If |sin x| > 23 , then from (7.13), it follows that |sin x| + |sin 2x| < 3,
whence
 √  √
   √
  3 3
|sin x| + |sin 2x| + |sin 4x| + · · · + sin(2k−1 · 4x) < 3 + 1 + (k − 1)  1 + (k + 1).
2 2

Therefore, the given inequality holds for every positive integer n.
7.20. Consider the function f n (α)  cos α + cos22α +· · ·+ cosnnα in 0, π2 and suppose
it attains its minimum value in this interval at the point αn .
Let us consider the following three cases.
(a) If αn  0, then f n (α) ≥ 1 + 21 + · · · + n1 > − 21 .
       
(b) If αn  π2 , then f n (α) ≥ f n π2 ≥ − 21 , as f 1 π2  0, f 2 π2  f 3 π2 
         
− 21 , f 4 π2  f 5 π2  − 21 + 41 , f 6 π2  f 7 π2  − 21 + 41 − 16 , and so on.
Proofs 99


(c) If 0 < αn < π2 , then f n (αn )  0, that is, − sin αn −sin 2αn −· · ·−sin nαn 
0.
We obtain that 2 sin α2n sin αn + · · · + 2 sin α2n sin nαn  0, whence cos α2n 
 
cos nαn + α2n .
 
Thus, sin α2n  ± sin nαn + α2n , and it follows that
 αn  αn  αn  αn αn αn
cos nαn  cos nαn + cos + sin nαn + sin  cos2 ± sin2 > 0.
2 2 2 2 2 2

We deduce that f n (α) ≥ f n−1 (αn )  cos αn + · · · + cos(n−1)α n


≥ − 21 , since
 π n−1
αn ∈ 0, 2 , and for (n − 1), the statement holds (for n  1, we have
f 1 (α)  cos α ≥ 0 > − 21 ).
7.21. We proceed by induction on n.
For n  2, we have aa21 + aa21  2 + (a1a−a
2
2)
1 a2
< 2 + a11a2 < 3.
Let n ≥ 3. Consider the following two cases.
(a) There exists i ∈ {2, . . . , n − 1} such that

(ai − ai−1 )(ai − ai+1 ) ≥ 0. (7.14)

Then aai−1
i
+ aai+1i ≤ aai−1
i+1
+ 2, since (ai − ai−1 )(ai − ai+1 ) ≤
|ai − ai−1 ||ai − ai+1 | < 1 < ai ai+1 .
Taking this into consideration and the induction hypothesis, we obtain
a1 a2 ai−1 ai an−1 an a1 ai−2 ai−1 ai+1
+ + ··· + + + ··· + + ≤ + ··· + + +
a2 a3 ai ai+1 an a1 a2 ai−1 ai+1 ai+2
an
+ ··· + + 2 < 2(n − 1) − 1 + 2  2n − 1,
a1

since from (7.14) it follows that |ai−1 − ai+1 | ≤


max(|ai − ai−1 |, |ai − ai+1 |) < 1.
(b) If (ai − ai−1 )(ai − ai+1 ) < 0, i  2, . . . , n − 1, then either a1 < a2 <
· · · < an , or a1 > a2 > · · · > an . In the first case, we have aa21 + aa23 + · · · +
an−1
an
+ aan1 ≤ 1 + ·
· · + 1 + aan1 < 2n − 1,
n−1
since an  (an − an−1 ) + (an−1 − an−2 ) + · · · + (a2 − a1 ) + a1 < na1 .
In the second case, we have aa21 + aa23 + · · · + aan−1
n
+ aan1 ≤ 2 + ·
· · + 2 +1  2n − 1,
n−1
since ai  ai+1 + |ai+1 − ai | < ai+1 + 1 < 2ai+1 , i  1, . . . , n − 1.
7.22. (a) We proceed by induction on n.√
For n  1, we need to prove that xx22−1 ≤ 21 , or (x2 − 2)2 ≥ 0.
Assume that the inequality holds for n  k, and let us prove that it holds for
n  k + 1, where√k ∈ N. We √
need to prove

that if 1 √ x1 ≤ x2 ≤ · · · ≤
x2 −x1 x3 −x2 xk+2 −xk+1
xk+1 ≤ xk+2 , then x2 + x3 + · · · + xk+2
≤ 4k+1
2
.

x3 xk+2 xk+1
x2 −1 x2 − x2
For the k +1 numbers 1 ≤ x3
x2
≤ ··· ≤ xk+2
x2
, we have x3 +· · ·+ xk+1 ≤
x2
√ x2
4k−3
2
, and therefore,
100 7 The Principle of Mathematical Induction
⎛ ⎞
 xk+2 xk+1 x3
√ √ √ x2 − x2 ⎟ x2 − 1
x2 − x1 x3 − x2 xk+2 − xk+1 x2 − x1 1 ⎜
+ + ··· + ≤ + √ ⎜ ⎟ + ... +
x2 x3 xk+2 x2 x2 ⎝ xk+2 ⎠ x3
x2 x2
 ⎛ ⎞
√ √  √
   2  √ 2
x2 − x1 1 4k − 3 x2 − x1 2 1 4k − 3 ⎠
≤ ·1+ √ ≤ + √ ⎝12 +
x2 x2 2 x2 x2 2
 √
2x2 − 1 4k + 1 4k + 1
 · ≤
x22 4 2

(see Chapter 4, inequality (4.1)). Hence, the given inequality holds for n 
k + 1.
Therefore the given inequality holds for every positive integer n.
(b) We proceed by induction

on n.
x2 −x1
For n  1, we have x2 ≤ 21 < 1.
Assume that the given inequality holds for n  k, and let us prove that it holds
for n  k + 1, where k ∈ N.
Let 1  x1 ≤ x2 ≤ · · · ≤ xk+1 ≤ xk+2 be positive integers. Then we have
√ √ 
x2 − x1 x3 − x2 xk+2 − xk+1 x − x1 x − x2 x − xk+1
+ + ··· + ≤ 2 + 3 + · · · + k+2
x2 x3 xk+2 x2 x3 xk+2
     
1 1 1 1 1 1
≤ + ··· + + + ··· + + ... + + ··· +
x1 + 1 x2 x2 + 1 x3 xk+1 + 1 xk+2
1 1 1
≤ + + ··· + .
2 3 xk+2

If xk+2 ≤ (k + 1)2 , then 21 + 13 + · · · + xk+2


1
≤ 21 + · · · + (k+1)
1
2 < 1+ 2 +· · ·+
1 1
(k+1)2
,
and therefore, the given inequality holds for n  k + 1.
If xk+2 > (k + 1)2 , then
    
 2
xk+2 − xk+1 xk+2 − 1 1 1 1 1 k 2 + 2k
≤  − < −  .
xk+2 xk+2 xk+2 xk+2 (k + 1)2 (k + 1)4 (k + 1)2

Hence, we obtain
√  
x2 − x1 xk+1 − xk xk+2 − xk+1 1 1
+ ··· + + ≤ 1 + + ··· +
x2 xk+1 xk+2 2 k2
 
xk+2 − xk+1 1 1 k 2 + 2k 1 1 2k + 1
+ ≤ 1 + + ··· + + < 1 + + ··· + +
xk+2 2 k 2 (k + 1)2 2 k 2 (k + 1)2
1 1 1 1 1 1 1
1+ + ··· + + + ··· + < 1 + + + ··· + .
2 k 2 (k + 1)2 (k + 1)2 2 3 (k + 1)2
  
2k+1

Hence, the given inequality holds for n  k + 1.


Therefore, the given inequality holds for every positive integer n.
Proofs 101

Remark If 1  x1 ≤ x2 ≤ · · · ≤ xn ≤ xn+1 are positive integers, then


⎛ 2 ⎞
√ √ n
x2 − x1 xn+1 − xn ⎝ 1⎠ 1
+ ··· + ≤ − .
x2 xn+1 i1
i 2

7.23. We proceed by induction on n.


β1 α1
For n  1, we have 0 < α1 ≤ β1 ≤ π, and therefore cos sin α1
≤ cos
sin α1
.
For n  2, we have α1 > 0, α2 > 0, β1 ≥ 0, β2 ≥ 0 and α1 + α2 ≤
β1 β2 α1 α2
β1 + β2 ≤ π , and we need to prove that cossin α1
+ cos
sin α2
≤ cos
sin α1
+ cos
sin α2
.
Let α1 and α2 be constants. Consider the expression sin α1 + sin α2 , where
cos x1 cos x2

α1 + α2 ≤ x1 + x2 ≤ π, x1 ≥ 0, x2 ≥ 0. Assume that this expression attains


its maximum value at x1  β1 , x2  β2 .
Let α1 ≤ α2 . Then one can assume that β1 ≤ β2 . Otherwise, we have cos β1 <
cos β2 , and therefore (cos β1 − cos β2 )(sin α2 − sin α1 ) ≤ 0, or
cos β1 cos β2 cos β2 cos β1
+ ≤ + .
sin α1 sin α2 sin α1 sin α2

If β1  0, then
cos β1 cos β2 cos α1 cos α2 1 cos(α1 + α2 ) cos α1 cos α2
+ − − ≤ + − −
sin α1 sin α2 sin α1 sin α2 sin α1 sin α2 sin α1 sin α2
α1 α1 α1
α1 2 sin 2 sin( 2 + α2 ) sin 2 sin(α1 + α2 )
2
 tan − − ≤ 0.
2 sin α2 cos α21 sin α2

If 0 < β1 ≤ β2 , then on decreasing the value of β1 , the value of the expression


cos β1 β2
sin α1
+ cos
sin α2
increases, and therefore β1 + β2  α1 + α2 . Let us prove that
sin β1 sin β2
sin α1
 sin α2 .
2 −x)
Note that the function f (x)  cos(β 1 +x)
sin α1
+ cos(β
sin α2
in [−β1 , β2 ] attains its
maximum value at the point x  0, and hence by Fermat’s theorem on
sin β1 sin β2
stationary points, f (0)  − sin α1
+ sin α2
 0.
Lemma. If αi > 0, βi > 0, i  1, . . . , n, α1 +· · ·+αn  β1 +· · ·+βn ≤
π , and
sin β1 sin βn
 ···   λ, (7.15)
sin α1 sin αn

then λ  1.
Proof . Let λ  1. Then without loss of generality one can assume that λ < 1.
Let α1 ≤ · · · ≤ αn . Then from (7.15) it follows that β1 ≤ · · · ≤ βn and
β1 < α1 , . . . , βn−1 < αn−1 ≤ π2 .
Note that

sin(β1 + β2 )  λ(sin α1 cos β2 + sin α2 cos β1 ) > λ(sin α1 cos α2


+ sin α2 cos α1 )  λ sin(α1 + α2 ).
102 7 The Principle of Mathematical Induction

In a similar way, we obtain

sin((β1 + β2 ) + β3 )  λ(sin(β1 + β2 ) cos β3 + sin β3 cos(β1 + β2 ))


> λ(sin(α1 + α2 ) cos β3 + sin α3 cos(β1 + β2 ))
> λ(sin(α1 + α2 ) cos α3 + sin α3 cos(α1 + α2 ))
 λ sin(α1 + α2 + α3 ),

and so on.
In general, we have sin(β1 + · · · + βn−1 ) > λ sin(α1 + · · · + αn−1 ).
sin βn
Therefore, sin(ϕ − βn ) > sin αn
sin(ϕ − αn ), where ϕ  α1 + · · · + αn 
β1 + · · · + βn .
Hence, sin αn sin(ϕ − βn ) > sin βn sin(ϕ − αn ), or

sin αn sin ϕ cos βn − sin αn sin βn cos ϕ >


sin βn sin ϕ cos αn − sin βn sin αn cos ϕ, sin ϕ sin(αn − βn ) > 0.

Since 0 < ϕ ≤ π, we must have ϕ  π and sin ϕ > 0, whence αn − βn > 0.


It follows that αn > βn .
We obtain β1 < α1 , . . . , βn−1 < αn−1 , βn < αn , and therefore β1 +· · ·+βn <
α1 + · · · + αn , which leads to a contradiction. Thus λ  1. This ends the proof
of the lemma.
Since sin α1  sin β1 and sin α2  sin β2 , it follows that α1  β1 and
α2  β2 , or α1  β1 and π − α2  β2 (α1 + α2  α1 + π − α2 , α2  π2 ,
whence α2  β2 ).
β1 β2 α1 α2
Therefore, cossin α1
+ cos
sin α2
 cos
sin α1
+ cos
sin α2
.
Let n ≥ 3 and suppose that the given inequality holds for n − 1. Let us prove
that it holds for n.
Let 0 < α1 ≤ · · · ≤ αn be constants. Consider the expression cos x1
sin α1
+· · ·+ cos xn
sin αn
,
where α1 + · · · + αn ≤ x1 + · · · + xn ≤ π, x1 ≥ 0, . . . , xn ≥ 0, and suppose
that this expression attains its maximum value at x1  β1 , . . . , xn  βn . Then
β1 ≤ β2 ≤ · · · ≤ βn (see the case n  2).
Without loss of generality one can assume that β1 > 0, for otherwise, for
n − 1 we would have
cos β2 cos β3 cos βn cos(α1 + α2 ) cos α3 cos αn
+ + ··· + ≤ + + ··· + . (7.16)
sin(α1 + α2 ) sin α3 sin αn sin(α1 + α2 ) sin α3 sin αn

Note that
1 cos β2 cos α1 cos α2 cos β2 cos(α1 + α2 )
+ − − ≤ − .
sin α1 sin α2 sin α1 sin α2 sin(α1 + α2 ) sin(α1 + α2 )
(7.17)

Indeed, since
Proofs 103
 
1 1 1 1
cos β2 − ≤ −
sin α2 sin(α1 + α2 ) sin α1 sin(α1 + α2 )

(α2 + (α1 + α2 ) ≤ α1 + α2 + α3 ≤ π, sin(α1 + α2 ) ≥ sin α2 ), it is sufficient


α1 α2
to prove that 1−cos
sin α1
+ 1−cos
sin α2
− 1−cos(α 1 +α2 )
sin(α1 +α2 )
≤ 0, or
α1 α2  α1 α2 
tan + tan − tan + ≤ 0,
 2 α1 2 
α2 α1
2 2
α2
− tan 2 + tan 2 tan 2 tan 2
≤ 0.
1 − tan α21 tan α22

The last inequality holds because 0 < α21 ≤ α22 < π4 . Summing (7.16) and
(7.17), we obtain the inequality for n. Then we have 0 < β1 ≤ β2 ≤ · · · ≤ βn ,
β1
and if the value of β1 decreases, then the value of the expression cos sin α1
+
cos βn
· · · + sin αn increases, and therefore α1 + · · · + αn  β1 + · · · + βn . Then
sin β1 sin βn
sin α1
 · · ·  sin αn
(see the case n  2), and therefore by the lemma, we
have α1  β1 , . . . , αn  βn .
7.24. We first prove by induction that if a > 0 and n ∈ N, then

2(1 + a n+1 )3 ≥ (1 + a 3 )(1 + a n )3 . (1)

For n  1, we need to prove that 2(1 + a 2 )3 ≥ (1 + a 3 )(1 + a)3 , which is


equivalent to the following obvious inequality: (a − 1)4 (a 2 + a + 1) ≥ 0.
Assume that the inequality holds for n  k, where k ∈ N, and let us prove
that it holds for n  k + 1.
We have 2(1 + a k+1 )3 ≥ (1 + a 3 )(1 + a k )3 , as (1 + a k+2 )(1 + a k ) ≥ (1 + a k+1 )2 ,
whence
 3  3
1 + a k+2 1 + a k+1
2(1 + a k+2 )3  2(1 + a k+1 )3 · ≥ (1 + a k )3 (1 + a 3 ) ·
1 + a k+1 1 + ak
 (1 + a 3 )(1 + a k+1 )3 ,

and therefore 2(1 + a k+2 )3 ≥ (1 + a 3 )(1 + a k+1 )3 .


Thus, inequality (1) holds.
Note that (13 + a 3 )(13 + b3 )(13 + c3 ) ≥ (1 + abc)3 (Problem 6.2) and
(2(a 2012 + 1)(b2012 + 1)(c2012 + 1))3  2(a 2012 + 1)3 · 2(b2012 + 1)3 · 2(c2012 + 1)3
≥ (1 + a 3 )(1 + a 2011 )3 · (1 + b3 )(1 + b2011 )3 · (1 + c3 )(1 + c2011 )3
≥ ((1 + abc)(a 2011 + 1)(b2011 + 1)(c2011 + 1))3 ,

and therefore 2(a 2012 + 1)(b2012 + 1)(c2012 + 1) ≥ (1 + abc)(a 2011 + 1)(b2011 +


1)(c2011 + 1).
7.25. (a) The inequality b12 ≥ b2 follows from the inequality of Problem 2.2.
(b) Let b0  1, bn+1  0. We then need to prove that bk2 ≥ bk−1 bk+1 , for
k  1, 2, . . . , n.
104 7 The Principle of Mathematical Induction

For the numbers a1 , . . . , an−1 , the numbers b1 , . . . , bn−1 are defined in a


similar way as we defined the numbers b1 , . . . , bn for the numbers a1 , . . . , an ,
and b0  1, bn  0.
For n  2, we have b0  1, b1  a1 +a 2
2
, b2  a1 a2 , b3  0, and hence
b1 ≥ b0 b2 ,b2 ≥ b1 b3 .
2 2

Assume that this statement holds for n − 1 numbers, and let us prove that it
holds for n (n ≥ 3) numbers.
k−1
b C k +b Cn−1 an
Note that bk  k n−1 Ck−1 k  n−k
n
bk + nk bk−1 · an , for k  1, 2, . . . , n.
n
For k  2, . . . , n − 1, we have
 2
n 2 (bk2 − bk−1 bk+1 )  (n − k)bk + kbk−1 · an
  
− (n − k + 1)bk−1 + (k − 1)bk−2 · an (n − k − 1)bk+1 + (k + 1)bk · an
  2 2
 (n − k)2 − 1 (bk − bk−1 bk+1 ) + (k 2 − 1)an2 (bk−1 − bk−2 bk )
+ (k − 1)(n − k − 1)(bk bk−1 − bk−2 bk+1 )an ≥ 0,

2 2
as bk · bk−1 ≥ bk−1 bk+1 · bk−2 bk , and therefore bk2 − bk−1 bk+1 ≥ 0.
This ends the proof.

Problems for Independent Study

Prove the following inequalities (1–20).


   
1. aa21 − aa21 + · · · + aan−1n
− aan−1
n
≤ aan1 − aan1 , where n ≥ 2, 0 < a1 ≤ · · · ≤ an .
2. (1 + a1 ) · · · (1 + an ) ≥ 1 + a1 + · · · + an , where a1 > −1, . . . , an > −1 and the
numbers a1 , . . . , an have the same sign.
3. C ≤ D ≤ 2C, where C  (a1 − b1 )2 + · · · + (an − bn )2 , D  (a1 − bn )2 +
(a2 − bn )2 + · · · + (an − bn )2 ,
a1 + · · · + ak
bk  , k  1, . . . , n.
k
√ √
4. n n > n+1 n + 1, where n ≥ 3, n ∈ N.
 k 2
5. (a) 1 + n1 < 1 + nk + nk 2 , where k ≤ n, n, k ∈ N.
 n
(b) 1 + mn m < 3, where m ∈ N, n ∈ N.
Hint. See Problems 5(a) and 3.6(c).
2 3 2k
6. 1 − x + x2! − x3! + · · · + (2k)!
x
> 0, where k ∈ N.
 
 1
n 
n
7. ai
≤ a1 an n(a1 + an ) −
1
ai , where 0 < a1 ≤ · · · ≤ an .
i1 i1
8. a1 +···+ak
k
< a1 +···+an
n
< ak+1 +···+an
n−k
, where a1 < · · · < an , n > k, n, k ∈ N.
Problems for Independent Study 105

9. a12 − a22 + · · · − a2n 2 2


+ a2n+1 ≥ (a1 − a2 + · · · − a2n + a2n+1 )2 , where a1 ≥ a2 ≥
· · · ≥ a2n+1 ≥ 0.
(n−1)xn2 +2xn +n−1
10. x1 + x1 (x2 − x1 ) + · · · + xn−1 (xn − xn−1 ) ≤ 2n
, where n ≥ 2.
11. 11 · . . . · n n > (2n)! , where n ≥ 5, n ∈ N.
12. (2m
m1!
1 )!
· · · (2m n )!
mn !
≥ 2 S , where m 1 , . . . , m n ∈ Z0 and m 1 + · · · + m n  S.
13. a+b + · · · + a+nb < √a(a+nb)
1 1 n
, where a > 0, b > 0 and n ∈ N.
 n
n−1
14. · 1 < 4, where n ≥ 2, n ∈ N.
n−k 2k−1
k1
 k k2
15. (a) 1 + n1 < 1 + nk + 2n 2 , where k, n ∈ N and (k − 1) < n;
2
       
n 1 n+1
(b) 1 + n1 1 + 4n 1
< 1 + n+1 1
1 + 4(n+1) , where n ∈ N.
 n  
16. cos 2i x  ≥ n , where n ∈ N.
2
i0
17. (a) sin α + sin22α + · · · + sinnnα ≥ 0, where n ∈ N and 0 ≤ α ≤ π ,
(b) cos α + cos22α + · · · + cosnnα ≥ −1, where n ∈ N.
18. (a) xx21 + xx23 + · · · + xxn1 ≥ xx21 + xx23 + · · · + xxn1 , where 0 < x1 ≤ · · · ≤ xn , n ≥ 2;
(b) x13 + · · · + xn3 ≥ x1 + · · · + xn , where x1 > 0, . . . , xn > 0 and x1 · . . . · xn  1.
x13 xn3 (x1 +···+xn )3
Hint. Note that 1·(x1 +...+x n)
+ · · · + 1·(x1 +···+x n)
≥ n·n(x 1 +···+x n )
.
19. (1 + a1 )(2 + a2 ) · . . . · (n + an ) ≤ 2 · n! , where n ≥ 2, a1 > 0, . . . , an > 0 and
a1 + · · · + an  1.
Hint. Let ai  iαi , i  1, . . . , n, and we need to prove that
(1 + α1 )(1 + α2 ) · · · (1 + αn ) ≤ 2, having that α1 + 2α2 + · · · + nαn  1.
Note that if (n − 1)αn−1 +nαn  (n −1)βn−1 , then (1+αn−1 )(1+αn ) ≤ 1+βn−1 .
20. (1 − a)(a k1 + a k2 + · · · + a kn )2 < (1 + a)(a 2k1 + a 2k2 + · · · + a 2kn ), where n ≥
2, n ∈ N,
0 < a < 1, 0 ≤ k1 ≤ · · · ≤ kn and k1 , k2 , . . . , kn ∈ N.
2k1 +1
Hint. Note that a k1 (a k2 + · · · + a kn ) < a 2k1 +1 + a 2k1 +2 + · · ·  a1−a .
21. For arbitrary positive integer n > 1 find the smallest possible value of C
if the inequality aa11−b +b1
1
+ · · · + aann−b
+bn
n
< C holds for all positive numbers
a1 , . . . , an , b1 , . . . , bn satisfying the equality a1 + · · · + an  b1 + · · · + bn .
1 ···x n
22. Prove that for an arbitrary positive integer n > 1, one has 1−x 1−y1 ···yn
< 1−x
1−y1
1
+
· · · + 1−yn , where 0 < yi ≤ xi < 1, i  1, . . . , n.
1−xn

Hint. Note that 1−ab 1−cd


< 1−cd
1−a 1−b
+ 1−cd < 1−a1−c
1−b
+ 1−d , where 0 < c ≤ a < 1, 0 <
d ≤ b < 1.
Chapter 8
A Useful Inequality

In 1997 Nairi Sedrakyan has published an article (in Russian) [17] called “On the
applications of a useful inequality,” in which the author proves a very useful inequality
and provides some applications.
In 1998, this inequality was re-published by the author, this time in Armenian, in
the book [16], in which the author devotes an entire chapter to its applications. In
2002 this book was published in Moscow in Russian [15].
Russian-speaking reader sometimes calls this inequality Sedrakyan’s inequality
in reference to Sedrakyan’s 1997 article [17] on its numerous applications, while
the English-speaking reader sometimes calls this inequality Engel’s form or Titu’s
lemma, in reference to the book [4] published in 1998, and the book [1], published
in 2003. Nevertheless, even though Sedrakyan in his article [17] stated and proved
this inequality without using the Cauchy–Bunyakovsky–Schwarz inequality, it turns
out that this inequality is nothing but another form of that inequality. Probably this
form was known even before this article, with the difference that in his article [17],
Sedrakyan noticed that written in this form, the inequality has very useful new appli-
cations, and he provided numerous ways in which this inequality can be used as a
mathematical proof technique to prove inequalities of various types. In this chapter
we consider that inequality, along with its generalizations and applications.

Lemma 1 (A useful inequality) Let a1 , a2 , . . . , an be real numbers and b1 , b2 , . . . , bn


positive real numbers. Then

a12 a2 (a1 + · · · + an )2
+ ··· + n ≥ . (8.1)
b1 bn b1 + · · · + bn

Moreover, equality holds if and only if a1


b1
 ···  an
bn

© Springer International Publishing AG, part of Springer Nature 2018 107


H. Sedrakyan and N. Sedrakyan, Algebraic Inequalities, Problem Books
in Mathematics, https://doi.org/10.1007/978-3-319-77836-5_8
108 8 A Useful Inequality

Generalizations of inequality (8.1).

Lemma 2 (Generalization 1)

a13 a3 a3 (a1 + a2 + · · · + an )3
+ 2 + ··· + n ≥ , (8.2)
b1 c1 b2 c2 bn cn (b1 + b2 + · · · + bn )(c1 + c2 + · · · + cn )

where ai > 0, bi > 0, > ci > 0, i  1, 2, . . . , n.

Lemma 3 (Generalization 2)

a1n ann (a1 + · · · + an )n


+ ··· + ≥ , (8.3)
b1n−1 bnn−1 (b1 + · · · + bn )n−1

where ai > 0, bi > 0, i  1, 2, . . . , n.

Lemma 4 (Generalization 3)
n n n
a1,1 a2,1 am,1
+ + ··· + ≥
a1,2 · a1,3 · · · a1,n a2,2 · a2,3 · · · a2,n am,2 · am,3 · · · am,n
 n
a1,1 + a2,1 + · · · + am,1
≥     ,
a1,2 + a2,2 + · · · + am,2 · a1,3 + a2,3 + · · · + am,3 · · · a1,n + a2,n + · · · + am,n
(8.4)

where ai, j > 0, i  1, 2, . . . , m, j  1, 2, . . . , n.

Proof of Lemma 1 Let us at first prove that for all real numbers a, b and positive
real numbers x, y, one has

a 2 b2 (a + b)2
+ ≥ . (8.5)
x y x+y

Note that this inequality is equivalent to the inequality (ay − bx)2 ≥ 0.


Obviously, here the equality holds here if and only if ay − bx  0, that is a
x
 by .
Now, let us proceed to the proof of this useful inequality.
Applying inequality (8.5) several times, we obtain

a12 a22 a32 a2 (a1 + a2 )2 a32 a2


+ + + ··· + n ≥ + + ··· + n
b1 b2 b3 bn b1 + b2 b3 bn
(a1 + a2 + a3 )2 an2
≥ + ··· +
b1 + b2 + b3 bn
≥ ···
(a1 + · · · + an )2
≥ .
b1 + · · · + bn

Obviously, equality holds if and only if a1


b1
 a2
b2
 ···  an
bn
.
8 A Useful Inequality 109

This ends the proof.


We provide proofs of Lemmas 2–4 after the remark following Lemma 6.
Let us consider the following examples and applications of this useful inequality
(Lemma 1).

Cauchy–Bunyakovsky–Schwarz inequality.  Let  x1 , x2 , . . . , xn and


y1 , y2 , . . . , yn be real numbers, then x12 + · · · + xn2 y12 + · · · + yn2 ≥ (x1 y1 + · · · + xn yn )2 .

Proof For real numbers a1 , a2 , . . . , an and positive real numbers b1 , b2 , . . . , bn , we


a2 a2
have b11 + · · · + bnn ≥ (ab11+···+a n)
2

+···+bn
.
 a2i  xi yi 2and  bi  yi . Then for2 all 1 ≤ i ≤ n, we obtain
2
 2Let us take
x1 + · · · + xn y1 + · · · + yn ≥ (x1 y1 + · · · + xn yn ) .
2

This ends the proof.

Remark We have obtained a very beautiful inequality (8.1). However, it


turns out that this is one of the forms of the Cauchy–Bunyakovsky–Schwarz √
inequality. Indeed, making a change of variables xi √abi and yi  bi we
 2 i  2 
obtain Cauchy–Bunyakovsky–Schwarz inequality x1 + · · · + xn y1 + · · · + yn2 ≥
2

(x1 y1 + · · · + xn yn )2 .
Note that in order to prove certain inequalities the form (8.1) is more convenient
than the classical form of Cauchy–Bunyakovsky–Schwarz inequality.

Example 8.1 Prove that 1


a+b
+ 1
b+c
+ 1
a+c
≥ 9
2(a+b+c)
, where a > 0, b > 0, c > 0.

Proof Note that according to Lemma 1 (for n  3), we have

1 1 1 12 12 12
+ +  + +
a+b b+c a+c a+b b+c a+c
(1 + 1 + 1)2 9
≥  .
(a + b + b + c + a + c) 2(a + b + c)

This ends the proof.

Example 8.2 Prove that  


12
a+b+c+d
≤ a+b
1 1
+ a+c + 1
a+d
+ 1
b+c
+ 1
b+d
+ 1
c+d
≤ 3 1
4 a
+ 1
b
+ 1
c
+ 1
d
, where a >
0, b > 0, c > 0, d > 0.

Proof Note that according to Lemma 1 (for n  6), we have


1 1 1 1 1 1
+ + + + +
a+b a+c a+d b+c b+d c+d
(1 + 1 + 1 + 1 + 1 + 1)2 12
≥  .
3(a + b + c + d) a+b+c+d

On the other hand, on applying Lemma 1 (for n  2) multiple times for each of
the following sets of parentheses, we obtain
110 8 A Useful Inequality
             
1 1 1 1 1 1 1 1 1 1 1 1 1 1 1 1
3 + + +  + + + + + + + + + + + ≥
a b c d a b a c a d b c b d c d

(1 + 1)2 (1 + 1)2 (1 + 1)2 (1 + 1)2 (1 + 1)2 (1 + 1)2


≥ + + + + + .
a+b a+c a+d b+c b+d c+d
Thus, it follows that

12 1 1 1 1 1 1
≤ + + + + +
a+b+c+d a+
 b a + c a +d b + c b + d c + d
3 1 1 1 1
≤ + + + ,
4 a b c d

which completes the proof.

A more general inequality. Let us try to obtain a more general inequality.


To this end, we write ai2 as the product of two factors.
On proving this inequality for n  2, we obtain new conditions. It turns out that
the following lemma holds.

Lemma 5 (Generalization 4) If ac11 ≥ · · · ≥ acnn , bc11 ≥ · · · ≥ bbnn (that is acii and


bi
ci
have the same order) and ci > 0, i  1, . . . , n, then the following inequality
holds:

n
n


n ai · bi
ai bi i1 i1
≥ . (8.6.1)
ci
n
i1 ci
i1

If ai  bi , then inequality (8.6.1) becomes inequality (8.1).

Proof In order to prove inequality (8.6.1), we use the following fact: if ac11 ≥ · · · ≥ acnn
and ci > 0, i  1, . . . , n, then ac11 +···+a k
+···+ck
≥ acnn for all k  1, . . . , n (see Problem 1.
11).
Let us prove inequality (8.6.1) by induction on n
For n  2, we have that ac1 b1 1 + ac2 b2 2 ≥ (a1 +ac21)(b +c2
1 +b2 )
, or (a1 c2 −a2 c1 )(b1 c2 −b2 c1 ) ≥
0, which follows from c1 ≥ c2 and c1 ≥ c2 .
a1 a2 b1 b1

Assume that inequality (8.6.1) holds for n = k Then we have a1cbi 1 + · · · + ack bk k ≥
(a1 +···+ak )(b1 +···+bk )
c1 +···+ck
.
From the above-mentioned fact and the inequality for n  2 it follows that ac1 b1 1 +
· · · + ack bk k + ak+1 bk+1
ck+1
≥ (a1 +···+a k )(b1 +···+bk )
c1 +···+ck
·bk+1
+ ak+1ck+1 ≥ (a1 +···+ac1k+1 )(b1 +···+bk+1 )
+···+ck+1
or ac1 b1 1 + · · · +
ak+1 bk+1
ck+1
≥ (a1 +···+ac1k+1 )(b1 +···+bk+1 )
+···+ck1
.
In a similar way, one can prove the following lemma.
8 A Useful Inequality 111

Lemma 6 (Generalization 5) If acii and bcii have opposite orders and ci > 0, i 
1, . . . , n, then the following inequality holds:

n
n


n ai · bi
ai bi i1 i1
≤ . (8.6.2)
ci
n
i1 ci
i1

Remark (Chebyshev’s inequality) If the conditions a1 ≥ · · · ≥ an , b1 ≥ · · · ≥


bn , and 0 < c1 ≥ · · · ≥ cn hold, then the conditions of inequality (8.6.1) hold, and
therefore this inequality holds as well.
From inequalities (8.1) and (8.6.1) we obtain the following well-known inequal-
ities:

n
1 n2
≥ n , where ci > 0, i  1, . . . , n, (8.7)
c
i1 i ci
i1

n ⎞2
ai2
1 2 ⎜ ⎟
n
a ≥⎜ ⎟ ,
i1
⎝ n ⎠ (8.8)
n i1 i


n
1
n n
ai bi ≥ ai · bi , (8.9)
i1
n i1 i1

where a1 ≥ · · · ≥ an and b1 ≥ · · · ≥ bn . Inequality (8.9) is called Chebyshev’s


inequality.
Note that inequality (8.7) follows from Chebyshev’s inequality (taking ai 
ci , bi  − c1i ).
Note also that inequality (8.8) follows from Chebyshev’s inequality (taking ai 
bi ).
Now, taking ai  ci xi , bi  ci yi , Pi  n
ci
, we obtain the following form
ci
i1
of Chebyshev’s inequality:

Chebyshev’s inequality (alternative form). If xi and yi have the same order,



n n
Pi  1, Pi > 0, i  1, . . . , n, then for the means M z  z i Pi , one has
i1 i1
M x · M y ≤ M(x y).

Remark Therefore, the Cauchy–Bunyakovsky–Schwarz inequality is a particular


case of Chebyshev’s inequality.
112 8 A Useful Inequality

Proof of Lemma 2 Indeed, inequality (8.2) is one of the forms of the Cauchy–Bun-
yakovsky–Schwarz inequality
 3     
a1 + a23 + · · · + an3 · b13 + b23 + · · · + bn3 · c13 + c23 + · · · + cn3
≥ (a1 b1 c1 + a2 b2 c2 + · · · + an bn cn )3 , (8.10)

where ai > 0, bi > 0, ci > 0, i  1, 2, . . . , n.


Note that in inequality (8.10), by making the change of variables ai  √3 xyii zi ,
√ √
bi  3 yi , ci  3 z i , we obtain for the numbers xi , yi , z i inequality (8.2).
In order to prove inequality (8.10), note that if the variables a1 , a2 , . . . , an are
replaced by variables λa1 , λa2 , . . . , λan , where λ is any positive number, then we
obtain an equivalent inequality.
Choose a number λ such that (λa1 )3 + (λa2 )3 + · · · + (λan )3  1.
Therefore, without loss of generality one can assume that a13 + a23 + · · · + an3 
1, b13 + b23 + · · · + bn3  1, c13 + c23 + · · · + cn3  1, and hence it is sufficient to
prove that 1 ≥ a1 b1 c1 + a2 b2 c2 + · · · + an bn cn Indeed, if x > 0, y > 0, z > 0,
then x 3 + y 3 + z 3 − 3x yz  21 (x + y + z) (x − y)2 + (y − z)2 + (z − x)2 ≥ 0, and
a 3 +b3 +c3
therefore ai bi ci ≤ i 3i i , i  1, 2, . . . , n.
Summing all these inequalities, we deduce that 1 ≥ a1 b1 c1 +a2 b2 c2 +· · ·+an bn cn .
This ends the proof.
Proof of Lemma 3 Lemma 3 is a direct consequence of Lemma 4.
Proof of Lemma 4 See Problem 45 in the end of this chapter.
Let us consider the following examples (applications of Lemma 2).
a3 b3 c3
Example 8.3 Prove that bc
+ ca
+ ab
≥ a + b + c, where a > 0, b > 0, c > 0 .
a3 b3 c3 (a+b+c)3
Proof According to inequality (8.2) we have that, bc
+ ca
+ ab
≥ (b+c+a)+(c+a+b)

a + b + c.
This ends the proof.
a3 3 3 (a+b+c)3
Example 8.4 Prove that x
+ by + cz ≥ 3(x+y+z)
where a > 0, b > 0, c > 0, x > 0,
y > 0, z > 0.
a3 b3 c3 a3 b3 c3
Proof According to inequality (8.2), we have x
+ y
+ z
 1·x
+ 1.y
+ 1.z

(a+b+c)3 (a+b+c)3

(1+1+1)(x+y+z) 3(x+y+z)
.
This ends the proof.
Example 8.5 Prove that a
b2 c2
+ b
c2 a 2
+ c
a 2 b2
≥ 9
a+b+c
, where a 2 + b2 + c2  3abc and
a > 0, b > 0, c > 0.
 2 3  2 3  2 3
Proof Since a b c a b c
+ 2 2 + 2 2  2 3 2 2 + 2 3 2 2 + 2 3 2 2,
2
b c 2 c a a b a ·a b c b ·a b c c ·a b c
then according to inequality (8.2), we have b2ac2 + c2ba 2 + a 2cb2 ≥
(a 2 +b2 +c2 )3  a+b+c
9
.
(a 2 +b2 +c2 )(a 3 b2 c2 +a 2 b2 c3 +a 2 b2 c3 )
This ends the proof.
Problems 113

Problems

Prove the following inequalities (8.1)–(8.8).

8.1. Nesbitt’s inequality: x2x+x 1


3
+ x3x+x
2
1
+ x1x+x
3
2
≥ 23 , where x1 > 0, x2 > 0, x3 > 0.
a
8.2. b+c b
+ c+d c
+ d+a d
+ a+b ≥ 2, where a > 0, b > 0, c > 0, d > 0.
8.3. x2 +xn + x3 +x1 +· · · + xnx+x
x1 x2 n−1
n−2
xn
+ x1 +x n−1
≥ 2, where n ≥ 4 and x1 > 0, . . . , xn > 0.
3 3 3
2 + b2 +bc+c2 + c2 +ca+a 2 ≥ , where a > 0, b > 0, c > 0.
a b c a+b+c
8.4. (a) a 2 +ab+b 3
(b) b + c + a ≥ b+c + a+b + 1, where a > 0, b > 0, c > 0.
a b c a+b b+c

(c) ab + bc + ac ≥ a+c + b+a + c+b , where a > 0, b > 0, c > 0.


 a b c  b+c a+bc+ab+c a+b
(d) 2 b + c + a ≥ b+c + a+b + a+b + a+c + a+c + b+c where a > 0, b > 0, c > 0.
a+c a+b b+c a+c
8.5. p−2a1 + · · · + p−2an ≥ n−2 , where p is the perimeter of a polygon with side
a1 an n

lengths a1 , . . . , an .
1
8.6. (a) a 3 (b+c) 1
+ b3 (a+c) 1
+ c3 (a+b) ≥ 23 , where abc = 1 and a,b,c > 0.

(b) 1+x1
1
+ · · · + 1
1+x1 +···+xn
< 1
x1
+ · · · + x1n , where xi > 0, i  1, . . . , n
y
(c) y 2x−z + z 2 −x + x 2z−y > 1, where 2 < x < 4, 2 < y < 4, 2 < z < 4.
 3  2    
8 x + y 3
8.7.  + z 3 ≥ 9x 2 + yz y 2 +x z z 2 +x y , where x > 0, y > 0 z > 0.
1 1 1 1 1
n n n n n
8.8. n
a i n
b i n
ci · · · n
di ≤ n
ai · bi · ci · · · di ,
i1 i1 i1 i1 i1
where 0 ≤ a1 ≤ · · · ≤ an , 0 ≤ b1 ≤ · · · ≤ bn , 0 ≤ c1 ≤ · · · ≤
cn , · · · , 0 ≤ d1 ≤ · · · ≤ dn .
8.9. Consider a sequence with positive terms (x k ) where 1  x0 ≥ x1 ≥ · · · ≥
xn ≥ · · · . Prove that there exists a positive integer n such that for every such
x2 x2 x2
sequence (x k ), one has x01 + x12 + · · · + xn−1 n
≥ 3.999.
8.10. Let M be an arbitrary point in the interior of a given triangle ABC. Con-
sider perpendiculars M A1 , M B1 , MC1 drawn from the point M to lines
BC, C A, AB, respectively. For which point M in the interior of triangle
ABC does the quantity MBCA1 + MC BA1 + MC AB
1
attain the smallest possible value?
8.11. Let G be the intersection point of the medians of triangle A1 A2 A3 and let C be
the circumcircle of triangle A1 A2 A3 Let the lines G A1 , G A2 , G A3 intersect
the circle C a second time at points B1 , B2 , B3 , respectively.
Prove that G A1 + G A2 + G A3 ≤ G B1 + G B2 + G B3 .

Prove that a1 +a2 +· · ·+a2k −2k 2k a1 · a2 · · · a2k ≥ (a1 −a 2 +···+a2k−1 −a2k )
2
8.12. 2(a1 +a2 +···+a2k )
, where
k ∈ N and a1 , a2 , . . . , a2k > 0.
√ √ √
8.13. Prove that a1 a2 + a1 a3 + · · · + an−1 an ≤ n2 ≤ n−1 2 (a1
+ a2 + · · · + an ),
where n ∈ N, n ≥ 2, a1 , . . . , an ≥ 0 and 1+a 1
1
+ · · · + 1
1+an
 n − 1.
8.14. Prove that 1+a1 +···+an−1 + 1+a1 +···+an−2 +an + · · · + 1+a2 +···+an ≤ 1, where n ∈ N, n ≥
1 1 1

2, a1 , . . . , an ≥ 0, and
 a1 . . .an  1.  
8.15. Prove that a 2 + bc b2 + cd c2 + da d 2 + ab ≥ (a +1)(b+1)(c+1)(d +1),
where abcd = 1 and a > 0, b > 0, c > 0, d > 0.
2 ≥ 3 , where abd = 1 and a > 0, b >
1 1 1 1
8.16. Prove that a 5 (b+2c) 2 + 5
b (c+2a)2
+ c5 (a+2b)
0, c > 0.
114 8 A Useful Inequality

8.17. Prove that a 3 +ba3 +c2 + a 2 +bb3 +c3 + a 3 +bc2 +c3 ≤ 1, where ab + bc + ac ≥ 3 and
a > 0, b > 0, c > 0.
8.18. Prove that
x3 y3 z3
(a) (1+y)(1+z) + (1+x)(1+z) + (1+x)(1+y) ≥ 34 , where x > 0, y > 0, z > 0 and
x yz  1. √
y 3 3 x yz
x
(b) 1−x + 1−y + 1−z z
≥ 1− √3 x yz , where 0 < x < 1, 0 < y < 1, 0 < z < 1.
y
x
(c) (y+z)(y+z−x) + (x+z)(x+z−y) z
+ (x+y)(x+y−z) ≥ 2(x+y+z)
9
, where x, y, z are the
side lengths of some triangle.
a3 b3 c3 d3
(d) b+c+d + a+c+d + a+b+d + a+b+c ≥ 13 , where a > 0, b > 0, c > 0, d > 0 and
ab +bc + cd + da  1.  
a3 a3 a3
(e) 1 + a12 1 + a22 · · · 1 + an2 ≥ (1 + a1 )(1 + a2 ) · · · (1 + an ), where
2 3 1
a1 , . . . , an > 0.  
  2
(f) x12 + · · · + xn2 x 2 +x1 x + · · · + x 2 +x1 n x1 ≥ n2 , where n ∈ N, n ≥
1 1 2 n
2, x1 , . . . , xn > 0.  
(g) (a1 a2 + a2 a3 + · · · + an a1 ) a 2a+a 1
+ a 2a+a
2
+ · · · + a 2a+a
n
≥ n+1
n
, where
2 2 3 3 1 1
n ∈ N, n ≥ 3, a1 , . . . , an > 0 and a1 + · · · + an  1.
x15 x25
8.19. Find the smallest value of the expression x2 +x3 +···+x n
+ x1 +x3 +···+x n
+ ··· +
xn5
x1 +x2 +···+xn−1
, where n ≥ 3, x1 > 0, x2 > 0, . . . , xn > 0, and x12 + x22 +
· · · + xn2  1.
8.20. Prove that for all positive numbers x, y, z such that x yz  1, the following
3 3 3
inequality holds: x 2x+y + y 2y+z + z 2z+x ≥ 23 .

Proofs
a2
8.1. In similar problems one often needs to write ab
instead of ab . It follows that
x12 x22 x2
 x1 (x2 +x3 ) + x2 (x3 +x1 ) + x3 (x13+x2 ) ≥ 2(x1(xx12 +x 2 +x 3 )
2
x1
+ x3x+x
x2 +x3
2
1
+ x1x+x
3
2 +x2 x3 +x3 x1 )
≥ 23
(here we have used that x1 x2 + x2 x3 + x3 x1 ≤ x12 + x22 + x32 ).
a2 b2 c2 d2 (a+b+c+d)2
8.2. We have that, a(b+c) + b(c+d) + c(d+a) + d(a+b) ≥ ab+2ac+ad+bc+2bd+cd ≥ 2.
In order to prove the last inequality, it is sufficient to expand (a + b + c + d)2
and use that a 2 + c2 ≥ 2ac, b2 + d 2 ≥ 2bd.
x2 x2 xn2
≥ 2(x1 x2 +x(x21x+x 2 +···+x n )
2
8.3. We have x1 (x21+xn ) + x2 (x32+x1 ) + · · · + xn (x1 +x n−1 ) 3 +···+x n−1 x n +x n x 1 )
 An .
If n ≥ 4, then An ≥ 2 (see the proof Problem 7.6).
4 4 4
8.4. (a) Using inequality (8.1), we obtain a 3 +aa2 b+ab2 + b3 +bb2 c+bc2 + c3 +c2ca+ca 2 ≥
(a 2 +b2 +c2 ) 2
≥ a+b+c .
(a+b+c)(a 2 +b2 +c2 ) 3
Here we have used that a 2 + b2 + c2 ≥ 13 (a + b + c)2 , or inequality (8.6.1) for
n = 3.
Proofs 115

≥ ab + (b+c)
2 2 2
(b) By inequality (8.1), it follows that ab + bc + ac  ab + bc
b c
+ ac bc+ac

a
b
+ c · a+b + a+b .
b b+c b+c

Hence, in order to complete the proof, one needs to prove that ab + bc · a+b b+c

  2
a+b
b+c
+1, which is true because it is equivalent to the inequality ac − b 2
≥ 0.
(c) Without loss of generality one can assume that (b − a)(b − c) ≤ 0.
On the other hand, by Problem 8.4 (b), we have ab + bc + ac ≥ a+b b+c
+ a+bb+c
+ 1.
Therefore, it is sufficient to prove that b+c + 1 ≥ b+c + a+c , which holds
a+b a+c a+b

because it is equivalent to the inequality (b − a)(b − c) ≤ 0.


(d) In a similar way, one can prove that ab + bc + ac ≥ a+b b+c
a+c
+ a+b b+c
+ a+c .
8.5. Without loss of generality one can assume that a1 ≥ · · · ≥ an , whence
0 < p − 2a1 ≤ · · · ≤ p − 2an .
a1 ·1
According to the remark of inequality (8.6.1), we have that p−2a 1
+ ··· +
an ·1 (a1 +···+an )n
p−2an
≥ np−2(a1 +···+an )
 n
n−2
.

( ) + ( b1 ) + ( 1c ) ≥ ( a1 + b1 + 1c )  ab+bc+ac ≥ 3 3 a 2 b2 c2 
1 2 2 2 2
8.6. (a) We have that, a(b+c) b(a+c) c(a+b)
a
2(ab+bc+ac) 2 2
3
2
.
1
(b) Set 1+x1 +···+x i
 yi , i  1, . . . , n, in which case 1 > y1 > · · · > yn > 0
yi−1 yi
and xi  yi−1 −yi , i  1, . . . , n, y0  1.
1

From inequality (8.1), we deduce that


  
 n  n  
1 1   yi−1 yi  yi2
+ ··· +     yi +
x1 xn y − yi
i1 i−1 i1
yi−1 − yi

 n
 yi2 y1 + · · · + yn
> ≥ √ > y1 + · · · + yn ,
y
i1 i−1
− yi 1 − yn


and therefore 1
x1
1
xn
+ ··· +
> 1+x1
1
+ · · · + 1+x1 +···+x
1
n
.
2 y2 2
x y
(c) We have that, y 2 −z + z 2 −x + x 2 −y  x y 2 −z + y z 2 −x + z xz2 −y ≥
z x
( ) ( ) ( )
(x+y+z)2 (x+y+z)2
≥ x y 2 +yz 2 +zx 2 −x z−yx−zy > 4x y+4yz+4zx−x z−yx−zy ≥ 1, and therefore y 2x−z +
y
z 2 −x
+ x 2z−y > 1.
8.7 We have
 3
   
x 2 + y 2 + z 2 + x y + yz + x z
2
x + yz 2
y + xy z + xy ≤
2
3
 
2 2  3
 x + y2 + z2 .
3 (8.11)

On the other hand, from inequality (8.1) we obtain 


x 3 +y 3 +z 3 x4 y4 z4 x 2 +y 2 +z 2 x 2 +y 2 +z 2
x 2 +y 2 +z 2
 x (x 2 +y 2 +z 2 )
+ x (x 2 +y 2 +z 2 )
+ x (x 2 +y 2 +z 2 )
≥ x+y+z
≥ 3
,
116 8 A Useful Inequality

whence
 2  3
3 x 3 + y3 + z3 ≥ x 2 + y2 + z2 . (8.12)

From (8.11) and (8.12) it follows that (x 2 + yz)(y 2 + x z)(z 2 + x y) ≤ 89 (x 3 +


y 3 + z 3 )2 .
8.8. Using Chebyshev’s inequality, we obtain
⎛ ⎞⎛ ⎞⎛ ⎞ ⎛ ⎞ ⎛ ⎞⎛ ⎞ ⎛ ⎞
n
1  ⎠⎝ 1  ⎠
n n 
n 
n 
n 
n
⎝1 ai ⎠ ⎝ bi ci · · · ⎝
1
di ⎠ ≤ ⎝
1
ai bi ⎠⎝
1
ci ⎠ · · · ⎝
1
di ⎠ ≤
n n n n n n n
i1 i1 i1 i1 i1 i1 i1
⎛ ⎞ ⎛ ⎞
1 n
1 n
1
n
≤ ⎝ ⎠
ai bi ci · · · ⎝ di ⎠ ≤ · · · ≤ ai bi ci · · · di .
n n n
i1 i1 i1

x2 x2 x2
≥ (x0x+x 1 +···+x n−1 )
2
8.9. From inequality (8.1), we obtain x01 + x12 + · · · + xn−1
n 1 +x 2 +···+x n
 Kn .
Let us prove that there exists a positive integer n 0 such that for n > n 0 , K n ≥
3.999.
We have
 2   2
x0 + x1 + · · · + xn−1 − 3.999(x1 + x2 + · · · + xn )  1 − x1 + x2 + · · · + xn−1 +
  3.999
+ 0.001 x1 + x2 + · · · + xn−1 − 3.999xn ≥ 0.001(n − 1)xn − 3.999xn ≥ 0, if n ≥ + 1.
0.001

Therefore, one can take n0 equal to 4000.


8.10. By transforming the given expression and using inequality (8.1), we obtain
that

BC 2 C A2 AB 2
+ +
BC · M A1 C A · M B1 AB · MC1
(BC + C A + AB)2 4 p2 p
≥  2 .
BC · M A1 + C A · M B1 + AB · MC1 2S r

Therefore, the smallest possible value of the expression MBCA1 + MC BA1 + MCAB
1
is
2p
equal to r , when BC·M A1  C A·M B1  AB·MC1 that is, when M A1  M B1 
BC CA AB

MC1
Therefore, the point M is the incenter of triangle ABC.
8.11. Let us denote by A1 , A2 , A3 and a1 , a2 , a3 the midpoints and the lenghts of
sides A2 A3 , A1 A3 , A1 A2 , respectively.
Letting a1 ≤ a2 ≤ a3 , one can easily prove that G A3 ≤ G A2 ≤ G A1 .
a2
We have 23 G A1 · B1 A1  41 a12 , and hence G B1  G2A1 + 6G1A1 .
a12 a2 a2
Hence, it is sufficient to prove that 3G A1
+ 3G2A2 + 3G3A3 ≥ G A1 + G A2 + G A3 .
Proofs 117

a2 a2 a2 a12 +a22 +a32


From inequality (8.6.1), it follows that 3G1A1 + 3G2A2 + 3G3A3 ≥ G A1 +G A2 +G A3

3(G A21 +G A22 +G A23 )
G A1 +G A2 +G A3
≥ GA + GA + GA ,
1 2 3
a12 a22 a32
and therefore, + 3 G A1
+ ≥ G A1 + G A2 + G A3 .
3 G A2 3 G A3
8.12. From Problem 2.1, it follows that
√ √ √ √ √ √
a1 a2 + a1 a2 + a3 a4 + a3 a4 + · · · + a2k−1 a2k + a2k−1 a2k

≥ 2k 2k a1 · a2 · · · a2k ,

and hence we obtain that


√ √ √ 2 √ √ 2
a1 + a2 + · · · + a2k − 2k 2ka1 · a2 · · · a2k ≥ a1 − a2 + · · · + a2k−1 − a2k 

(a1 − a2 )2 (a2k−1 − a2k )2 (a1 − a2 )2 (a2k−1 − a2k )2


≥ √ √ 2 + · · · + √ √ 2 ≥ 2(a + a ) + · · · + 2(a .
a1 + a2 a2k−1 + a2k 1 2 2k−1 + a2k )

(a1 −a2 )2 (a2k−1 −a2k )2


From inequality (8.1), it follows that 2(a1 +a2 )
+ ··· + 2(a2k−1 +a2k )

(a1 −a2 +···+a2k−1 −a2k )2
2(a1 +a2 +···+a2k )
,
√ (a1 −a2 +···+a2k−1 −a2k )2
It follows that a1 + a2 + · · · + a2k − 2k 2k
a1 · a2 · · · a2k ≥ 2(a1 +a2 +···+a2k )
,
2
8.13. We have n − 1  1+a 1
1
+ · · · + 1+a 1
n
≥ (1+a1 )+···+(1+a
n
n)
, and hence
2 (a1
n−1
+ a2 + · · · + an ) ≥ 2 .
n
√ √
Note that 1  1+a a1
+ · · · + an
≥ ( a1 +···+ an )2 , and therefore n ≥ √a a +
(1+a )+···+(1+a n)
1 2
√ √1 1+a n 1 2
a1 a3 + · · · + an−1 an .
8.14. Letting S  a1 + · · · + an we need to prove that 1+S−a S−a1
1
+ · · · + 1+S−a
S−an
n
≥ n − 1.
From inequalities (8.1) and (3.5), it follows that
√ 2 √ 2 √ √ 2
S − a1 S − an S − a1 S − an S − a1 + · · · + S − an
+ ··· +  + ··· + ≥ 
1 + S − a1 1 + S − an 1 + S − a1 1 + S − an n + (n − 1)S
√   √ 
(n − 1)S + 2 S − ai S − a j (n − 1)S + 2 S − ai − a j + ai a j
i  j i  j
 ≥ 
n + (n − 1)S n + (n − 1)S
  √ √
(n − 1)S + n(n − 1)S − 2 ai + a j + 2 ai a j (n − 1) S + 2
2
ai a j
i  j i  j i  j
  ≥
n + (n − 1)S n + (n − 1)S
 √
(n − 1)2 S + 2Cn2 Cn2 ai a j
i  j (n − 1)2 S + 2Cn2
≥   n − 1,
n + (n − 1)S n + (n − 1)S

whence we obtain S−a1


1+S−a1
+ · · · + 1+S−a
S−an
n
≥ n − 1.
3 (a+1)3
8.15. Since a +bc  a·1 + a·d , it follows from inequality (8.2) that a 2 +bc ≥ 2a(d+1)
2 a 1
.
(b+1)3 (c+1)3
In a similar way, we obtain b + cd ≥ 2b(a+1) , c + da ≥ 2c(b+1) , d + ab ≥
2 2 2

(d+1)3
2d(c+1)
.
Multiplying these inequalities, we obtain that
 2    
a + bc b2 + cd c2 + da d 2 + ab ≥ (a+1) (b+1) 8(c+1) (d+1) .
2 2 2 2
118 8 A Useful Inequality

√ √ √ √
Since (a + 1)(b + 1)(c + 1)(d + 1) ≥ 2 a · 2 b · 2 c · 2 d  8,
    
we have a 2 + bc b2 + cd c2 + da d 2 + ab ≥ (a+1) (b+1) 8(c+1) (d+1) ≥
2 2 2 2

(a + 1)(b + 1)(c + 1)(d + 1), and therefore, (a 2 + bc)(b2 + cd)(c2 + da)(d 2 +


ab) ≥ (a + 1)(b + 1)(c + 1)(d + 1).
8.16. Since
1 1 1
+ + 
a 5 (b + 2c)2 b5 (c + 2a)2 c5 (a + 2b)2
 1 3  1 3  1 3
a b c
 + + ,
(ab + 2ac)(ab + 2ac) (bc + 2ab)(bc + 2ab) (ac + 2bc)(ac + 2bc)

then from inequality (8.2), it follows that 1


a 5 (b+2c)2
+ 1
b5 (c+2a)2
+ 1
c5 (a+2b)2

ab+bc+ac
9
.

3 3 ab·bc·ac
Note that, ab+bc+ac
9
≥ 9
 1
3
, and therefore, 1
a 5 (b+2c)2
+ 1
b5 (c+2a)2
+
1
c5 (a+2b)2
≥ 1
3
.
a3 3 3 (a+b+c)3
8.17. From inequality (8.2), it follows that, a 3 + b3 + c2  1·1
b
+ 1·1 c
+ 1·c ≥ 3(2+c)
,
and therefore, a 3 +ba3 +c2 ≤ (a+b+c)
6a+3ac
3.

In a similar way, we obtain b


a 2 +b3 +c3
≤ 6b+3ab
, c
(a+b+c)3 a 3 +b2 +c3
≤ (a+b+c)
6c+3bc
3.

Summing these inequalities, we deduce that a 3 +b3 +c2 + a 2 +b3 +c3 + a 3 +bc2 +c3
a b

6a+3ac
(a+b+c)3
6b+3ab
+ (a+b+c)3 +
6c+3bc
(a+b+c)3
.
Note that 6a+3ac
(a+b+c)3
6b+3ab
+ (a+b+c)3 + 6c+3bc
(a+b+c)3
 6
(a+b+c)2
+ 3(ab+bc+ac)
(a+b+c)2 (a+b+c)

≤ 6
+ 1
3(ab+bc+ac) a+b+c
≤ 3+3
 1, and therefore,
2 1 a
+ b
+ c
a 3 +b3 +c2 a 2 +b3 +c3 a 3 +b2 +c3
≤ 1.
8.18 (a) From inequality (8.2), it follows that,
x3 y3 z3 (x + y + z)3
+ + ≥ 
(1 + y) · (1 + z) (1 + z) · (1 + x) (1 + x) · (1 + y) (1 + y + 1 + z + 1 + x)(1 + z + 1 + x + 1 + y)
 2
x+y+z 3 √ x+y+z 1
 (x + y + z) ≥ , as x + y + z ≥ 3 3 x yz  3 and ≥ .
3+x +y+z 4 3+x +y+z 2

3 3 3
y
x
Therefore, (1+y)(1+z) + (1+x)(1+z) z
+ (1+x)(1+y) ≥ 34 .
(b) From inequality (8.2), it follows that
 √ 3  √ 3 √ 
3x  √ 3
3y 3z 3x+ √ 3 y+ √ 3z 3 √
x y z 27 3 x yz
+ +  + + ≥ ≥  √  
1−x 1−y 1−z 1 · (1 − x) 1 · (1 − y) 1 · (1 − z) 3(3 − x − y − z) 3 3 − 3 3 x yz
√ 
3 3 x yz √ √ √ 3 √ √ √ √
  √  , as 3 x + 3 y + 3 z ≥ 3 3 x · 3 y · 3 z and x + y + z ≥ 3 3 x yz.
1 − 3 x yz


y 3 3 x yz
x
Therefore, 1−x + 1−y z
+ 1−z ≥ 1− √3 x yz .
(c) From inequality (8.2), it follows that
x y z
+ + 
(y + z)(y + z − x) (x + z)(x + z − y) (x + y)(x + y − z)
x3 y3 z3
  +  +   ≥
(x y + x z) x y + x z − x 2 (x y + yz) x y + yz − y 2 (x z + yz) x z + yz − z 2
(x + y + z)3 9
≥   ≥ ,
2(x y + yz + x z) 2x y + 2yz + 2x z − x 2 − y 2 − z 2 2(x + y + z)
Proofs 119

 
as x 2 + y 2 + z 2 − x y − yz − x z  0, 5 (x − y)2 + (y − z)2 + (z − x)2 ≥ 0 and

(x + y + z)2 (x + y + z)2
≥ 3, ≥ 3.
x y + yz + x z 2x y + 2yz + 2x z − x 2 − y 2 − z 2

y
x
Therefore, (y+z)(y+z−x) + (x+z)(x+z−y) z
+ (x+y)(x+y−z) ≥ 9
2(x+y+z)
.
(d) From inequality (8.2), it follows that
a3 b3 c3 d3
+ + + ≥
1 · (b + c + d) 1 · (a + c + d) 1 · (a + b + d) 1 · (a + b + c)
(a + b + c + d)3 (a − b + c − d)2 + 4(ab + bc + cd + da) 1
≥  ≥ ,
4(3a + 3b + 3c + 3d) 12 3
3 3 3 3
a
and therefore, b+c+d b
+ a+c+d c
+ a+b+d d
+ a+b+c ≥ 13 .
(e) From inequality (8.2), it follows that
3
13 x3 (1+x)3
1 + xy 2  1·1 + y·y ≥ (1+y)(1+y) , whence we obtain
    
a13 a23 an3 (1 + a1 )3 (1 + a2 )3 (1 + an )3
1+ 2 1 + 2 ··· 1 + 2 ≥ · · · ·
a2 a3 a1 (1 + a2 )2 (1 + a3 )2 (1 + a1 )2
 (1 + a1 ) (1 + a2 ) · · · (1 + an ) .

(f) From inequality (8.2), it follows that, 


13 3
n3
x1 ·(x1 +x2 )
+ · · · + xn ·(x1n +x1 ) ≥ 2(x +···+x 2 , and therefore (x 1 + · · · + x n )
2 2 1
+
n) x12 +x1 x2
 1
 
· · · + x 2 +x1 n x1 ≥ 2(x( 1+···+x )n2) ≥ n2 , since 1 n n ≥ x1 +···+x
n 3 x 2 +···+x 2 2
2 x 2 +···+x 2 n
.
n
 2
1

n
  n
n2
Therefore, x1 + · · · + xn x 2 +x x + · · · + x 2 +xn x1 ≥ 2 .
2 1 1
1 1 2 n

(g) For A  a1 a2 + a2 a3 + · · · + an−1 an + an a1 < n1 , from inequality (8.2), it


follows that
a1 a2 a an a3 a3 a3
+ + · · · + n−1 +   1  +  2  + ··· +  n  ≥
a22 + a2 a32 + a3 an2 + an a12 + a1 a1 a2 a1 a2 + a1 a2 a3 a2 a3 + a2 an a1 an a1 + an
 
a + a2 + · · · + an 3 1 1 n
≥  1   > · and therefore,
,
A A + a1 + a2 + · · · + an A(A + 1) A n+1
⎛ ⎞
  a1 a2 a an ⎠ n
a1 a2 + a2 a3 + · · · + an−1 an + an a1 ⎝ + + · · · + n−1 + ≥ .
a22 + a2 a32 + a3 an2 + an a12 + a1 n+1

For A ≥ n1 , from inequality (3), we have


     
a1 2 a2 2 an 2
a1 a2 an−1 an a2 a3 a1
+ + ··· + 2 +  + + ··· + ≥
a22 + a2 a32 + a3 an + an a12 + a1 a2 + 1 a3 + 1 a1 + 1
   2 
a1 a2 an     
a2 + a3 + a1 n2 a1 a2 an a1 a2 an
≥ ≥ , as + + ··· + ≥nn · ···  n.
n+1 n+1 a2 a3 a1 a2 a3 a1

Hence, we obtain that,


 
  a1 a2 an−1 an 1 n2 n
a1 a2 + a2 a3 + · · · + an−1 an + an a1 + + · · · + + ≥ ·  .
a22 + a2 a32 + a3 an2 + an a12 + a1 n n+1 n+1
120 8 A Useful Inequality

x15 x5 x5
8.19. Letting x1 + x2 + · · · + xn  S, + x1 +x3 +···+x
2
n
+ · · · + x1 +x2 +···+x
x2 +x3 +···+xn
n
n−1
 A,
( x12 )
3
( x22 )
3
( xn2 )
from inequality (8.2), it follows that A  x1 (S−x1 ) + x2 (S−x2 ) + · · · + xn (S−xn ) ≥
3

(x12 +x22 +···+xn2 )3  1 x3


≥ 1 , as 1 1 + 2 + · · · + n ≥
x3 x3
S·(S−x1 +S−x2 +···+S−xn ) (n−1)S 2 n(n−1) 1·x1 1·x2 1·xn
S3 2

nS
 Sn .
We have obtained A ≥ 1
n(n−1)
and for x1  x2  · · ·  xn  √1n , we have
A  n(n−1) , and therefore, the smallest possible value of the given expression
1

1
is equal to n(n−1) .
3 3 3 (x+y+z)3
8.20. From inequality (8.2), it follows that x 2x+y + y 2y+z + z 2z+x ≥ 3 x 2 +y+y 2 +z+z 2 +x .
 2 (  )
It is sufficient to prove that, 2(x + y + z) ≥ 9 x + y + z + x + y + z .
3 2 2
√ 
Letting x + y + z  d, we have√x y + yz + zx  (x y + yz + zx)2 ≥

3x y · yz + 3x y · zx + 3yz · zx  3d.
One
 needs to prove  that, 2(x + y + z)3 + 18(x y + yz + zx) ≥
9 (x + y + z)2 + x + y + z .

Let us prove that, 2d 5 + 18 3√≥ 9d 3 + 9d. √ √ √
Note that, 2d 5 −9d 3 −9d +18 3  (d − 3)2 (2d 3 +4 3d 2 +9d +6 3) ≥ 0.

Problems for Independent Study

Prove the following inequalities (1–22, 24–45, 47, 48).



n
ai2
n
1. 2 ≥
ai +ai+1
ai , where an+1  a1 , ai > 0 i  1, . . . , n.
i1 i1
n 
2
xi + x1i ≥ ( n ) , where x1 + · · · + xn  1, xi > 0, i  1, . . . , n.
2
n 2 +1
2.
i1
2 2 2 2 2 2 3 3 3
3. a + b + c ≤ a 2c +b
+ b 2a
+c
+ a 2b +c
≤ bc a
+ acb c
+ ab , where a > 0, b > 0, c > 0.
(a1 +···+an )(b1 +···+bn )
4. (a) a1 +b1 + · · · + an +bn ≤ (a1 +···+an )+(b1 +···+bn ) , where ai > 0, bi > 0, i  1, . . . , n,
a1 b1 an bn
2
n2
(b) a1 +ba11 +b1
+aa1 b1
+ · · · + an +bann +bn
+aan bn
≥ a(a1 +···+a
n
n )+2n
+ a(b1 +···+bn )+2n
,
where a > 0, ai > 0, bi > 0, i  1, . . . , n,

n
n

n ai bi
(c) √ai 2bi 2 ≤  i1 i1
2  n 2 , where ai > 0, bi > 0, i  1, . . . , n,
i1 ai +bi
n
ai bi
i1 i1
 
(d) a1 +b1
a1 +b1 +na1 b1
+ ··· + an +bn
+ n2 a1a+···+a
an +bn +nan bn
1 +···+an
n +2
+ b1 +···+bn
b1 +···+bn +2
≥ n, where ai >
0, bi > 0, i  1, . . . , n,
n 2 (c1 +···+cn )
(e) a1c·b1 1 + · · · + anc·bn n ≥ (a1 +···+a n )(b1 +···+bn )
, where ai > 0, bi > 0, ci > 0, i 
1,
 . . . , n, 
aj bj
ai
ci
− cj
bi
ci
− cj
≤ 0, i, j ∈ {1, . . . , n}.
Problems for Independent Study 121

n2
(f) √ 1
a1 b1 −c1 d1
+ ··· + √ 1
an bn −cn dn
≥  √ √ ,
(a1 +···+an )(b1 +···+bn )−( c1 d1 +···+ cn dn )
2

where ai > 0, bi > 0, ci > 0, di > 0, ai bi ≥ ci di , i  1, . . . , n.



Remark Letting ai bi − ci di  xi , we have
    2
1 1
√ + ··· + √ (a1 + · · · + an )(b1 + · · · + bn ) − c1 d1 + · · · + cn dn
a1 b1 − c1 d1 an bn − cn dn
  
     2
1 1  2 2
(a + · · · + a ) x1 + · · · + xn + (a + · · · + a ) c1 d1 + · · · + cn dn −

 + ··· + 1 n 1 n c1 d1 + · · · + cn dn .
x1 xn a1 an a1 an

5. x1
+ 3x+x
2
+· · ·+ xxnn−1 + xn ≥ n , where n  5 or n  6, xi > 0, i  1, . . . , n.
x2 +x3 2x 4
2
 +x1 x1 +x22  2
a a a
6. 1 + a12 1 + a23 · · · 1 + an1 ≥ (1 + a1 ) · · · (1 + an ), where n ≥ 2, a1 >
0, . . . , an > 0.
a2 12 a2 (1+a)2
Remark Note that 1 + b
 1
+ b
≥ 1+b
, where b > 0.

7. a
b+c
b
+ c+a c
+ a+b + ab+bc+ca
a 2 +b2 +c2
≤ 25 , where a, b, c are the side lengths of some triangle.
Remark Note that
b+c−a c+a−b a+b−c
+ + 
b+c c+a a+b
(b + c − a)2 (c + a − b)2 (a + b − c)2 (a + b + c)2
 + + ≥  2 .
(b + c)(b + c − a) (c + a)(c + a − b) (a + b)(a + b − c) 2 a + b2 + c2

8. a
b+2c+3d
+ b
c+2d+3a
c
+ d+2a+3b d
+ a+2b+3c ≥ 23 , where a > 0, b > 0, c > 0 d > 0.
n √
n xi n
9. √ xi
1+xi
≥ √
i1
n−1
, where n ≥ 2, x i > 0, i  1, . . . , n, and xi  1.
i1 i1
2 2
x1 x xn2
10. 1+x2 +x3 +···+xn
+ 1+x1 +x32+···+xn + · · · + 1+x1 +x2 +···+x n−1
≥ 3n−24
, where n ≥ 2,
xi > 0, i  1, . . . , n and x1 + · · · + xn  2.
11. x1k + · · · + xnk ≥ x1 + · · · + xn , where k ∈ N and n ≥ 2, xi > 0, i 
1, . . . , n, x1 · · · xn  1.
√ √
12. a1 + · · · + an ≤ a1 + · · · + an , where n ≥ 2, ai > 0, i  1, . . . , n and
a1 · · · an  1.
(x+y)z
13. (a) (x+y) 2
+z 2
≤ 4z+3x+3y
4z
, where x > 0, y > 0, z > 0;
(b+c−a)2 (c+a−b)2 (a+b−c)2
(b) (b+c)2 +a 2
+ (c+a)2 +b2
+ (a+b) ≥ 35 , where a > 0, b > 0, c > 0.
√ √ 
2
+c2
14. (a) √ x1√
+ √ 2√ + · · · + √ n√ ≥ 1
x x
x1 + · · · + xn ,
x1 + x2 x2 + x3 xn + x1 2
where n√≥ 3, x1 > √ 0, . . . , x n √
> 0.
(b) 4b√bc−ca √a + 4c√ca−a
b√
b
+ √
a c
4a b−b c
√ ≥ 1, where a, b, c ∈ (1, 2).

√ √
 4b(√ac−ac
ab)
2
b a
Remark Note that √ √
4b c−c a
.
(a1 +···+an )2
15. 2(a12 +···+an2 )
≤ a2a+a 1
3
+ a3a+a
2
4
+ · · · + a1a+a n
2
, where n ≥ 3, a1 > 0, . . . , an > 0.
9 9 9 9 9 9
x +y y +z
3 z 3 +z 6 + z 6 +z 3 x 3 +x 6 ≥ 2, where x > 0, y > 0, z > 0 and
z +x
16. x 6 +x 3 y 3 +y 6
+ y 6 +y
x yz  1.
122 8 A Useful Inequality

y
17. x
1+x 2
+
1+y 2
+ 1+zz 2 ≤ 3 4 3 , where x 2 + y 2 + z 2  1.
a3 b3 c3 2 2 2
18. b+2c
+ c+2a + a+2b ≥ a +b3 +c , where a > 0, b > 0, c > 0.
19. 2n
3n+1
≤ n+11 1
+ n+2 + · · · + 2n1
≤ 4(n+1)
3n+1
, where n ∈ N
 √ √ √ 2
20. a1
+ a 2 +1 + · · · + a 2 +1 ≥ 5 a1 a1 + a2 a2 + · · · + an an ,
a22 +1
a2 an 4
where n ≥
3 1
4, a1 > 0, . . . , an > 0 and a12 + · · · + an2  1.
21. a
2a+b
b
+ 2b+c c
+ 2c+a ≤ 1, where a > 0, b > 0, c > 0.

b2 c2 (b+c)2
Remark We have 1
2
− a
2a+b
+ 1
2
+ b
2b+c
 2b(2a+b)
+ 2c(2b+c)
≥ 2(2ab+(b+c)2 )
≥ c
2c+a
.

22. b2ac2 + c2ba 2 + a 2cb2 ≥ a+b+c


9
, where a > 0, b > 0, c > 0 and a 2 + b2 + c2  3abc.
23. Solve the following system of equations:

x1 + x2 + · · · + xn  n,
.
x14 + x24 + · · · + xn4  x13 + x23 + · · · + xn3

1
24. a(b+a) 1
+ b(b+c) 1
+ c(c+a) ≥ 2(a+b+c)
27
2 , where a > 0, b > 0, c > 0.

25. a b+b c+c d+d a ≥ abcd(a + b + c + d), where a > 0, b > 0, c > 0, d > 0
4 4 4 4

2 ≥ 4(ab+bc+ac) where a > 0, b > 0, c > 0.


a b c 9
26. b(b+c)2 +
c(a+c)2
+ a(a+b)

a3
Remark We have a
b(b+c)2
 ab·a(b+c)2
.
   
27. 3 x 2 − x + 1 y 2 − y + 1 z 2 − z + 1 ≥ (x yz)2 − x yz + 1.
 3
Remark Prove that 3 t 2 − t + 1 ≥ t 6 + t 3 + 1.
       
28. 2 a 2012 + 1 b2012 + 1 c2012 + 1 ≥ (1 + abc) a 2011 + 1 b2011 + 1 c2011 + 1 ,
where a > 0, b > 0, c > 0.
2 ≥ 4(a+b+c) , where a > 0, b > 0, c > 0.
a b c 9
29. (b+c)2 +
(a+c)2
+ (a+b)

a3
Remark Note that a
(b+c)2
 (ab+ac)·(ab+ac)
.

30. x 8 + y 8 + z 8 ≥ x 2 y 2 z 2 (x y + yz + zx).

 (y 2 ·z)2 + (z 2 ·x)2 + (x 2 ·y)2 .


3 3 3
x 8 +y 8 +z 8 x2 y2 z2
Remark Note that x yz  0, whence x 2 y2 z3

x3 y3 3
31. + + z ≥ 14 , where x + y + z  1 and x > 0, y > 0, > z > 0.

x+yz y+zx z+x y 
32. abc
a+b−c
abc
+ b+c−a + a+c−babc
≥ a + b + c, where a, b, c are the side lengths of
some triangle.

Remark Note that abc(a + b + c) ≥ a 3 (b + c − a) + b3 (a + c − b) + c3 (a + b − c).


√ √ √
33. 3 ab + 3 cd ≤ 3 (a + c + d)(a + c + b), where a > 0, b > 0, c > 0, d > 0.
Problems for Independent Study 123
√ √
( 3 ab+ 3 cd)3
Remark Note that ab
(a+c)b
+ cd
d(a+c)
≥ (a+c+d)(b+a+c)
.

34. (a 5 − a 2 + 3)(b5 − b2 + 3)(c5 − c2 + 3) ≥ (a + b + c)3 , where a > 0, b > 0, c > 0.

Remark Note that x 5 − x 2 + 3 ≥ x 3 + 2, where x > 0.

35. 3(a 3 + b3 + c3 ) ≥ (a 2 + b2 + c2 )3 , where a > 0, b > 0, c > 0.

Remark Note that 3(a 3 + b3 + c3 ) ≥ (a + b + c)(a 2 + b2 + c2 ).


     
a3 a3 a3
36. 1 + a12 · 1 + a22 · · · 1 + an2 ≥ (1 + a1 ) · (1 + a2 ) · · · (1 + an ), where a1 >
2 3 1

0, 3a2 > 0,. .3. , an > 0. 3   2     


37. a1 + 1 · a2 + 1 · · · an + 1 ≥ a1 a2 + 1 · a22 a3 + 1 · · · an2 a1 + 1 , where
a1 > 0, a2 > 0, . . . , an > 0.
     3
Remark We have x 3 + 1 x 3 + 1 y 3 + 1 ≥ x 2 y + 1 , where x > 0, y > 0.
(x1 +x2 +···+xn )3
38. x1
+ x2
+···+ xn
≥ 2, where 0 < x1 < 1, 0 <
1−x12 1−x22 1−xn2 (x1 +x2 +···+xn )2 −(x12 +x22 +···+xn2 )
x2 < 1, . . . , 0 < xn < 1.  n
39. n n (x1n + 1) · (x2n + 1) · · · (xnn + 1) ≥ x1 + x2 + · · · + xn + 1
x1
+ 1
x2
+ ··· + 1
xn
,
where x1 · x2 · · · xn  1
and x1 < 0, x2 < 0, . . . , xn < 0.

Remark Note that (x1n + 1) · (1 + x2n ) · · · (1 + xnn ) ≥ (x1 · 1 · · · 1 + 1 · x2 · · · xn )n 


 n
x1 + x11 .
√ √ √
40. x1 3 1 + xn − x2 + x2 · 3 1 + x1 − x3 +· · ·+ xn · 3 1 + xn−1 − x1 ≤ x1 + x2 +· · · xn ,
where x1 > 0, x2 > 0, . . . , xn > 0.
√ √ √ √
Remark Note that x1 · 3 1 + xn − x2  3 x1 · 3 x1 · 3 x1 + xn x1 − x1 x2 .
  2
41. (x12 + x22 + · · · + xn2 ) x 2 +x1 x + x 2 +x1 x + · · · + x 2 +x1 n x1 ≥ n2 ,
1 1 2 2 2 3 n
where x1 > 0, x2 > 0, . . . , xn > 0.
13 13 13 n3
Remark Note that x12 +x1 x2
+ xn2 +x2 x3
+ ··· + xn2 +xn x1
≥ 2(x1 +x2 +···+xn )2
.

x13 xn3
42. (ax1 +bx2 )(ax2 +bx1 )
+ · · · + (axn +bx1 )(ax 1 +bx n )
≥ x1(a+b) 2 , where a
+···+xn
> 0, b > 0, x1 >
0, x2 > 0, . . . , xn > 0.
a13 a3  2 3
43. b1
+ · · · + bnn ≥ 1, where ai > 0, bi > 0 (i  1, . . . , n) and a1 + · · · + an2 
b12 + · · · + bn2 .

 (a3 1 ) . + · · · + (an3 n )2 .
3 3
a13 an3 a2 a2
Remark Note that b1
+ ··· + bn 1
b1 ·b12 bn ·bn

 2  3
44. 4 x 3 + y 3 + z 3 + x yz ≥ x 2 + y 2 + z 2 + t 2 , where x > 0, y > 0, z > 0,
and t 2  maxx (x,y,z)
yz
.
124 8 A Useful Inequality
√ 2

Remark Note that x + y + z + x yz  ( x ) + ( y ) + ( z ) +


2 2 2 3
x2 y2 z2 x 2 y2 z2
3 3 3 √
3 x yz .
n
a1,1 an an
45. a1,2 ·a1,3 ···a1,n
+ 2,1
a2,2 ·a2,3 ···a2,n
+ ... + m,1
am,2 ·am,3 ···am,n

(a1,1 +a2,1 +···+am,1 ) n
, where ai, j > 0, i  1, 2,
(a1,2 +a2,2 +···+am,2 )·(a1,3 +a2,3 +···+am,3 )···(a1,n +a2,n +···+am,n )
. . . , m, j  1, 2, . . . n.

Remark Without loss of generality one can assume that a1,1 + a2,1 + · · · + am,1 
a1,2 + a2,2 + · · · + am,2  · · ·  a1,n + a2,n + · · · + am,n  1.
an
Note that ak,2 ·ak,3k,1···ak,n + ak,2 + · · · + ak,n ≥ nak,1 .

46. Find the smallest possible value of the expression a4 +2b4 +3c4 , if a +b +c  1.

Remark Note that

a4 b4 c4 (|a| + |b| + |c|)4


a 4 + 2b4 + 3c4  +   +   ≥    ≥
13 3 1
3
3 1
3  3
3 1 1
2 3 1+ 2 + 3 3

(|a + b + c|)4 6
≥    √ √ √ 3 .
 3 3
2+ 3+ 3 6
3

1 + 3 21 + 3 13

(b+c)5
+ (c+a) + (a+b)
5 5
47. a b c
≥ 32
9 (ab
+ bc + ca), where
a > 0, b > 0, c > 0 and a + b + c  1.
(b+c)5 (c+a)5
(b+c) (a+b)5
(c+a)
+ (a+b)
5 5 5
Remark Note that a
+  1·1·1·a
b
+
+ 1·1·1·b
c 1·1·1·c
≥ 32
27
.
 √  
48. k−1
2 − 1 (a1 + a2 + · · · + an ) < k 2a1k + 22 a2k + · · · + 2n ank , where
a1 > 0, . . . , an > 0 and k ≥ 3, k ∈ N.

Remark Note that


a1k a2k ank
2a1k + 22 a2k + · · · + 2n ank   √ k−1 + · · · +  k−1
√ k−1 +  k−1 √ k−1 ≥
k−1
1/ 2 1/ 2 2 1/ 2n

(a1 + a2 + · · · an )k (a1 + a2 + · · · + an )k
≥  √ k−1 >  k−1 √ k−1 .

k−1 k−1 √ k−1 k−1 √
1/ 2 + 1/ 22 1/ 2 + 1/ 22 + · · · + 1/ 2n + · · ·
Problems for Independent Study 125

49. Let G be the intersection point of the medians of triangle A1 A2 A3 , and let C be
the circumcircle of triangle A1 A2 A3 . Let the lines G A1 , G A2 , G A3 intersect
the circle C a second
 time at points B1 , B2 , B3 , respectively. Prove that G B1 +
G B2 + G B3 ≥ A1 A22 + A2 A23 + A3 A21 .

Remark See the proof of Problem 8.11.


Chapter 9
Using Derivatives and Integrals

Historical origins. Derivatives and integrals are foundational proof technique tools
in mathematics. The derivative of a function y  f (x) of a variable x is a measure
of the rate at which the value y of the function changes with respect to the change
of the variable x. In general, in the literature the following two distinct notations are
commonly used for derivatives:
1. Leibniz’s notation, introduced by German mathematician and philosopher Got-
tfried Wilhelm von Leibniz, born 1 July 1646 in Leipzig, Holy Roman Empire
(now Leipzig, Germany), died 14 November 1716 in Hanover, Holy Roman
Empire (now Hanover, Germany). According to Leibniz’s notation, the deriva-
tive of y with respect to x is denoted by ddyx , where dy represents the change in
y and d x represents the change in x.
2. Lagrange’s notation, introduced by the Italian–French mathematician Joseph
Louis Lagrange, born 25 January 1736 in Turin, Kingdom of Sardinia (now Turin,
Italy), died 10 April 1813 in Paris, France. According to Lagrange’s notation, the
derivative of a function f (x) with respect to x is denoted by f  (x), or sometimes
it is denoted by f x (x).
In this chapter, for simplicity and brevity we use Lagrange’s notation for the
derivative f  (x).
Suppose we need to prove the inequality

f (x) ≥ g(x) (9.1)

on an interval [a, b]  I or [a, +∞)  I , where the functions f (x) and g(x) are
defined and continuous on I .

Theorem 9.1 If functions f (x), g(x) are differentiable on the domain I , f (a) ≥
g(a), and on I we have that h  (x) ≥ 0, where h(x)  f (x) − g(x), then f (x) ≥ g(x)
holds for all x ∈ I .

© Springer International Publishing AG, part of Springer Nature 2018 127


H. Sedrakyan and N. Sedrakyan, Algebraic Inequalities, Problem Books
in Mathematics, https://doi.org/10.1007/978-3-319-77836-5_9
128 9 Using Derivatives and Integrals

Proof If on the domain I we have h  (x) ≥ 0, then the function h(x) on this domain
does not decrease, and consequently, h(x) ≥ h(a) at each point x of the domain I ,
that is, f (x) − g(x) ≥ f (a) − g(a) ≥ 0. Therefore, f (x) ≥ g(x).
This ends the proof.

Remark If on the domain I we have h  (x) > 0(x  a), then for x ∈ I and x > a,
one has f (x) > g(x).
Now let us consider the following examples in order to see how Theorem 9.1 can
be applied.

Example 9.1 Prove that 2x+1 > x + 2 if x ≥ 1.

Proof Let us consider the function h(x)  2x+1 − x − 2 on [1 , + ∞).


We have h(1)  1 and h  (x)  2x+1 ln 2 − 1. Note that the function y  2x is
increasing on [1 , +∞), whence h  (x) ≥ 4 ln 2 − 1 > 0.
Therefore, if x ≥ 1, then h(x) ≥ h(1) or 2x+1 ≥ x + 3, and hence 2x+1 > x + 2.
This ends the proof.


k 
k
Example 9.2 Prove that ( am an
m+n
) ≥ 0.
n1 m1


k 
k
x m am .x n an
Proof Let us consider the function h(x)  ( m+n
) on [0 , +∞).
n1 m1

k 
k
We have xh  (x) ( x m am · x n a n )  (xa1 + · · · + x k ak )2 ≥ 0, and therefore,
n1 m1
for x > 0, we have h  (x) ≥ 0.

k 
k
Hence h(1) ≥ h(0)  0, that is, ( am an
m+n
) ≥ 0.
n1 m1
This ends the proof.

Theorem 9.2 If f (b) ≥ g(b) and on the domain I one has h  (x) ≤ 0,(h(x) 
f (x) − g(x)), then on I one has f (x) ≥ g(x), where I  [a, b] or I  (−∞, b].

Proof If h  (x) ≤ 0 on I , then the function h(x) in this domain is not increasing, and
it attains its minimum value at the point x  b. On the other hand, h(b) ≥ 0, and
therefore for every x in I , one has h(x) ≥ 0. Therefore, on I , one has f (x)−g(x) ≥ 0,
or equivalently, f (x) ≥ g(x).
This ends the proof.

 Integrals are another crucial proof technique tool in mathematics. The notation
was also introduced by Gottfried Wilhelm von Leibniz in 1675. It represents the
first letter, s, of the word sum (summa in Latin). Let us provide the following useful
theorem as an application of integrals to prove inequalities.

Theorem 9.3 If for every x in the domain I one has the inequality f (x) ≥ g(x),
then
9 Using Derivatives and Integrals 129

x x
f (t)dt ≥ g(t)dt, (9.2)
a a

where I  [a, b] or I  [a, +∞).

Proof If inequality (9.1) holds, then F  (x) ≥ G  (x), where F  (x)  f (x), G  (x) 
g(x), and F(a)  G(a)  0.
x
It follows that F(x) ≥ G(x). Hence, using that F(x)  f (t)dt, G(x) 
a
x
g(t)dt, we obtain inequality (9.2).
a
This ends the proof.

Now let us consider the following example in order to see how Theorem 9.2 can
be applied.

Example 9.3 Prove that ln(2sin x) > 21 x(π − x) − 5


72
π 2, if x ∈ (π/6, π/2).

Proof Let us consider the inequality cot x > π2 − x, which is seen to be valid if we
use the fact that tan α > α for 0 < α < π2 and substitute α by π2 − x.

Integrating the inequality under consideration, we obtain


x x  π 
cot tdt > 2
− t dt, and hence
π π
6 6
    2 
ln(sin x) − ln 21 > π2 · x − x2 − π2 · π6 − 21 · π6
2
, or
ln(2sin x) > 21 x(π − x) − π .
5 2
72
This ends the proof.

Problems

Prove the following inequalities (9.1–9.22, 9.27–9.32)


9.1. 3a 3 + 7b3 ≥ 9ab2 , where a ≥ 0, b ≥ 0.
9.2. 2n−1 (x n + y n ) ≥ (x + y)n , where x > 0, y > 0, n ∈ N.
2
9.3. (a) cos x ≥ 1 − x2 ,
3
(b) sin x ≥ x − x3! , where x ≥ 0,
2 4
(c) cos x ≥ 1 − x2! + x4! ;
3 5
(d) sin x ≥ x − x3! + x5! , where x ≥ 0 (use that sin x ≤ x).

9.4. x − sin x ≤ 1 − cos x ≤ x 2 − sin x, where 0 ≤ x ≤ π2 .
9.5. tan x + sin x ≥ 2x, where 0 ≤ x < π2 .
2 3
x 2n+1 x 2n+2 2 3
x 2n+1
9.6. x − x2 + x3 − . . . + 2n+1 − 2n+2 ≤ ln(1 + x) ≤ x − x2 + x3 − . . . + 2n+1
, where
x ≥ 0,n ∈ N.
130 9 Using Derivatives and Integrals

ln(cos x) ≤ − x2 , where 0 ≤ x < π2 .


2
9.7.
9.8. sin x ≤ x(π−x) 2
, where 0 ≤ x ≤ π2 .
9.9. tan x − 3 ≤ x, where 0 ≤ x < π2 .
tan3 x

9.10. (x + x1 )arccot x > 1, where x > 0.


9.11. 1
n+1
+ n+2 1
+ · · · + 3n
1
< ln 3, where n ∈ N.
9.12. 3 cos x
1+2 cos x
< sin x
x
< 3
4−cos x
, where 0 < x ≤ π2 .
p q
9.13. Young’s inequality: ab ≤ ap + bq , where a > 0, b > 0, p > 0, q > 0 and
1
p
+ q1  1.
a −b p
> aa n−b
p n n
9.14. a p +b p +bn
, where a > b > 0, p > n.
(1 + x ) − (1 + x t )− t ≤ x, where x ≥ 0, t ≥ 2.
t 1t 1
9.15.
 ≤ e +2 b(ln b − 1), where b ≥ 1.
a
9.16. ab
ln x ≤ x x−1 , where x ≥ 1.
(ln x) 2
9.17. 2+ 3
9.18. 2sin x + 2cos x ≥ 3, where 0 ≤ x ≤ π2 .
k+1  k+1 n k+1
9.19. nk+1 < 1k + 2k + · · · + n k < 1 + n1 · , where n, k ∈ N.
 n n k+1
9.20. (a) e < n! , where n ∈ N,
 n
(b) n! < n ne , where n ≥ 7, n ∈ N.
1
9.21. (a α + bα ) α > (a β + bβ ) β , where a > 0, b > 0, 0 < α < β.
1

 a+b  a a  b b
9.22. a+b
c+d
≤ c · d , where a > 0, b > 0, c > 0, d > 0.
9.23. Find the integer part of the expression √314 + √31 + · · · + √3 1 6 .
5 10
9.24. Prove that 1
2
+ 1

3 2
+ 1

4 3
+ ··· + 1√
(n+1) n
< 2, where n ≥ 2, n ∈ N.
9.25. Prove that

(a) Bernoulli’s inequality: (1 + x)α > 1 + α x, if α > 1, x > −1, x  0,


(b) Bernoulli’s inequality: (1 + x)α < 1 + α x, if 0 < α < 1, x > −1, x  0.
(c) (S − x1 )x1 + . . . + (S − xn )xn > n − 1, where n ≥ 2, x1 > 0, . . . , xn > 0,
and x1 + . . . + xn  S.

9.26. Let f (x)  a1 sin x + a2 sin 2x + . . . + an sin nx, where a1 , · · · , an are real
numbers and n is a positive integer. We have that | f (x)| ≤ |sinx| for all real
numbers x. Prove that |a1 + 2a2 + · · · + nan | ≤ 1.
2
9.27. x √ ≥ (1 + x)ln √
2
(1 + x), where x > −1.
9.28. n+1
n + 1 < n, where n ≥ 3, n ∈ N.
n

9.29. a1 b1x +a2 b2x +· · ·+an bnx ≥ a1 +· · ·+an , where ai > 0, bi > 0, i  1, . . . , n, x >
0, and b1a1 · . . . · bnan  1.
 x+1 x+1
9.30. x x > a a+1 , where a > 0, x > a1 .
 α  1  β β 
1
a1 +···+anα α a1 +···+an β
9.31. n
≥ n
, where a1 > 0, · · · , an > 0, α ≥ β, α  0, β 
0.α ≥√β, α  0, β  0.
9.32. (a) ab < ln a−ba−ln b
< a+b 2
, where a > 0, b > 0, a  b,
(b) x+2 < ln(x + 1) < 2(x+1) , where x > 0,
2x x(x+2)

(c) x−1 ln x
< √1x , where x > 0, x  1,
Problems 131

2
(d) 2x+1 < ln(1 + x1 ) < 2x(x+1)
2x+1
, where x > 0,
(e) |x − y| ≤ |ln x − ln y| , where 0 < x ≤ 1, 0 < y ≤ 1,
(f) ln 1y < ln x−y
x−ln y
, where 0 < y < x ≤ 1,
 
(g) ln 1 + 1y < x−y ln x−ln y
, where x > 0, y > 0 and x + y ≤ 1.
2 3 2k
9.33. (a) Prove that 1 − x + x2! − x3! + · · · + (2k!) x
> 0, where k ∈ N.
(b) Prove that if a polynomial P(x) of degree n is nonnegative for x, then
P(x) + P  (x) + P  (x) + · · · + P (n) (x) ≥ 0 for all values of x.
9.34. Prove that a r − br + cr ≥ (a − b + c)r , where a ≥ b ≥ c ≥ 0 and r ≥ 1.
9.35. Prove that (1 − x1k )m + . . . + (1 − xnk )m ≥ n − 1, where x1 ≥ 0, . . . , xn ≥
0, k, m ∈ N, k ≥ m, and x1 + . . . + xn ≤ 1.
n
2
 n
9.36. Prove that ai ≤ ij
i+ j−1
· ai a j .
i1 i, j1

Proofs

9.1. Let us consider the function h(a)  3a 3 + 7b3 − 9ab2 on [0, +∞). If b > 0,
then h(0)  7b3 > 0, h  (a)  9a 2 − 9b2  9(a − b)(a + b).
Note that the function h(a) is decreasing on [0, b], and is increasing on [b, +∞),
and since h(b)  3b3 + 7b3 − 9b3  b3 > 0, we have h(a) > 0 on [0, +∞).
Therefore, 3a 3 + 7b3 > 9ab2 .
If b  0, then obviously 3a 3 + 7b3 ≥ 9ab2 .
9.2. Consider the function h(y)  2n−1 (x n + y n ) − (x + y)n on [0, +∞). If x > 0,
then h  (y)  n2n−1 y n−1 − n(x + y)n−1  n((2y)n−1 − (x + y)n−1 ), h  (y)  0
if y  x, h  (y) < 0 for 0 ≤ y < x, and h  (y) > 0 for y > x. It follows that
the function h(y) is decreasing on [0, x], and increasing on [x, +∞). On the
other hand, h(x)  0, and hence on [0, +∞) we have that h(y) ≥ 0, that is,
2n−1 (x n + y n ) ≥ (x + y)n .
9.3. (a) If x > 0, then integrating the inequality sin x ≤ x, we obtain
x x 2 2
sin t dt ≤ t dt, or −cos x + 1 ≤ x2 − 0, that is, cos x ≥ 1 − x2 .
0 0
x2
If x  0, then cos x  1  1 − 2
.
|x|2 x2
Since cos x  cos |x| ≥ 1 − 2
1− 2
, it follows that

x2
cos x ≥ 1 − . (9.3)
2
x x  t2

(b) By Theorem 9.3, for inequality (9.3) we have cos tdt ≥ 1− 2
dt,
0 0
and hence the given inequality holds.
132 9 Using Derivatives and Integrals

(c) If x > 0, then by part (b) and using to Theorem 9.3, we obtain
x x  t3

sin tdt ≥ t − 3! dt, and thus the given inequality holds. If x  0,
0 0
x2 x4
then cos x  1  1 − 2!
+ 4!
.
Since cos x  cos |x| ≥ 1 − |x| + |x|
2 4 2 4

2! 4!
 1 − x2! + x4! , it follows that
x2 x4
cos x ≥ 1 − 2! + 4! .
(d) The proof follows from part (c) and Theorem 9.3. √  
9.4. Let us evaluate sin x −cos x. We have sin x +cos x  2 sin x + π4 , and note

that for 0 ≤ x ≤ π2 , on integrating the inequality 1 ≤ sin x + cos x ≤ 2, we
x x x √
obtain 1 dt ≤ (cos t + sin t) dt ≤ 2dt, therefore x ≤ sin x −cos x +1 ≤
√ 0 0 0
x 2.
9.5. According to inequality (3.2), we have cos x + cos12 x ≥

2 cos x · cos2 x  2 cos x ≥ 2, and from Theorem 9.3, it follows that
1 1

x   x
cos t + cos12 t dt ≥ 2dt, and hence the given inequality holds.
0 0
9.6. Let us first us prove that if x ≥ 0, then
1
1 − x + x 2 − · · · + x 2n − x 2n+1 ≤ ≤ 1 − x + x 2 − · · · + x 2n . (9.4)
1+x
2n+2
Using the formula for the sum of a geometric progression, we obtain 1−x 1+x

1+x 2n+1
1
1+x
≤ 1+x , or 1 − x 2n+2
≤1≤1+x 2n+1
(x ≥ 0).
From Theorem 9.3 and inequality (9.4), it follows that the given inequality
holds.  2 
9.7. Since (ln(cos x))  − tan x, − x2  −x, from the inequality − tan x ≤
 
−x 0 ≤ x < π2 and Theorem 9.3 it follows that the given inequality holds.

9.8. Consider the function F(x)  sin x − x(π−x)
2
on 0, π2 and the function G(x) 
F  (x)  cos x + x − π2 on [0, π/2]. Since G  (x) ≥ 0, it follows that for x ≤ π2 ,
we have G(x) ≤ G(π/2), or F  (x)  cos x + x − π2 ≤ 0, and since x ≥ 0, we
have F(x) ≥ F(0)  0.
It follows that sin x − x(π−x) ≥ 0.
2 
9.9. Consider the function f (x)  x −tan x + tan3 x on 0, π2 . We have that f  (x) 
3

1 − (tan x) + tan2 x · (tan x)  tan4 x ≥ 0, and therefore, f (x) ≥ f (0)  0,
3
that is, x ≥ tan x − tan3 x .

9.10. Let us prove that for x > 0, we have arccot x > 1+x x
2.

Consider the function f (x)  arccot x − 1+x 2 on (0, +∞) .


x

We have f  (x)  − 1+x1 1−x 2


2 − (1+x 2 )2  − (1+x 2 )2 < 0, and hence the function
2

f (x) is decreasing on (0, +∞). It follows that f (x) > 0 as lim f (x)  0.
x→+∞

n+k 
n+k
9.11. We have 1
n+k
dx < 1
x
d x, k  1, 2, . . . , 2n, and hence
n+k−1 n+k−1
Proofs 133

n+1 n+2 3n


1 1 1 1 1 1
+ + ... + < dx + dx + . . . + dx
n+1 n+2 3n x x x
n n+1 3n−1
3n
1 
 d x  ln x 3n
n  ln 3.
x
n

9.12. Let us prove first that if 0 < x ≤ π2 , then sinx x > 1+2
3 cos x
cos x
, or for 0 < x < π2 ,
tan x + 2 sin x > 3x.  
Indeed, consider the function f (x)  tan x + 2 sin x − 3x on 0, π2 . Since
 π
f (0)  0 and the derivative of the function f (x) on 0, 2 is positive, we have
 x·cos x
f  (x)  cos12 x +2 cos x −3  cos12 x +cos x +cos x −3 > 3 3 coscos 2x −3  0,
since cos2 x  cos x.
1

Therefore, the function f (x) on 0, π2 is increasing, and hence f (x) > f (0).
 
The inequality sinx x < 4−cos3
x
on 0, π2 is equivalent to the inequality 4sin x −
sin x cos x < 3x In order to prove this, let us consider the function F(x) 
4sin x − sin x cos x − 3x in the given domain.
As for 0 < x < π2 , we have that

F  (x)  4 cos x − cos 2x − 3  4 cos x − 2 cos2 x − 2


 −2(cos x − 1)2 < 0.

Then on 0, π2 , the function F (x) is decreasing. It follows that F (x) <
F (0)  0.

9.13. Consider the function f (x)  x p−1


pb
+ bq−1
qx
in (0, +∞) . We have f  (x) 
q
x −b
, and therefore, f (x) > 0 for x > b p and f  (x) < 0

p−1 p−2 q−1 p q

p
· x b − bq x 2  qbx 2
q
for 0 < x <
 b p .q   q 
Hence on 0, b p , the function f (x) is decreasing, and on b p , +∞ it is
increasing.
q
Therefore, the function f (x) attains its smallest value for x  b p , that is,
 q  p−1
bp
x p−1 bq−1 bq−1 xp bq
pb
+ qx
≥ pb
+ q  1
p
+ 1
q
 1, or p
+ q
≥ xb. Thus for x  a
qb p
we obtain the given inequality.
9.14. Let us denote ab by c. In this case, we have c > 1 and aa p−b  cc p−1 , a −b 
p p p n n

+b p +1 a n +bn
c −1
. Consider the function f (x)  ccx−1 on (−∞, +∞). We have f  (x) 
n x

cn +1 +1
2c x ln c
(c x +1)2
> 0, and therefore, the function f (x) is increasing on the given domain.
Hence, if p > n, then f ( p) > f (n), that is, cc p−1 > ccn−1
p n

+1 +1
.
9.15. One can easily prove that the √
given inequality is equivalent to the following
1 2
inequality: (1 + x t ) t ≤ x+ 2x +4 . The proof follows from Problem 9.21 and

1 x+ x 2 +4
inequality (1 + x 2 ) 2 ≤ 2
.
134 9 Using Derivatives and Integrals

9.16. Consider the function f (x)  e x + b(ln b − 1) − xb in (−∞, +∞).


We have f  (x)  e x − b, and therefore, the function f (x) is increasing on
[ln b, +∞) and decreasing on (−∞, ln b]. Hence for x  ln b, the function
attains its smallest value, that is, f (x) ≥ f (ln b). Therefore, f (a) ≥ eln b +
b(ln b − 1) − b ln b  0.
9.17. Consider the function f (x)  x x−1 − 2 ln x − ln3 x in [1, +∞).
2 3

2
We have f  (x)  1 + x12 − 2x − lnx x  (x−1)
2 2

x2
− lnx x .
In order to find the sign of the function f  (x), let us consider the sign of
2
the function g(x)  (x−1) x
− ln2 x in the given domain. We have g  (x) 
x 2 −1
x 2 −1 −2ln x
x2
− 2lnx x  x 2x .
Now let us consider the function F(x)  x x−1 − 2ln x on [1, +∞).
2

2
Since F  (x)  x x+1
2
2 − 2x  (x−1)
x2
≥ 0, we have x ≥ 1, F(x) ≥ F(1)  0,
and therefore, g (x) ≥ 0, whence g(x) ≥ g(1)  0, and f  (x) ≥ 0, that is,


f (x) ≥ f (1)  0.

9.18. Consider the function f (x)  2sin x + 2cos x − 3 on 0, π4 .
We have f  (x)  2cos x · cos x ln 2(2sin x−cos x − tan x) ≥ 0, since 2sin x−cos x −
tan x ≥ 0.
Indeed, let us consider the function F(x)  sin x − cos x − log2 tan x in
0, π4 . We obtain
√ 
1 1 2 1
F  (x)  cos x + sin x −  sin(x + π/4) sin 2x −
sin x cos x ln 2 sin x cos x 2 ln 2

1 2 1
≤ ( − ) < 0,
sin xcos x 2 ln 2

and therefore, F(x) ≥ F(π/4)  0 or 2sin x−cos x − tan x ≥ 0.


It follows that f  (x) ≥ 0, and hence f (x) ≥ f (0)  0.
If π4 ≤ x ≤ π2 , then f ( π2 − x) ≥ 0, and using that f (x)  f ( π2 − x), we
deduce that f (x) ≥ 0.
9.19. We have

2 3 n+1
1 + 2 + ··· + n <
k k k k
x dx + x dx + · · · + xk dx
k

1 2 n
n+1
(n + 1)k+1 1 (n + 1)k+1 1 n k+1
 xkdx  − <  (1 + )k+1 · .
k+1 k+1 k+1 n k+1
1

In a similar way, we obtain


1 2 n k n x k+1 n n k+1
1k +2k +· · ·+n k > x k d x + x k d x +· · ·+ x dx  xkdx  k+1 0
|  k+1
.
0 1 n−1 0
Proofs 135

9.20. (a) We have


2 3 n n
ln 2 + ln 3 + · · · + ln n > ln x d x + ln x d x + · · · + ln x d x  ln x d x
1 2 n−1 1
 (x ln x − x) |n1  n ln n − n + 1 > n ln n − n,
 n n
and therefore, n! > .
e
 7 7
(b) One can easily verify that 7! < 7 e
. Letting n ≥ 8, we have
9 
n+1 
n+1
ln 2 + ln 3 + · · · + ln n < ln 7! + ln x d x + · · · + ln x d x  ln 7! + ln x d x
8 n 8
 ln 7! + (x ln x − x) |n+1
8 < (n + 1) (ln (n + 1) − 1) − 8 (ln 8 − 1) + 8 ln 7 − 7 < (n + 1) ln n − n,

 n
or n! < n ne (see Problem 3.16(b))
1
9.21. Consider the function f (x)  (a x + b x ) x on (0, +∞).
We have
x
+b x
 a x ln a x + b x ln b x − ln(a x + b x )a
1
f (x)  (a + b ) ·x x x .
x 2 (a x + b x )

Consider the function F(t)  t ln t + c ln c − (t + c)ln(t + c) on (0, c], where


c > 0.
We have F  (t)  1 + ln t − 1 − ln(t + c)  ln t+c t
< ln 1  0, and therefore,
F(t) ≤ F(c)  2c ln c − 2c ln 2c < 0.
Let a x ≥ b x . Taking c  a x , t  b x , we obtain a x ln a x + b x ln b x − (a x +
b x )ln(a x + b x ) < 0, hence f  (x) < 0, and thus f (x) is a decreasing function
on (0, +∞).
1
Therefore, if β > α > 0, then (a α + bα ) α > (a β + bβ ) β .
1

a
9.22. Consider the function f (x)  (x+d) x
a+b on (0, +∞).

We have f  (x)  (x+d)


a−1
a+b+1 · (ad − bx). Therefore, if 0 < x < , then
x ad
b
 
f (x) > 0, for otherwise, if x > b , then f (x) < 0. Hence, the function
ad
 
f (x) is increasing on 0, adb
, and decreasing on ad b
, +∞ .
Thus, it follows that at the point x  ad b
, function f (x) attains its greatest
value.     ca
We deduce that f (x) ≤ f ad b
, and therefore f (c) ≤ f ad b
, or (c+d) a+b ≤

( adb ) , and we obtain  a+b a+b ≤  a a ·  b b .


a

(d+ adb )
a+b c+d c d
136 9 Using Derivatives and Integrals

9.23. We have

4 5 106
1 1 1 1 1 1

3
+√
3
+ ··· + √
3
< √
3
dx + √
3
dx + · · · + √
3
dx
4 5 106 x x x
3 4 106 −1

106  √ 
1 3√3 3√ 3
9
 √ dx  10 − 9  14997 + 3 1 − < 14997.
3
12
3
x 2 2 2
3

In a similar way, we obtain

5 6 6 +1
10
1 1 1 1 1 1

3
+√
3
+ ··· + √
3
> √
3
dx + √
3
dx + · · · + √
3
dx
4 5 106 x x x
4 5 106

 +1
10 6

1 3 3√ 3√3 3√
 √  6 + 1)2 − > 12 −
3 3 3
d x (10 16 10 16
3
x 2 2 2 2
4

3 4 3√ 3√ 8 − 3 3 16
 10 − 16  1500 − 16  14996 + > 14996.
3 3

2 2 2 2

It follows that 14996 < √314 + · · · + √3 1 6 < 14997, and therefore, the integer
10
part of the given number is equal to 14996.
9.24. We have
1 1 1 1
+ √ + √ + ··· + √
2 3 2 4 3 (n + 1) n
1 1 1 1 1 1 1
< + √ + √ + √ + √ + √ + ··· + √
2 3 2 4 3 5 4 5 5 6 6 n n
5 n
1 1 1 1 1 1
< + √ + √ + √ + √ dx + · · · + √ dx
2 3 2 4 3 5 4 x x x x
4 n−1
n
1 1 1 1 1 1 1 1 1 2 1 1
 + √ + √ + + √ dx  + + √ + √ + 1 − √ < 1.6 + √ + √ < 2,
2 3 2 4 3 10 x x 2 10 3 2 4 3 n 3 2 4 3
4

since
√ √
1 1 2 2+ 3 2 · 1.5 + 1.8
√ + √  <  0.4.
3 2 4 3 12 12

Therefore, 1
2
+ 1

3 2
+ ··· + 1√
(n+1) n
< 2.
Proofs 137

9.25. (a) Consider the function f (x)  (1 + x)α − 1 − αx in (−1, +∞) .


Since f  (x)  α(1+ x)α−1 −α and α > 1, it follows that for −1 < x < 0
we have f  (x) < 0. It then follows that f (x) > f (0)  0, and for x > 0
we have f  (x) > 0, whence f (x) > f (0)  0.
(b) See the proof of (a).
(c) Consider the following two cases.
(1) max(x1 , . . . , xn ) ≥ 1. Let max(x1 , . . . , xn )  xi , j  i. Then we
x
have (S − x j )x j ≥ xi j ≥ 1,and therefore, (S − x1 )x1 + . . . + (S −
xn ) > n − 1.
xn

(2) max(x1 , . . . , xn ) < 1. Then from inequality (c) in Problem 9.25, it


follows that
S − xk S − xk
(S − xk )xk  ≥
(1 + S − xk − 1)1−xk 1 + (1 − xk )(S − xk − 1)
S − xk S − xk
 > , where k  1, . . . , n.
S − xk (S − xk ) S

Therefore, (S − x1 )x1 + . . . + (S − xn )xn > S−x1


S
+ ... + S−xn
S
 n − 1.

9.26. We have − |sin x| ≤ f (x) ≤ |sin x| . If 0 ≤ x ≤ π2 , then − sin x ≤ f (x) ≤


sin x, whence − sinx x ≤ f (x)
x
≤ sinx x , and therefore, − lim sinx x ≤ lim f (x)
x

x→0 x→0
limsin x
 1 (x > 0) .
x→0 x
On the other hand,
f (x) f (x) − f (0)
lim  lim  f  (0)  a1 + 2a2 + . . . + n · an .
x→0 x x→0 x −0

It follows that −1 ≤ a1 + 2a2 + . . . + n · an ≤ 1.


9.27. If x ≥ 0, then the given inequality is equivalent to the following inequality:
√x
x+1
≥ ln(1 + x). In order to prove this, let us consider the function f (x) 
√x
1+x
− ln(1 + x) in (−1, +∞) .
√ 2
Since f  (x)  2(√x+1(x+1)
x+1−1)
≥ 0, we have for x ≥ 0 that f (x) ≥ f (0)  0.
For −1 < x ≤ 0, the given inequality is equivalent to the following inequality:
f (x) ≤ f (0)  0.
9.28. Consider the function f (x)  lnx x in (0, +∞) . Since f  (x)  1−ln x2
x
, in
(e, +∞) the function f (x) is decreasing, and hence √ if n + 1 >
√ n ≥ 3, then
f (n + 1) < f (n), or ln(n+1)
n+1
< lnn n , and therefore n+1 n + 1 < n n.
9.29. Let us first prove that for x ≥ 0, one has f (x)  b x − 1 − x ln b ≥ 0, since
f  (x)  ln b(b x − 1).
If b ≥ 1, then ln b ≥ 0 and b x − 1 ≥ 0, and therefore f  (x) ≥ 0.
If 0 < b < 1, then ln b < 0 and b x − 1 ≤ 0, and therefore f  (x) ≥ 0.
Hence, we obtain that f  (x) ≥ 0, that is f (x) ≥ f (0). It follows that b x ≥
1 + x ln b, and therefore
138 9 Using Derivatives and Integrals

a1 b1x + a2 b2x + · · · + an bnx ≥ a1 (1 + x ln b1 ) + a2 (1 + x ln b2 ) + · · · + an (1 + x ln bn )


 a1 + a2 + · · · + an + x ln(b1a1 · b2a2 · · · · ·bnan )  a1 + a2 + · · · + an .

9.30. The given inequality is equivalent to the following inequality: x ln x > ln a +


x+1
(x + 1) ln a+1 .

Consider the function f (x)  x ln x − ln a − (x + 1) ln a+1
x+1
in a1 , +∞ .
As for x > a1 , we have f  (x)  ln x(a+1) > 0, and then the function f (x) in
x+1  
the given domain is increasing. Hence for x > a1 , we have f (x) > f a1  0.
a1x +···+anx
ln
9.31. Consider the function f (x)  x
n
in (−∞, 0) and (0, +∞). We have
a1x ln a1 +···+anx ln an a x +···+anx
· x − ln 1 n
a1x +···+anx
f  (x) 
x2
 
n a1x ln a1x + · · · + anx ln anx a x + · · · + anx a x + · · · + anx
   − 1 · ln 1 ≥ 0,
x 2 a1x + . . . + anx n n n

since the function f (t)  t ln t is convex in (0, +∞).


Indeed, f  (t)  ln t + 1 and f  (t)  1t > 0.
 α 1/α  β β 1/β
a +···+a α a +···+a
Therefore, if α ≥ β > 0 or 0 > α ≥ β, then 1 n n ≥ 1 n n .
 α 
α 1/α   1/α √
≥ n a1α · · · anα
a +···+a
If α > 0 > β, then 1 n n  n a1 · · · an ,
 β

β 1/β
1/β
a1 +···+an β β √
n
≤ n
a 1 · · · a n  n a1 · · · an , and therefore

1/α  β β
1/β
a1α + · · · + anα a1 + · · · + an
≥ .
n n

9.32. (a) Without loss of generality one can assume that a ≥ b.


In this case, the given inequalities are equivalent to the following inequal-
ities:
   
2 ab − 1 a a b
a < ln < − .
b
+ 1 b b a

Consider the function f (x)  ln x − 2 · x−1


x+1
in [1, +∞) .
 (x−1)2
Note that f (x)  x(x+1)2 > 0 (x > 1), and therefore, the function f (x)
is increasing on [1, +∞) and for x > 1, we have f (x) > f (1)  0.
Taking x  ab , we obtain (ab +1 ) < ln ab . In order to prove the second
2 a −1
 b
inequality, let us define ab  x and consider the function g(x) 
2 ln x − x + x1 in [1, +∞)
Proofs 139

2
We have g  (x)  − (x−1)x2
< 0 (x > 1), and therefore, the function g(x)
is decreasing on [1, +∞), and then for x > 1, we have g(x) < g(1)  0,
a b
that is, ln b < b − a .
a

(b) Let us take a  x + 1 and b  1. From Problem 9.32(a), it follows that


2x
< ln(x + 1) < √x+1 x
. Note that √x+1
x
< x(x+2) .
x+2
√ 2(x+1)

(c) Taking a  x, b  1 and using that ab < ln a−b a−ln b
, we obtain x <
x−1
ln x
, and hence x−1
ln x
< √1x .
(d) Taking
√ a  x + 1, b  x and using Problem 9.32(a), we obtain
x(x + 1) < ln 1x+1 < x+(x+1)
2
, and therefore 2x+1
2
< ln x+1
x
< √x(x+1)
1
.
x
Note that √ 1
x(x+1)
< 2x(x+1)
2x+1
.
(e) Using that ln a−ln b < 2 for the numbers x, y and ln a−b
a−b a+b
> 0, we
  a−ln b
 x−y 
see that  ln x−ln y  < 2 (x  y), or |ln x − ln y| > x+y · |x − y| ≥
x+y 2

|x − y| , since 0 < x, y ≤ 1, and for x  y, we obtain that the given


inequality holds.
(f) Let I  (0, 1], x1  y, x2  x, x3  1, and f (x)  − ln x. Then
from Problem 7.12(a), it follows that x ln y − ln y ≥ y ln x − ln x >
y ln y − ln x.   x−y
(g) From Problem 9.25(b), it follows that 1 + 1
< 1 + x−y  xy ; hence
  y y

we obtain ln 1 + 1y < ln x−y


x−ln y
.
9.33. (a) Let us define

x2 x 2k
pk (x)  1 − x + − ... + , k  0, 1, 2, . . . .
2! (2k)!

If x ≤ 0, then pk (x) > 0. Now let us prove that if x > 0, then pk (x) −
e−x > 0. For k  0, we have p0 (x) − e−x  1 − e−x > 0.
Assume that for k  n (if x > 0), we have pn (x) − e−x > 0, and let us
prove that it holds for k  n + 1. (That is, the following inequality holds:
pn+1 (x) − e−x > 0.)
Indeed, let f (x)  pn+1 (x) − e−x . Then for x > 0 we have f  (x) 
pn (x) − e−x > 0, and therefore, f  (x) > f  (0)  0, whence for x > 0
it follows that f (x) > f (0)  0.
Therefore, we have obtained pn+1 (x) > e−x if x > 0, and hence for
x > 0 it follows that pk (x) > 0.
(b) Since for every value of x we have p(x) ≥ 0, it follows that n is an even
number. On the other hand, the polynomial F(x)  P(x) + P  (x) + · · · +
P (n) (x) has degree equal to n.
So the polynomial F(x) has the smallest value.
Let min F(x)  F(x0 ), in which case F  (x0 )  0. Thus, it follows
(−∞,+∞)
that F  (x0 )  P  (x0 ) + P  (x0 ) + · · · + P (n) (x0 ) + P (n+1) (x0 )  P  (x0 ) +
P  (x0 ) + · · · + P (n) (x0 )  0, and F(x0 )  P(x0 ) + P  (x0 ) + P  (x0 ) +
· · · + P (n) (x0 )  P(x0 ) ≥ 0. Therefore, F(x) ≥ F(x0 ) ≥ 0, and hence
F(x) ≥ 0 for all values of x.
140 9 Using Derivatives and Integrals

x 2n x 2n 2n−1
Taking P(x)  (2n)!
, we obtain (2n)!
x
+ (2n−1)! + · · · + 1 ≥ 0 (see Problem
9.33(a)).
9.34. Consider the function f (x)  a r − br + x r − (a − b + x)r , in [0, b] .
Then f  (x)  r x r −1 − r (a − b + x)r −1  r x r −1 (1 − (1 + a−b
x
)r −1 ) ≤ 0, since
a−b
x
≥ 0 and r − 1 ≥ 0.
Hence, in [0, b] the function f (x) is nonincreasing, and hence f (x) ≥ f (b) 
0, that is, f (c) ≥ 0 or a r − br + cr ≥ (a − b + c)r .
9.35. Note that for 0 ≤ x ≤ 1 we have 0 ≤ x k ≤ x m ≤ 1, and then (1 − x k )m ≥
(1−x m )m , and so it is sufficient to prove that (1−x1m )m +. . .+(1−xnm )m ≥ n−1.
Let n ≥ 3 and x1 ≤ . . . ≤ xn . Then x1 + 2x2 ≤ 1. Let us prove that

(1 − x1m )m + (1 − x2m )m ≥ 1 + (1 − (x1 + x2 )m )m .

Consider the function f (x)  (1 − x m )m − (1 − (x + x2 )m )m in [0, h] , where


h  min(x2 , 1 − 2x2) (if h  0, then x1  0, and hence (1) holds). 
Since f  (x)  m 2 (x + x2 − (x + x2 )m+1 )m−1 − (x − x m+1 )m−1 and x2 ≥
x2 ((x + x2 ) + x2 )m ≥ x2 ((x + x2 )m + . . . + x2m ) ≥ x2 ((x + x2 )m + . . . + x m ), we
have x + x2 − (x + x2 )m+1 ≥ x − x m+1 .
It follows that f  (x) ≥ 0, and hence f (x1 ) ≥ f (0).
Therefore, (1− x1m )m −(1−(x1 + x2 )m )m ≥ 1−(1− x2m )m . From inequality (1)
it follows that it is enough to prove that (1 − x1m )m + . . . + (1 − xnm )m ≥ n − 1,
for n  2.
Let 0 ≤ x1 ≤ x2 and x1 + x2 ≤ 1. Let us prove that (1 − x1m )m + (1 − x2m )m ≥ 1.
We have (1−x1m )m +(1−x2m )m ≥ (1−x1m )m +(1−(1−x1 )m )m and 0 ≤ x1 ≤ 21 .

Consider the function g(x)  (1 − x m )m + (1 − (1 − x)m )m in 0, 21 .
 
Since g  (x)  m 2 (1 − x − (1 − x)m+1 )m−1 − (x − x m+1 )m−1 and 1 − 2x 
(1−2x)(1−x +x)m ≥ (1−2x)((1−x)m +. . .+x m ), we have 1−x −(1−x)m+1 ≥
x − x m+1 .
Hence g  (x) ≥ 0, and therefore, g(x1 ) ≥ g(0), that is, (1 − x1m )m + (1 − (1 −
x1 )m )m ≥ 1.
It follows that (1 − x1m )m + (1 − x2m )m ≥ 1.
9.36. If we set ai  bii , i  1, . . . , n, then we need to prove that p(1) ≥ 0, where
n

n
bi b j  bi i−1 2
p(x)  i+ j−1
· x i+ j−1
− x i
· x .
i, j1 i1
We have

2 ⎞

n


n ⎜
bi
i
· xi ⎟
⎜ ⎟
p  (x) 
i1
bi b j x i+ j−2 − ⎜ ⎟
⎝ x ⎠
i, j1
Proofs 141



2

n 
n 
n


n 2 bi
i
· x i
bi · x i−1
x− bi
i
· x i
i1 i1 i1
 bi b j x i+ j−2 −
i, j1
x2
⎛
n 
n ⎞2
bi · x i − bi
· xi  n
2
⎜ i1 i ⎟  b
⎜ ⎟ 
i1 i
⎝ ⎠ bi − · x i−1
,
x i1
i

for x  0.
Therefore, for x > 0, we have p  (x) ≥ 0, and hence p(x) is a nondecreasing
function on [0, +∞) . It follows that p(1) ≥ p(0)  0.

Problems for Independent Study

Prove the following inequalities (1–13, 16–20).

1. 2 sin x ≥ π2 + (x − π2 ) cos x, where 0 ≤ x ≤ π2 .


2. x p−1 > x q−1 , where p > q > 0.
p q

√ √ √ √
3. 1 + 2 + · · · + n > 23 n n, where n ∈ N.
4. e x ≥ x e , where x > 0.
5. sinα α > 1 − α6 , where 0 < α < π2 .
2

6. 2 sin α + tan α > 3α, where 0 < α < π2 .


7. sin x + sin22x + sin33x > 0, where 0 < x < π .
 b+1  a b
8. a+1
b+1
≥ b , where a > 0, b > 0.
−ea
< e 2c−1 (b + a) + 1, where 0 ≤ a < b ≤ c.
b c
9. eb−a
10. (a) sinα α > sinβ β , where 0 < α < β < π2 ;
(b) tanα α < tanβ β , where 0 < α < β < π2 .
11. π
2
x < sin x < x, where 0 < x < π/2.
12. (sin x)−2 ≤ x −2 + 1 − π42 , where 0 < x < π2 .
(a) tan x > x + x3 , where 0 < x < π2 ,
3
13.
(b) x cos x < 0, 6, where 0 < x < π2 ,
 3
(c) sinx x ≥ cos x, where 0 < x < π2 .
14. Let pn and qn be the respective perimeters of regular n-gons inscribed in and
circumscribed about a circle with radius 21 . Let us divide the interval ( pn , qn )
into three equal parts. To which part does the number π belong?
15. Find real values of x such that a x ≥ x a , where x ≥ 0 and a ≥ 1.
16. a+b
(a) a+b+2ab a+c
+ a+c+2 ≥ 2bc+3b+c+2
2(b+1)(c+1)
, where a > 0, b > 0, c > 0;
a+b c+d a+c
(b) a+b+2ab + c+d+2cd + a+c+2 b+d
+ b+d+2 ≥ 2, where a > 0, b > 0, c > 0, d > 0.
17. x p + x − p + 2 p ≤ (x + x −1 ) p + 2, where x > 0 and p ≥ 2.
142 9 Using Derivatives and Integrals

3(x 2 −1)
18. (a) ln x > , where x > 1,
 √
x 2 +4x+1 
(b) a−b
ln a−ln b
< 3 2 ab + a+b
1
2
, where a > 0, b > 0, a  b.
2 2 π
19. (cos x) > (sin x)
cos x
, where 0 < x <
sin x
4
.

Remark According to the Bernoulli’s inequality (see Problem 9.25(a)), we have


cos2 x
cos2 x
(cos x) sin2 x > 1 + (cos x − 1) · sin2 x
.
 p p  q
1
q
1
20. a1 + . . . + an p b1 + . . . + bn q ≥ a1 b1 + . . . + an bn , where ai > 0, bi >
0, i  1, . . . , n, p > 0, q > 0 and 1p + q1  1.
p p
Remark Use the inequality of Problem 9.13 with the condition a1 + . . . + an  1
q q
and b1 + . . . + bn  1.
Chapter 10
Using Functions

Consider functions f : Rn → R and g : Rn → R. Assume that we need to prove


the inequality f (x1 , x2 , . . . , xn ) ≥ g(x1 , x2 , . . . , xn ) for some values of variables
x1 , x2 , . . . , xn ∈ R.
Rewriting this inequality as f (x1 , x2 , . . . , xn ) − g(x1 , x2 , . . . , xn ) ≥ 0, we study
the dependence on xi (1 ≤ i ≤ n) of the function F(x)  f (x1 , x2 , . . . , xn ) −
g(x1 , x2 , . . . , xn ), where the variables x1 , x2 , . . . , xi−1 , xi+1 , . . . , xn are considered
constants. Let us consider the following example.

Example 10.1 Prove that


  π
x12 + x22 + · · · + xn2 cos ≥ x1 x2 + x2 x3 + · · · + xn−1 xn ,
n +1
where n ≥ 2.

Proof Let us consider the following function:


π   π
F(x1 )  cos x12 − x2 x1 + x22 + · · · + xn2 cos − x2 x3 − · · · − xn−1 xn .
n +1 n +1

This is a quadratic expression in x1 . Note that the quadratic expression ax 2 +bx +c


for a > 0 attains its smallest value at the point x  − 2ab
. Hence since cos n π+ 1 > 0, it
follows that F(x1 ) attains its smallest value at the point x1  2 cosx2 π , and therefore,
n+1
 
π x2
F(x1 )  (x1 + x2 + · · · + xn ) cos
2 2 2
− x1 x2 − · · · − xn−1 xn ≥ F .
n +1 2 cos n π+ 1

One can easily prove that


   

x2 sin n+1 π
F  x22 + x32 + ··· + xn2 cos − x2 x3 − · · · − xn−1 xn .
2 cos n π+ 1 2 sin 2π
n+1 cos π
n+1
n +1

© Springer International Publishing AG, part of Springer Nature 2018 143


H. Sedrakyan and N. Sedrakyan, Algebraic Inequalities, Problem Books
in Mathematics, https://doi.org/10.1007/978-3-319-77836-5_10
144 10 Using Functions

Let us consider the following quadratic function: G(x2 )  F x2
2 cos n π+ 1
. We obtain
   
2π 4π
sin n+1 sin n+1 π
G(x2 ) ≥ G 3π
x3  3π π
x32 + · · · + xn2 cos − x3 x4 − · · · − xn−1 xn .
sin n+1 2 sin n+1 cos n+1
n +1

In a similar way, for variables x3 , . . . , xn , we obtain F(x1 ) ≥ G(x2 ) ≥ · · · ≥


sin (n+1)π
2 sin nnπ
n+1
π xn
2
cos n π+ 1  0, and therefore, (x12 + x22 + · · · + xn2 ) cos n π+ 1 ≥ x1 x2 +
+ 1 cos n + 1
· · · + xn−1 xn .
One can use the following properties of functions in proving a large number of
inequalities.

Property 1 If a function f (x) is defined on [a, b] and is decreasing on [a, c] and


increasing on [c, b], then on [d, e], then the function f (x) attains its maximum value
at one of the endpoints of [d, e] (a ≤ d < e ≤ b).

Property 2 If the function f (x) is defined on [a, b] and is increasing on [a, c], and
is decreasing on [c, b], then on [d, e], the function f (x) attains its smallest value at
one of the endpoints [d, e] (a ≤ d < e ≤ b).

One can easily note that if the function f  (x) is increasing on [a, b], then the
function f (x) attains its greatest value on [a, b] at the point a or b, and if f  (x) is
decreasing on [a, b], then f (x) attains its greatest value on [a, b] at the point a or b.
These properties of functions can be used to prove a large number of inequalities.

Example 10.2 Prove that a 2 + b2 + c2 ≤ a 2 b + b2 c + c2 a + 1, where 0 ≤ a ≤ 1,


0 ≤ b ≤ 1, 0 ≤ c ≤ 1.

Proof Consider the function f (a)  a 2 (1 − b) − c2 a + b2 + c2 − bc2 − 1 in [0, 1].

If b  1, then f (a) is a quadratic trinomial in a, and the branches of its graph


are directed upward. Hence, the function f (a) attains its greatest value at oneof the
endpoints in [0, 1]. Since f (0)  b2 + c2 − b2 c − 1  (1 − c) b2 − (1 + c) ≤ 0,
f (1)  1 − b − c2 + b2 + c2 − b2 c − 1  b(b − 1) − b2 c ≤ 0, it follows that f (a) ≥ 0
on [0, 1].
This ends the proof.
If b  1, then the proof can be done similarly.

Example 10.3 Prove that x1 + x2 + x3 − x1 x2 − x2 x3 − x3 x1 ≤ 1, where 0 ≤ xi ≤ 1,


i  1, 2, 3.

Proof Let us consider the following monotonic function f (x)  x + x2 + x3 − x x2 −


x2 x3 − x x3  x(1 − x2 − x3 ) + x2 + x3 − x3 x2 , which attains its greatest value at one
of the endpoints [0, 1]. We have f (0)  x2 + x3 − x2 x3  1 + (1 − x3 )(x2 − 1) ≤
1, f (1)  1 − x3 − x2 + x2 + x3 − x2 x3  1 − x2 x3 ≤ 1. Therefore, in [0, 1] one has
f (x) ≤ 1 or f (x1 )  x1 + x2 + x3 − x1 x2 − x2 x3 − x3 x1 ≤ 1.
This ends the proof.
Problems 145

Problems

Prove the following inequalities (10.2–10.14).



10.1. Prove that, if


ax 2 + bx + c
≤ 1 holds for all numbers
x belonging
to [−1,
1], then for those x one has the following inequality:
cx 2 − bx + a
≤ 2.
10.2. (a + b + c)2 < 4(ab + bc + ac), where a, b, and c are the side lengths of some
triangle.
10.3. b + ac + 1 + c + ab + 1 + a + bc + 1 + (1 − a)(1 − b)(1 − c) ≤ 1, where 0 ≤ a ≤ 1,
0 ≤ b ≤ 1, 0 ≤ c ≤ 1.  
10.4. Schweitzer’s inequality: (a + b + c + d + e) a1 + b1 + 1c + d1 + 1e ≤ 25 +
 2
p
6 q
− qp , where 0 < p ≤ a, b, c, d, e ≤ q.
n  n

n
10.5. (a) Chebyshev’s inequality: m i ai ( m i bi ) ≤ m i ai bi , where a1 ≤
i1 i1 i1

n
a2 ≤ · · · ≤ an , b1 ≤ b2 ≤ · · · ≤ bn , m i  1, m i > 0(i  1, . . . , n).
n  n  i1
n
(b) m i ai m i bi ≥ m i ai bi , where a1 ≤ a2 ≤ · · · ≤ an , b1 ≥
i1 i1 i1

n
b2 ≥ · · · ≥ bn , m i  1, m i > 0(i  1, . . . , n).
i1
10.6. (1 + a1 )(1 + a2 ) · · · (1 + an ) ≥ 1 + a1 + a2 + · · · + an , where the numbers
ai > −1, i  1, . . . , n, and the numbers a1 , . . . , an have the same sign.
10.7. (x1 + · · · + xn ) x11 + · · · + x1n ≤ (a+b)
2

4ab
· n 2 , where x1 , . . . , xn ∈ [a, b], 0 <
a < b.
10.8. (1 − a)(1
ab
− b)
≥ (1 − c(a
c)(1 − a − b + c)
+ b − c)
, where 0 < a ≤ c ≤ b, a + b < 1.
10.9. 1 +1 a + 1 +1 b ≥ 1 +1 m + 1 +1 ab , where 1 < a < m < b.
m 
10.10. abc ≤ 18 ( pa + qb + r c), where a, b, c > 0, p, q, r ∈ 0; 21 , a + b + c 
p + q + r  1.
10.11. ai + a j > ak , where n ≥ 3, i < j < k, 0 < a1 < . . . < an and
 2 2  
a1 + · · · + an2 > (n − 1) a14 + · · · + an4 .
10.12.
x +y+z ≥
x y+yz+zx,

where
x
≥ 0, y ≥ 0,
z ≥
0 and x y+yz+zx
+x yz ≤ 4.
10.13.
b2 − 4ac

B 2 − 4 AC
, if
ax 2 + bx + c

Ax 2 + Bx + C
, for all real

values of x.
10.14. 1 + √23 (x 2 − x + 1)(y 2 − y + 1)(z 2 − z + 1) ≥ (x yz)2 − x yz + 1.
10.15. (a) (1 − x1 · · · xn )λ + (1 − y1λ ) · · · (1 − ynλ ) ≥ 1, where λ > 1, 0 < xi <
1, 0 < yi < 1, xi + yi  1, i  1, . . . , n.
(b) (1− p n )m +(1−q m )n ≥ 1, where p+q  1, p > 0, q > 0, m, n ∈ N.
146 10 Using Functions

Proofs

10.1. We have |cx 2 − bx + a|≤ |cx 2 − c|+|c − bx + a| |c|(1 − x 2 ) + |c − bx + a|≤


|c|+|c − bx + a|.
If x  0, then from
2|ax + bx
+ c|≤ 1 it follows that |c|≤ 1.
2

We deduce that
cx − bx + a
≤ 1 + |c − bx + a|.
Note that the function f (x)  |c − bx + a| in [−1, 1] attains its greatest
value at one of the endpoints. Therefore,
f (x) ≤ max(
f (−1), f (1)) 
max(|c + b + a|, |c − b + a|) ≤ 1, whence
cx 2 − bx + a
≤ 1 + 1  2.
10.2. Since a, b and c are the side lengths of some triangle, we have |b − c| < a <
b + c. Without loss of generality, one can assume that b ≥ c.
Consider the function f (a)  (a + b + c)2 − 4(ab + bc + ac) in [b − c; b + c].
We have f (a)  a 2 −2a(b + c) +b2 +c2 −2bc. The graph of this function is a
parabola with branches pointing upward. Consequently, in the given domain,
f (a) attains its largest value at one of the points b−c, b+c, that is, f (b − c) 
4c(c − b) ≤ 0, f (b + c)  −4bc < 0. Hence in (b − c, b + c) we have
f (a) < 0.
10.3. Consider the function f (a)  b + ac + 1 + c + ab + 1 + a + bc + 1 +(1 − a)(1 − b)(1 − c)
in [0, 1]. Obviously, the derivative is increasing:
1 b c
f  (a)  − − − (1 − b)(1 − c)
b + c + 1 (c + a + 1)2 (a + b + 1)2

in [0, 1], and therefore, it satisfies Property 1 above, that is, f (a) attains its
greatest value at one of the points 0, 1.
Substituting a by 0 or 1 and considering the obtained function as a function
depending on b, we obtain that it also attains its greatest value at one of the
points b  0, b  1.
Since the given inequality is symmetric with respect to a, b, and c, we obtain
that the expression in the left-hand side attains its greatest value at one of the
following triples: (0, 0, 0), (0, 0, 1), (0, 1, 1), (1, 1, 1). Note that in all these
cases, the left-hand side is equal to 1.
Alternative proof According to Problem 1.26, we have
a b c b+c
+ + + (1 − a)(1 − b)(1 − c) ≤ a − a +
1+b+c 1+a+c 1+a+b 2
abc a + c abc a + b abc
+ +b−b + +c−c + + (1 − a)(1 − b)(1 − c)  1.
3 2 3 2 3
 
10.4. Consider the function f (a)  (a + b + c + d + e) a1 + b1 + 1c + d1 + 1e , or
f (a)  Aa + Ba + AB + 1, where A  b1 + 1c + d1 + 1e , B  b + c + d + e.
Since f  (a)  A − aB2 (B > 0), the function f (a) is increasing in [ p, q], and
therefore, it attains its greatest value at one of the points p, q.
It follows that f (a) ≤ max( f ( p), f (q)).
Proofs 147

Consider the function g(b)  ( p + b + c + d + e) 1p + b1 + 1c + d1 + 1e or

h(b)  (q + b + c + d + e) q1 + b1 + 1c + d1 + 1e in [ p, q].
In a similar way we obtain that g(b) ≤ max(g( p), g(q)) and h(b) ≤
max(h( p), h(q)).
Continuing these arguments with respect to variables c, d, e, we get
that the expression attains its greatest value when some of the vari-
ables are equal to p and the others are equal  to q. Therefore,
 
(a + b + c + d + e) a1 + b1 + 1c + d1 + 1e ≤ (mp + nq) m 1p + n q1 , where m ≥
0, n ≥ 0, m + n  5, m, n ∈ Z.   2
Now let us prove that (mp + nq) mp + qn ≤ 25 + 6 p
q
− q
p
.
  2
mnp mnq p q
m 2 + n2 + + ≤ 25 − ,
q p q p
   
p q p q
(m + n)2 + mn + − 2 ≤ 25 + 6 + −2 ,
q p q p
   
p q p q
25 + mn + − 2 ≤ 25 + 6 + −2 ,
q p q p

since one can easily note that mn ≤ 6.

Remark Problem 10.4 can be generalized in the following way.


Prove that if 0 < p < a1 , a2 , . . . , an < q, then
  2
(a1 + · · · + an ) a11 + · · · + a1n ≤ n 2 + ψ(n) p
q
− q
p
(see Problem
10.7), where
⎧ 2
⎨ n , n  2k(k  0, 1, 2, . . .),
4
ψ(n) 
⎩ n 2 −1 , n  2k + 1(k  0, 1, 2, . . .)
4

10.5. (a) Consider the function


⎛ ⎞
⎜ ⎟
⎜ ⎟ n 
⎜  n ⎟   n
⎜ ⎟
f (x)  ⎜m 2 + x m i ai ⎟ m i bi − m i ai bi − m 2 xb2
⎜ ⎟
⎜ i 1 ⎟ i1
i 1
⎝ ⎠
i  2 i  2

in [a1 , a3 ]. Since f (x) is a linear function, it attains its greatest value at


one of the points a1 , a3 . In the expression M  f (a2 ), substituting a2 by
a1 or a3 and in the obtained expression substituting a3 by x, in a similar
148 10 Using Functions

way, one can prove that this expression attains its greatest value at one
of the points a1 , a4 .
Continuing these assumptions for the numbers
a4 , . . . , an−1 , b2 , . . . , bn−1 , we obtain M ≤ (αa1 + βan )(γ b1 + δbn ) −
ka1 b1 − (α − k)a1 bn − (δ − α + k)bn an − (β + α − δ − k)an b1 , where
α is the sum of the m i whose corresponding ai were substituted by a1 ,
and γ is the sum of the m i whose corresponding bi were substituted by
b1 and α + β  1, γ + δ  1, k  min(α, γ ).
We have
M ≤ αγ a1 b1 + (αδ − α)a1 bn + (βγ − β − α + δ)an b1 + (βδ − δ + α)an bn −
  
− ka1 b1 + ka1 bn + kan b1 − kan bn  αγ a1 b1 − αγ a1 bn − αγ an b1 + αγ an bn − k a1 − an b1 − bn 
  
 (αγ − k) a1 − an b1 − bn ≤ 0,

as k  α or k  γ and 0 ≤ α ≤ 1, 0 ≤ γ ≤ 1.
(b) Let us rewrite (a) for numbers a1 ≤ a2 ≤ · · · ≤ an and −b1 ≤ −b2 ≤
· · · ≤ −bn .   n 

n
n
It follows that m i ai m i (−bi ) ≤ m i ai (−bi ), and on mul-
i1 i1 i1
tiplying both sides by −1, we obtain the given inequality.
10.6. If a1 > 0, . . . , an > 0, then in this case it is sufficient to expand the left-hand
side of the inequality.
If −2 ≤ a1 < 0, . . . , −2 ≤ an < 0, then let us consider the function
f (x)  (1 + x)(1 + a2 ) · · · (1 + an ) − 1 − x − a2 − · · · − an in [−2, 0]. Since
it is a linear function, it attains its smallest value at one of the points −2, 0.
One can easily prove that the expression (1 + a1 ) · · · (1 + an ) − 1 − a1 −
· · · − an attains its smallest value if some of the numbers are equal to −2 and
the others are equal to 0.
Hence, if a1  · · ·  an  0, then we have (1 + a1 ) · · · (1 + an ) − 1 − a1 −
· · · − an  1 − 1  0.
If some of the numbers are equal to −2, then (1 + a1 ) · · · (1 + an ) − 1 − a1 −
· · · − an  (−1)k − 1 + 2k ≥ 0, where k is thenumber of −2’s.
10.7. Consider the function f (x)  (x + x2 + · · · + xn ) x1 + x12 + · · · + x1n in [a, b].
Note that one can rewrite this function in the following way: f (x)  1 + Ax +
Bx + AB, where A  x2 + · · · + xn , B  x12 + · · · + x1n .
Since f  (x)  B − xA2 (B > 0), it follows that f  (x) is increasing in [a, b],
and therefore it attains its greatest value at one of the points a, b.
It follows that f (x1 ) ≤ max( f (a), f (b)).
In a similar way, let us consider the following function:
g(x)  (b + x + x3 + · · · + xn ) b1 + x1 + x13 + · · · + x1n , or g(x)  (a +

x + x3 + · · · + xn ) a1 + x1 + x13 + · · · + x1n in [a, b]. We obtain g(x2 ) ≤
max(g(a), g(b)).
Proofs 149

Repeating thesame for the variables


x3 , . . . , xn , we obtain that the expression
(x1 + · · · +xn ) x1 + · · · + xn attains its greatest value if some of the variables
1 1

equal a and the others equal b.


Let us take x1  · · ·  xk  a, xk+1  · · ·  xn b.
It is sufficient to prove that (ka + (n − k)b) ak + n −b k ≤ (a4ab + b)2
· n2.
This inequality holds because ((n − 2k)a − (n − 2k)b) ≥ 0. 2

10.8. Consider the function f (x)  (1 − x(a x)(1 − a − b + x)


in [a, b].
 1+ b − x)  1 
One can easily prove that f (x)  a + b − 1 x + a + b1 − x + 1.
 1 
We have f  (x)  (2xx−2 (a(a++bb))(a
− x)2
+ b)
− 1 , and hence the function f (x) is
a+b a+b
a+b 
decreasing on a, 2 and increasing in 2 , b . It follows that the function
f (x) attains its greatest value on [a, b] at one of the points a, b.
Since f (a)  f (b)  (1 − a)(1 ab
− b)
, we have f (x) ≤ (1 − a)(1 ab
− b)
, where by
taking x = c, we obtain the given inequality.
10.9. Consider the function f (x)  1 +1 x + 1 +1 ab  1 +1 x + 1 − x ab + ab
in [a, b].
√ √ x
(ab − 1)(x − ab)(x + ab)
Since f  (x)  , we see that the function f (x) is
 √ (x+ 1) (x + ab) √ 
2 2

decreasing on a, ab and increasing on ab, b . Therefore, f (x) ≤


max( f (a), f (b)).
Since f (a)  f (b)  1 +1 a + 1 +1 b and a ≤ m ≤ b, we have f (m) ≤ 1 +1 a + 1 +1 b .
10.10. Note that 21 −q ≤ p ≤ 21 , and so for the function f ( p)  pa+qb+(1− p−q)c,
we have
         
1 1 1 1 1 1
f ( p) ≥ min f −q , f  min − q a + qb + c, a + qb + −q c
2 2 2 2 2 2
 
1 1 1 1 1 1
≥ min a + c, b + c, a + b .
2 2 2 2 2 2
 
Let min 21 a + 21 c, 21 b + 21 c, 21 a + 21 b  21 (a + c). Therefore,
1
pa + qb + r c ≥ (a + c) ≥ (a + c)2(a + c)(1 − (a + c))  2(a + c)2 b ≥ 8abc.
2
10.11. Letting m ∈ {1, . . . , n}, we see that the given inequality is equivalent to the
following inequality:
  
4 − 2a 2 a 2 + · · · + a 2 − a 2 + (n − 1) a 4 + · · · + a 4 − a 4 − a 2 + · · · + a 2 − a 2 2 < 0,
(n − 2)am m 1 n m 1 n m 1 n m

which means that the quadratic function


  
2 + (n − 1) a 4 + · · · + a 4 − a 4 − a 2 + · · · + a 2 − a 2 2
(n − 2)x 2 − 2x a12 + · · · + an2 − am 1 n m 1 n m

for x  am2 attains a negative value, whence D > 0, and so we have


 2 2  
a1 + · · · + an2 − am2 > (n − 2) a14 + · · · + an4 − am4 .
150 10 Using Functions
 2 
Repeating the same process, we obtain ai2 + a 2j + ak2 > 2 ai4 + a 4j + ak4 ,
or (ai + a j + ak )(ai + a j − ak )(ai − a j + ak )(−ai + a j + ak ) > 0, and therefore,
ai + a j > ak .
√ √ √
10.12. If max(x y, yz, zx) ≤ 1, then x + y + z ≥ x y + yz + zx ≥ x y + yz + zx,
and therefore, x + y + z ≥ x y + yz + zx.
If x y > 1, let us consider the following two cases.
(a) If x + y ≥ 4, then x + y + z ≥ 4 ≥ x y + yz + zx.
(b) If x + y < 4, then z ≤ x 4+−y +x yx y , and one needs to prove that z ≤ x + y − xy
x + y −1
.

Prove that x 4+−y +x yx y ≤ xx++yy−−x1y , or f (b)  b2 − (a − 1)b + 4a − 4 − a 2 ≤ 0,


where a  x + y, b  x y.   2
2
Since 1 ≤ b ≤ a4 , we have f (b) ≤ max f (1), f a4 .

Note
 that f (1)  (a − 1)(2 − a) ≤ 0, since a ≥ 2 x y ≥ 2, and
a2 (a − 2)2 (a + 4)(a − 4)
f  ≤ 0, and therefore f (b) ≤ 0.
4 16



10.13. Note that if x  0, then


a + bx + xc2

A + Bx + xC2
, and hence letting
x → +∞, we obtain |a| ≤ |A|. (1)
Consider the following cases.

(a) If A  0, then from (1) it follows that a  0, and therefore,


b + c









x

B + C
, whence |b| ≤ |B|. It follows that
b2 − 4ac

b2

B 2


2 x


B − 4 AC
.
(b) If A  0, let B 2 −4 AC > 0,
and let x 1 , x 2
be the

roots of the polynomial

Ax 2 + Bx + C. Then from
ax 2 + bx + c

Ax 2 + Bx + C
it follows
that axi2 + bxi + c  0, i  1, 2, and hence a  b  c  0 or



(x2 − x1 )2  b −a 24ac  B −A24 AC . Therefore,


b2 − 4ac

B 2 − 4 AC
.
2 2

(c) If A  0, let B 2 − 4 AC ≤ 0,
and without
loss of generality one can
assume that A > 0. Therefore,
ax 2 + bx + c
≤ Ax 2 +Bx+C, and hence,
( A − a)x 2 + (B − b)x + C − c ≥ 0 and ( A + a)x 2 + (B + b)x + C + c ≥ 0,
for all real values of x.

We have (B − b)2 − 4( A − a)(C − c) ≤ 0 and (B + b)2 − 4( A + a)(C + c) ≤ 0,


and hence(B − b)2 − 4( A − a)(C − c) + (B + b)2 − 4( A + a)(C + c) ≤ 0.

Therefore, b2 − 4ac ≤ 4 AC − B 2 
B 2 − 4 AC
.
In order to complete the proof, it is left to consider the case b2 − 4ac < 0.
Without loss of generality one can assume that a > 0, and thus ax

2
+ bx + c

4ac − b
≤ 4 A , and hence b − 4ac

4 AC − B

2 2
2 2
Ax + Bx + C, and it follows
that 4a

4ac − b ≤ 4 AC − B 

B − 4 AC .
2 2 2

10.14. If x yz  0, then 1 + √2
3
(x 2 −x +1)(y 2 −y+1)(z 2 −z+1) ≥ 1 + √2
3
· 43 · 43 >
1  (x yz)2 − x yz + 1.
If x yz  0, then without loss of generality one can assume that yz > 0.
Proofs 151

Let k  1 + √23 , A  (y 2 − y + 1)(z 2 − z + 1), B  yz. Then one needs


to prove that

(k A − B 2 )x 2 − (k A − B)x + k A − 1 ≥ 0. (1)

Note that k A ≥ k · 43 y 2 · 43 z 2 > y 2 z 2  B 2 , and then it suffices to prove that


D  (k A − B)2 − 4(k A − B 2 )(k A − 1) ≤ 0, or 3k 2 A2 − 2k(2B 2 − B + 2)A +
3B 2 ≥ 0.
Let us prove that

2B 2 − B + 2 + 2 (B − 1)2 (B 2 + B + 1)
A≥ . (2)
3k
Note that

A  (y + z)2 − (B + 1)(y + z) + B 2 − B + 1. (3)



Let t  B+ √1 .
B
Consider the following two cases.

(a) If t > 4, then from (3), it follows that A ≥ 43 (B−1)2 , and let us prove that
  
9k
(B − 1)2 ≥ 2B 2 − B + 2 + 2 (B − 1)2 (B 2 + B + 1), 9k4 B + B1 − 2 ≥
4
      
2 B + B1 − 1 + 2 B + B1 − 2 B + B1 + 1 , or 9k4 t 2 − 4 ≥ 2(t 2 −

2) − 1 + 2  (t 2 − 4)(t 2 − 1). Then the obtained inequality holds because
9k 2
t − 4 − (4t 2 − 10) > 9k4 − 4 16 − 9k + 10 > 0, and therefore,
4 
2(t 2 − 2) − 1 + 2 (t 2 −  4)(t − 1) ≤ 2(t − 2) − 1 + (t − 4) + (t − 1) 
2 2 2 2

4t − 10 < 4 t − 4 .
2 9k 2

(b) If t ≤ 4, we have y + z ≥ 2 yz ≥ B 2+ 1 , and then from (3)
 √ 2 √
it follows that A ≥ 2 B − 2(B + 1) B + B 2 − B + 1, and
 √
let us prove that 3k B 2 + 3B + 1 − 2(B + 1) B ≥ 2B 2 − B + 2 +
 
2 (B − 1)2 (B 2 + B + 1), 3k(t − 1)2 ≥ 2t 2 − 5 + 2 (t 2 − 4)(t 2 − 1),
 √ 2  √  √ √
or t − 1 − 3 9 + 4 3 t 2 − 18 + 6 3 t + 12 + 2 3 ≥ 0.
This inequality holds because
 √ 2  √  √
D  18 + 6 3 − 4 9 + 4 3 12 + 2 3 < 0.

10.15. (a) We proceed by induction.


For n  1 we have (1 − x1 )λ + 1 − y1λ  1.
Let n ≥ 2 and suppose that the inequality holds for n − 1 numbers.
Consider the function f (x)  (1 − (1 − x) · x2 · · · xn )λ + (1 − x λ ) · (1 −
y2λ ) · · · (1 − ynλ ) on [0, 1].
Since f  (x)  λx2 · · · xn (1 − (1 − x) · x2 · · · xn )λ−1 − λx λ−1 ·
(1 − y2λ ) · · · (1 − ynλ ), we have 0 ≤ x < x0 , where x0 
152 10 Using Functions

1 − x2 ··· xn
  1 , and then we obtain f  (x) > 0, and if x0 <
(1 − y2λ ) ··· (1 − ynλ ) λ−1
x2 ··· xn −x2 ··· xn

x ≤ 1, then we obtain f  (x) < 0.


(Note that (1 − y2λ ) · · · (1 − ynλ ) > (1 − y2 ) · · · (1 − yn )  x2 · · · xn ,
whence 0 < x0 < 1.)
It follows that f (y1 ) ≥ min( f (0); f (1)).
Since f (0)  (1 − x2 · · · xn )λ + (1 − y2λ ) · · · (1 − ynλ ) ≥ 1 and f (1)  1,
we have f (y1 ) ≥ 1.
Hence, the given inequality holds for every positive integer n.
(b) If m  1, then (1 − p n )m + (1 − q m )n  1.
If m > 1, then in Problem 10.15(a), taking λ  m, x1  · · ·  xn 
p, we obtain (1 − p n )m + (1 − q m )n ≥ 1.
This ends the proof.

Problems for Independent Study

Prove the following inequalities (2–12, 14–20).

1. Given that 0 ≤ p ≤ 1, 0 ≤ r ≤ 1, and the identity ( px + (1 − p)y)(r x + (1 −


r )y)  ax 2 + bx y + cy 2 , prove that one of the numbers a, b, c is not less than
4/9. 
2. x1 + · · · + xn − x1 x2 − x2 x3 − · · · − xn−1 xn − xn x1 ≤ n2 , where 0 ≤ xi ≤
1 (i  1, . . . , n) and n ≥ 3, n ∈ N.
3. (x
 1n
x2 + · ·· + xn + 1)2 ≥ 4(x12 + · · · + xn2 ), where x1 , . . . , xn ∈ [0, 1].
+ 
2
ai
n
bi2
 √ √ 2
AB ab
ab +
4. i1
n

i1
2 ≤ 2
AB
, where 0 < a ≤ ai ≤ A, 0 < b ≤ bi ≤ B, i 
ai bi
i1
1, . . . , n.
5. a 2 b2 + b2 c2 + c2 a 2 > 21 (a 4 + b4 + c4 ), where a, b, c are the side lengths of some
triangle.
6. (a) x 2 + y 2 + z 2 + x y + yz + zx + x + y + z + 38 ≥ 0,
(b) x y(1 − z) + yz(1 − x) + x z(1 − y) ≤ 1, where 0 ≤ x, y, z ≤ 1.
7. 2n ≥ (1 + a1 ) · · · (1 + an ) + (1 − a1 ) · · · (1 − an ) ≥ 2, where 0 ≤ ai ≤ 1, i 
1, . . . , n.
8. 2a+b+ 2b+c + 2c+a < 1 + 2a+b+c+1, where a > 0, b > 0, c > 0.
2 2 2
9. min (a − b)2 , (b − c)2 , (a − c)2 ≤ a + b2 + c .
10. a 4 + ab + b2 ≥ 4a 2 b + a 3 b, where a, b ≥ 0.
11. a B + bC + c A < k 2 , where a > 0, b > 0, c > 0, A > 0, B > 0, C > 0 and
a + A  b + B  c + C  k.
12. bca+ 1 + cab+ 1 + abc+ 1 ≤ 2, where a, b, c ∈ [0, 1].
13. Prove that the perimeter of the quadrilateral section of a regular tetrahedron with
edge length a is smaller than 3 a.
Problems for Independent Study 153



n n
 m 2
ai
ai xi x
i1 i
14. i1
 n 2

≤ 1
4
M
m
+ M
, where a1 , . . . , an > 0 and 0 < m ≤ xi ≤ M,
ai
i1
i  1, . . . , n.
15. 8ac + 8bd ≤ 3a 2 + 3b2 + 3c2 + 3d 2 + ab + bc + cd + da, where 0 ≤ a ≤ b ≤ c ≤ d.
2
+b2 −c2
16. (a) a a+b−c ≤ a + b − 2, where 2 ≤ a, b, c ≤ 3.
a12 + a22 − a32 a2 + a2 − a2 a2 + a2 − a2
(b) a1 + a2 − a3
+ a22 + a33 − a44 + · · · + ann + a11 − a22 ≤ 2a1 + 2a2 + · · · + 2an − 2n, where
n ≥ 3, 2 ≤ ai ≤ 3, i  1, . . . , n.
a(c − d) + 2d
17. 1
2
< b(d − c) + 2c
≤ 2, where 1 ≤ a ≤ 2, 1 ≤ b ≤ 2, 1 ≤ c ≤ 2, 1 ≤ d ≤ 2.
18. αγ − β 2 ≤ 0, where aγ − 2bβ + cα  0 and ac − b2 > 0.
19. (a1 + · · · + an − b1 − · · ·
− bn )2 +
2|a1 b2 −
a2 b1 | +
· · · + 2|a1 bn − an b1 | +
· · · + 2|an−1 bn − an bn−1 | ≤
a12 − b12
+ · · · +
an2 − bn2
+ 2|a1 a2 − b1 b2 | + · · · +
2|a1 an − b1 bn | + · · · + 2|an−1 an − bn−1 bn |, where n ≥ 2 and ai > 0, bi >
0, i  1, . . . , n.
20. a1
1 + a1 + ··· + an − a1
+ · · · + 1 + a1 + ···an+ an − an + (1 − a1 ) · · · (1 − an ) ≤ 1, where n ≥ 2,
0 ≤ ai ≤ 1, i  1, . . . , n.
Chapter 11
Jensen’s Inequality

Historical origins. Jensen’s inequality is named after the Danish mathematician


Johan Ludwig William Valdemar Jensen, born 8 May 1859 in Nakskov, Denmark,
died 5 March 1925 in Copenhagen, Denmark. This inequality was proved in a paper
that Jensen published in 1906. He never received an academic degree in mathematics.
In 1876 he entered Copenhagen College of Technology and studied mathematics
among various other subjects. Mathematics was his favorite subject, and he published
his first paper in mathematics when he was still a student. Jensen studied advanced
topics of mathematics later by himself. He never held an academic position. In 1881
he began work as an engineer for a Copenhagen division of the international Bell
company (from 1882 Copenhagen telephone company) and did mathematics in his
spare time. From 1890 to 1924 he was head of the technical department of the
Copenhagen telephone company, and from 1892 to 1903 he was president of the
Danish Mathematical Society.

Theorem 11.1 (Jensen’s inequality) Suppose the following inequalities hold for all
a and b in D( f )  I :
 
f (a) + f (b) a+b
≥ f , (11.1)
2 2
  
f (a) + f (b) a+b
≤ f . (11.2)
2 2

Then for all α1 , . . . , αn ∈ Q + 1 , where α1 + · · · + αn  1, and all numbers


x1 , . . . , xn ∈ I , one has the following inequalities:

α1 f (x1 ) + · · · + αn f (xn ) ≥ f (α1 x1 + · · · + αn xn ), (11.3)


[α1 f (x1 ) + · · · + αn f (xn ) ≤ f (α1 x1 + · · · + αn xn )]. (11.4)

1Q is the set of the positive rational numbers


+

© Springer International Publishing AG, part of Springer Nature 2018 155


H. Sedrakyan and N. Sedrakyan, Algebraic Inequalities, Problem Books
in Mathematics, https://doi.org/10.1007/978-3-319-77836-5_11
156 11 Jensen’s inequality

Proof See Problem 7.8.


Note that if on I the function f (x) satisfies (11.1), then f (x) is said to be Jensen
convex or midconvex on I , and if f (x) satisfies (11.2), then f (x) is said to be Jensen
concave or midconcave on I .
  
Example 11.1 Prove that cos x1 +···+cos
n
xn
≤ cos x1 +···+x
n
n
, where x1 , . . . , xn ∈ 0, π2 .

Proof Prove that for all numbers a, b ∈ 0, π2 inequality (11.2) holds, that is,
 
cos a+cos b
2
≤ cos a+b
2
. Indeed, cos a+cos
2
b
 cos a+b
2
cos a−b 2
≤ cos a+b2
.
Assume that α1  · · ·  αn  n . From (11.4), it follows that
1

cos x1 + · · · + cos xn x1 + · · · + xn
≤ cos .
n n
This ends the proof.

Theorem 11.2 If for all numbers a and b from D( f )  I and for all α, β such that
α ≥ 0, β ≥ 0, α + β  1, one has

α f (a) + β f (b) ≥ f (αa + βb), (11.5)


[α f (a) + β f (b) ≤ f (αa + βb)], (11.6)

then for all α1 , . . . , αn , x1 , . . . , xn , one has

α1 f (x1 ) + · · · + αn f (xn ) ≥ f (α1 x1 + · · · + αn xn ), (11.7)


[α1 f (x1 ) + · · · + αn f (xn ) ≤ f (α1 x1 + · · · + αn xn )], (11.8)

where α1 ≥ 0, . . . , αn ≥ 0, α1 + · · · + αn  1, and x1 , . . . , xn ∈ I .

Proof See Problem 7.9.


Note that if on I the function f (x) satisfies (11.5), then f (x) is said to be convex
in I , and if f (x) satisfies (11.6), then f (x) is said to be concave in I .

Theorem 11.3 If for a function f (x) in D( f )  I one has

f  (x) ≥ 0, (11.9)

or f  (x) ≤ 0 , (11.10)

then (11.7) or (11.8) holds.

Proof Assume that (11.9) holds. Let us first prove that (11.5) holds.
11 Jensen’s inequality 157

Indeed, from the finite increments formula, it follows that


α f (x1 ) + β f (x2 ) − f (αx1 + βx2 )  α( f (x1 ) − f (αx1 + βx2 )) + β( f (x2 ) − f (αx1 + βx2 ))
 α f  (c1 )(x1 − αx1 − βx2 ) + β f  (c2 )(x2 − αx1 − βx2 )
 
 αβ f  (c2 ) − f  (c1 ) (x2 − x1 )  αβ f  (c)(c2 − c1 )(x2 − x1 ),

where x1 < c1 < αx1 + βx2 < c2 < x2 and c1 < c < c2 .
Hence, the sign of the left-hand side is the same as the sign of f  (c), and thus
(11.5) holds.
According to Theorem 11.2, we have that (11.7) holds.
This ends the proof.
n  n k−1 n 

k

k−1 k
Example 11.2 Prove that ai cik
≥ ai ci , where ai >
i1 i1 i1
0, ci > 0 i  1, . . . , n, and k ∈
/ (0, 1).

Proof Consider the function f (x)  x k in (0, +∞).


k
cik
Since f  (x)  k(k − 1)x k−2 ≥ 0, then taking αi 

n , xi  ai
ci
, where
ckj
j1
i  1, . . . , n, we obtain by Theorem 11.3 that
⎛ ⎞k
 k ⎜
ai ⎟
n n
cik ai ⎜ cik ⎟
· ≥⎜ · ⎟ .

k
n
ci ⎝
k ci ⎠
n
i1 cj i1 cj
j1 j1

This ends the proof.

Problems

Prove the following inequalities (11.1–11.21)


11.1. sin x1 +···+sin xn
n
≤ sin x1 +···+x
n
n
, where x1 , . . . , xn ∈ [0, π ].
2
+b2 +c2
11.2. a+b+c
≤a a/(a+b+c)
·b b/(a+b+c)
· cc/(a+b+c) ≤ a a+b+c , where a, b, c ∈ N.
 3 b−c a  b  
a−b c
11.3. 1+ a · 1 + c−a
b
· 1 + c
≤ 1, where a, b, c are the side lengths
of some triangle and a, b, c ∈ Q.
11.4. a 2 b(a − b) + b2 c(b − c) + c2 a(c − a) ≥ 0, where a, b, c are the side lengths
of some triangle.
11.5. a 3 b + b 3 c + c3 a ≥ a 2 bc + b2 ca + c2 ab, where a > 0, b > 0, c > 0.
   
11.6. 1 + x1 1 + 1y 1 + 1z ≥ 64, where x > 0, y > 0, z > 0, and x+y+z  1.
11.7. 12
√ √ √ √
a+ b+ c+ d
≤ √a+1 √b + √a+1 √c + √a+1√d + √ 1√
b+ c
+ √ 1√
b+ d
+ √ √ ,
1
c+ d
where a > 0, b > 0, c > 0, d > 0.
158 11 Jensen’s inequality

mp nq
11.8. + ≥ mn, where m > 0, n > 0, p > 0, q > 0, and 1
+ 1
 1.

p
√q
√  p q
11.9. a(a + c − b) + b(a + b − c) + c(b + c − a) ≤ (a 2 + b2 + c2 )(a + b + c),
where a, b, c are the side lengths of some triangle.
n  n   n 2

2

11.10. ai ci ≥
2
ai ci .
i1 i1
 i1
 k

n
n k−1

n
11.11. k
ai k
ci ≤ k−1
ai ci , where a1 , a2 , . . . , an , c1 , . . . , cn >
i1 i1 i1
0, and 0 < k < 1.
n 1  n  q1

p p
q

n
11.12. (a) ai bi ≥ ai bi , where ai > 0, bi > 0, i 
i1 i1 i1
1, . . . , n, p > 0, q > 0, and 1
p
+ q1  1;
 p

n p
ai
ai
(b) p−1 ≥ 
i1

n
 p−1 , where ai > 0, bi > 0, i  1, . . . , n, and p > 1.
bi
i1 bi
i1
11.13. a1
+ a1 −a2a+···+a
−a1 +a2 +···+an
2
n
+ · · · + a1 +a2 +···+a
an
n−1 −an
≥ n
n−2
, where a1 , a2 , . . . , an
are the side lengths⎛of some ⎞ n-gon.

n m
dim bim di bi
11.14. i1
n
· i1
n
≥ ⎝ i1
n
⎠ , where d1 > 0, . . . , dn > 0, b1 >

0, . . . , bn > 0, and m ≥ 2.
11.15. (a) x2x+x 1
3
+ x3x+x2
1
+ x1x+x
3
3
≥ 23 , where x1 > 0, x2 > 0, x3 > 0;
(b) x2 +xn + x3 +x1 + · · · + xnx+x
x1 x2 n−1
n−2
xn
+ x1 +x n−1
≥ 2, where n ≥ 4, x1 >
0,
 · · · , x n > 0. 
 
11.16.  xy + yz + xz − xy − yz − xz  ≤ 2 1 − xxy+yz+x z
2 +y 2 +z 2 , where x, y, z are the side
lengths of some triangle.
11.17. (a) b+c+d a
+ a+c+db c
+ a+b+d d
+ a+b+c ≥ 43 , where a > 0, b > 0, c > 0, d > 0;
(b) a2 +a3 +···+an + a1 +a3 +···+an + · · · + a1 +a2 +···+a
a1 a2 an
n−1
≥ n−1
n
,
where n ≥ 2, a1 > 0, . . . , an > 0.
a
11.18. b+c + c+db
+ a+d c d
+ a+b ≥ 2, where a > 0, b > 0, c > 0, d > 0.
2 2
b2 +c2 2
+a 2 a3 b3 c3
11.19. a + b + c ≤ 2c + 2a + c 2b
a +b
≤ bc + ca + ab , where a > 0, b > 0, c > 0.
a3 b3 c3
11.20. a 2 +ab+b2 + b2 +bc+c2 + c2 +ca+a 2 ≥ 3 , where a > 0, b > 0, c > 0.
a+b+c
 a+b  a a  b b
11.21. a+bc+d
≤ c · d , where a > 0, b > 0, c > 0, d > 0.
11.22. Let (xn ) be a sequence with positive terms such that 1  x0 ≥ x1 ≥ x2 ≥
· · · ≥ xn ≥ · · ·.
Prove that there exists n such that for every such sequence (xn ), one has

x02 x12 x2
+ + · · · + n−1 ≥ 3.999.
x1 x2 xn

11.23. Perpendiculars M A1 , M B1 , MC1 are drawn from the inner point M of the
triangle ABC to the sides BC, C A, AB, respectively. For which point M
of the triangle ABC is the value of the expression MBCA1 + MC BA1 + MC
AB
1
the
smallest?
Problems 159

11.24. Let a1 , . . . , an , b1 , . . . , bn > 0 and J ⊆ {1, . . . , n}. Prove that if for every
J one has m n 2  n 




2 m+1
bi ≥ ai , where m ∈ N, then ai bi ≥ ai .
i∈J i∈J i1 i1
11.25. Let a1 > 0, . . . , an > 0, b1 > 0, . . . , bn > 0. Prove
(a) Radon’s inequality:
p p
a1 an (a1 + · · · + an ) p
+ ··· + ≥ ,
p−1
b1
p−1
bn (b1 + · · · + bn ) p−1

where p( p − 1) ≥ 0,
p p
a1 (a1 +···+an ) p
(b) p−1 + · · · + ap−1
n
≤ (b1 +···+bn ) p−1
,
b1 bn
p p
a1 an (a1 + · · · + an ) p
+ ··· + ≤ ,
b1
p−1
bn
p−1 (b1 + · · · + bn ) p−1

where p( pp − 1) p+1
< 0,
p+1
p p
p+1 a1
(c) p · b p + · · · + abn p > a1 · ab11 + . . . + an · ab11 +···+a n

 p+11
n
p+1 
+···+bn
p 1 (a1 +···+an ) p+1
p+1
b1 · ab11 + · · · + bn ab11 +···+a
+···+bn
n
+ p+1 . (b1 +···+bn ) p ,
where p > 0.
11.26. (a) Given the interval G and a function f (x) defined on the interval I, suppose
that for  all2numbers
 x1 and x2 from the interval I, where x1 + x2 ∈ G, one
has f x1 +x 2
≤ f (x1 )+ f (x2 )
2
.
Prove that if the x1 , . . . , xn are numbers from the interval I such that
x1 + x1 , . . . , xn + xn ∈ G, where the numbers x1 , . . . , xn are the numbers
 
x1 , . . . , xn written in some other order, then f x1 +···+x n
n
≤ f (x1 )+···+
n
f (xn )
.
Prove that
(b) (n−1)a a1
+ · · · + (n−1)a an
≤ 1, where n ≥ 2, ai > 0, i  1, . . . , n,
1 +a2 1
n +a
(c) √1+x +· · ·+ √1+x ≥ min 1, √1+λ
1 1 n
, where n ≥ 2, xi > 0, i  1, . . . , n,
1 n
and x1 . . . xn  λ (λ > 0),
n
x1 +...+xn n
1−
(d) (1−xx11)···(1−x
···xn
n)
≥ n
x1 +···+xn , where n ≥ 2, xi > 0, i  1, . . . , n,
n
and xi + x j ≤ 1, i, j ∈ {1, . . . , n}.
2 + 1+y 2 + 1+z 2 ≤ 10 , where x + y + z  1.
1 1 1 27
11.27. Prove that 1+x
α + (1+b)α + (1+c)α + (1+d)α ≥ 2 , where α ≥ 2,
1 1 1 1 2−α
11.28. Prove that (1+a)
a > 0, b > 0, c > 0, d > 0, and abcd  1.
160 11 Jensen’s inequality

Proofs

11.1. For all a, b ∈ [0, π ] we have


sin a+sin b
2
≤ sin a+b 2
.
We have sin a+sin 2
b
 sin a+b2
cos a−b 2
≤ sin a+b 2
.
Taking α1  · · ·  αn  n and using inequality (11.4), we deduce that
1
sin x1 +···+sin xn
n
≤ sin x1 +···+xn
n
.
11.2. Since for all a > 0, b > 0 one has log a+log 2
b
≤ log a+b 2
, it follows that if
we set α1  a+b+c , α2  a+b+c , α3  a+b+c , then by inequality (11.4), we
a b c

have α1 log a + α2 log b + α3 log c ≤ log(α1 a + α2 b + α3 c), thus a a/(a+b+c) ·


2
+b2 +c2
bb/(a+b+c) · cc/(a+b+c) ≤ a a+b+c .
Let us consider the function f (x)  x ln x in (0, +∞).
Since f  (x)  x1 > 0, we have that (11.7) holds for the function f (x).
Taking α1  α2  α3  13 , it follows that 13 (a ln a + b ln b + c ln c) ≥
 a+b+c
3 (a
1
+ b + c) · ln a+b+c 3
. Therefore, a a · bb · cc ≥ a+b+c 3
, or a a/(a+b+c) ·
b b/(a+b+c)
·c c/(a+b+c)
≥ 3 .
a+b+c

11.3. Let f (x)  log x, x1  1 + b−c a


, x2  1 + c−a b
, x3  1 + a−b c
,
α1  a
,α  b
, α3  c
, hence
  a+b+c 2  
a+b+c   a+b+c
1
a log 1 + b−c
+ b log 1 + c−a
+ c log 1 + a−b

a+b+c a  a  b  c
log a+b+c 1 + b−c + b
1 + c−a
+ c
1 + a−b
,
a a 
a+b+c b
c−a b
 a+b+ca−b c c
and therefore 1 + b−c a
· 1 + b
· 1+ c ≤ 1.
11.4. Consider the function f (x)  x 2 . One can easily prove that for all numbers
m and n, one has α f (m) + β f (n) ≥ f (αm + βn), where α ≥ 0, β ≥ 0,
and α + β  1.
According to Theorem 11.2 inequality (11.7) holds, and taking
x1  c, x2  b, x3  a, α1  a+c−b a+b+c
, α2  b+c−a
a+b+c
, α3  a+b−c
a+b+c
, we obtain

a+c−b b+c−a a+b−c


c2 + b2 + a2
a+b+c a+b+c a+b+c
 2
ac + c2 − bc + b2 + bc − ab + a 2 + ab − ac
≥ ,
a+b+c

whence
   
c2 (a + c)2 − b2 + b2 (b + c)2 − a 2
   2
+ a 2 (a + b)2 − c2 ≥ a 2 + b2 + c2 ,

or

a 2 b(a − b) + b2 c(b − c) + c2 a(c − a) ≥ 0.

11.5. Hint. Take f (x)  x 2 , x1  a + b − c, x2  a + c − b, x3  b + c − a,


Proofs 161

a b c
α1  , α2  , α3  .
a+b+c a+b+c a+b+c
Alternative proof. Note that
 
a 3 b + b3 c + c3 a − a 2 bc + b2 ca + c2 ab
 ab(a − c)2 + bc(b − a)2 + ac(b − c)2 ≥ 0.
 
11.6. Let us consider the function f (t)  ln 1 + 1t on (0, +∞).
Since f  (t)  t12 − (t+1)1
2 > 0, inequality (11.7) holds. Taking
α1  α2  α3  13 ,t1  x, t2  y, t3  z, we obtain
   
1
3
ln 1 + x1 + ln 1 + 1y + ln 1 + 1z ≥ ln 1 + x+y+z
3
; hence from x + y +
    3
z  1, it follows that 1 + x1 1 + 1y 1 + 1z ≥ 1 + x+y+z
3
 64.
 
11.7. Let us consider the function f (x)  x (0, +∞). We have 2 a + b1 ≥ a+b
1 1 1 2
,
√ √ √ √
and so by Theorem 11, taking x1  a + b, x2  a + c,
√ √ √ √ √ √
x3  a + d, x4  b + c, x5  b + d,
√ √ 1
x6  c + d, α1  · · ·  α6  ,
6
we obtain that the given inequality holds.
11.8. Let us consider the function f (x)  ln x in (0, +∞). Since f  (x)  −1 x2
< 0,
inequality (11.6) holds. Taking α  1p , β  q1 , a  m p , b  n q , we
obtain that the given inequality holds.√
11.9. Let us consider the function f (x)  x in (0, +∞). One can easily prove
that for the function f (x), the condition (11.10) holds.
Taking α1  a+c−b a+b+c
, α2  a+b−c
a+b+c
, α3  c+b−a
a+b+c
, x1  a, x2  b, x3  c,
we deduce inequality (11.8) in the given form.
Hint. For a  9, b  4, c  1 the inequality does not hold.
11.10. Let c12 + · · · + cn2
 0. Consider the function f (x)  x 2 . Since f  (x) > 0,
on taking
 2
ci2 ai
αi  , xi  ,
c1 + · · · + cn2
2 ci

where i  1, . . . , n, using Theorem 11.3, we obtain


 2  n 2

n
ci2 ai ci2 ai
· ≥ · ;
i1
c12 + · · · + cn2 ci i1
c1 + · · · + cn ci
2 2

hence the given inequality holds.


If c1  · · ·  cn  0, then we obtain an obvious inequality.
162 11 Jensen’s inequality

11.11. The proof is similar to Problem 11.2, using f  (x) < 0.


11.12. (a) The proof is similar to Problem 11.2, on substituting k by p, and ci by
q

bip .
(b) According to Problem 11.12a, we have
⎛ ⎛ ⎞ p ⎞ 1p  p 
n  p−1  p−1
p−1

n
a p
n
⎝ ⎝ p−1 ⎠ ⎠
i
bi p
≥ ai ,
p
i1 bi i1 i1

and therefore,
 n  n  p−1  n p
ap
i
p−1
bi ≥ ai ,
i1 bi i1 i1

or
 p


n p ai
ai i1
≥  p−1 .
i1
p−1
bi

n
bi
i1

11.13. Consider the function f (x)  x1 in (0, +∞).


Taking xi  a1 +· · ·+ai−1 −ai +· · ·+an , αi  ai
a1 +···+an
, where i  1, . . . , n
(11.7), we obtain

1
n
ai a1 + · · · + an
· ≥ n ,
a1 + · · · + an a1 + · · · + ai−1 − ai + · · · + an
 
i 1 ai a1 + · · · + ai−1 − ai + · · · + an
i1

whence

n
ai (a1 + · · · + an )2 n
≥   ≥ ,
a1 + · · · + ai−1 − ai + · · · + an (a1 + · · · + an )2 − 2 a12 + · · · + an2 n−2
i1

since
 
n a12 + · · · + an2 ≥ (a1 + · · · + an )2 (see Problem 2.2).
 n k

n

11.14. In Problem 11.2, taking ci  1, we obtain n k−1


ai ≥
k
ai ,
i1 i1

n

n
⎞ k
aik ai
or i1
n
≥ ⎝ i1n ⎠ . Now taking k  m
2
, ai  di2 , it follows that

n

n
⎞ m2
dim di2
i1
n
≥ ⎝ i1n ⎠ .

n

n
⎞ m2
bim bi2
In a similar way, we deduce that i1
n
≥ ⎝ i1n ⎠ .
Proofs 163

Multiplying the obtained inequalities term by term and using the inequality
of Exercise 11.10, ⎛ ⎞ ⎛ ⎞

n m/2

n m
bim dim bi2 · di2 bi di
we obtain i1
n
· i1
n
≥⎝ i1 i1
n2
⎠ ≥⎝ i1
n
⎠ .

11.15. Consider the function f (x)  x1 in (0, +∞).


(a) Taking αi  x1 +xx2i +x3 , i  1, 2, 3, we obtain
x1 1 x2 1 x3 1
· + · + ·
x1 + x2 + x3 x2 + x3 x1 + x2 + x3 x1 + x3 x1 + x2 + x3 x1 + x2
1
≥ ,
x1
·
x1 +x2 +x3 (x 2
+ x3 ) + x1 +x2 +x3 (x1 + x3 ) + x1 +xx23 +x3 (x1 + x2 )
x2

and hence.

x1 x2 x3 (x1 + x2 + x3 )2
+ + ≥  .
x2 + x3 x1 + x3 x1 + x2 2 x1 x2 + x2 x3 + x1 x3

x 2 +x 2 +x 2  x +x +x 2
From the inequality 1 32 3 ≥ 1 32 3 , or
x12 + x22 + x32 ≥x1 x2 + x2 x3 + x1 x3 , we have

(x1 + x2 + x3 )2 1 x12 + x22 + x32 3


  · +1≥ ,
2 x1 x2 + x2 x3 + x1 x3 2 x x
1 2 + x x
2 3 + x x
1 3 2

and therefore x2x+x 1


3
+ x1x+x
2
3
+ x1x+x
3
2
≥ 23 .
(b) αi  (x1 +···+xn ) , i  1, . . . , n, and we have that
xi

x1 1 x2 1 xn 1
· + · + ··· + ·
x1 + · · · + xn x2 + xn x1 + · · · + xn x1 + x3 x1 + · · · + xn x1 + xn−1
1
≥       ,
x1 x2 +xn +x2 x3 +x1 +···+xn x1 +xn−1
x1 +···+xn

whence
x1 x2 xn (x1 + · · · + xn )2
+ + ··· + ≥  
x2 + xn x1 + x3 x1 + xn−1 x1 x2 + x1 xn + x2 x3 + x2 x1 + · · · + xn x1 + xn−1
(x1 + · · · + xn )2
   ≥2
2 x1 x2 + x2 x3 + · · · + xn−1 xn + xn x1

(see the solution to Problem 7.6).


164 11 Jensen’s inequality

Hence, it follows that


x1 x2 xn
+ + ··· + ≥ 2.
x2 + xn x1 + x3 x1 + xn−1

11.16. Let A  xy + yz + xz − xy − yz − xz ≥ 0. Consider the function f (x)  x1 in


(0, +∞).
Let us take α1  y+z−x , α2  y+x−z , α3  z+x−y .
y+z+x
y+z+x y+z+x
≥ x 2 +y2 +z2 , or 3 − A ≥ x(x+y+z)
2
y+z−x x+y−z z+x−y
2 +y 2 +z 2 ,
1 1
Hence, x+y+z y
+ x + z
x+y+z
(x+y+z)2 x y+x z+yz
whence A ≤ 3 − x 2 +y 2 +z 2  2 1 − x 2 +y 2 +z 2 .
 
One can easily prove that x y + x z + yz > 21 x 2 + y 2 + z 2 , and thus it follows
that A < 1 .
11.17. (b) We have
a1 1 a2 1 an 1
· + · + ··· + ·
a1 + · · · + an a2 + · · · + an a1 + · · · + an a1 + a3 + · · · + an a1 + · · · + an a1 + · · · + an−1
1
≥      ;
a1 a2 +···+an +a2 a1 +a3 +···+an +···+an a1 +···+an−1
a1 +···+an

therefore,
a1 an
+ ··· +
a2 + · · · + an a1 + · · · + an−1
(a1 + · · · + an )2 n
≥  2 ≥ ,
(a1 + · · · + an ) − a1 + · · · + an
2 2 n−1
 
since n a12 + · · · + an2 ≥ (a1 + · · · + an )2 .
11.18. We have
a 1 b 1 c 1
· + · + ·
a+b+c+d b+c a+b+c+d c+d a+b+c+d d +a
d 1 1
+ · ≥ a(b+c)+b(c+d)+c(d+a)+d(a+b) .
a+b+c+d a+b a+b+c+d

Therefore,
a b c d (a + b + c + d)2
+ + + ≥ ≥ 2,
b+c c+d d +a a+b ab + ac + bc + bd + cd + ac + ad + db

since (a + b + c + d)2 − 2(ab + ac + bc + bd + cd + ac + ad + bd) 


(a − c)2 + (b − d)2 ≥ 0.
11.19. Consider the function f (x)  x1 in (0, +∞).
Proofs 165

Taking

α1  α6  a
2(a+b+c)
, α2  α3  b
2(a+b+c)
, α4  α5  c
2(a+b+c)
,
x1  ac , x2  bc , x3  ab , x4  ac , x5  bc , x6  ab ,

we obtain
a 1 b 1 b 1 c
· + · + · +
2(a + b + c) ac 2(a + b + c) bc 2(a + b + c) ab 2(a + b + c)
1 c 1 a 1 1
· a + · b + · b ≥ 2c+2a+2b  1.
c
2(a + b + c) c
2(a + b + c) a 2(a+b+c)

Similarly, one can obtain the second inequality:


     
a 3 b3 c3 1 2 1 1 1 2 1 1 1 2 1 1
+ +  a · + b2 · ac + a · + c2 · + c · + b2 · ac
bc ca ab 2 bc 2 bc ab 2 ab
a b a c c b
2 2 2
a 2 + b2 a 2 + c2 c 2 + b2
1 1 1
≥ · + · + ·
2 2abc 2 2abc 2 2abc
a 2 + b2 a 2 + b2 a 2 + c2 a 2 + c2 b2 + c2 b2 + c2
 · + · + ·
2c 2ab 2b 2ac 2a 2bc
a 2 + b2 a 2 + c2 b2 + c2
≥ + + .
2c 2b 2a

11.20. Consider the function f (x)  1


x
in (0, +∞). Taking

a2 b2 c2
α1  a 2 +b2 +c2
, α2  a 2 +b2 +c2
, α3  a 2 +b2 +c2
,
a 2 +ab+b2 b2 +bc+c2 c2 +ca+a 2
x1  a
, x2  b
, x3  c
,

we obtain

a2 1 b2 1 c2 1
· a 2 +ab+b2
+ · b2 +bc+c2
+ · c2 +ac+a 2
a 2 + b2 + c2 a
a 2 + b2 + c2 b
a 2 + b2 + c2 c
1
≥ a 3 +a 2 b+ab2 +b3 +b2 c+bc2 +c3 +c2 a+a 2 c
,
a 2 +b2 +c2

or
 2 2
a3 b3 c3 a + b2 + c2 a+b+c
+ + ≥   ≥ .
a 2 + ab + b2 b2 + bc + c2 c2 + ac + a 2 (a + b + c) a 2 + b2 + c2 3

11.21. Consider the function f (x)  − ln x on (0, +∞). It is a convex function,


and therefore,
166 11 Jensen’s inequality
 
a c b d a c b d
− ln · + · ≤− ln − ln ,
a+b a a+b b a+b a a+b b
or
 a+b a a  b b
a+b
≤ · .
c+d c d

Remark If a1 > 0, . . . , an > 0, b1 > 0, . . . , bn > 0, then


   a1  an
a1 + · · · + an a1 +···+an a1 an
≤ · ... · .
b1 + · · · + bn b1 bn

11.22. Consider the function f (x)  1


x
on (0, +∞). Taking αi 
xi−1
x0 +x1 +···+xn−1
, i  1, . . . , n,
we obtain
⎛ ⎞
x02 x2 x2 x0 1 xn−1 1 ⎠  
+ 1 + · · · + n−1  ⎝ · x + ··· + · x · x0 + x1 + · · · + xn−1
x1 x2 xn x0 + x1 + · · · + xn−1 1 x0 + x1 + · · · + xn−1 x n
x0 n−1
 
x0 + x1 + · · · + xn−1 2
≥  kn .
x1 + x2 + · · · + xn−1 + xn

Now let us prove that there exists a positive integer n 0 , such that for n > n 0
we have kn ≥ 3.999.
Indeed, if n > 0,001
3,999
+ 1, we have
 2  
x0 + x1 + · · · + xn−1 − 3, 999 x1 + x2 + · · · + xn−1 + xn ≥ 0, 001(n − 1)xn − 3, 999xn ≥ 0,

and one can take n 0  4000.


11.23. Consider the function f (x)  x1 in (0, +∞). Taking α1 
BC
AB+BC+AC
, α2  AB+BC+AC
AC
, α3  AB+BC+AC
AB
, x 1  M A1 , x 2 
M B1 , x3  MC1 , we obtain
 
BC CA AB BC 1 CA 1 AB 1
+ +  · + · + ·
M A1 M B1 MC1 BC + AC + AB M A1 BC + AC + AB M B1 BC + AC + AB MC1
(BC + AC + AB)2 4 p2 2 p2
(BC + AC + AB) ≥   ,
BC · M A1 + AC · M B1 + AB · MC1 2S S

BC CA AB
and thus the smallest value of the expression M A1
+ M B1
+ MC1
is equal to
2
2p
, and equality holds when M A1  M B1  MC1 .
S
11.24. Without loss of generality one can assume that a1 ≥ a2 ≥ · · · ≥ an .
According to Problem 14.10a, we have


n
ai bi  b1 (a1 − a2 ) + (b1 + b2 ) (a2 − a3 ) + · · ·
i 1
+ (b1 + · · · + bn−1 ) (an−1 − an ) + (b1 + · · · + bn ) an .
Proofs 167

It follows that

n
ai bi ≥ a1m (a1 − a2 ) + (a1 + a2 )m (a2 − a3 ) + · · ·
i 1
+ (a1 + · · · + an−1 )m (an−1 − an ) + (a1 + · · · + an )m an
 A.

Since the function f (x)  x m is convex on [0, +∞), we have


 m  m
a1 (a1 − a2 ) (a1 + a2 )(a2 − a3 ) (a1 + · · · + an−1 )(an−1 − an ) (a1 + · · · + an )an a12 + a22 + · · · + an2
≥ + + ··· + + .
a1 a1 a1 a1 a1

Hence, we deduce that


⎛ ⎞2 2m  m−1

n a12 + a22 + · · · + an2 a12 + a22 + · · · + an2 m+1
⎝ ai bi ⎠ ≥ A2 ≥  a12 + a22 + · · · + an2 ≥
i1 a12m−2 a12

m+1
≥ a12 + a22 + · · · + an2 .

 2

n  m+1
Thus, it follows that ai bi ≥ a12 + a22 + · · · + an2 .
i1
11.25. (a) Consider the function f (x)  x 1− p in (0, +∞). Since f  (x)  p( p −
1)x − p−1 ≥ 0, it follows that for the function f (x) one has inequality (11.7).
Taking αi  a1 +···+a
ai
n
, i  1, . . . , n, we obtain
 1− p  1− p
a1 b1 an bn
· + ··· + ·
a1 + · · · + an a1 a1 + · · · + an an
 
a1 b1 an bn 1− p
≥ · + ··· + · ,
a1 + · · · + an a1 a1 + · · · + an an
or
p p
a1 an (a1 + · · · + an ) p
+ ··· + ≥ .
b1
p−1
bn
p−1 (b1 + · · · + bn ) p−1

(b) The proof is similar to Problem 11.25a, using f  (x) < 0.


(c) Defining Bi  ab11 +···+bi
+···+ai
, i  1, . . . , n, then if i  2, . . . n, we obtain
168 11 Jensen’s inequality

bi  Bi (a1 + · · · + ai ) − Bi−1 (a1 + · · · + ai−1 ), and therefore,


bi a1 + · · · + ai Bi−1
p+1
 p − (a1 + · · · + ai−1 ) · p+1 . (11.11)
Bi Bi Bi

By to the Problem 9.13, we have


 p
 p+1
p
  p+1
p+1
Bi−1 1
p p
Bi p+1
Bi−1 1
p+1
+ ≥ p,
p
p+1 Bi

and thus it follows that


Bi−1 p+1 1
p+1
≥ p − p−1
. (11.12)
Bi p Bi p Bi

From (11.11) and (11.12) we obtain p+1 ai


p
· B p − bp+1 i
≥ a1 +···+a
p B1
p
i
− a1 +···+a
p
p B1−1
i−1
,i 
i Bi
2, . . . , n, and p+1
p
· a1
p
B1
− b1
p+1  a1
p B1
p ≥
a1
p B1
p , and summing all n inequalities,
B1
we obtain
   
p+1 a1 an b1 bn a1 a1 + · · · + an
· p + ··· + p − p+1
+ ··· + p+1
 p ≥ p .
p B1 Bn B1 Bn p B1 p Bn
(11.13)
Therefore, from (11.13) it follows that
 
p+1 a1 an b1 bn
· p + ··· + p > p+1 + · · · + p+1 , (11.14)
p B1 Bn B1 Bn

and according to Problem 11.25a, it follows that


p+!p p+!p p+!p
b1 bn
a1
p
B1
an
p
Bn
a1
p
B1
+ ··· + an
p
Bn
p+1
+ ··· + p+1
 1p + · · · + 1p ≥ 1p .
B1 Bn a1
p+1 p+1
an a1
p+1 p+1

b1
p
bn
p
b1
p + ··· + an
bn
p

(11.15)

From (11.14) and (11.15) we obtain


 p  p  p p+1 p+1

a1 a1 + · · · + an p+1 a1 an
a1 · + · · · + an · < p + ··· + p .
b1 b1 + · · · + bn p b1 bn
(11.16)
Proofs 169

From (11.13), it follows that


 p 

a1 a1 + · · · + an p
a1 · + · · · + an ·
b1 b1 + · · · + bn
   p+1   
p a1 a1 + · · · + an p+1
≥ b1 · + · · · + bn
p+1 b1 b1 + · · · + bn
1 (a1 + · · · + an ) p+1
+ . .
p + 1 (b1 + · · · + bn ) p (11.17)

Remark

1. If p < −1 and a1 ,…, an , b1 ,…, bn are arbitrary positive numbers, then


 p  p  p p+1 p+1

a1 a1 + · · · + an p+1 a1 an
a1 · + · · · + an · < p + ··· + p .
b1 b1 + · · · + bn p b1 bn

2. If p > 1 and a1 ,…, an are arbitrary positive numbers, then


a + · · · + a p n a + · · · + a p
p 1 n 1 n
a1 + · · · + + ·
n p−1 n
 p
p p
< · (a1 + · · · + anp ).
p−1

3. If p > 1 and b1 ,…, bn are arbitrary positive numbers, then


 p
1 1 p
b11 + · · · + (b1 · . . . · bn ) n < · (b1 + · · · + bn ).
p−1

4. If b1 ,…, bn are arbitrary positive numbers, then


1 1
b11 + · · · + (b1 · . . . · bn ) n < e · (b1 + · · · + bn ).

x +x 
11.26. (a) Consider the numbers xi  i 2 i , i  1, . . . , n. Note that min(xi , xi ) ≤
xi ≤ max(xi , xi ) and min(xi + x j , xi + x j ) ≤ xi + x j ≤ max(xi + x j , xi + x j ),
for all i, j ∈ {1, . . . , n}. Therefore, xi ∈ I and xi + x j ∈ G.
If we prove that
  
x1 + · · · + xn f (x1 ) + · · · + f (xn )
f ≤ , (11.18)
n n
  f (xi )+ f (xi )
then since f xi ≤ , i  1, . . . , n, and f (x1 ) + · · · + f (xn ) 

2
  
f (x1 ) + · · · +  x1 +···+xn
f (xn ), n  x1 +···+x
n
n
, we have f x1 +···+x
n
n
≤ f (x1 )+···+
n
f (xn )
.
Assume that inequality (11.18) holds for m numbers, and let us prove that it
holds for 2m numbers.
170 11 Jensen’s inequality

Indeed, we have
⎛ ⎞
  x 
x 1 +x 2 +x 
x1 + x2 + · · · + x2m−1
 
+ x2m + · · · + 2m−12 2m
f  f⎝ 2 ⎠
2m m
  x  +x 
x +x
f 1 2 2 + · · · + f 2m−12 2m

m
f (x 1 )+ f (x 2 ) f (x  )+ f (x  )
+ ··· + 2m−1 2m
≤ 2 2
m
f (x1 ) + f (x2 ) + . . . + f (x2m−1
  )
) + f (x2m
 .
2m
Now let us prove that if inequality (11.18) holds for k (k > 2) numbers,
then it holds for k − 1 numbers.
Let x1 , . . . , xk−1

∈ I and xi + x j ∈ G, where i, j ∈ {1, . . . , k − 1}.
x  +···+x 
Note that min(x1 , . . . , xk−1

) ≤ 1 k−1 k−1  x  ≤ max(x1 , . . . , xk−1 
), and
   
therefore,
x ∈ I, 2x , x + x i ∈ G, for all i  1, . . . , k − 1, whence
x1 +···+xk−1

+x  f (x  )+···+ f (xk−1

)+ f (x  )
f ≤ 1 . Thus it follows that
 k

k
x1 +···+xk−1 f (x1 )+···+ f (xk−1

)
f k−1
≤ k−1
. Since inequality (11.18) for
two numbers
holds, from this statement it follows that it holds for n numbers.
Remark If G  R, then we obtain Jensen’s inequality.
(b) Let us prove that if n ≥ 2, yi > 0, i  1, . . . , n, and y1 · . . . · yn 
1
(n−1)n
, then
1 1
+ ··· + ≤ n − 1. (11.19)
1 + y1 1 + yn

Indeed, let us consider the function f (x)  1+3 1


on R, and G 
x
 
(−∞, 0]. Note that if x1 + x2 ∈ G, then − x1 +x2 − − 1+31 x1 − 1+31 x2 
2
  1+3 2
x1 +x2 x1 x2 2
3 2 −1
3 2 −3 2

x1 +x2
 ≤ 0, and hence
(1+3x1 )(1+3x2 ) 1+3 2
x + x f (x1 ) + f (x2 )
1 2
f ≤ .
2 2
If yi y j ≤ 1 for all i and j, then xi + x j ∈ G, where yi  3xi , i  1, . . . , n,
and thus for x1  x2 , . . . , xn−1

 xn , xn  x1 , the condition of Problem
11.25a holds. Therefore,
1 1 n
+ ··· + ≤ √  n − 1.
1 + y1 1 + yn 1 + n y1 · . . . · yn
Proofs 171

If there exist numbers i and j such that yi y j > 1, then


1 1 1 1 1 1
+ ··· + ≤ + + < 1+ < 1+(n − 2)  n − 1.
1 + y1 1 + yn 1 + yi 1 + yj 1 + yk 1 + yk
k
i,k
 j k
i,k
 j

For yi  (n−1)a
ai+1
i
, i  1, . . . , n, an+1  a1 , from inequality (11.19), it
follows that
a1 an
+ ··· + ≤ 1.
(n − 1)a1 + a2 (n − 1)an + a1

(c) Consider the function f (x)  √1


1+3x
in R, and G  [2, +∞). Note that
if x1 + x2 ∈ G, then
√ √ 2 
1 + 3x1 + 1 + 3x2  2 + 3x1 + 3x2 + 2 (1 + 3x1 )(1 + 3x2 )
x1 +x2

≥ 2 + 3x1 + 3x2 + 2 1 + 3 2 ,

whence
√ √ 2 x1 +x2

1 + 3x1 + 1 + 3x2 1+3 2
x1 +x2
x1 +x2

≥ 4 + 3x1 + 3x2 + 2 · 3 2 1+3 2
≥ 4(1 + 3x1 )(1 + 3x2 ),

since
 x1 +x2  x1 +x2   x +x  x x 2 2
1 2 1
4 + 3x1 + 3x2 + 2 · 3 2 1+3 2 − 4(1 + 3x1 )(1 + 3x2 )  3 2 − 3 3 2 − 3 2 ≥ 0.

It follows that
x +x f (x1 ) + f (x2 )
1 2
f ≤ .
2 2

If yi y j ≥ 9 for all i and j, then xi + x j ∈ G, where yi  3xi , i  1, . . . , n,


and therefore for x1  x2 , . . . , xn−1
 xn , xn  x1 , the conditions of
Problem 11.25a hold.
Hence
1 1 n n
√ + ··· + √ ≥ √ √ .
1 + y1 1 + yn 1 + y1 · . . . · yn
n
1+λ

If there exist numbers i and j such that yi y j < 9, then


1 1 1 1
√ + ··· + √ ≥√ +√ > 1,
1 + y1 1 + yn 1 + y1 1 + y2
172 11 Jensen’s inequality

since
  2 √
1 + y1 + 1 + y2 ≥ 4 + y1 + y2 + 2 y1 y2 > (1 + y1 )(1 + y2 ).

(d) Consider the function f (x)  ln 1−x


x
on (0, 1), and G  (0, 1]. Note
  x +x
1− 1 2
that if x1 , x2 ∈ (0, 1) and x1 + x2 ∈ G, then f x1 +x 2
2
 ln x1 +x22 ≤
1−x 1−x
ln x 1 +ln x 2
2 2

1
2
2
 f (x1 )+ f (x2 )
2
, since 2−(x1 +x2 )
x1 +x2
≤ (1−x1 )(1−x2 )
x1 x2
, or (x 1 −x 2
2 ) (1−
x1 − x2 ) > 0, whence
x + x f (x1 ) + f (x2 )
1 2
f ≤ .
2 2
Thus, from Problem 11.25a, it follows that
x + ··· + x f (x1 ) + · · · + f (xn )
1 n
f ≤ ,
n n
or
 n
(1 − x1 ) · . . . · (1 − xn ) 1− x1 +···+xn
≥ n
.
x1 · . . . · xn x1 +···+xn
n

11.27. Without loss of generality one can assume that x ≤ y ≤ z. Let us consider
three cases.
(a) If x ≤ − √13 , then y + z ≥ 1 + √1 > 3 . Therefore, z > 3 . It follows that
3 2 4
1
1+x 2 + 1
1+y 2 + 1
1+z 2 < 1 + 1 + 16
25
< 27
10
.
(b) If y ≥ √13 , then z ≥ √13 , whence, 1+x1
2 + 1+y 2 + 1+z 2 ≤ 1 + 4 + 4 < 10 .
1 1 3 3 27

(c) If x, y ∈ − √13 , √13 , then consider the function f (t)  1+t1 2 in

− √13 , √13 .
2
−1) 2
Note that f  (t)  2(1+t(1+t)(3t
2 )2 < 0, and thus by Jensen’s inequality, it
follows that 1+x 2 + 1+y 2 ≤
1 1 2
2 
2
2 . Therefore, 1+x 2 + 1+y 2 + 1+z 2 ≤
1 1 1
1+( x+y 2 ) 1+( 1−z
2 )

2 + 1+z 2 ≤ 10 , since the last inequality is equivalent to the inequality


2 1 27
1+( 1−z
2 )
(3z − 1)2 (3z 2 − 4z + 5) ≥ 0.
11.28. Let us prove that if α ≥ 2 and a > 0, b > 0, then

1 1 22−α
α
+ α
≥ α (11.20)
(1 + a) (1 + b) 1 + (ab) 2

On setting x  1
1+a
, y 1
1+b
, we see that inequality (11.20) can be rewritten
as
Proofs 173
 α α α α  α α α α
x 2 y− 2 + x − 2 y 2 x 2 y 2 + (1 − x) 2 (1 − y) 2 ≥ 22−α .
α α α α
Without loss of generality one can assume that y 2 (1 − y) 2 ≥ x 2 (1 − x) 2 .
Thus by to the Cauchy–Bunyakovsky–Schwarz inequality, it follows that
 α α α α  α α α α
x 2 y − 2 + x − 2 y 2 x 2 y 2 + (1 − x) 2 (1 − y) 2
  2
α α α α α
≥ x 2 + y 2 (1 − y) 2 x − 2 (1 − x) 2
 α α 2
≥ x 2 + (1 − x) 2 ,
 α α 2
and by Jensen’s inequality, it follows that x 2 + (1 − x) 2 ≥
  α2 2
2 x+(1−x)
2
 22−α .
α α α α α α α α
Therefore, (x 2 y − 2 + x − 2 y 2 )(x 2 y 2 + (1 − x) 2 (1 − y) 2 ) ≥ 22−α .
Note that
1 1 1 1 22−α 22−α
α
+ α
+ α
+ α
≥ α + α
(1 + a) (1 + b) (1 + c) (1 + d) 1 + (ab) 2 1 + (cd) 2
22−α 22−α
 α +  1  α2  2
2−α
.
1 + (ab) 2
1 + ab

Problems for Independent Study

1. Prove that if α, β, γ are the angles of some triangle, then


(a) cos α + cos β + cos γ ≤ 23 ,
(b) sin21α/2 + sin21β/2 + sin21γ /2 ≥ 12,

(c) cos1α/2 + cos1β/2 + cos1γ /2 ≥ 2 3,
(d) sin1α/2 + sin 1β/2 + sin 1γ /2 ≥ 6.
2. Prove that if α, β, γ are the angles of some acute triangle, then
√ √ √
cos2 α 1+8 cos2 β 1+8 cos2 γ
(a) 1+8 sin α
+ sin β
+ sin γ
≥ 6,
(b) cos α + cos β + cos γ ≥ 6.
1 1 1

3. Prove that if for a convex quadrilateral ABC D one has





sin 2A sin 2B sin 2C sin 2D  41 , then ABC D is a rectangle.


4. Prove that x2 +x3 + x3 +x4 + · · · + xxnn−1
x1 x2
+x1
+ x1x+x
n
2
≥ n2 , where 3 ≤ n ≤ 6 and
xi > 0, i  1, · · · , n.
5. Prove that among all convex n-gons inscribed in a given circle, the one with
greatest area is the regular n-gon.
174 11 Jensen’s inequality

6. Prove that among all convex n-gons inscribed in a given circle, the one with the
greatest perimeter is the regular n-gon.
7. Prove that 1+xa a1
1
a2
+ 1+xa 2
+· · ·+ 1+xa
an
n
≤ n+x
n
, where x > 0, a1 ≥ 0, · · · , an ≥ 0,
and a1 + · · · + an  1.
8. Prove that sin α sin β + sin β sin γ + sin γ sin δ + sin δ sin α ≤ 2, where α >
0, β > 0, γ > 0, δ > 0, and α + β + γ + δ  π.
9. Let x1 , x2 , · · · , xn be arbitrary real numbers.
(a) Prove that, if λ ≥ 2 or λ < 0, then it holds true the following inequality
⎛ ⎞λ/2

n
(±x1 ± x2 ± · · · ± xn )λ ≥ 2n ⎝ xi2 ⎠ ,
i1

(b) Prove that if 0 < λ < 2, then one has the following inequality:
n λ


2 2
|±x1 ± x2 ± · · · ± xn |λ ≤ 2n xi (on the left-hand side, the summa-
i1
tion is over all combinations of plus and minus signs).
2 2 2  2 2
a −b
10. Prove that 2
≥ a +b
2
− a+b 2
, where a ≥ 21 , b ≥ 21 .

n
xi + x1i ≥ ( na−1) , where a > 0, x1 > 0, . . . , xn > 0, and
a n 2 +1
a
11. Prove that
i1

n
xi  1.
i1
12. (a) Prove that a1 x1 + · · · + an xn ≥ x1a1 · . . . · xnan , where a1 + · · · + an  1 and
xi > 0, ai ≥ 0, i  1, . . . , n,

n
n
(b) Let ai j ≥ 0 and ai j  1, j  1, 2, . . . , n, ai j  1, i  1, 2, . . . , n.
i1 j1
Prove that
(a11 x1 + a12 x2 + · · · + a1n xn ) (a21 x1 + a22 x2 + · · · + a2n xn )
· . . . · (an1 x1 + an2 x2 + · · · + ann xn ) ≥ x1 x2 · . . . · xn ,

where x1 ≥ 0, · · · , xn ≥ 0.
(c) Prove that (ux + vy + wz)(vx + wy + uz)(wx + uy + vz) ≥ (y + z − x)(z +
x − y)(x + y − z),
where u + v + w  1, u ≥ 0, v ≥ 0, w ≥ 0, x ≥ 0, y ≥ 0, z ≥ 0.
(1−x)(1−y)
z
(1−y)(1−z)
x
(1−z)(1−x)
y
13. Prove that (1−x)(1−y)
z
· (1−y)(1−z)
x
· (1−z)(1−x)
y
≥ 256
81
,
where x > 0, y > 0, z > 0 and x + y + z  1
14. Find the largest and smallest values of the expression
(a) a
a+b
+ b
b+c
− a
a+c
, where a > 0, b > 0, c > 0,

n−1
xj
(b) x j +x j+1
− x1
x1 +xn
, where n ≥ 3, x1 > 0, . . . , xn > 0.
j1
Problems for Independent Study 175
√ √
x +···+ x
15. Prove that √1−x x1
+ · · · + √1−xxn
≥ 1√n−1 n , where n ≥ 2, xi > 0, i 
1 n
1, . . . , n, and x1 + · · · + xn  1.
x1 +···+xn
Hint. Prove that √1−x x1
+ · · · + √1−x
xn
≥ n · √ xn1 +···+xn .
1 n 1− n
3
2 + · · · + 1−x 2 ≥ n · , where 0 ≤ xi < 1, i 
x1 xn (x1 +···+xn )
16. Prove that 1−x (x1 +···+xn )2 −(x12 +···+xn2 )2
1 n
1, . . . , n.
Hint. Consider the function f (x)  1−x 1
2 in [0, 1), taking αi  x +···+x ,
1
xi
n
i
1, . . . , n.
17. Prove that (a1 b1 + a2 b2 + · · · + an bn )k + (a1 b2 + a2 b3 + · · · + an b1 )k + · · · +
+(a1 bn + a2 b1 + · · · + an bn−1 )k ≤ (b1 + b2 + · · · + bn )k (a1k + a2k + · · · + ank ), where
n ≥ 2, ai > 0, bi > 0, i  1, · · · , n, k > 1. 
an an
18. For which values of λ does the inequality a n +λa11·...·an +· · ·+ a n +λa1n·...·an ≥ √1+λ n
1 n
hold for all positive numbers a1 , . . . , an ?
Chapter 12
Inequalities of Sequences

In this section we consider inequalities related to sequences, in particular sequences


given by recurrence relations, which are proved in various ways. While most inequal-
ities can be proved by well-known proof techniques, there are not many tools for
inequalities involving sequences. Below we provide some examples of problems on
inequalities of sequences and demonstrate the proof techniques with which such
problems can be attacked.

Problems
x2
12.1. Prove that if xn+1  xn + nn2 , n  1, 2, . . ., and 0 < x1 < 1, then the sequence
(xn ) is bounded. √
12.2. Prove that if a1  1, an  an−12
1
+ an−1 , n  2, 3, . . . , 10, then 0 < a10 − 2 <
10−370 .
12.3. Consider the sequence (u n ) such that u 1  1, u n+1  u n + u1n , n  1, 2, . . ..
Prove that 14.2 < u 100 < 14.22.
12.4. Consider the sequence (u n ), such that u 1  1, u n+1  u n + u12 , n  1, 2, . . ..
n
Prove that 30 < u9000 < 30.01.
12.5. Prove that if u 0  0.001, u n+1  u n (1 − u n ), n  0, 1, . . ., then u 1000 <
1
.
2000  2
a
12.6. Consider the sequence a1  1, an+1  a2n + 4n + a1n , n  1, 2, . . .. Prove
that this sequence is unbounded.
12.7. Given that a0  an  0, ai > 0, i  1, . . . , n − 1, n ≥ 2, and as−12+as+1 >
as cos πk , k ∈ N, prove that n ≥ k.

© Springer International Publishing AG, part of Springer Nature 2018 177


H. Sedrakyan and N. Sedrakyan, Algebraic Inequalities, Problem Books
in Mathematics, https://doi.org/10.1007/978-3-319-77836-5_12
178 12 Inequalities of Sequences

12.8. Let x1 ∈ [0, 1), and for n  1, 2, . . ., suppose that



⎨ 0, if xn  0,
xn+1   
⎩ x1 − x1 if xn  0.
n n

Prove that x1 + x2 + · · · + xn < FF21 + FF23 + · · · + FFn+1


n
, where F1  F2  1 and
Fn+2  Fn+1 + Fn , n  1, 2, . . . .
12.9. Let n ≥ 2 be a positive integer. Find the smallest possible value of the sum
a0 + a1 + · · · + an if a0 , a1 , . . . , an are nonnegative numbers such that a0  1
and ai ≤ ai+1 + ai+2 , i  0, 1, . . . , n − 2.
12.10. Given a sequence of positive numbers x1 , x2 , . . . , xn , . . . such that xnn 
 j
n−1
xn , n  1, 2, . . ., prove that 2 − 2n−1 1
≤ xn < 2 − 21n .
j0
12.11. Let a0 , a1 , a2 , . . . , an , . . . be an infinite
√ sequence of positive numbers.
Prove that the inequality 1 + an > n 2.7 × an−1 holds for infinitely many
numbers n.
12.12. Consider the following sequence an  max(x n (1 − x) + (1 − x)n x), n 
[0;1]
1, 2, . . .. Prove that an+1 ≥ 21 an , n  1, 2, . . ..
12.13. Given that a0  an+1  0 and |ai−1 − 2ai + ai+1 | ≤ 1, i  1, . . . , n,
prove that ak ≤ k(n+1−k)
2
, k  1, . . . , n.
12.14. Find the smallest value of the number C such that there exists a sequence
(an ) of positive numbers such that the inequality a1 +· · ·+am+1 ≤ Cam holds
for all positive integers m.

Proofs

12.1. Let us prove by induction that


nx1
xn ≤ √ √ , n  1, 2, 3, . . . .
(1 − x1 )n + x1

If n  1, then we have x1 ≤ √ x1 √
(1− x1 )+ x1
 x1 .
Assume that the given inequality holds for n  k, that is, xk ≤ (1−√xkx1 )k+
1 √
x1
.
Let us prove that the given inequality holds for n  k + 1, that is, xk+1 ≤
x2
√ (k+1)x1 √ . Since x k+1  x k + k2 and x k ≤ √ kx1 √ , we have
(1− x1 )(k+1)+ x1 k (1− x1 )k+ x1
Proofs 179

kx1 x2
xk+1 ≤ √ √ + √ 1 √ 2 
k(1 − x1 ) + x1 (k(1 − x1 ) + x1 )
√ √
k 2 (1 − x 1 ) + k x1 + x1 (k + 1)x1
 x1 √ √ 2 ≤ √ √ , as
(k(1 − x1 ) + x1 ) (1 − x1 )(k + 1) + x1
√ √
k+1 k 2 (1 − x1 ) + k x1 + x1
√ − √ √ √ 
k(1 − x1 ) + 1 k 2 (1 − x1 )2 + 2k(1 − x1 ) x1 + x1
√ √
x1 (1 − x1 )2
 √ √ √ √ > 0.
(k(1 − x1 ) + 1)(k 2 (1 − x1 )2 + 2k(1 − x1 ) x1 + x1 )

Therefore, for every positive integer n we have 0 < xn ≤ (1−√xnx1 )n+ 1 √


x1

nx√
1
 1− x1 , and hence the sequence (xn ) is bounded.
x√
1
n(1− x1 )

12.2. Prove that an − 2 > 0, where
n 2, 3, . . .. √ √
We have an  0.5 an−1 + an−12
≥ an−1 · an−1 2
 2, and hence an − 2 ≥
0. There is never
√ equality, since all terms of the sequence are rational numbers;
hence an − 2 > 0. √
Now let us consider the sequence b n  an −
2. √
√ √ a 2 −2 2a +2
We have bn  an − 2  0, 5 an−1 + an−1 2
− 2  n−1 2an−1 n−1 
√ 2
(an−1 − 2)2 bn−1 b2
2an−1
 n  2, 3, . . ., and thus bn < 2n−1
2an−1
, √ .
2
√ √ b2 b22
Since b2  a2 − 2  23 − 2 < 10 1
and bn < 2n−1
√ , we have b3 <
2

2 2
<
1√
102 2 2
, b4 < √1 √ ,
104 (2 2)2 2 2
and continuing in a similar way, we deduce that
1 1 1 1 1 1
b10 < · √ · √ · · · √  256 · √ .
1028 (2 2)27 (2 2)26 2 2 10 (2 2)255

Now let us prove that


1 1 1 √ 255 √
· √ < or 10 114
< (2 2)  2 382
2.
10256 (2 2)255 10370

Since 210 > 103 , it follows that 10114 < 2380 < 2382 2. √
Hence b10 < 101256 · (2√12)255 < 101370 , and thus it follows that a10 − 2 < 10−370 .
12.3. We have u 2n+1  u 2n + 2 + 1
u 2n
. Thus, it follows that
1 1 1
u 22  u 21 + 2 + 2
, u 23  u 22 + 2 + 2 , . . . , u 2n  u 2n−1 + 2 + 2 .
u1 u2 u n−1

Summing these equalities, we obtain


1 1 1
u 2n  2n + + + · · · + 2 (n ≥ 3). (12.1)
u 22 u 23 u n−1
180 12 Inequalities of Sequences

Since u n−1 > u n−2 > · · · > u 2  2, we have 1


u 22
+ 1
u 23
+ ··· + 1
u 2n−1
< n4 .
Therefore,
9
2n ≤ u 2n < n (n ≥ 2). (12.2)
4
From the conditions (12.1) and (12.2), it follows
that for n  100 we have
200 + 49 21 + 13 + · · · + 99
1
< u 2100 < 200 + 21 21 + 13 + · · · + 99
1
.
Let us estimate from below the following sum; 21 + 13 + · · · + 99 1
.
We have
 
1 1 1 1 1 1 1 1 1 1 1 1 1
+ + ··· +  + + + + + + + + + +
2 3 99 2 3 4 5 6 8 10 7 9 11
 
1 1 1 1 1 1 232 652
+ + + ··· + + + + ··· + > 1.675 + 0.346 + + > 3.989.
12 13 33 34 35 99 23 · 23 65 · 67

a2 a2 (a1 +···+an )2
In this proof, we have used that b11 + · · · + bnn ≥ b1 +···+bn
, where b1 >
0, . . . , bn > 0.
Now let us estimate from above the following sum: 1
2
+ 1
3
+ ··· + 1
99
.
According to Problem 7.1b we have
  
1 1 1 1 1 1 1 1 1 1 1 1
+ + ··· +  + + ··· + + ··· +
+ + + + ··· + +
12 13 99 12 13 24 25 48 49 50 98 99
  
1 1 1 1 1 1 1 1 1
< + + ··· + + + + ··· + + + + ··· +
12 13 24 49 25 48 97 50 98
1 25 25 25 13
< + + +  < 2.2.
12 36 36 36 6

It follows that
  
1 1 1 1 1 1 1 1 1 1 1 1 1 1 1 1
+ + ··· +  + + + + + + + + + + + + ··· + <
2 3 99 2 3 4 5 6 8 10 7 9 11 12 13 99

1 1 1
< 2.02 + + + ··· + < 4.22.
12 13 99


Since 200 + 49 21 + 13 + · · · + 99
1
< u 2100 < 200 + 21 21 + 13 + · · · + 99
1
, we have
14.22 < 200 + 49 · 3.989 < u 2100 < 200 + 21 · 4.22 < 14.222 , hence 14.2 <
u 100 < 14.22.
12.4. We have u 3n+1  u 3n + 3 + u33 + u16 , where n  1, 2, . . ..
n n
Therefore, u 32  u 31 +3+ u33 + u16 , u 33  u 32 +3+ u33 + u16 , . . ., u 3n+1  u 3n +3+ u33 + u16 .
1 1 2 2 n n
Summing these equalities, we
obtain

u 3n+1  3n + 3 u13 + · · · + u13 + u16 + · · · + u16 ; hence


1 n 1
√ n

u 3n+1 > 3n+ u33  3(n+1), or u n+1 > 3 3(n + 1), thus u 9000 > 3 3 · 9000  30.
1
Proofs 181

On the other hand,


 
1 1 1 1
u 3n+1  3n + 3 3 + · · · + 3 + + · · · + <
u1 un u 61 u 6n
 
1 1 1 1
< 3(n + 1) + 1 + 3 + ··· + + + · · · + 
3·2 3·n 32 · 22 32 · n 2
 
1 1 1 1 1
 3(n + 1) + 1 + + ··· + + + · · · + .
2 n 9 22 n2

Since 1
2
+ 1
3
+ ··· + 1
n
< ln n (see Problem 12.3) and
1 1 1 1 1 1
+ + ··· + 2 < + + ··· +
22 32 n 1·2 2·3 (n − 1)n
  
1 1 1 1 1 1
 1− + − + ··· + −  1 − < 1,
2 2 3 n−1 n n

we must have u 3n+1 < 3(n + 1) + 1 + ln n + 19 ; hence u 39000 < 27002 + ln 9000 <

1 3
27002 + ln 225 < 27027 < 30 + 100 .
Therefore, u 9000 < 30.01
12.5. Since u n+1  u n (1 − u n ), we have u n+1
1
 u n (1−u
1
n)
 u1n + 1−u
1
n
.
Therefore, u 1  u 0 + 1−u 0 , u 2  u 1 + 1−u 1 ,…, u 1000  u 999 + 1−u1 999 .
1 1 1 1 1 1 1 1

Summing these equalities, we obtain


1 1 1 1
 + + ··· +
u 1000 u0 1 − u0 1 − u 999
1 1
 103 + + ··· +
1 − u0 1 − u 999
> 10 + 1 + · · · + 1  2000;
3

hence u 1000 < 2000


1
.
In this proof, we have used the inequalities 0 < u n < 1, which can be proved
in the following way:
2
u n + (1 − u n ) 1
u n+1  u n (1 − u n ) ≤  < 1 (n  0, 1, 2, . . .).
2 4

If u n ≤ 0, then from the equality u n  u n−1 (1 − u n−1 ) it follows that u n−1 ≤


0. Continuing in a similar way, we deduce that u 0 ≤ 0, which leads to a
contradiction; hence u n > 0.
12.6. Assume that the sequence (an ) is bounded, that is, m < an < M (n 
1, 2, 3, . . .),
where M > 0, since an > 0.
We have an+1 2
− an+1 an  a1n or a1n  an+1
1
+ a 2 a12 .
n n+1
Hence 1
a1
 1
a2
+ 1
, 1
a12 a22 a2
 1
a3
+ 1
a22 a32
, …, 1
an
 1
an+1
+ 1
2 .
an2 an+1
182 12 Inequalities of Sequences

Summing these equalities, we deduce that


1 1 1
1  an+1
1
+ a 21a 2 + a 21a 2 + · · · + a 2 a12 > 4 + 4 + · · · + 4  n
M4
, or M 4 > n,
1 2 2 3 n n+1 M
 M  M 
n
which leads to a contradiction, and hence the sequence (an ) is bounded.
12.7. Assume that n < k. Thus, πk < πn , and it follows that cos πk > cos πn .
Hence we obtain a2 > 2a1 cos πn , a1 +a3 > 2a2 cos πn , …, an−2 > 2an−1 cos πn .
Let us multiply both sides of the first inequality by sin πn , and both sides of
the second inequality by sin 2π n
, and so on, ending by multiplying both sides
of the last inequality by sin (n−1)π
n
.
Summing all these inequalities, we obtain 0 > 0, which is a contradiction.
Therefore, our assumption is incorrect, since n ≥ k.
12.8. Without loss of generality, one can assume that xn  0.

We have 
xk  xk−11
k  2, . . . , n, whence xk ∈ (0, 1), k  1, 2, . . . , n, and
xk  1 
≤ 1
xk+1 +1
 f (xk+1 ) k  1, 2, . . . , n −1 (∗), where f (x)  1
x+1
.
xk+1 + x1
k

Lemma If the function f (x) is decreasing on an interval I, and the function


x + f (x) is increasing there, then the function g(x)  x + f (x) + f ( f (x)) +
· · · + f ( f (. . . f (x) . . .)) is increasing on I, where m ∈ N.
  
m
Note that the function f ( f (. . . f (x) . . .)), k ∈ N is increasing on I, and hence
  
2k
the function f ( f (. . . f (x) . . .)) + f ( f (. . . f (x) . . .)) is increasing on I, and
     
2k 2k+1
the function g(x)  x + f (x) + f ( f (x)) + f ( f ( f (x))) + . . . is the sum of
increasing functions. Thus, it follows that this function is increasing on I.
One can easily verify that the function f (x)  x+11
on I  [0, 1] satisfies the
assumptions of the lemma.
Therefore, by the lemma and (∗), we have
x1 + x2 + · · · + xn ≤ f (x2 ) + x2 + x3 + · · · + xn ≤ x3 + f (x3 ) + f ( f (x3 ))+
+ x4 + · · · + xn ≤ . . . ≤ xn + f (xn ) + f ( f (xn )) + · · · + f ( f (. . . f (xn ) . . .)) ≤
  
n−1
F1 F2 Fn
≤ 1 + f (1) + f ( f (. . . f (1) . . .))  + + ··· + ,
   F2 F3 Fn+1
n−1

as 1  F1
F2
and f Fi
Fi+1
 Fi+1
Fi+2
, i  1, 2, . . . , n − 1.
Proofs 183

12.9. Let us prove the following lemma by induction.

Lemma If n ≥ 2, c0  0, cn ≥ 0, and ci ≤ ci+1 + ci+2 , i  1, 2, . . . , n − 2,


then c0 + c1 + · · · + cn ≥ 0.
Indeed, for n  2 and n  3, we have
c0 + c1 + c2 ≥ 2c0  0 and c0 + c1 + c2 + c3 ≥ 2c0 + c3 ≥ 0.
Assume that for n ≤ k the lemma holds. Let us prove that the lemma holds
for n  k + 1 (k ≥ 3).
Consider the following two cases.

(a) c1 ≥ 0. Since 0 ≤ c1 ≤ c2 + c3 , for the numbers 0, c2 , c3 , . . . , ck+1 the


assumptions of the lemma hold, and therefore 0 + c2 + c3 + · · · + ck+1 ≥ 0,
whence c0 + c1 + c2 + · · · + ck+1 ≥ c2 + c3 + · · · + ck+1 ≥ 0.
(b) c1 < 0. We then have c2 ≥ −c1 > 0, and for the numbers
0, c3 , c4 , . . . , ck+1 the assumptions of the lemma hold. Hence 0 + c3 +
c4 + · · · + ck+1 ≥ 0. On the other hand,

c0 + c1 + c2 + c3 + · · · + ck+1 ≥ 2c0 + c3 + · · · + ck+1 ≥ 0.

This ends the proof of the lemma.

Note that the numbers ci  ai − FFn−i n


i  0, 1, . . . , n, where F0  0, F1  1,
and Fi+2  Fi+1 +Fi , i  0, 1, . . . , n−2, satisfy the assumptions of the lemma.
Therefore, c0 + c1 + · · · + cn ≥ 0, or a0 + a1 + · · · + an ≥ FFnn + FFn−1
n
+ · · · + FFn0 . On
the other hand, the numbers ai  FFn−i n
satisfy the assumptions of the problem,
and therefore, the smallest value of the sum a0 + a1 + · · · + an is equal to
Fn +Fn−1 +···+F0
Fn
. One can prove by induction that F0 + F1 + . . . + Fn  Fn+2 − 1.
12.10. Note that x 1  1 and xn > 1 for n  2, 3, . . .. Indeed, for n > 1 we have
x n −1
xnn  xnn −1 , or xnn (2 − xn )  1.
Let xn  2 − yn . Then 0 < yn < 1 for n  2, 3, . . . and yn (2 − yn )n  1. We
need to prove that 21n < yn ≤ 2n−1 1
for n  2, 3, . . .. We have yn  (2−y1 n )n > 21n
(for n  2, 3, . . .).
By Bernoulli’s inequality, we have
1  yn (2 − yn )n  yn (1 + (1 − yn ))n ≥ yn (1 + n(1 − yn )), whence yn ≤ n1 ,
and therefore yn  2n 1−1 yn n ≤ 2n 1−1 nyn ≤ 2n−1 1
, for n  2, 3, . . ..
( 2) ( 2 ) √
12.11. We argue by contradiction. Assume that the inequality 1 + an > n 2.7 × an−1
holds for finitely many numbers n. Thus,√there exists a positive integer n 0
n≥ n 0 , one has 1 + an > 2.7 × an−1 .
n
such that for all
1 n
We have lim 1 + n  e; thus there exists a positive integer n 1 such that for
n→∞ n
n ≥ n 1 , 1 + n1 > 2.7 (e  2.718281828 . . .). Hence, for n ≥ max(n 0 , n 1 ),

we have 1 + an ≤ an−1 · n 2, 7 < n+1 a , and therefore n+1
n n−1
1 an
+ n+1 < an−1
n
. It
follows that m+1 < m − m+1 , m+2 < m+1 − m+2 , . . . , m+k < m+k−1 − am+k
1 am−1 am 1 am am+1 1 am+k−2 m+k−1
.
am−1
Summing these inequalities, we obtain m+1 + m+2 + · · · + m+k < m −
1 1 1
am+k−1
m+k
< am−1 m
, and hence m+1 1
1 + 21 + · · · + k1 < am−1 m
, and it follows that the
184 12 Inequalities of Sequences

sequence xk  1 + 21 + · · · + k1 is bounded, which leads to a contradiction,


since x2n − xn > 21 , where n ∈ N.
Indeed, x2n  xn + n+11
+ · · · + 2n
1
> xn + n · 2n
1
 xn + 21 .
12.12. We proceed by induction.
We have a1  21 , a2  41 , and therefore a2 ≥ 21 a1 .
Let am+1 ≥ 21 am , where m ∈ N, and let us prove that am+2 ≥ 21 am+1 .
Let x0 ∈ [0, 1] be such that am+1  x0m+1 (1 − x0 ) + (1 − x0 )m+1 x0 . Then
am+2 ≥ x0m+2 (1 − x0 ) + (1 − x0 )m+2 x0  (x0m+1 (1 − x0 ) + (1 − x0 )m+1 x0 )(x0 + (1 − x0 ))
a a
− x0 (1 − x0 )(x0m (1 − x0 ) + (1 − x0 )m x0 ) ≥ am+1 − x0 (1 − x0 )am  m+1 + m+1 − x0 (1 − x0 )am
2 2
2
am+1 am am+1 1 am+1
≥ + − x0 (1 − x0 )am  + am − x0 ≥ ,
2 4 2 2 2

and therefore, am+2 ≥ 21 am+1 .


Hence, an+1 ≥ 21 an , n  1, 2, . . ..
12.13. Let us set ai+1 − ai  bi , i  0, 1, . . . , n, and bi+1 − bi  ci , i 
0, 1, . . . , n − 1. Then |ci | ≤ 1, i  0, 1, . . . , n − 1. We have bi+1 
c0 + · · · + ci + b0 , i  0, 1, . . . , n − 1, and ai+1  b0 + · · · + bi  (i + 1)b0 +
ic0 + (i − 1)c1 + · · · + ci−1 . Therefore, 0  an+1  (n + 1)b0 + nc0 + · · · + cn−1 ,
whence, b0  − n+1 n
c0 − · · · − n+1
1
cn−1 and
 
(i + 1)n (i + 1)(n − i + 1)
ai+1  i − c0 + · · · + 1 − ci−1
n+1 n+1
n−i 1
− ci − · · · − cn−1 ,
n+1 n+1

where we note that i − k − (i+1)(n−k)


n+1
≤ 0, for k  0, . . . , i − 1. Thus, ai+1 is
maximal if c0  · · ·  cn−1  −1, in which case

ai+1  (i + 1)b0 − i − (i − 1) − · · · − 1
1 + ··· + n i(i + 1) (i + 1)(n − i)
 (i + 1) · −  .
n+1 2 2

Therefore, ak ≤ k(n+1−k)
2
, k  1, . . . , n.
12.14. Let the sequence of the positive numbers (an ) be such that
a1 + · · · + am+1 ≤ Cam , (1) m  1, 2, . . . , and let us prove that C ≥ 4.
Indeed, for m  2, 3, . . ., we have Sm+1 ≤ C(Sm − Sm−1 ), where Sn 
a1 + · · · + an (n ∈ N). √ √
Therefore,

C Sm ≥ Sm+1 + C Sm−1 ≥ 2 Sm+1 · C Sm−1 , whence C ≥
2 Sm+1 ·Sm−1
Sm
.
Proofs 185

n √ √  
n+2 ·Sn
Therefore, C ≥ 2 SSn+1 · · · · · 2 SS23 ·S1  2n · SSn+2 ·S1
n+1 ·S2
> 2 n
· S1
S2
, and
√ √ 
hence C ≥ 2 · n q, where q  SS21 . Thus letting n → +∞, it follows that

C ≥ 2, and we obtain that C ≥ 4.
If we prove that for C  4 there exists a sequence of positive numbers (an )
such that inequality (1) holds, then we obtain that the smallest possible value
of the number C is 4.
For the sequence an  2n−1 , we have a1 + · · · + am+1  1 + · · · + 2m 
2m+1 − 1 < 4 · 2m−1  4am , and hence a1 + · · · + am+1 < 4am .
This ends the proof.

Problems for Independent Study

u 2 +2
1. Consider the sequence (u n ) such that u 1  109 , u n+1  2u n
, n  1, 2, . . ..
√ −13
n

Prove that 0 < u 36 − 2 < 10 .


2. Let a1 , a2 , . . . , an be real numbers such that a1  0, |a2 | |a1 +1|, |a3 | |a2 +1|,
…, |an | |an−1 + 1|. Prove that a1 +a2 n+···+an ≥ − 21 .
3. Consider the sequence a1 , a2 , . . . , an , . . . such that a1  1, an+1  an + a12 ,
n
n  1, 2, . . ..
Is this sequence bounded?
4. Suppose the sequence (an ) is nondecreasing and a0  0. Given that the sequence
bn  an −an−1 , n  1, 2, . . ., is not increasing, prove that the sequence cn  ann ,
n  1, 2, . . ., is not increasing.
xn4 +9
5. Consider the sequence (xn ) such that x1  2, xn+1  10x n
, n  1, 2, . . .. Prove
that 5 < xn ≤ 4 , for all n > 1.
4 5
 2
a
6. Consider the sequence a1  1, an+1  a2n + 4n + a1n , n  1, 2, . . .. Prove that
a250 < 10.
7. Consider the sequence x1  2, xn+1  xn2 − xn + 1, n  1, 2, . . ..
Prove that x11 + x12 + · · · + x1n < 1.

8. Consider the sequences (an ) and (bn ) such that a1  1, an+1  an + an2 + 1,
bn  a2nn , n  1, 2, . . .. Prove that the sequence (bn ) is bounded. √ √
9. Given that R1  1, Rn+1  1+ Rnn , n  1, 2, . . ., prove that n ≤ Rn ≤ n +1.
a2
10. Given that an+1  an + nn , prove that there exists a number A such that 0 <
n( A − an ) < A3 .
11. Consider the following sequence: a0  2, an+1  an + a1n , n  0, 1, . . ..
Prove that 12 < a70 < 12.25.
12. Consider the following sequences: (an ) (an > 0, n  1, 2, . . .) and x1  1,
x2  2, xn+2 + an xn+1 + xn  0, n  1, 2, . . .. Prove that the sequence (xn )
contains an infinite number of positive terms and an infinite number of negative
terms.
186 12 Inequalities of Sequences

13. Consider the following sequence: a1  1, an  an−1 + an−1 1


, n  2, 3, . . ..
√ √
Prove that 2n − 1 ≤ an ≤ 3n − 2.
xn4 +9
14. Consider the following sequence: x1  2, xn+1  10x n
, n  1, 2, . . ..
−10 −10
Prove that 1 − 10 < x100 ≤ 1 + 10 .
15. (a) Consider the following sequence: a1 ≥ 2, an+1  an 2
− 2, n  1, 2, . . ..
a2
Prove that 1
+ 1
+ 1
+ ··· + 1
a1 a2
< a21 − 41 − 1.
a1 a1 a2 a1 a2 a3 ...an

a2
(b) Let a > 2 and a0  1, a1  a, an+1  a 2 n − 2 an , n  1, 2, . . ..
√ n−1

Prove that 1
a0
+ 1
a1
+ ··· + 1
ak
< 1
2
2 + a − a2 − 4 .
16. For which values of x1 are all terms of the sequence xn+1  2xn2 − 1, n 
1, 2, . . ., negative?
1
n+1
17. Consider the following sequence: xn  i
. Prove that xn k −1 > kxn−1 , where
i2
n, k ∈ N and n > 1, k > 1. 
18. Consider the following sequence: a1  1, an+1  (a1 + · · · + an )2 + 1, n 
1, 2, . . ..
Prove that a11 + a12 + · · · + a1n < 2.5.
19. Consider the following sequence: u 1  1, u n  n! + n−1
n
u n−1 , n  2, 3, . . ..
Prove that u11 + u12 + · · · + u1n < 1.
20. Consider a monotonic sequence (xn ) of positive numbers. Prove that xx21 + · · · +
+ xxn1 ≥ n + (x2x−x + · · · + (xnx−x
2 2
xn−1 n−1 )
xn
1)
1 x2 n−1 x n
, where n ≥ 2.
Hint. Let x1  1 + α1 , . . . , xn−1  1 + αn−1 . Then xxn1  (1 + α1 ) . . . (1 + αn−1 ).
x2 xn
 
21. Let 0 ≤ ai ≤ a, i  1, 2, . . ., and ai − a j  ≥ i+1 j , for all i < j. Prove that
a ≥ 1.
Hint. Let ai1 ≤ . . . ≤ ain , where {i 1 , . . . , i n }  {1, . . . , n}. Then by inequality
(8.4) (see Chapter
 8) and the assumptions
 the problem, it follows that a ≥
 of(n−1)
  
ain − ai1  ain − ain−1 + · · · + ai2 − ai1 ≥ n(n+1) . 2

22. Let the sequence (u n ) be such that |u m+n − u m − u n | ≤ mn , for all m >
n, m, n ∈ N.
Prove that u n  nu 1 , n  1, 2, . . ..
Hint. Prove that lim (u m+n − u n )  u m , then u m+1  lim (u m+1+n − u n ) 
n→∞ n→∞
 lim (u m+1+n − u n+1 ) + lim (u n+1 − u n )  u m + u 1 .
n→∞ n→∞
Chapter 13
Algebraic Inequalities in Number Theory

Inequalities arise not only in algebra, but very often in number theory as well.
In this chapter we consider inequalities in number theory and prove them using
algebraic inequalities. In order to explain how a large number of problems may be
attacked, we provide a list of problems and their proof techniques.

Problems

Prove the following inequalities (13.1–13.3).


√   √
13.1. mn < 2 1 − 4n1 2 , where mn < 2, m, n ∈ N.
 √   √  √
 
13.2. n d + 1 sin π dn  ≥ 1, where n, d ∈ N and d ∈ / N.
13.3. Let S, p, q ∈ N, and suppose that S is divisible by q and leaves a remainder
1 on division by p. Prove that S ≤ pq − q(q− p−1)
p
if q > p, S < pq.
13.4. Let m, k ∈ N and suppose that all prime divisors of the number m are
k+1
less than or equal to an integer n such that m ≤ n 2 . Prove that m can be
represented as the product of k positive integers each of which is less than
or equal to n.
13.5. Let n ∈ N and let σ (n) denote the sum of all divisors of the number n (note
that σ (3)  4, σ (6)  12, σ (12)  28). Prove that if σ (a) > 2a and b ∈ N,
then σ (ab) > 2ab.
13.6. Find all numbers αsuch that for1every positive integer n there exists a positive
integer m such as α − mn  < 3n .
13.7. A number is called an interesting number if it is equal to the product of two
primes. What is the greatest number of consecutive interesting numbers?
 there is an infinite number of positive integers n such that S(2 ) >
n
13.8. Prove
 n+1that
S 2 , where S(a) is the sum of the digits of the positive integer a.
13.9. Let the positive integer n be represented as a sum of natural numbers. We
denote the number of all such representations by P(n) (for example, P(4) 

© Springer International Publishing AG, part of Springer Nature 2018 187


H. Sedrakyan and N. Sedrakyan, Algebraic Inequalities, Problem Books
in Mathematics, https://doi.org/10.1007/978-3-319-77836-5_13
188 13 Algebraic Inequalities in Number Theory

5, since 4  4, 4  3 + 1, 4  2 + 2, 4  2 + 1 + 1, 4  1 + 1 + 1 + 1).
Prove that P(n + 1) + P(n − 1) ≥ 2P(n), n  2, 3, . . . .
13.10. Let the positive integer a be divided (long division) by a positive integer b
yielding the number c0 .c1 c2 c3 . . .. Given that cn+1  cn+2
  · · ·  cn+k  9
for some n ∈ {0, 1, . . .}, k ∈ N, prove that, (a) k ≤ 10 b
, (b) k ≤ log b .
13.11. Find for every positive integer n the greatest number f (n) such that from
the numbers 1, 2, . . . n one can choose f (n) numbers such that no two of
them have ratio equal to 2.
13.12. Let n be a positive integer. Prove that n can be represented as the sum of the
squares of m positive integers, where 3 log4 n + 5 ≤ m ≤ n − 14.
13.13. Letb a and
 b be distinct positive integers greater than or equal to 3. Prove that
a − ba  > 0.62ba−1 .

Proofs
√ √
√ −m
2 2
13.1. We have 2− m
 2n−m
≥ n √2n+m
 1 2n
, since 2n 2 − m 2 is a
n ( 2n+m )
√ ( )
n n

positive integer. On the other hand, m < 2n, and therefore 2 − mn ≥

√1
n ( 2n+m )
> n √2n+1√
 √1 . Hence, we have obtained that 2 − mn >
( √  2n ) 2 2n 2
√1 , or m < 2 1 − 4n1 2 .
2 2n 2

n
√ √
13.2. Let dn  k, whence k < dn < k + 1.
Consider the following two cases.

(a) If k < dn < k + 21 , then
 √ 
 √   √ 

√ 

n d + 1 sin(π dn)  n d + 1 sin π dn − k  ≥
 √  √   √  dn 2 − k 2 √
2n d + 2
≥ n d + 1 2 dn − k  2n d + 2 √ ≥ √ > 1.
dn + k dn + k

Here we have used that sin x ≥ π2 x, where 0 ≤ x ≤ π2 (this can be


proved using derivatives).  √  
√  √ 
(b) If k + 21 < dn < k + 1, then n d + 1 sin π k + 1 − dn  ≥

2n √d+2
k+1+ dn
> 1.
13.3. By to the assumption of the problem, we have that S can be represented in
the following way: S  p(q − i) + 1  pq − ( pi − 1)  pq − q j, where
i, j ∈ N.
We have pi − 1  q j and q > p, and hence i > j, where i ≥ j + 1 and
q j ≥ p( j + 1) − 1, or (q − p) j ≥ p − 1. Therefore, since q > p, we have
S  pq − q j ≤ pq − q(q− p−1)
p
.
13.4. Consider all possible representations of the number m  a1 · · · · · ak , where
a1 ≤ a2 ≤ . . . ≤ ak and a1 , a2 , . . . , ak ∈ N. Note that the number of such
Proofs 189

representations is finite. Let m  a1 a2 · · · ak be the representation among


these representations such that ak is the smallest possible.
If ak ≤ n, then this ends the proof. Assume that ak > n. Then ak is not a
prime number, since m|ak . Let p be the smallest prime divisor of ak . Therefore,

p ≤ ak .
We have m  ( pa1 )a2 · · · · · ak−1 · apk , and hence pa1 ≥ ak , where a1 ≥
√ √
ak > n.
√ k−1 k+1
Therefore, m  a1 a2 · · · · · ak−1 ak > n · n  n 2 , which leads to a
contradiction.
13.5. Let the numbers 1  a1 < · · · < ak  a be all the divisors of a. Then
the numbers ba1 < · · · < bak are the divisors of ab, whence σ (ab) ≥
ba1 + · · · + bak  bσ (a) > 2ab.  
13.6. Without loss of generality one can assume that α ∈ [0, 1), since α − mn  
 
{α} − m−[α]n .
n
Note that α  0 satisfies the condition of the problem, and it is sufficient to
take m  0. Let us prove that if 0 < α < 1, then there exists a positive integer
n such that for every integer m we have α − mn  ≥ 3n 1
.
p
If α ∈ Q, then α  q , where 0 < p < q, p, q ∈ Z. Assume that α satisfies
the condition of the problem. If n  q, q + 1, . . . , 2q, then the numbers
  1p, (q +1 1)α,
qα  . . . , 2qα  2 p belong
 to the union of thesets
p − 3 , p + 3 , p + 1 − 13 , p + 1 + 13 , . . . , 2 p − 13 , 2 p + 13 . Since the
number of these numbers is equal to q + 1, and the number of intervals is
equal to p + 1, we have according to the Dirichlet’s principle that there exist
i, j ∈ {q, q + 1, . . . , 2q} i  j, such that the numbers αi , α j belong to the
same interval. Hence we obtain that α < 23 . On the other hand, there exists a
number l ∈ {q, q + 1, . . . , 2q} such that the numbers αl and α(l + 1) belong
to different intervals. Therefore α > 13 . Hence if 13 < α < 23 , then for n  1
there does not exist an integer m such that |α − m| < 13 . This leads to a
contradiction.
If α is an irrational number, then there exists n 0 ∈ N such that l + 25 < αn 0 <
l + 35 , where l ∈ Z (see Problem
 14.25).

 m0 
Assume that m 0 ∈ Z and α − n 0  < 13 .
If l  m 0 , then |αn 0 − m 0 |  |αn 0 − l + l − m 0 | ≥ |l − m 0 | − |αn 0 − l| >
1 − 35  25 > 13 .
If l  m 0 , then |αn 0 − m 0 |  |αn 0 − l| > 25 > 13 .
Thus, only integers satisfy the condition of the problem.
   
Alternative solution. Let n ∈ N, k, m ∈ Z and α − m  < 1 and α − k  <
n 3n 2n
1
6n
.      
If k  2m, then |k − 2m|≥ 1; thus 1
2n
≤  mn − k 
2n
≤ α − k  m
2n
+ n − α <
1
3n
1
+ 6n  2n
1
,
190 13 Algebraic Inequalities in Number Theory
 
which leads to a contradiction. Let k  0, 1, 2, . . ., α − m2kk  < 3·21 k , and
m k ∈ Z. Then m 0  m21  · · ·  m2kk  · · ·, and hence |α − m 0 |< 3·21 k .
Therefore α  m 0 ∈ Z.  
If α ∈ Z, then it is obvious that for m  αn we have m ∈ Z and α − mn  
0 < 3n1
.
13.7. Let us provide an example of three consecutive interesting numbers. 33 
3 · 11, 34  2 · 17, 35  5 · 7.
Let us prove that there do not exist four consecutive interesting numbers. We
argue by contradiction. Assume that there exist four consecutive interesting
numbers, Then one of these numbers is divisible by 4. Therefore, that number
is equal to 4. Thus, it follows that one of the numbers is equal to either 5 or 3,
neither of which is an interesting number. This is thedesired contradiction.
13.8. Assume that n satisfies the inequality S(2n ) > S 2n+1 . Therefore, there exists
a positive integer n 0 such that for all n ≥ n 0 , n ∈ N,
   
S 2n ≤ S 2n+1 (1)
 
Note that S(2n )  S 2n+1 is impossible, for otherwise, we would have that
2n  2n+1 − 2n is divisible by 9, which leads to a contradiction.
Let us prove the following lemmas.
 
Lemma 1 If n ≥ n 0 , then S 2n+6 ≥ S(2n ) + 27.
Indeed, on dividing the numbers 2n , 2n+1 , 2n+2 , . . . , 2n+6 by 9, we obtain
as remainders seven consecutive terms of the following periodic sequence:
1, 2, 4, 8, 7, 5, 1, 2, 4, 8, 7, 5, 1, 2, . . .. Hence from (1), it follows that
                  
S 2n+6 − S 2n  S 2n+6 − S 2n+5 + S 2n+5 − S 2n+4 + . . . + S 2n+1 − S 2n ≥

≥ 1 + 2 + 4 + 8 + 7 + 5  27.

Using Lemma 1 and induction, one can easily prove the following Lemma 2.
 
Lemma 2 S 2n 0 +6k ≥ S(2n 0 ) + 27k, where k ∈ N.
 
Lemma 3 S 2n 0 +6k < 9m + 18 k, where k ∈ N, and m is the number of
digits of 2n 0 .
Indeed, we have 2n 0 < 10m , where 2n 0 +6k < 10m+2k ; hence the number of
the digits of 2n 0 +6k is not greater than (m + 2k), and therefore, S 2n 0 +6k <
9(m + 2k).
Note that Lemmas 2 and 3 contradict each other.
This leads to a contradiction.
13.9. Let x1 , x2 , . . . , xk ∈ N, x1 ≥ x2 ≥ · · · ≥ xk , and n  x1 + x2 + · · · + xk .
Therefore, n + 1  x1 + x2 + · · · + xk + 1; hence one can create a one-to-one
correspondence between partitions of the number n and the partitions of the
number n + 1 that contain 1. It follows that
   
x1 , x2 , . . . , xk ←→ x1 , x2 , . . . , xk , 1 .
Proofs 191

Hence, the number p(n + 1) − p(n) is equal to the number of the partitions
of n + 1 not containing 1.
If n  y1 + y2 +· · ·+ yl , where y1 ≥ y2 ≥ · · · ≥ yl ≥ 2, then n +1  (y1 + 1)+
y2 + · · · + yl , and the number of partitions of the number n + 1 not containing
1 is greater than or equal to the number of similar partitions of the number n.
Therefore, p(n + 1) − p(n) ≥ p(n) − p(n − 1), or p(n + 1) + p(n − 1) ≥
2 p(n), n  2, 3, . . ..
13.10. Let

 b
a 
. . . c c . . .
.. n+1 n+2
.
10an+1

cn+1 b
10an+2

cn+2 b

..
.

We have that 10an+1 − 9b  an+2 , 10an+2 − 9b  an+3 , . . . , 10an+k − 9b 


an+k+1 ,
where 0 < an+i < b, i  1, 2, . . . , k,an+k+1 ≥ 0, whence 9b 10

an+1 , an+2 , . . . , an+k < b.
b
(a) Assume that k > 10 . Then among the numbers an+1 , an+2 , . . . , an+k
there are two equal numbers. Therefore, b
A  max an+i − min an+i ≤ b−1− 9b 10
 10
b
−1, hence A ≤ 10 −1 <
1≤i≤k 1≤i≤k
k − 1. It follows that there exist numbers p < q such that an+ p  an+q .
Hence, we obtain
10an+ p − 9b  an+ p+1 , 10an+ p+1 − 9b  an+ p+2 , . . . , 10an+q−1 − 9b 
an+q  an+ p , and summing these equations, we deduce that an+ p + . . . +
an+q−1  (q − p)b, which leads to a contradiction. Hence, we obtain
that k ≤ 10 b
.
(b) Define an+i  b − bn+i i  1, 2, . . . , k + 1. It then follows that 1 ≤
bn+1 , bn+2 , . . . , bn+k < b and 1 ≤ bn+k+1 ≤ b.
Note that bn+2  10bn+1 , bn+3  10bn+2 , . . . , bn+k+1  10bn+k  , and

therefore, bn+k+1  10k bn+1 , where 10k ≤ 10k bn+1 ≤ b, or k ≤ log b .

Remark If b  10k and a  99 . . . 9, then



a
b
 0, 99 . . . 9.

k log b

13.11. Consider the set X  {4α q | 4α q ≤ n, α ∈ Z0 , q odd}. It is clear that the ratio
of two elements of the set X is never equal to 2, and therefore f (n) ≥ |X |,
where we denote by |X | the cardinality of the set X.
Let Υ ⊂ {1, 2, . . . , n} and |Υ | > |X |, and let us put every number a
from X into correspondence with the pair (a, 2a). We have that 2a ∈ / X and
|{(a, 2a)}|  |X |. Therefore, for some a ∈ Υ , we have 2a ∈ Υ . It follows
that f (n) ≤ |X |; hence f (n)  |X |.
192 13 Algebraic Inequalities in Number Theory

Let n  2k1 +2k2 +. . .+2km , where k1 > k2 > . . . > km ≥ 0 and k1 , k2 , . . . , km


are integers.
Consider the following sets:

X 0  {2k1 + 2k2 + · · · + 2ki |i ∈ {1, 2, . . . , m}, ki even},


X i  {2r1 + 2r2 + · · · + 2rs |s ≥ i, r1 > r2 > . . . > rs ≥ 0, rs even and
r1  k1 , r2  k2 , . . . , ri−1  ki−1 , ri < ki , } i  1, 2, · · · , m.

Obviously, X 0 , X 1 , . . . , X m are mutually disjoint sets and X  X 0 ∪ X 1 ∪


· · · ∪ X m . Therefore, |X |  |X 0 | + |X 1 | + · · · + |X m |.
Let us denote by λ the number of even numbers among the numbers
k1 , k2 , . . . , km
and the number of odd numbers by μ.
We have |X 0 |  λ. Let us prove that
⎧ k +1

⎪ 2 i −1
⎨ if ki is odd,
|X i |  3


ki +1
−2
⎩2 if ki is even.
3

Indeed, let ki be an even number. We have that 2ri + ... + 2rs ≤ 2ki −1 + ... +
20  2ki − 1. Note that |X i | is one less than the number of numbers from
1, 2, 3, · · · , 2ki whose canonical decomposition contains an even power of 2,
that is,

2ki −2ki −1 + 2ki −2 − 2ki −3 + · · · + 22 − 2 + 1 − 1


(−2)ki +1 − 1 2ki +1 − 2
 −1 .
−2 − 1 3

If ki is odd, then the proof can be obtained in a similar way. Thus, it follows
that f (n)  λ + 23 n − 23 λ − μ3  23 n + λ−μ  23 n + (−1) +···+(−1)
k1 km

3 3
.
13.12. We have that n  1 + .
2
. . + 1, n  3 + 3 + 1 + .
2 2 2 2
. . + 1 and n  22 + 22 +
2

n n−18
32 + 12 + .
. . + 12 . Therefore, n can be represented as the sum of the squares
n−17
of m positive integers, where m ∈ {n − 16, n − 14, n}.
On the other hand, if n  4k + 4k + 4k + 4k + x12 + . . . + xm−4 2
, then n 
4 + x1 + . . . + xm−4 , where k  0, 1, . . . ; hence n can be represented as the
k+1 2 2

sum of the squares of m − 3 positive integers.


Thus, taking into consideration that n  12 + · · · + 12 , it follows that n can
n
be represented as the sum of the squares of m positive integers, where m ∈
{n, n − 3, n − 6, . . .} and m ≥ c0 +c1 +· · ·+ck , where ck ck−1 . . . c0 represents
n written in the base-4 number system.
Proofs 193
 
Therefore, c0 +c1 +· · ·+ck ≤ 3(k + 1) and 4k ≤ n < 4k+1 ; hence k  log4 n .
In order to complete the proof, it is sufficient to note that
{n, n − 3, n − 6, . . .} ∪ {n − 14, n − 17, n − 20, . . .} ∪ {n − 16, n − 19, n − 22, . . .} ⊃
   
⊃ n − 14, n − 15, n − 16, . . . , 3 log4 n + 5 .

13.13. Let us prove the following lemmas.


 n
Lemma 1 If n ∈ N, then 1 + n1 < 3.
By mathematical induction, one can easily prove that if k ∈ N and n 
1, 2, . . . , k, then
 
1 n n n2
1+ <1+ + 2 (1)
k k k
 n
If n  1, then we have 1 + k1  1 + k1 < 1 + k1 + k12 .
Assume that for n  m ≤ k − 1, inequality (1) holds, and let us prove that it
holds for n  m + 1.
 m+1    m    2

Note that 1 + k1  1 + k1 · 1 + k1 < 1 + k1 · 1 + mk + mk 2 < 1 +
+ (m+1)
2
m+1
k k2
, since m 2 < (m + 1)k.
This ends the proof of inequality (1).
 k
For n  k we obtain 1 + k1 < 3.

Lemma 2 If a, b ∈ N and a > b ≥ 3. Then ba−1 > a b−1 .  n


For n ∈ N and n ≥ 3, using Lemma 1, we deduce that n ≥ 3 > 1 + n1 .
Therefore,
1 1
n n > (n + 1) n+1 . (2)
1 1
Thus, from (2) it follows that b b > a a , and therefore ba > a b > b · a b−1 ,
whence ba−1 > a b−1 .
From Lemma 2 it follows that without loss of generality one can assume that
a > b. Let a  b + c, where c ∈ N.
Consider the following two cases:

(a) b ≥ 4.
We obtain
     
 b     c b 
a − ba   (b + c)b − bb+c   bb  1 + − bc  
b
    c−1 
 c  bc  c−1  c  bc c  bc c b
 bb b − 1 +  b + bc−2 1 + + ... + 1 + ≥ bb |b − (1 + ) c |bc−1 
b b b b
      
 c  bc      c  bc 
 ba−1 b − 1 +
b , therefore, a b − ba  > ba−1 b − 1 +
b . (3)
194 13 Algebraic Inequalities in Number Theory

For c > 1, using Problem 2.1 for (1 + c/b), . . . , (1 + c/b), 1, we deduce



  c−1 
c−1
 c−1  b
that ( b ) c ( b ) > c 1 + bc
b
1+ c +...+ 1+ c +1
, or 1 + c−1b
> 1 + bc c .
 b  b
Hence, we have 1 + bc c ≤ 1 + b1 < 3, and therefore,
  b 
 c c
b − 1 + b  > 1.
 
From (3), it follows that a b − ba  > ba−1 .
(b) If b  3, then by induction one can easily prove that 2.38 · 3a−1 > a 3 ,
where a  4, 5, . . ..

This ends the proof.

Problems for Independent Study

Prove the following inequalities (1–4).


√  √ √ 
   p
1.  a − p+aq p+q  ≤ q
p
, where a, p, q ∈ N, a ∈
/ N and  a − q
≤ 1.
2. (a) S(mn) ≤ S(m)S(n), where n, m ∈ N.
(b) S(m + n) ≤ S(m) + S(n), where n, m ∈ N
(see Problem 13.8).
3. S(1981n ) ≥ 19, where n ∈ N.
4. S(1998n ) > 106 for all positive integers n starting from some number.
5. Prove that for every number M there exists a positive integer n such that SS(n) >
(n 2 )
M.
6. Prove that 52l can be represented as the sum of the squares of m numbers, where
l ∈ N and 1 ≤ m ≤ 2l .
7. Find the smallest positive integer n such that for every partition of the positive
integers 1, 2, . . . , n into two groups, in one of them there are three numbers
forming a geometric progression.
8. Let a1 , a2 , . . . , ak be integers such that 1 < a1 < a2 < · · · < ak ≤ n and
a1 · a2 · · · ak is not divisible by ai2 , i  1, 2, . . . , k. Prove that k ≤ π(n), where
π (n) is the number of primes less than or equal to n.
S(n)
9. Let k be a given positive integer. Prove that the sequence S(kn) is bounded if and
α β
only if k  2 · 5 , where α, β ∈ {0, 1, . . .}.
10. Let x ≥ 1 and let A(x) be the least common multiple of the numbers 1, 2, . . . , [x].
Prove that
 
(a) A x2 · [x]! ≥ A(x), where x ≥ 2,
([ x2 ]!)2
(b) A(x) < 5 , x

(c) A(2n) · A( 2n ) ≥ A(n) · (2n)! , where n ≥ 2, n ∈ N,


3 √  (n!)2
(d) A(2n) > A(n) A 2n , where n ∈ N, n ≥ 100,
Problems for Independent Study 195

(e) there exists a prime number belonging to (n, 2n), where n ∈ N, n > 1.

Remark If we assume that


√there
 is no primenumber belonging to (n, 2n),
√ 
prove that A(2n)|A(n)A 2n (i.e., A(n)A 2n divides A(2n)).

11. Prove that d(1)


12
+ d(2)
22
+ · · · + d(n)
n2
< 2.78, where d(k) is the number of the positive
integer divisors of k.
 ∞ 2  ∞  ∞   ∞ 
 1  1  1  d(k)
Remark k 2  i 2 j 2  k 2 .
k1 i1 j1 k1

12. Find all numbers α such that the following  condition


 holds: for every positive
integer n there exists an integer m such as α − mn  ≤ 3n
1
.
13. Let a1 ≤ a2 ≤ · · · ≤ an be positive integers such that no sum of some of them
is equal to the sum of some of the others. Prove that
(a) a1 + a2 + · · · + ak ≥ 2k − 1, k  1, 2, . . . , n,
(b) a11 + a12 + · · · + a1n ≤ 1 + 21 + · · · + 2n−1
1
.
14. Let a1 , . . . , an are distinct positive integers. Prove that

a13 + · · · + an3 ≥ (a1 + · · · + an )2 .

Remark Let a1 < · · · < an and ai  i + bi . Then b1 ≤ · · · ≤ bn .


Chapter 14
Miscellaneous Inequalities

In this chapter we consider miscellaneous inequalities, and we mostly use various


proof techniques not included in the previous chapters.

Problems

Prove the following inequalities (14.1–14.5).


  
14.1. a12 + b21 + · · · + an2 + b2n ≥ (a1 + · · · + an )2 + (b1 + · · · + bn )2 .
 √
14.2. (a−b) − ab ≤ √(a−b)
2 2
a2 +b2
2(a+b)
≤ 2 2(a+b)
, where a, b > 0.
1 1 1 1
14.3. x2n − x1n ≤ (x2 − α) n − (x1 − α) n , where 0 ≤ α ≤ x1 ≤ x2 , n ∈ N.
  
14.4.  n a1n + · · · + akn − n bn1 + · · · + bnk  ≤ |a1 − b1 | + · · · + |ak − bk |, where n ≥
2, n ∈ N, a1 > 0, . . . , ak > 0, b1 > 0, . . . , bk > 0.
14.5. 1+1√3 + √5+1√7 + · · · + √9997+1√9999 > 24.
14.6. Given that a2 − 4a + b2 − 2b + 2 ≤ 0, c2 − 4c + d 2 − 2d + 2 ≤ 0 and
e2 − 4e + f 2 − 2f + 2 ≤ 0, find the greatest possible value of the expression
(a − c)(f − d ) + (c − e)(b − d ).
14.7. Prove that if an increasing sequence of positive numbers a1 , a2 , . . . , an , . . .
is unbounded, then there exists a sufficiently large index k such that the
following inequalities hold:
(a) a1
a2
+ aa23 + · · · + aak+1
k
< k − 21 ,
(b) a1
a2
+ a3 + · · · + ak+1 < k − 1985.
a2 ak

  1r   1r

n 
n
14.8. Minkowski’s inequality: Prove that (ai + bi ) r
≤ air +
i1 i1
 n  1r
 r
bi , where ai ≥ 0, bi ≥ 0, i  1, . . . , n.
i1

© Springer International Publishing AG, part of Springer Nature 2018 197


H. Sedrakyan and N. Sedrakyan, Algebraic Inequalities, Problem Books
in Mathematics, https://doi.org/10.1007/978-3-319-77836-5_14
198 14 Miscellaneous Inequalities

14.9. Young’s inequality: Prove that if the function f (x) is increasing and contin-
a b
uous on [0, +∞) and f (0)  0, then f (x)dx + f −1 (y)dy ≥ ab, where the
0 0
function f −1 (x) is the inverse function of the function f (x) and a > 0, b > 0.
14.10. (a) Prove that x1 y1 + · · · + xn yn  x1 (y1 − y2 ) + (x1 + x2 )(y2 − y3 ) + . . . +
(x1 + · · · + xn−1 )(yn−1 − yn ) + (x1 + . . . + xn )yn .
(b) Prove that if x1 ≥ x2 ≥ · · · ≥ xn and y1 ≥ y2 ≥ · · · ≥ yn , then
x1 yn + x2 yn−1 + · · · + xn y1 ≤ x1 yi1 + x2 yi2 + · · · + xn yin ≤ x1 y1 + · · · + xn yn ,
where i1 , i2 , . . . , in is some permutation of the numbers 1, 2, . . . , n.
14.11. Prove that

(a) x1 + · · · + xn ≥ n, where n ≥ 2, x1 > 0, . . . , xn > 0, and x1 · · · xn  1,


(b) a1 (a12 +1) +· · ·+ an−1 (a1 n +1) + an (a11 +1) ≥ 1+a1n···an , where n ≥ 2, (n−3)(ai −1) ≥
0, ai > 0, i  1, . . . , n,

(c) 3 + (A + M + S) + A1 + M1 + S1 + MA + MS + AS ≥ 3(A+1)(M +1)(S+1)


AMS+1
, where
A
> 0, M >

10, S > 0,

(d) a1 + 1b + 1c 1+a + 1+b 1


+ 1+c 1
≥ 1+abc
9
, where a > 0, b > 0, c > 0,
√ √ √
a+ b+ c
(e) √a
a+b
+ √b
b+c
+ √c
c+a
≥ √
2
, where a > 0, b > 0, c > 0.

14.12. Schur’s inequality: Prove that ϕ(x1 ) + · · · + ϕ(xn ) ≥ ϕ(y1 ) + · · · + ϕ(yn ),


where y1 ≥ y2 ≥ · · · ≥ yn , x1 + · · · + xk ≥ y1 + · · · + yk , k  1, . . . , n − 1,
x1 + · · · + xn  y1 + · · · + yn , and for every x in I, one has ϕ  (x) > 0
(x1 , . . . , xn , y1 , . . . , yn ∈ I ).
14.13. Prove that b1 + · · · + bn ≥ a1 + · · · + an , where a1 ≥ · · · ≥ an > 0, b1 ≥ a1 ,
b1 b2 ≥ a1 a2 , . . . , b1 · . . . · bn ≥ a1 · . . . · an . √
14.14. Let α, β, γ be angles of an acute triangle. Prove that 2 < sin α2 + sin β2 +
sin γ2 ≤ 23 .
√ √
√ α, β, γ be √
14.15. Let angles of an obtuse triangle. Prove that 0 < sin α + sin β +
sin γ < 1 + 4 8. √
14.16. Prove that 1 + √12 + √13 + · · · + √1n > 2 n − 2, where n ∈ N.
14.17. Prove that a > 0, b > 0, c > 0, d > 0, if S1  a + b + c + d , S2 
ab + bc + cd + ad + ac + bd > 0, S3  abc + bcd + cda + abd > 0,
S4  abcd > 0.

n
14.18. Prove that n! < n ne , where n ≥ 8 and n ∈ N.

14.19. Prove that 1 + 21 · · · 1 + 21n < 3, where n ∈ N.


1

14.20. Prove that if g is a convex function on [0, a1 ] (see Chapter 11) and a1 ≥
a2 ≥ · · · ≥ a2m ≥ a2m+1 ≥ 0, then g(a1 ) − g(a2 ) + g(a3 ) − · · · + g(a2m+1 ) ≥
g(a1 − a2 + a3 − · · · + a2m+1 ).
14.21. Let the absolute value of the polynomial p(x) on [−1, 1] be less than or equal
to 1. Prove that

(a) |a| + |b| + |c| ≤ 3, where p(x)  ax2 + bx + c,


(b) a2 + b2 + c2 ≤ 5, where p(x)  ax2 + bx + c,
(c) |a| ≤ 4, where p(x)  ax3 + bx2 + cx + d .
Problems 199

14.22. Let the polynomial p(x)  xn + an−1 xn−1 + · · · + a0 (n > 1) have n negative
roots. Prove that a1 p(1) ≥ 2n2 a0 .
14.23. The complete graph on n vertices has each of its edges colored in one of
two given colors. We denote by t(n) the number of triangles whose sides are
colored in the same color. Prove that


⎪ k(k−1)(k−2) , if n  2k,

⎨ 3

t(n) ≥ 23 k(k − 1)(4k + 1), if n  4k + 1,





⎩ 2 k(k + 1)(4k − 1), if n  4k + 3.
3

14.24. Consider a graph having n (n ≥ 3) vertices. The number of its edges is


2
greater than n4 . Prove that the number of triangles formed by these edges is
n
not less than 2 .
14.25. Prove that if α, (α > 0) is an irrational number and 0 < a < b < 1, then
there exists a positive integer n such that a < {nα} < b.
14.26. Let x1 + · · · + xn  0 and xi ∈ [m, M ], i  1, . . . , n. Prove that

n
(a) xi2 ≤ −mMn,
i1
n

(b) xi4 ≤ −mMn · m2 + M 2 + mM .


i1

m
14.27. Prove that x2 + y2 ≥ 2m xm ym + (xm − ym )2 , where m ∈ N.
14.28. Prove that cos(α − β)cos(β − γ )cos(γ − α) ≥ 8 cosα cosβ cosγ , where
α, β, γ arethe angles of some triangle. 
14.29. Prove that sin x + sin22x + · · · + sinnnx  < 3.
14.30. Prove that a1 +a
2
· 2 · · · an−12+an · an +a
2 a2 +a3
2
1
≤ a1 +a32 +a3 · a2 +a33 +a4 · · · an +a31 +a2 , where
0 < a1 ≤ a2 ≤ · · · ≤ an and n ≥ 3.
14.31. Prove that if x, y, z ≥ 0 and x2 + y2 + z 2  1, then

y
(a) 1 ≤ x
+ z
+ 1−xy ≤ 323,
1−yz 1−zx
y

(b) 1 ≤ x
1+yz
+ 1+zx
z
+ 1+xy ≤ 2.

14.32. For which values of λ does the inequality


a b c 3
√ +√ +√ ≥√ (14.1)
a2 + λbc b2 + λca c2 + λab 1+λ

hold for arbitrary positive numbers a, b, c?


14.33. Prove that for every triangle with sides a, b, c, one has
√ √ √
a2 + ab + b2 + b2 + bc + c2 + c2 + ca + a2

≤ 5a2 + 5b2 + 5c2 + 4ab + 4bc + 4ca.
200 14 Miscellaneous Inequalities

14.34. Prove that


x13 +···+xn3 (1−x1 )3 +···+(1−xn )3
(a) x1 ···xn
≥ (1−x1 )···(1−xn )
, where n ≥ 3, 0 ≤ xi ≤ 21 , i  1, . . . , n,
x14 +···+xn4 (1−x1 )4 +···+(1−xn )4
(b) x1 ···xn
≥ (1−x1 )···(1−xn )
, where n ≥ 4, 0 ≤ xi ≤ 21 , i  1, . . . , n.

14.35. Let n ≥ 2 and let a1 , . . . , an be distinct positive integers. Prove that

a12015 + · · · + an2015 12015 + · · · + n2015


≥ .
a1 + · · · + an2000
2000 12000 + · · · + n2000
√    √ 
14.36. Prove that 1 + x2 + 1 + y2 + (1 − x)2 + (1 − y)2 ≥ 1 + 5 (1 − xy),
where 0 ≤ x ≤ 1, 0 ≤ y ≤ 1.
14.37. Prove that if xi > 0, i  1, . . . , n and x1 · · · xn  1, then

(a) √1
1+x1
+ ··· + √1
1+xn
≤ 2
2
· n, where n  2, 3, . . .,
(b) √1
1+x1
+ ··· + √1
1+xn
< n − 1, where n ≥ 4.

14.38. Prove that


n
xiα −xi
x1 +···+xn +xiα −xi
≥ 0, where n ≥ 2, xi > 0, i  1, . . . , n, x1 · · · xn ≥ 1,
i1
and α ≥ 1.

Proofs

14.1. Consider the points


A1 (0, 0), A2 (a1 , b1 ), A3 (a1 + a2 , b1 + b2 ), . . . , An+1 (a1 + · · · + an , b1 + · · · + bn ).

Since A1 A2 + A2 A3 + · · · + An An+1 ≥ A1 An+1 , it follows that


   
a12 + b21 + a22 + b22 + · · · + an2 + b2n ≥ (a1 + · · · + an )2 + (b1 + · · · + bn )2 .
 √
a2 +b2
14.2. Let us rewrite the expression 2
− ab in the following way:
  
 a2 +b2
√ a2 +b2

√ − ab + ab
a2 + b2 2 2 (a − b)2
− ab   √   .
2 a2 +b2 a2 +b2

2 + ab 2 2 + ab

√ 
a2 +b2
It is left to prove that + ab ≤ a + b. The left-hand side of this
a+b
√ ≤ 2
 2
2
a2 +b2
√ 2 2 2
inequality obviously holds, since 2
+ ab > a +b 2
+ ab  (a+b)
2
.
In
 order to prove the right-hand side, let us prove the following inequality:

− 2 ≤ 2 − ab, or (a−b)  ≤  (a−b)
2 2
a2 +b2
√ .
a+b a+b
2 2 2 a+b a +b 4 + ab
4 2 + a+b
2 2
Proofs 201
 √
a2 +b2
Note that this inequality holds, because 2
≥ ab.
14.3. We have
1 1 x2 − x1
x2n − x1n   n−1  n−2  1 n−1
1 1 1
x2n + x2n · x1n + · · · + x1n

x2 − x1
≤      
1 n−1 1 n−2 1 1 n−1
(x2 − α) n + (x2 − α) n · (x1 − α) n + · · · + (x1 − α) n
1 1
 (x2 − α) n − (x1 − α) n .
 
14.4. Let us rewrite the expression n a1n + · · · + akn − n bn1 + · · · + bnk in the following
way:
   an + · · · + an − bn − · · · − bn 
n n n 1 1
 a1 + · · · + ak − b1 + · · · + bk   ,
n n n k k
An−1 + An−2 B + · · · + Bn−1
 
where A  n a1n + · · · + akn , B  n bn1 + · · · + bnk .
Therefore,
    
 
(a1 − b1 ) a1n−1 + · · · + bn−1
1 + · · · + (ak − bk ) akn−1 + akn−2 bk + · · · + bn−1
k 
|A − B| 
An−1 + An−2 B + · · · + Bn−1
   n−1 
 an−1 + · · · + bn−1  a n−1 
   k + · · · + bk 
≤ |a1 − b1 | ·  1n−1 1
 + · · · + |a k − b k | ·  
A + · · · + Bn−1   An−1 + · · · + Bn−1 
≤ |a1 − b1 | + · · · + |ak − bk |,

as A > ai > 0, B > bi > 0 (i  1, . . . , n).


14.5. We have
 
1 1 1
2· √ +√ √ + ··· + √ √
1+ 3 5+ 7 9997 + 9999
1 1 1 1 1 1
 √ + √ +√ √ +√ √ + ··· + √ √ +√ √
1+ 3 1+ 3 5+ 7 5+ 7 9997 + 9999 9997 + 9999
1 1 1 1 1 1
> √ ++√ √ +√ √ +√ √ + ··· + √ √ +√ √
1+ 3 3+ 5 5+ 7 7+ 9 9997 + 9999 9999 + 10001
√ √ √ √ √ √ √ √
3−1 5− 3 7− 5 10001 − 9999 10001 − 1 100 − 1
 + + + ··· +  > > 48,
2 2 2 2 2 2

whence 1+1√3 + √5+1√7 + · · · + √9997+1√9999 > 24.


√ b), B(c, d ), C(e, f ) are inside the circle with
14.6. We have that the points A(a,
center O(2, 1) and radius 3.
202 14 Miscellaneous Inequalities

→ →
Consider the vectors BA{a − c, b − d }, BC{e − c,⎛
f − d} ⎞and

→ → → → →
BD{f − d , c − e}. Note that BC · BD  0, and therefore, ⎝BA, BD⎠ 
⎛ ⎞ ⎛ ⎞
∧ ∧
 ◦  → → → →
90 − β , or ⎝BA, BD⎠  |90◦ + β|, where ⎝BA, BC ⎠  β.
 
→ →
It follows that BA · BD  BA · BD · |cos(90◦ ± β)|  BA · BD sin β 

BA · BC sin β  2SABC . √ √
Hence, |(a − c)(f − d ) + (c − e)(b − d )|  2SABC ≤ 2 · 9 4 3  9 2 3 , since

among all triangles inscribed in a circle of radius 3, the one with the largest
area is an equilateral triangle (see the proof of Problem 2.7).
It is left to note

that the expression (a − c)(f − d )+(c − e)(b − d ) can assume
the value 9 2 3 .

Thus, the greatest possible value of the given expression is 9 2 3 .
14.7. (a) If we define aa21  1 + α1 , aa23  1 + α2 , . . . , aak+1
k
 1 + αk , then we have
−1 < α1 < 0, . . . , −1 < αk < 0.
We have (1 + α1 ) · · · (1 + αk )  aak+11
.
Since the sequence (ak ) is unbounded, there exists a number k0 such that
k ≥ k0 aak+1 1
< 21 , and therefore, 21 > (1 + α1 ) · · · (1 + αk ) ≥ 1 + α1 +
· · · + αk (see Problem 10.6), and hence aa21 + aa23 + · · · + aak+1
k
< k − 21 .
ak1
(b) Let the numbers k1 , k2 , . . . , k2·1985 be such that a2 + · · · + ak +1 < k1 − 21 ,
a1
1
ak1 +1 a ak
+ · · · + ak k2+1 < (k2 − k1 ) − 21 , . . . , aks−1 +2 + · · · + aak k+1 < (ks − ks−1 ) −
+1 s
ak1 +2 2 s−1 s

2 (S
1
 2 · 1985).
Summing these inequalities, we obtain
a1
a2
+ aa23 + · · · + aak k+1
s
< ks − S · 21  ks − 1985; hence for k ≥ kS , it follows
s
that aa21 + aa23 + · · · + aak+1
k
< k − 1985, since aam+1
m
< 1.

n n n
14.8. (ai + bi ) 
r
ai (ai + bi ) + bi (ai + bi ) .
r−1 r−1
i1 i1 i1
By Problem 11.12, we have
 n 1/r  n (r−1)/r

n  

r−1 r/(r−1)
ai (ai + bi ) r−1
≤ air · (ai + bi )
i1 i1 i1

and
 n 1/r  n (r−1)/r

n  

r−1 r/(r−1)
bi (ai + bi ) r−1
≤ bri · (ai + bi ) .
i1 i1 i1
Proofs 203

It follows that
 n (r−1)/r ⎛ 1/r  n 1/r ⎞

n  
n 
(ai + bi ) ≤
r
(ai + bi ) r ⎝ air + bri ⎠,
i1 i1 i1 i1

or
 n 1/r  n 1/r  n 1/r
  
(ai + bi ) r
≤ air + bri .
i1 i1 i1

a b
14.9. For b < f (a) we have that f (x)dx  S1 + S2 , f −1 (y)dy  S3 , where
0 0
a b −1
ab  S1 + S3 , and therefore, ab ≤ f (x)dx + f (x)dx.
0 0
For the case b ≥ f (a) the proof is similar.
14.10. (a)

x1 (y1 − y2 ) + (x1 + x2 ) (y2 − y3 ) + · · ·


+ (x1 + · · · + xn−1 ) (yn−1 − yn )
+ (x1 + · · · + xn ) yn  x1 y1 + (x1 + x2 − x1 ) y2
+ · · · + ((x1 + · · · + xn ) − (x1 + · · · + xn−1 )) yn
 x1 y1 + x2 y2 + · · · + xn yn .

(b) We have

x1 yi1 + · · · + xn yin  yi1 (x1 − x2 ) + yi1 + yi2 (x2 − x3 ) + · · · + yi1 + · · · + yin−1 (xn−1 − xn )

+ yi1 + · · · + yin xn ≤ y1 (x1 − x2 ) + (y1 + y2 )(x2 − x3 )


+ · · · + (y1 + · · · + yn−1 )(xn−1 − xn ) + (y1 + · · · + yn )xn  x1 y1 + · · · + xn yn .

The second inequality can be proved in a similar way.


14.11. (a) Let x1  aa21 , x2  aa23 , . . . , xn−1  aan−1 n
, where a1 > 0, . . . , an > 0.
In this case, xn  a1 . Using Problem 14.10(b) for the numbers
an

a1 , a2 . . . , an and a11 , a12 , . . . , a1n , we obtain a1 · a11 + a2 · a12 + · · · + an · a1n ≤


a1 · a12 + a2 · a13 + · · · + an · a11 , or n ≤ x1 + x2 + · · · + xn .
(b) Note that
 
1 1
(1 + a1 · · · an ) + ··· +
a1 (a2 + 1) an (a1 + 1)
1 1 a2 · · · an a1 a2 · · · an−1
 + ··· + + + ··· +  A.
a1 (a2 + 1) an (a1 + 1) a2 + 1 a1 + 1

For n ≥ 3 we obtain A ≥ 1
a1 (a2 +1)
+· · ·+ an (a11 +1) + aa11+1 + a2 +1 +· · ·+ aan1+1
a2 a2 a3 an
 B.
204 14 Miscellaneous Inequalities

One can easily prove that a1 , b and 1


, a , (a, b
b+1 a+1
> 0) have the same
order; hence
1 ab 1 a 1
+ ≥ · + ·b (1)
a(b + 1) a + 1 a a+1 b+1

(see Problem 14.10(b)).


By (1), we obtain
     
1 a2 1 a3 1 a1
B≥ + + + + ··· + +
a1 + 1 a2 + 1 a2 + 1 a3 + 1 an + 1 a1 + 1
   
1 a1 1 an
 + + ··· + +  n.
a1 + 1 a1 + 1 an + 1 an + 1
 
Therefore, (1 + a1 · · · an ) a1 (a12 +1) + · · · + an (a11 +1) ≥ n.
For n  2, we have a1 , a2 ≤ 1; hence
 
1 1 a1 a2 1
(1 + a1 a2 ) +  + +
a1 (a2 + 1) a2 (a1 + 1) a1 + 1 a2 + 1 a1 (a2 + 1)
1 a1 a2 1 1
+ ≥ + + +  2.
a2 (a1 + 1) a1 + 1 a2 + 1 a1 + 1 a2 + 1

3+A+M +S+ 1 + 1
+1+ A
+M +S
(c) It is sufficient to prove that A M S M
(A+1)(M +1)(S+1)
S A
 A(M1+1) + M (S+1)
1
+
1
S(A+1)
; then it is sufficient to use inequality of Problem 14.11(b) for n  3.
1
(d) Let a(1+b) 1
+ b(1+c) 1
+ c(1+a) ≤ a(1+c)
1 1
+ b(1+a) 1
+ c(1+b) . Then since
1
a(1+b)
+ b(1+c) + c(1+a) ≤ a(1+a) + b(1+b) + c(1+c) (see Problem 14.10(b)).
1 1 1 1 1

Hence
  
1 1 1 1 1 1
+ + + +
a b c 1+a 1+b 1+c
 
1 1 1 9
≥3 + + ≥
a(1 + b) b(1 + c) c(1 + a) 1 + abc

(see Problem 14.10(b)).


(e) It is sufficient to note that √2ab
a+b
, √2bc
b+c
, √2ac
a+c
and √ 1 , √1 , √1
a+b b+c c+a
have opposite order. Thus it follows that
 2
a b c a2 b2 c2 2ab 1
√ +√ +√  + + +√ ·√
a+b b+c c+a a+b b+c c+a a+b b+c
2bc 1 2ac 1 a2 b2 c2 2ab 1
+√ ·√ +√ ·√ ≥ + + +√ ·√
b+c c+a a+c a+b a+b b+c c+a a+b a+b
   
2bc 1 2ac 1 a+b+c a+b ab b+c bc
+√ ·√ +√ ·√  + + + +
b+c b+c a+c c+a 2 4 a+b 4 b+c
√ √ √ 2
c +a ca  a+b+c √ √ √ a+ b+ c
+ + ≥ + ab + bc + ca  √ ,
4 c+a 2 2
Proofs 205
√ √ √
a b c a+ b+ c
or √ +√ +√ ≥ √ .
a+b b+c c+a 2

14.12. Let us begin by proving that ϕ(u) ≥ ϕ(x)+(u−x)ϕ  (x). Consider the function
f (x)  ϕ(u) − ϕ(x) + (x − u)ϕ  (x). Then f  (x)  (x − u)ϕ  (x), and hence in
the case x > u, it follows that f (x) is increasing, and for x < u it follows that
f (x) is decreasing. Thus, it follows that f (x) ≥ f (u)  0, or ϕ(u) ≥ ϕ(x) +
(u − x)ϕ  (x), and therefore ϕ(x1 ) ≥ ϕ(y1 ) + (x1 − y1 )ϕ  (y1 ), . . . , ϕ(xn ) ≥
ϕ(yn ) + (xn − yn )ϕ  (yn ).
We obtain ϕ(x1 ) + · · · + ϕ(xn ) ≥ ϕ(y1 ) + · · · + ϕ(yn ) + (x1 − y1 )ϕ  (y1 ) + · · · +
(xn − yn )ϕ  (yn ).
Now let us prove that (x1 − y1 )ϕ  (y1 ) + · · · + (xn − yn )ϕ  (yn ) ≥ 0.
By Problem 14.10(a), we have

(x1 − y1 )ϕ  (y1 ) + · · · + (xn − yn )ϕ  (yn )  (x1 − y1 ) ϕ  (y1 ) − ϕ  (y2 )




+ (x1 − y1 + x2 − y2 ) ϕ (y2 ) − ϕ  (y3 ) + · · · + (x1 − y1 + x2 − y2 + · · · + xn − yn )ϕ  (yn ) ≥ 0,

since x1 + x2 + · · · + xk ≥ y1 + y2 + · · · + yk and ϕ  (yk−1 ) − ϕ  (yk ) ≥ 0


(k  1, . . . , n − 1) (ϕ  (x) is increasing).
We deduce that ϕ(x1 ) + · · · + ϕ(xn ) ≥ ϕ(y1 ) + · · · + ϕ(yn ).
14.13. Consider the numbers ln a1 , . . . , ln an and ln b1 , . . . , ln bn−1 , ln bn , where
1 ···an
bn  ba1 ···b , and the function ϕ(x)  ex Then from Problem 14.12,
n−1

it follows that ϕ(ln b1 ) + · · · + ϕ ln bn ≥ ϕ(ln a1 ) + · · · + ϕ(ln an ), or


b1 + · · · + bn−1 + bn ≥ a1 + · · · + an .
1 ···an
On the other hand, bn  ba1 ···b ≤ bn , and therefore, b1 +· · ·+bn ≥ a1 +· · ·+an .
n−1

14.14. Consider the function f (x)  − sin 2x in 0, π2 .


We have f  (x)  14 sin 2x > 0 if x ∈ 0, π2 .


Let α ≥ β ≥ γ . Then we have α ≥ π3 , α +β ≥ π3 + π3 , α +β +γ  π3 + π3 + π3 ,
and
by the
inequality of Problem 14.12, we have f (α)+f (β)+f (γ ) ≥
therefore,

f π3 + f π3 + f π3 , or sin α2 + sin β2 + sin γ2 ≤ 23 .


On the other hand, π2 > α, π2 + π2 > α + β, π2 + π2 + 0  α + β + γ , whence



f π2 + f π2 + f (0) > f (α) + f (β) + f (γ ), or 2 < sin α2 + sin β2 + sin γ2 .
√ √ √
14.15. The proof of the inequality sin α + sin β + sin γ > 0 is√obvious. Let us
prove the second inequality. Consider the function f (x)  − sin x in (0, π ).
Therefore f  (x)  24sin
2 2
x+cos

sin x sin x
x
> 0 if x ∈ (0, π ).
Since in the case α ≥ β ≥ γ one has α > π2 , α + β > π2 + π4 , α + β + γ 
π π π
+ + , it follows by the inequality of Problem 14.12 that f (α)+f (β)+f (γ ) >
2 4
4

√ √ √ √
f π2 + f π4 + f π4 , or sin α + sin β + sin γ < 1 + 4 8.

Alternative proof. Note that x + y ≤ 2(x2 + y2 ). Therefore for α > π2 , we
have
√    
sin α + sin β + sin γ < 1 + sin β + sin γ ≤ 1
 
 β +γ √ √
+ 2(sin β + sin γ ) ≤ 1 + 4 sin < 1 + 2 2  1 + 8.
4

2
206 14 Miscellaneous Inequalities

14.16. We have
1 1 1 2 2 2
1 + √ + √ + ··· + √ > √ +√ √ + ··· + √ √
2 3 n 1+ 2 2+ 3 n+ n+1
√  √ √   √ √  √ √
 2 2 − 1 + 2 3 − 2 + · · · + 2 n + 1 − n  2 n + 1 − 2 > 2 n − 2.

14.17. Consider the polynomial P(x)  (x − a)(x − b)(x − c)(x − d ). Since


P(x)  x4 − S1 x3 + S2 x2 − S3 x + S4 and P(a)  0, we must have

a4  S1 a3 − S2 a2 + S3 a − S4 . (1)

If a ≤ 0, then S1 a3 − S2 a2 + S3 a − S4 < 0, and a4 ≥ 0, which leads to a


contradiction with (1). Therefore, a > 0. In a similar way one can prove that
b > 0, c > 0, d > 0.
14.18. Consider the sequence an  n n!n n .
(e)
Thus, it follows that
an+1 e 1

<
an 1 n+1 1
1+ n 1+ n


n
(see Problem 9.32(d)), and therefore, an+1 < √n+1 · an .
We have
√ √ √
n−1 n−1 n−2
an < √ · an−1 < √ ·√ · an−2
n n n−1
√ √ √
n−1 n−2 1 a1 e
< ··· < √ ·√ · · · √ · a1  √  √ .
n n−1 2 n n

Hence an < √en , and therefore, if n ≥ 8, then an < √e8 < 1.


14.19. By induction, one can easily prove that 2n ≥ n2 − 1, n ∈ N.
It follows that
         
1 1 1 1 1
1 + 1 ··· 1 + n ≤ 1 + · 1+ 2 ··· 1 + 2
2 2 2 2 −1 n −1
2 2
3 2 n 3n
 · ···  < 3.
2 1·3 (n − 1) · (n + 1) n+1

14.20. Let a, b, c ∈ [0, a1 ] and a ≥ b ≥ c. If b ≤ a+c


2
, then the following conditions
hold: a ≥ a−b+c, a+c  (a−b+c)+b and a−b+c ≥ b. Therefore, by Problem
2.8, it follows that g(a) + g(c) ≥ g(a − b + c) + g(b), or g(a) − g(b) + g(c) ≥
g(a − b + c).
If b ≥ a+c2
, then a ≥ b, a + c  b + (a − b + c) and b ≥ a − b + c. Therefore,
g(a) + g(c) ≥ g(a − b + c) + g(b).
Proofs 207

It follows that g(a) − g(b) + g(c) ≥ g(a − b + c).


Hence using Problem 7.5, we obtain the proof of the given inequality (taking
f (x)  g(x) − g(0)).
14.21. (a), (b) Let p(1)  a + b + c  m, p(−1)  a − b + c  n. Then |m| ≤
1, |n| ≤ 1, and |c|  |p(0)| ≤ 1.
Note that b  m−n 2
, a  m+n 2
− c, and therefore,
m + n   m − n 
 
|a| + |b| + |c|   − c +   + |c|
2 2 
 m + n   m − n 
    + 2|c| ≤ max(|m|, |n|) + 2|c| ≤ 3.
≤ +
2 2 

We have

m2 + n2
a2 + b2 + c2  + 2c2 − mc − nc
2
m2 + n2 m2 + c2 n2 + c2
≤ + 2c2 + +  m2 + n2 + 3c2 ≤ 5.
2 2 2

(c) Consider the polynomial F(x)  a + bx + cx2 + dx3 . Then by Lagrange’s


formula, 1
we have
(x + 1)(x − 1)(x − 2) (x + 2)(x − 1)(x − 2)
F(x)  F(−2) · + F(−1) ·
−12 6
(x + 2)(x + 1)(x − 2) (x + 2)(x + 1)(x − 1)
+ F(1) · + F(2) · .
−6 12

Hence a  F(0)  F(−2) + 23 F(−1) + 23 F(1) − 16 F(2).


−6

On the other hand, F(x)  x3 p 1x , x  0, and therefore F(2) 


1
1

8p 2 , F(−2)  −8p − 2 , F(1)  p(1), F(−1)  −p(−1).



Thus, a  43 p − 21 − 23 p(−1) + 23 p(1) − 43 p 21 , and it follows that





|a| ≤ 43 p − 21  + 23 |p(−1)| + 23 |p(1)| + 43 p 21  ≤ 43 + 23 + 23 + 43  4. Note
that the polynomial F(x)  4x3 − 3x satisfies the assumption of the problem
(if we set x  cosα, then F(x)  cos3α).
14.22. Let the roots of the polynomial P(x) be the numbers −b1 , −b2 , . . . , −bn ,
where bi > 0 i  1, 2, . . . , n.
We have P(x)  (x + b1 ) · · · (x + bn ), and hence a0  b1 · · · bn and a1 
a0 b11 + · · · + b1n . Thus, we need to prove that

1 If P(x) is a polynomial of degree n, and x1 , x2 , . . . , xn , xn+1 are distinct real numbers, then
(x − x2 ) · · · (x − xn+1 ) (x − x1 )(x − x3 ) · · · (x − xn+1 )
P(x)  P(x1 ) · + P(x2 ) ·
(x1 − x2 ) · · · (x1 − xn+1 ) (x2 − x1 )(x2 − x3 ) · · · (x2 − xn+1 )
(x − x1 ) · · · (x − xn )
+ · · · + P(xn+1 ) · .
(xn+1 − x1 ) · · · (xn+1 − xn )
208 14 Miscellaneous Inequalities
 
1 1
+ ··· + (1 + b1 )(1 + b2 ) · · · (1 + bn ) ≥ 2n2 , n  2, 3, . . . .
b1 bn

By Cauchy’s inequality, it follows that b11 + · · · + b1n ≥ √n b bn ···b , and 1 + bi 


1 1  1 2 n

+ ... + +bi ≥ n · (n−1)n−1 , i  1, . . . , n. Therefore,


n bi

n − 1  n − 1
n−1
   n−1
1 1 1
+ ··· + (1 + b1 ) · · · (1 + bn ) ≥ n2 1 +
b1 bn n−1
 
1
≥ n2 1 + (n − 1) ·  2n2 .
n−1

In order to complete the proof, it is sufficient to use Bernoulli’s inequality.


14.23. We call two segments having a common vertex a bird, and the segments them-
selves wings. The number of non-single-color triangles is equal to Cn3 − t(n).
Each non-single-color triangle contains birds with wings of distinct colors.
Therefore, the number of birds with wings of distinct colors is equal to 2(Cn3 −
t(n)).
On the other hand, if through some vertex, k segments of the first color and
n − 1 − k segments of the second color pass, then the number of birds with
wings of distinct colors containing that vertex (that is, both wings contain that
vertex) is equal to k(n − 1 − k).
Note that

⎨ n−1 2 , if n is even,
k(n − 1 − k) ≤ 2
⎩ n n − 1
, if n is odd.
2 2

Thus, it follows that



⎨ (n−1)2 n ,

if n is odd,
4
2 Cn − t(n) ≤
3

⎩ n n − 1 , if n is even.
2

2 2

Hence, we obtain
− n (n−2)
2
if n  2k, then t(n) ≥ n(n−1)(n−2)
6 8
 k(k−1)(k−2)
3
,
n(n−1)2
if n  4k + 1, then t(n) ≥ n(n−1)(n−2)
6
− 8  3 k(k − 1)(4k + 1),
2
2
if n  4k + 3, then t(n) ≥ n(n−1)(n−2)
6
− n(n−1)
8
 23 k(k + 1)(4k − 1).
14.24. We proceed by induction.
If n  3, 4, then the statement of the problem is true.
Assume that the statement holds for n ≤ k − 1 points. Let us prove that it
holds for n  k points (k ≥ 5).
Consider the following two cases.
Proofs 209

(a) Every edge is part of a triangle.


k2
In this case, the number of triangles formed is greater than 12 , and there-
k 
fore not less than 2 , k  5, 6, . . ..
(b) There exists an edge AB that does not belong to any triangle. Let us denote
all other vertices of the graph by A1 , A2 , . . . , Ak−2 . Hence, each vertex
Ai is not connected to both A and B (i  1, 2, . . . , k − 2). Therefore, the
2
number of edges with vertices A1 , A2 , . . . , Ak−2 is greater than k4 − (k −
2
1)  (k−2)
4 
, and hence the number of triangles formed by these vertices
 k 
is at least k−22
 2 − 1.
Let one of the triangles be A1 A2 A3 . If none of the vertices Ai is connected
to A or B, then we will not consider the segment A1 A2 . Then the points
2
A1 , A2 , . . . , Ak−2 are connected to (k−2) segments. Therefore, there exist at
k  4  
least 2 − 1 triangles, and with A1 A2 A3 , there exist k2 triangles. It is left to
consider the case in which A is connected to the vertices A1 , A2 , . . . , Am , and
B is connected to the rest of them. If the segment Ai Aj is an edge for some
(i, j), i < j ≤ m or m < i < j, then this ends the proof. Otherwise, the number
of edges with vertices A1 , A2 , . . . , Ak−2 is not greater than m(k − 2 − m),
which leads to a contradiction, since m(k − 2 − m) ≤ (k−2)
2

4
.
14.25. Note that the numbers {nα} are distinct numbers. We argue by contradiction.
Assume that {iα}  {jα}, i  j. Then iα  jα + k, k ∈ Z, and hence
α  i−j k
∈ Q, which leads to a contradiction.
Let m ∈ N and m1 < min(a, b − a, 1 − b). There exist positive inte-
gers i > j such that |{iα} − {jα}| < m1 (it is sufficient to divide the seg-
ment
 [0, 1] into m+ 1equal parts and use Dirichlet’s principle). Therefore,
(i − j)α − [iα] + jα  < 1 , and hence {(i − j)α} < 1 , or {(i − j)α} >
m m
1 − m1 . Let {(i − j)α} < m1 , (i − j)α  x + k, k ∈ Z, x  {(i − j)α}.
Consider the numbers x, 2x, 3x, . . . . One of these numbers belongs to (a, b),
so suppose a < lx < b, l ∈ N. We have that l(i − j)α  lx + kl; hence
{l(i − j)α}  lx ∈ (a, b).
Let {(i − j)α} > 1 − m1 , (i − j)α  x + k, k ∈ Z, x  {(i − j)α}. We have
1 − x < m1 .
Consider the numbers 1 − x, 2(1 − x), . . . . One of these numbers belongs
to (1 − b, 1 − a), so let 1 − b < l(1 − x) < 1 − a, l(i − j)α  lx + lk 
−l(1 − x) + l + lk. Therefore, l(k + 1) + a − 1 < l(i − j)α < l(k + 1) + b − 1,
or a < {l(i − j)} < b.
n n
14.26. (a) We have (xi − m)(M − xi ) ≥ 0, and therefore xi2 ≤ −nmM .
i1 i1

n
(b) Note that M ≥ 0 ≥ m, whence (M (xi − m)(M 3 − xi3 ) − m(xi3 −
i1
m3 )(M − xi )) ≥ 0.
210 14 Miscellaneous Inequalities


n

Thus, we deduce that (M −m) xi4 ≤ −n M 4 m − m4 M . Therefore, if M 


i1

n

m, then we have xi4 ≤ −mMn m2 + M 2 + mM . If m  M  0, then it


i1

n

follows that xi4  0  −mMn m2 + M 2 + mM .


i1
14.27. We have
 m

Cm1 x2m−2 y2 + x2 y2m−2 + . . . + Cmm−1 x2 y2m−2 + y2 x2m−2


x2 + y2 − x2m − y2m 
2
≥ Cm1 xm ym + Cm2 xm ym + · · · + Cmm−1 xm ym  (2m − 2)xm ym .

k
m−k 2
m−k 2
k 
Here we have used that x2 · y2 + x · y ≥ 2 x2m · y2m ≥
2xm · ym , k 
1, · · ·
,mm − 1.
m
Therefore, x2 + y2 − x2m − y2m ≥ (2m − 2)xm ym , or x2 + y2 ≥
(x − y ) + 2 x y .
m m 2 m m m

This ends the proof.


14.28. If α ≤ π2 , β ≤ π2 , γ ≤ π2 , then we have sin α cos(β − γ ) 
√ √
2 (sin
1
2γ + sin 2β) ≥ sin 2γ sin 2β, and thus cos(β − γ ) ≥ sin sin
2γ sin 2β
α
.
Therefore,
√ √ √
sin 2α sin 2β sin 2γ sin 2β sin 2α sin 2γ
cos(α − β) cos(β − γ ) cos(γ − α) ≥ · ·
sin γ sin α sin β
 8 cos α cos β cos γ .

Let α ≤ β < π2 < γ .


If γ ≤ π2 + α or γ ≥ π2 + β, then cos(α − β) cos(β − γ ) cos(γ − α) ≥ 0 >
8 cos α cos β cos γ .
If π2 + α < γ < π2 + β, then 0 < − cos(γ − α) < − cos γ and 0 < cos(α −
β) cos(γ − β) < cos(β − α) < 4 cos(β − α) + 4 cos(α + β)  8 cos α cos β;
hence cos(α − β) cos(β − γ ) cos(γ − α) > 8 cos α cos β cos γ .
14.29. Consider the following two cases.
 
(a) if sin 2x  0, we have x  2π n, n ∈ Z and sin x + sin22x + · · · + sinnnx  
0 < 3.  
(b) if sin 2x  0, then there exists a positive integer m such that m+1
1
< sin 2x  ≤
1
m
.
For n ≤ m we have
 
 
sin x + sin 2x + · · · + sin nx  ≤ |sin x| + |sin 2x| + · · · + |sin nx|
 2 n  2 n
 x
 
≤ |sin x| + |sin x| + · · · + |sin x| ≤ m|sin x| ≤ 2msin  ≤ 2 (1)
   2
n

(see Problems 1.13 and 7.4(a)).


Proofs 211
 
Now let us estimate the following expression:  sin(m+1)x
m+1
+ · · · + sinnnx  for n >
m. By Problem 14.10, we have that
 
sin(m + 1)x sin nx 1 1
+ ··· +  sin(m + 1)x −
m+1 n m+1 m+2
 
1 1
+ +(sin(m + 1)x + sin(m + 2)x) −
m+2 m+3
 
1 1
+ · · · + (sin(m + 1)x + · · · + sin(n − 1)x) −
n−1 n
1
+ (sin(m + 1)x + · · · + sin nx)
n
Thus, it follows that
   
 sin(m + 1)x sin nx  1 1
 + · · · + ≤ |sin(m + 1)x| −
 m+1 n  m+1 m+2
 
1 1
+ |sin(m + 1)x + sin(m + 2)x| −
m+2 m+3
 
1 1 1
+ · · · + |sin(m + 1)x + · · · + sin(n − 1)x| − + |sin(m + 1)x + · · · + sin nx|
n−1 n n
 
1 1 1 1 1 1 1
≤   − + ··· + − +    < 1.
sin x  m + 1 m+2 n−1 n n (m + 1)sin x 
2 2

Hence
 
 sin(m + 1)x sin nx 
 + ··· + < 1. (2)
 m+1 n 

Here we have used the following inequality:


1
|sin kx + sin(k + 1)x + · · · + sin px|≤  x  .
sin 
2

Let us prove this inequality. We have


|sin kx + sin(k + 1)x + · · · + sin px|
 
2 sin x sin kx + · · · + 2 sin x sin px
 2   2
2sin 2x 
             
 
cos k − 21 x − cos k + 21 x + cos k + 21 x − cos k + 1 + 21 x + · · · + cos p − 21 x − cos p + 21 x
  
2sin 2x 
     
 1 1 
cos k − 2 x − cos p + 2 x 2 1
   ≤     .
2sin x 
2 22sin x  2
sin x 

From inequalities (1) and (2), it follows that


212 14 Miscellaneous Inequalities
 
 
sin x + sin 2x + · · · + sin nx  < 3.
 2 n 

14.30. We need to prove that


 n
(a1 + a2 + a3 )(a2 + a3 + a4 ) · · · (an + a1 + a2 ) 3
A ≥ . (1)
(a1 + a2 )(a2 + a3 ) · · · (an + a1 ) 2

Let us prove (1) by mathematical induction.


(a) For n  3, we have 
a1 + a2 + a3  a1 +a
2
2
+ a2 +a3
2
+ a3 +a1
2
≥ 3 3 a1 +a
2
2
· a2 +a3
2
· a3 +a1
2
. Therefore,
(a1 +a2 ) (a2 +a3 ) (a3 +a1 )  3
(a1 + a2 + a3 )(a2 + a3 + a1 )(a3 + a1 + a2 ) · 33 3
≥ 2 2
 2
.
(a1 + a2 )(a2 + a3 )(a3 + a1 ) (a1 + a2 )(a2 + a3 )(a3 + a1 ) 2

(b) Let n ≥ 4 and suppose that inequality (1) holds for n − 1 numbers. Let
us first prove inequality (1) if a1  a2 or an−1  an .
If a1  a2 , we have
(2a2 + a3 )(a2 + a3 + a4 ) · · · (an−1 + an + a2 )(an + 2a2 )
A
2a2 (a2 + a3 ) · · · (an−1 + an )(an + a2 )
(2a2 + a3 )(an + 2a2 ) (a2 + a3 + a4 ) · · · (an−1 + an + a2 )(an + a2 + a3 )
 ·
2a2 (an + a2 + a3 ) (a2 + a3 ) · · · (an−1 + an )(an + a2 )
 n−1  n−1  n
(2a2 + a3 )(an + 2a2 ) 3 3 3 3
≥ · ≥ · ≥ ,
2a2 (an + a2 + a3 ) 2 2 2 2

since the inequality (2a2 + a3 )(an + 2a2 ) ≥ 3a2 (an + a2 + a3 ) is equivalent to


the inequality (a2 − a3 )(a2 − an ) ≥ 0.
If an−1  an , the proof is similar.
Now let us prove that a1 < a2 and an−1 < an . If we substitute the sequence of
numbers a1 , a2 , . . . , an−1 , an by the numbers a1  a1 + x, a2 , . . . , an−1 , an −
x  an , where 0 ≤ x ≤ min(a2 − a1 , an − an−1 ), then the value of A does not
increase, and therefore,
(a1 + a2 + a3 + x)(a2 + a3 + a4 ) · · · (an−2 + an−1 + an − x)(an−1 + an + a1 )(an + a1 + a2 )
A≥ ,
(a1 + x + a2 )(a2 + a3 ) · · · (an−1 + an − x)(an + a1 )
or
(a1 + a2 + a3 )(an−2 + an−1 + an ) (a1 + a2 + a3 + x)(an−2 + an−1 + an − x)
≥ .
(a1 + a2 )(an−1 + an ) (a1 + x + a2 )(an−1 + an − x)
(2)

Note that the last inequality can be rewritten in the following way: ax2 +bx+c ≤
0, where a  (a1 +a(a2 +a 3 )(an−2 +an−1 +an )
1 +a2 )(an−1 +an )
− 1 > 0, c  0, and therefore, in order
to prove (2) it is sufficient to prove it for the values x  0 and x  a4 − a1
Proofs 213

(n  4), x  a5 + a4 − a2 − a1 (n  5), and x  a2 − a1 (n ≥ 6), since


min(a2 − a1 , an − an−1 ) ≤ a2 − a1 ≤ a4 − a1 ≤ a4 − a1 + a5 − a2 .
If x  0, the proof of (2) is obvious.
If n  4, then x  a4 − a1 , and we need to prove that
(a1 +a2 +a3 )(a2 +a3 +a4 )
(a1 +a2 )(a3 +a4 )
≥ (a2 +a3 +a4 )(a2 +a3 +a1 )
(a2 +a4 )(a1 +a3 )
, or (a4 − a1 )(a3 − a2 ) ≥ 0.
If n  5, then x  a5 + a4 − a2 − a1 , and the proof of (2) is obvious.
For n ≥ 6, then x  a2 − a1 , and we need to prove that ax + b ≤ 0. Therefore,
 
(a1 + a2 + a3 )(an−2 + an−1 + an )
− 1 (a2 − a1 )
(a1 + a2 )(an−1 + an )

(a1 + a2 + a3 )(an−2 + an−1 + an )
+ an +an−1 +an−2 −a1 −a2 −a3 −
a1 + a2

(a1 + a2 + a3 )(an−2 + an−1 + an )
+ ≤ 0,
an−1 + an
or
(a3 an−2 + a3 (an−1 + an ) + an−2 (a1 + a2 ))(a2 − a1 )
(a1 + a2 )(an−1 + an )
 
(an−1 + an − a1 − a2 )(a3 an−2 + a3 (an−1 + an ) + an−2 (a1 + a2 ))
+ an−2 − a3 − ≤ 0,
(a1 + a2 )(an−1 + an )

(a3 an−2 + a3 (an−1 + an ) + an−2 (a1 + a2 ))(2a2 − an − an−1 ) + (an−2 − a3 )(a1 +


a2 )(an−1 + an ) ≤ 0,
2a2 an−2 (a1 + a2 + a3 ) + (an−1 + an )(a2 a3 − a3 an−2 − a3 an−1 − a3 an − a3 a1 ) ≤ 0,
2a2 an−2 (a1 + a2 + a3 ) ≤ (a3 an−1 + a3 an )(an−2 + an−1 + an + a1 − a2 ), where
the last inequality holds because 2a2 an−2 ≤ a3 an−2 + a3 an−2 ≤ a3 an−1 + a3 an
and a1 + a2 + a3 ≤ an−2 + an−1 + an + a1 − a2 .
Taking x  min(a2 − a1 , an − an−1 ), we obtain for the numbers
a1 , a2 , . . . , an−1 , an the case a1  a2 or an−1  an , such that a1 ≤ a2 ≤
(a +a2 +a3 )(a2 +a3 +a4 )···(an +a1 +a2 )
n
· · · ≤ an−1 ≤ an Thus, it follows that A ≥ 1 (a   ≥ 23 .
1 +a2 )(a2 +a3 )···(an +a1 )
This ends the proof of the given inequality.
14.31. (a) Note that yz < 1, xz < 1, xy < 1; therefore
y y
x
1−yz
+ 1−zx z
+ 1−xy ≥ x + y + z ≥ x2 + y2 + z 2  1, and hence 1−yz x
+ 1−zx +
z
1−xy
≥ 1.
y2 +z 2 z 2 +x2 x2 +y2
Note that yz ≤ 2
, xz ≤ 2
, xy ≤ 2
; therefore,

x y z x y z
+ + ≤ 2 +z 2 + 2 +x 2 +
1 − yz 1 − zx 1 − xy 1 − y
1− 2z 2
1 − x +y
2

2 2
2x 2y 2z
 + + .
1 + x2 1 + y2 1 + z 2
214 14 Miscellaneous Inequalities

On the other hand, note that


  
2 2 x y 2(x − y)2 (x + y)(1 − xy)
− − − ≤ 0,
1 + x2 1 + y2 1 + x2 1 + y2 (1 + x2 )2 (1 + y2 )2

and thus from inequality (8.5.1) (see Chapter 8) we obtain


 √
2x 2y 2z 6(x + y + z) 3 3 3
+ + ≤ ≤ 3(x 2 + y2 + z 2 )  .
1 + x2 1 + y2 1 + z 2 3 + x2 + y2 + z 2 2 2

y
It follows that x
1−yz
+ 1−zx
+ z
1−xy
≤ 3 3
2
(see also Problem 5.17).
y x2 y2 z2
(b) We have x
1+yz
+ 1+zx + 1+xy  x+xyz + y+xyz + z+xyz
z
, xyz  0.
By inequality (8.4), we have
x y z (x + y + z)2 (1 + xy + yz + zx) + xy + yz + zx
+ + ≥ 
1 + yz 1 + zx 1 + xy x + y + z + 3xyz x + y + z + 3xyz
x + y + z + (1 − x)(1 − y)(1 − z) + xyz + xy + yz + zx x + y + z + xyz + xy + yz + zx
 ≥ > 1;
x + y + z + 3xyz x + y + z + 3xyz

y
x
hence 1+yz + 1+zx z
+ 1+xy > 1.
y
If xyz  0, for example x  0, so that y2 + z 2  1 and 1+yz
x
+ 1+zx z
+ 1+xy 
y + z ≥ y + z  1, let us√prove that if x, y, z ≥ 0 and x + y + z  1,
2 2 2 2 2
y
x
then 1+yz + 1+zx z
+ 1+xy ≤ 2.
Note that if a ≥ 0, then 1+a 1
≤ 1 − a + a2 , and therefore,
x y z
+ + ≤ x(1 − yz + y2 z 2 ) + y(1 − xz + x2 z 2 ) + x(1 − xy + x2 y2 )
1 + yz 1 + zx 1 + xy
 x + y + z − 3xyz + xyz(yz + xz + xy).

y
Moreover, we have xy + yz + zx ≤ x2 + y2 + z 2 , whence 1+yz
x
+ 1+zx z
+ 1+xy ≤
x + y + z − 2xyz. √
Let us prove that x+y+z−2xyz ≤ 2, where x, y, z ≥ 0 and x2 +y2 +z 2 
1.
Indeed,√let max(x, y, z)  √ z. Then 3z 2 ≥ x2 + y2 + z 2  1; therefore,
2z + 2 2z − 1 ≥ 2z +√2 2z 2 − 1 >
2 2
√ 3z − 12 ≥ 0,
2
and√thus from the
inequality 4z − 8z + 4 2z − 1  ( 2z − 1) (2z 2 + 2 2z − 1) ≥ 0,
4 2
√ 2 √
z(x + y) − 2√1 z + 4z −8z 4z
4 2
it follows that +4 2z−1
≥ 0, or z(x + y)2 −


(x + y) + z 3 − 2z + 2 ≥ 0, x + y + z − (x + y)2 + z 2 − 1 z ≤ 2,

x + y + z − 2xyz ≤ 2.
14.32. Let us prove that if inequality (14.1) holds for all positive numbers a, b, c,
then λ ≥ 8 or λ  0.
Indeed, if λ  0, then for a  1, b  c  1n , from (14.1) we obtain
1 1 1 3
 +√ +√ ≥√ .
1 + λ n2 1 + nλ 1 + nλ 1+λ
Proofs 215

For n → +∞ we have that 1 ≥ √1+λ 3


, and therefore, λ ≥ 8 (if λ < 0, then

the expression 1 + nλ is not defined for n > − λ1 ).
Let us prove that for λ ≥ 8 and a > 0, b > 0, c > 0 inequality (14.1) holds.

Lemma 1 If x > 0, y > 0, and xy ≤ 9, then √1+x


1 1
+ √1+y ≥ 1.
√ √ 2 √
Indeed, we have 1 + x + 1 + y  2 + x + y + 2 (1 + x)(1 + y) ≥ 2 + x +
√ √ √ √ √
y +2+2 xy ≥ (1+x)(1+y), and therefore, 1 + x + 1 + y ≥ 1 + x 1 + y.

Lemma 2 If x > 0, y > 0, and xy > 9, then √1+x 1 1


+ √1+y ≥ √ 2√ .
1+ xy
√ √ 2 √

We need to prove that 1 + x + 1 + y 1 + xy ≥ 4(1 + x)(1 + y).


√ √ 2 √

We have 1 + x + 1 + y 1 + xy ≥ 4 + x + y + 2 xy 1 + xy (see
the proof of Lemma 1). √

Now, let us prove the following inequality: 4 + x + y + 2 xy 1 + xy ≥


4(1 + x)(1 + y). Note that it is equivalent to the following obvious inequality:

√ √
2
xy −3 x − y ≥ 0.  2
y
Define 2
1+x2
− 2
1+y2
x
1+x2
− 1+y2
 − 2(x−y) (x+y)(1−xy)
(1+x2 )2 (1+y2 )2
≤ 0. Then x >
0, y > 0, z > 0, xyz  λ . Let us prove that for λ ≥ 8 the following
3

inequality holds:
1 1 1 3
√ +√ +√ ≥√ .
1+x 1 + y 1+z 1+λ

Let max(x, y, z)  z. Then z ≥ λ. If xy ≤ 9, then by Lemma 1 we have


1 1 1 1 1 3
√ +√ +√ >√ +√ ≥1≥ √ .
1+x 1 + y 1+z 1+x 1 + y 1+λ

If xy > 9, then by Lemma 2, we have √1


1+x
1
+ √1+y ≥√ 2
√ , √1
1+z
1
+ √1+λ ≥
1+ xy
√ 2
√ (since zλ ≥ λ2 ≥ 64 > 9).
1+ zλ
Summing these inequalities, we obtain
1 1 1 1 2 2
√ +√ +√ +√ ≥ √ + √ .
1+x 1+y 1+z 1+λ 1 + xy 1 + zλ
√ √ √ √
Since xy ≥ 3 and zλ ≥ λ, we have xy · zλ ≥ 3λ ≥ 24 > 9, and
therefore, by Lemma 2, we have √ 2√ + √ 2√ > √ 4√4  √1+λ
4
.
1+ xy 1+ zλ 1+ xyzλ
Hence √1
1+x
+ √1
1+y
+ √1
1+z
+ √1
1+λ
> √1+λ
4
, which ends the proof of inequality
(14.1).
216 14 Miscellaneous Inequalities

Remark In a similar way, one can prove that n ≥ 2, a1 > 0, . . . , an > 0.


Then
 
a1n−1 ann−1
+ · · · + ≥ 1.
a1n−1 + (n2 − 1)a2 · · · an ann−1 + (n2 − 1)a1 · · · an−1

14.33. Let us first prove the following lemma.


Lemma If x, y, z are the lengths of some triangle, then
 √ y + z  y + z 2
x2 + xy + y2 x2 + xz + z 2 ≤ x2 + x + .
2 2
Indeed, we need to prove that
2


2 2
x + xy + y2 x2 + xz + z 2 ≤ x2 + x y+z 2
+ y+z
2
. This inequality holds
because it is equivalent to the following obvious inequality:
y−z
2  y+z
2 
2 2
+ yz + x(y + z − x) ≥ 0.
This ends the proof of the lemma.
From the lemma, we have
√ √ √ 2
a2 + ab + b2 + b2 + bc + c2 +
c2 + ac + a2
√ √
 2a2 + 2b2 + 2c2 + ab + bc + ca + 2 a2 + ab + b2 + b2 + bc + c2
√ √ √ √
+ 2 a2 + ab + b2 c2 + ac + a2 + 2 b2 + bc + c2 c2 + ac + a2
(a + c)2
≤ 2a2 + 2b2 + 2c2 + ab + bc + ca + 2b2 + b(a + c) +
2
2 2
(b + c) (a + b)
+ 2a2 + a(b + c) + + 2c2 + c(a + b) +
2 2
 5a2 + 5b2 + 5c2 + 4ab + 4bc + 4ca.

It follows that
√ √ √
a2 + ab + b2 + b2 + bc + c2 + c2 + ac + a2

≤ 5a2 + 5b2 + 5c2 + 4ab + 4bc + 4ca.

Remark 1. One can prove that if a, b, c ≥ 0, then


√ √ √
a2 + ab + b2 + b2 + bc + c2 + c2 + ca + a2

≤ 5a2 + 5b2 + 5c2 + 4ab + 4bc + 4ca.
√ √
2. If a  −1, b  c  1, then 1 + 3+1> 11.
Proofs 217

14.34. (a) Let us begin by proving that if the given inequality holds for n  3,
then it holds for all n  4, 5, . . . .
Indeed, we have
⎛ ⎞
x13 + · · · + xn3 3 3 3
1 ⎝ x1 + x 2 + x 3 1 xn3 + x13 + x23 1
 · + ··· + · ⎠
x1 · · · xn 3 x1 x2 x3 x4 · · · xn xn x1 x2 x3 · · · xn−1

1 (1 − x1 )3 + (1 − x2 )3 + (1 − x3 )3 1
≥ · + ···
3 (1 − x1 )(1 − x2 )(1 − x3 ) x4 · · · xn

(1 − xn )3 + (1 − x1 )3 + (1 − x2 )3 1
+ ·
(1 − xn )(1 − x1 )(1 − x2 ) x3 · · · xn−1

1 (1 − x1 )3 + (1 − x2 )3 + (1 − x3 )3
≥ + ···
3 (1 − x1 ) · · · (1 − xn )

(1 − xn )3 + (1 − x1 )3 + (1 − x2 )3
+
(1 − x1 ) · · · (1 − xn )

(1 − x1 )3 + · · · + (1 − xn )3
 ,
(1 − x1 ) · · · (1 − xn )

x3 +···+x3 3 3
and therefore, 1x1 ···xn n ≥ (1−x1 ) +···+(1−xn )
(1−x1 )···(1−xn )
.
Let us prove that the given inequality holds for n  3.
 
x13 + · · · + xn3 1 x13 + x23 + x33
1 xn3 + x13 + x23 1
 · + ··· + ·
x1 · · · xn 3 x1 x2 x3 x4 · · · xn xn x1 x2 x3 · · · xn−1
 
1 1 1 1  
≥3+ +   +   (x1 − x2 )2 + (x2 − x3 )2 + (x3 − x1 )2
2 x1 x2 x2 x3 x3 x1

(1 − x1 )3 + (1 − x2 )3 + (1 − x3 )3
 ,
(1 − x1 )(1 − x2 )(1 − x3 )

x13 +x23 +x33


where xi  1 − xi , i  1, 2, 3. Therefore, x1 x2 x3

(1−x1 )3 +(1−x2 )3 +(1−x3 )3
(1−x1 )(1−x2 )(1−x3 )
.
(b) It is sufficient to prove the given inequality for n  4 (see the proof
of Problem 14.34(a)).
Let x1 ≥ x2 ≥ x3 ≥ x4 ; then x1 x1 ≥ x2 x2 ≥ x3 x3 ≥ x4 x4 and
x1
x
≥ xx2 ≥ xx3 ≥ xx4 , where xi  1 − xi , i  1, 2, 3, 4. It follows that
1 2 3 4
 
x14 + x24 + x34 + x44 (x1 − x2 )4 4(x1 − x2 )2 x x x x (x − x4 )4 4(x3 − x4 )2
 + +2 1 2 + 3 4 + 3 +
x1 x2 x3 x4 x1 x2 x3 x4 x3 x4 x3 x4 x1 x2 x1 x2 x3 x4 x1 x2
 
(x1 − x2 )4 4(x1 − x2 )2 x1 x2 x3 x4 (x3 − x4 )4 4(x3 − x4 )2
≥     + +2   +   +     +
x1 x2 x3 x4 x3 x4 x3 x4 x1 x2 x1 x2 x3 x4 x1 x2

(1 − x1 )4 + (1 − x2 )4 + (1 − x3 )4 + (1 − x4 )4
 ,
(1 − x1 )(1 − x2 )(1 − x3 )(1 − x4 )

since

x1 x2 x3 x4 x x x x
+ − 1 2 − 3 4
x3 x4 x1 x2 x3 x4 x1 x2
 
1 1
 (x1 x1 · x2 x2 − x3 x3 · x4 x4 )   − ≥ 0.
x1 x2 x3 x4 x1 x2 x3 x4
218 14 Miscellaneous Inequalities

Another proof of this inequality can be obtained using the following


identity:
 
x14 + x24 + x34 + x44 1 (x1 − x2 )4 + (x2 − x3 )4 + (x3 − x4 )4 + (x4 − x1 )4

x1 x2 x3 x4 2 x1 x2 x3 x4
  
2(x1 − x2 )2 2(x2 − x3 )2 2(x3 − x4 )2 2(x4 − x1 )2 (x1 − x3 )2 (x2 − x4 )2
+ + + + + +2 +2 .
x3 x4 x1 x4 x1 x2 x2 x3 x1 x3 x2 x4

14.35. We need to prove that B  a12015 +· · ·+an2015 −A(a12000 +· · ·+an2000 ) ≥ 0,


2015 2015
where A  112000 +···+n+···+n2000
.
If {a1 , . . . , an }  {1, . . . , n}, then B  0.
If {a1 , . . . , an }  {1, . . . , n}, then there exist numbers i and j (i, j ∈
{1, . . . , n}) such that ai ≥ n + 1 and ak  j, for k  1, . . . , n.
Then ai2015 − Aai2000 > j 2015 − Aj 2000 , as ai2000 > j 2000 and ai15 ≥
(n + 1)15 > A.
Therefore, on substituting ai by j, we see that the value of B is decreas-
ing. After several such √  that B≥ 0.
steps we obtain
14.36. We need to √ prove that 1 + x2 + 1 + y2 + (1 − x)2 + (1 − y)2 −
(1 − xy) ≥ 5(1 − xy).
Note that
     
1 + x2 + 1 + y2 + (1 − x)2 + (1 − y)2 − (1 − xy)  xy − 1 + x2 − 1 1 + y2 − 1
     
+ (1 − x)2 + (1 − y)2 + 1 + x2 · 1 + y2 ≥ (1 − x)2 + (1 − y)2 + 1 + x2 · 1 + y2 ,


as a ≥ 1 + a2 −1 ≥ 0, for a ≥ 0. √  √
Let us prove that (1 − x)2 + (1 − y)2 + 1 + x2 · 1 + y2 ≥ 5(1 −
xy).
If xy ≥ 21 , then
 √  √ 
(1 − x)2 + (1 − y)2 + 1 + x2 · 1 + y2 ≥ 1 + x2 · 1 + y2

3 5 √
≥ 1 + xy ≥ > ≥ 5(1 − xy).
2 2

If xy < 21 , then by inequality (14.1), we have


   
  2
(1 − x)2 + (1 − y)2 + 1 + x2 · 1 + y2  1 − 2xy + (1 − x − y)2 + (1 − xy)2 + (x + y)2
 
 2 
≥ 1 − 2xy + 1 − xy + 1  3 − 4xy + x2 y2 + 2(1 − xy) 1 − 2xy
 √
≥ 3 − 4xy + x2 y2 + 2(1 − xy)(1 − 2xy)  5(1 − xy),

 √  √
and therefore, (1 − x)2 + (1 − y)√2 + 1 + x2 · 1 + y2 ≥ 5(1−xy).
1+ x √
14.37. (a) For n  2 we have √1+x 1
+ √1+x1
 √1+x1 ≤ 2.
1 2 1
For n  3 let us prove that there exists an acute triangle with angles α, β, γ
tan α β γ
such that x1  tan β
, x2  tan γ , and then x3  tan
tan
tan α
.
Proofs 219


Indeed,
 it is sufficient
 to take tan α  (1 + x2 + x1 x2 )x1 , tan β 
1+x2 +x1 x2
x1
, tan γ  1+x2 +x1 x2
x
· x2 .
1
 1  
β cos α sin γ cos β sin α cos γ
We need to prove that sinsin γ
+ sin α
+ sin β
≤ √32 , or equiva-
lently,
   
sin β sin(2α) + sin γ sin(2β) + sin α sin(2γ ) ≤ 3 sin α sin β sin γ .

According to the Cauchy–Bunyakovsky–Schwarz inequality, we have


  
sin β sin(2α) + sin γ sin(2β) + sin α sin(2γ )


≤ sin2 β + sin2 γ + sin2 α (sin(2α) + sin(2β) + sin(2γ ))




 2 sin2 β + sin2 γ + sin2 α sin α sin β sin γ ≤ 3 sin α sin β sin γ ,
√ √ √
√ therefore, sin β sin(2α) + sin γ sin(2β) + sin α sin(2γ ) ≤
and
3 sin α sin β sin γ
(b) The proof is by induction.
For n  4, from the inequality √1+a
1 1
+ √1+b < 1 + 1+ab
1
, where a > 0, b > 0,
it follows that
1 1 1 1 1 1
√ +√ +√ +√ <1+ +1+  3.
1 + x1 1 + x2 1 + x3 1 + x4 1 + x1 x2 1 + x3 x4

Assume that n ≥ 5 and that the inequality holds for n − 1 numbers. Let us
prove that it holds for n numbers.
We have
1 1 1 1 1 1
√ + ··· + √ +√ < √ + ··· + √ +1+
1 + x1 1 + xn−1 1 + xn 1 + x1 1 + xn−2 1 + xn−1 xn
1 1 1
< √ + ··· + √ +√ + 1 < n − 2 + 1  n − 1,
1 + x1 1 + xn−2 1 + xn−1 xn

whence
1 1
√ + ··· + √ < n − 1.
1 + x1 1 + xn

Now let us prove that √1+a1


+ √ 1 < 1 + 1+ab
1
, where a > 0, b > 0.
√ √ 1+b
Define 1 + a  1 + x, 1 + b  1 + y, and note that x > 0, y > 0. We
need to prove that (2 + x + y)(1 + x2 y2 + 2x2 y + 2xy2 + 4xy) < (1 + x + y +
xy)(2 + x2 y2 + 2x2 y + 2xy2 + 4xy),
or equivalently,

2xy + 2x2 y + 2xy2 < x + y + x3 y3 + 2x2 y3 + 2x3 y2 + 3x2 y2 . (1)


220 14 Miscellaneous Inequalities

Indeed, we have
x y
+ 2x3 y2 ≥ 2x2 y, + 2x2 y3 ≥ 2xy2 ,
2 2

x y 3 3
+ + 3x y ≥ 3
2 2
· xy > 2xy, x3 y3 > 0.
2 2 4

Summing these inequalities, we obtain inequality (1).


14.38. Consider the following two cases.
(a) 1 ≤ α ≤ 2 + n−1
1
. Then note that

xiα − xi xi − xi2−α xi − xi2−α


 ≥ , i  1, . . . , n.
x1 + · · · + xn + xiα − xi xi + x1 +···+xn
− xi2−α x1 + · · · + xn
xα−1 i

Summing these inequalities, we obtain


n
xiα − xi  xi − x2−αn
≥ i
. (1)
i1
x1 + · · · + xn + xiα − xi i1
x1 + · · · + xn

If 1 ≤ α ≤ 2 − 1
n−1
, then by Problem 3.42(a), it follows that

n
xi − xi2−α
≥ 0.
x + · · · + xn
i1 1

If 2 − 1
n−1
≤α ≤2+ 1
n−1
, then by Problem 3.42(b), again it follows that

n
xi − xi2−α
≥ 0. (2)
x + · · · + xn
i1 1


n
xiα −xi
Hence, from (1) and (2) we have x1 +···+xn +xiα −xi
≥ 0.
i1
(b) α ≥ 2 + n−1
1
.
This solution was proposed by N.Nikolov.
If we prove that there exists a number γ ≥ 1 such that

γ 
n
γ
nxi − xi
xiα
− xi i1
≥ , i  1, . . . , n, (3)
x1 + · · · + xn + xiα − xi n
γ
(n − 1) xi
i1


n
xiα −xi
then summing these inequalities, we will obtain x1 +···+xn +xiα −xi
≥ 0.
i1
Proving inequality (3) is equivalent to proving the following inequality:
Proofs 221

γ
nxi (xiα−1 − 1) (n − 1)xi
≥n  − 1. (4)
x1 + · · · + xn  γ γ
xi − xi
i1

It is sufficient to prove that


⎛ ⎞
α−1 γ
nxi x
⎝ α−1 i α−1 − 1
⎠ ≥  (n − 1)xi
− 1, (5)
x1 + · · · + xn x n · · · x n n
γ γ
1 n xi − xi
i1

α−1 α−1
since x1 n · · · xn n ≥ 1.
Without loss of generality, one can assume that xi  1.
Now let us prove that there exists a number γ ≥ 1 such that the following
inequality holds
 
n 1 1
− 1 ≥ − 1, (6)
x1 + · · · + xn G A

n
γ
xi −1 α−1 α−1
where A  n−1 , G  x1 n · · · xn n .
i1

Taking γ  (n−1)(α−1)
n
, we obtain that A ≥ G.
If G ≥ 1, then 0 ≥ G1 − 1 ≥ A1 − 1 and x1 +···+x
n
n
≤ 1, which ends the proof of
inequality (6).
If G ≤ 1 and A ≤ 1, then G1 − 1 ≥ A1 − 1 ≥ 0 and x1 +···+x n
≥ 1, since by
x1 +···+xn −1
γ n

Jensen’s inequality, we have 1 ≥ A ≥ n−1


, which completes the
proof of inequality (6). 1

If G ≤ 1 and A > 1, then x1 +···+x


n
n G
− 1 ≥ 0 > A1 − 1.

Problems for Independent Study

Prove the following inequalities (1–16).


√ √ √
1. 3x2 + 2x + 1 + 3x2 − 4x + 2 ≥ 351 .
 √ 2  √ √
2. 2 + 2 − 1 ≤ x2 + y2 ≤ 1 and (2 − 2 + 2)2 ≤ (x − 2/2)2 + (y −
√ √ √ √ √
2/2) ≤ 2 − 2,where (x + 2/2) + (y + 2/2)2  2 + 2, x ∈ [0, 1].
2 2

3. a12 + (1 − a2 )2 + a22 + (1 − a3 )2 + · · · + an2 + (1 − a1 )2 ≥ n 2/2, where
n√≥ 2. √  √ √
4. 2 + 4 − 2 2 + · · · + 2n − 2 (n − 1)n ≥ n(n + 1), where n ∈ N.
2 √ 2
5. (a) (a−b)
8a
≤ a+b
2
− ab ≤ (a−b)8b
, where a ≥ b > 0.
222 14 Miscellaneous Inequalities
 √ √ 
(b) (a + c)2 + (b + d )2 ≤ a2 + b2 + c2 + d 2 ≤ (a + c)2 + (b + d )2 +
√ |ad −bc| , where a > 0, b > 0, c > 0, d > 0.
(a+c)2 +(b+d )2
√ √ √ √
6. 0 < 4n + 2 − n  − n + 1 < 1/16√ n , where n ∈ N.
3

7. (a) x + xy + y + y + yz + z ≥ x + √
2 2 2 2 2 xz + z 2 ;
√ √
(b) c a2 − ab + b2 + a b2 − bc + c2 ≥ b a2 + ac + c2 , where a > 0, b >
√ c > 0.
0, √ √
8. 1 − cos(x3 − x2 ) + 1 − cos(x2 − x1 ) ≥ 1 − cos(x3 − x1 ).
3 √ 3 √
9< x4 + 1dx + 4 x4 − 1dx < 9.0001.
4
9.
0 0
x13 x23 xn3
10. +
x12 +x1 x2 +x22 x22 +x2 x3 +x32
+ ··· + xn2 +xn x1 +x12
≥ 13 (x1 + . . . + xn ), where n ≥ 3, x1 >
0, . . . , xn > 0.
x17 x27 x7
11. x14 +2x13 x2 +2x1 x23 +x24
+ + · · · + x4 +2x3 x +2x
n
x24 +2x23 x3 +2x2 x33 +x34 3 4 ≥ 6 (x1 + · · · + xn ),
1 3 3
n n 1 n x1 +x1
where n ≥ 3, x1 > 0, . . . , xn > 0.
12. 3n + 4n + · · · + (n + 2)n < (n + 3)n , where n ≥ 6 and n ∈ N.
13. 1
< ln 2014
2015
< 2014
1
.
2015

14. tan x ≥ x , where x ∈ 0, π2 .


3

n 
n
15. tan αi ≥ (n − 1) cot αi , where n ≥ 3, 0 < αi < π2 , i  1, . . . , n and
i1 i1
n
cos αi  1.
2
i1
16. a + b3 + c3 + 6abc > 41 (a + b + c)3 , where a > 0, b > 0, c > 0.
3

100 2
100  100  100
17. Compare the numbers 13 + 3 and √12 + 1 − √12 .
18. Prove that if a + b + c + d  0 and a2015 + b2015 + c2015 + d 2015  0, then
a3 + b3 + c3 + d
3
 0.
2 2 √
19. Prove that if α a +b 2
+ (1 − α) ab ≤ a+b
2
holds for every pair of positive
numbers a, b, then α ≤ 21 .
20. Prove that if |a1 | + · · · + |an | < ε < 21 , then |1 − (1 + a1 ) · · · (1 + an )| < 2ε.
21. The function f is defined on the set Q and f (0) · f (1) < 0. Prove that there exist
rational numbers r1 and r2 , such that f (r1 ) − f (r2 ) > (r1 − r2 )2 .
22. Given that a ≥ 0, b ≥ 0, c ≥ 0 and a + b + c ≤ 1, find the greatest value of
the expression |a − b| · |b − c| · |c − a|.
23. Let α, β, γ be the angles of some triangle, and a, b, c be its sides. Prove that

(a) 0 < sin α + sin β + sin γ ≤ 3 2 3 ,
√ √ √
1
(b) 0 < sin α + sin β + sin γ ≤ 3 · 34 4 ,

(c) 0 < sin α sin β sin γ ≤ 3 3
,
8 √
γ
(d) 2 < cos α2 + cos β2 + cos 2
≤ 3 3
2
,
(e) cos α cos β cos γ ≤ 18 ,

(f) 0 < cos α2 cos β2 cos γ2 ≤ 3 3
8
,
2
+b2 +c2
(g) 13 ≤ a(a+b+c) 2 < 2,
1
Problems for Independent Study 223

(h) 1
4
≤ ab+bc+ca
(a+b+c)2
≤ 13 ,
(i) 1
4
≤ (a+b)(b+c)(c+a)
(a+b+c)2
≤ 27
8
.
24. Let α, β, γ be the angles of an acute triangle. Prove that

(a) 2 < sin α + sin β + sin γ ≤ 3 2 3 ,
√ √ √
1
(b) 2 < sin α + sin β + sin γ ≤ 3 · 34 4 ,
(c) 34 ≤ sin2 α2 + sin2 β2 + sin2 γ2 < 1,
(d) 1 < cos α + cos β + cos γ ≤ 23 ,
(e) 2 < cos2 α2 + cos2 β2 + cos2 γ2 ≤ 49 ,

(f) 21 < cos α2 cos β2 cos γ2 ≤ 3 8 3 ,
m+2
(g) tanm α + tanm β + tanm γ ≥ 3 2 , where m ≥ 1.
25. Prove that for an obtuse triangle with angles α, β, γ and sides a, b, c the
following inequalities hold:

(a) 0 < sin α + sin β + sin γ ≤ 1 + 2,
(b) 0 < sin α sin β sin γ ≤ 21 ,

(c) 0 < cos α2 cos β2 cos γ2 ≤ 1+ 2
4
,
α cos β cos γ
(d) 1+cos
sin α sin β sin γ
≥ 2,
2
+b2 +c2
(e) 1
3
≤ a(a+b+c) 2 ≤ 8,
3

(f) 5
16
< ab+bc+ca
(a+b+c)2
≤ 13 ,
(g) 9
32
< (a+b+c)2 ≤ 27
(a+b)(b+c)(c+a) 8
.
26. Prove that if 0 < a1 ≤ . . . ≤ an and 0 < b1 ≤ . . . ≤ bn , then
(a1 + b1 ) · · · (an + bn ) ≤ (a1 + bi1 ) · · · (an + bin ) ≤ (a1 + bn ) · · · (an + b1 ), where
the numbers i1 , . . . , in are a permutation of the numbers 1, . . . , n.
27. Given that the assumptions of Problem 26 hold for the numbers ai , bi , i 
1, . . . , n, and g is a nondecreasing convex function (see Chapter §11). Prove
that
           
b1 bn bi1 bin bn b1
g + ··· + g ≤g + ··· + g ≤g + ··· + g .
a1 an a1 a1 a1 an

 n on [0, a1] and g(0) ≤ 0. Prove that


28. Let g be a convex function
n 
(−1)i−1 g(ai ) ≥ g (−1)i−1 ai , where 0 < an ≤ . . . ≤ a1 .
i1 i1
29. 
Proven that, if a1 >  0, . . ., an > 0, then
n n
an−1 an−1
a1
a2
+ · · · + an
+ aan1 ≥ aa21 + · · · + an
+ an
a1
, where n ≥ 2.
30. Suppose that for the polynomial p(x)  ax + bx + c the following assumptions
2

hold:
224 14 Miscellaneous Inequalities

0 ≤ p(−1) ≤ 1, 0 ≤ p(0) ≤ 1, 0 ≤ p(1) ≤ 1. Prove that for all x ∈ [0, 1] the


following inequality holds: p(x) ≤ 98 .
31. Given that for a trinomial p(x)  ax2 + bx + c on [0, 1] the inequality |p(x)| ≤ 1
holds, prove that |b| ≤ 8.
   p(x)  ax +bx+c on [−1, 1] the inequality |p(x)| ≤ 1
2
32. Given that for a trinomial
holds, prove that p (x) ≤ 4 on [−1, 1].  
Remark. For every polynomial p(x) the following inequality holds: p (x) ≤
2n2
b−a
max|p(x)|, where x ∈ [a, b] and n is the degree of p(x).
[a;b]
33. Given that the degree of the polynomial p(x) is not greater than 2n and that for
every integer k ∈ [−n, n] the inequality |p(k)| ≤ 1 holds, prove that for all
x ∈ [−n, n] the following inequality holds: |p(x)| ≤ 22n .
34. Let x0 < x1 < · · · < xn be integers. Prove that at least one of the numbers
|p(x0 )|, |p(x1 )|, . . . , |p(xn )| is not smaller than 2n!n , where p(x)  xn + a1 xn−1 +
· · · + an .
35. Let x1 < . . . < xk , k ≥ 3, be given numbers belonging to [−1, 1]. Prove that
1 1
+ ··· + ≥ 2k−2 .
|(x1 − x2 ) · · · (x1 − xk )| |(xk − x1 ) · · · (xk − xk−1 )|

36. Prove that if a > 0, b > 0, c > 0, then


a b c
√ + √ + √ ≤ 1.
a+ (a + b)(a + c) b + (b + a)(b + c) c + (c + a)(c + b)

37. Let p(x)  ax3 + bx2 + cx + d and suppose that for all x belonging to [−1, 1]
the following inequality holds: |p(x)| ≤ 1. Prove that |a| + |b| + |c| + |d | ≤ 7.
38. Prove that
(a) xλ (x − y)(x − z) + yλ (y − x)(y − z) + z λ (z − x)(z − y) ≥ 0, where x >
0, y > 0, z > 0;
g(x)(f (x) − f (y))(f (x) − f (z)) + g(y)(f (y) − f (x))(f (y)
(b)
− f (z)) + g(z)(f (z) − f (x))(f (z) − f (y)) ≥ 0,
where f and g are monotonic functions and E(g) ⊆ [0, +∞).
39. Let each of the edges of the complete graph on n vertices be colored in one of
three colors. Prove that there exists a single-color connected subgraph having
at least 2n vertices.
√ √ √ √
40. Prove that a1 + · · · + an ≤ a1 + C(n) · · · an + C(n), where n ≥ 2,
n−1
a1 ≥ 0, . . . , an ≥ 0, C(n)  n−2 .
n n−1
√ π
41. Prove that 0 ≤ sin x + . . . + sin(nx)
< 2 2, where 0 ≤ x ≤ .
n 2 √
Hint. If x  0, then there exists a positive integer m such that 2
2(m+1)
< sin x
2


2
2m
.
Problems for Independent Study 225

42. Let f : R → R. Prove that f is constant if


(a) for all real numbers x and y the following inequality holds: |f (x) − f (y)| ≤
C|x − y|α , where C > 0, α > 1,
(b) for all real numbers a, b, c, d forming an arithmetic progression, the
following inequality holds: |f (a) − f (d )| ≥ A|f (c) − f (b)|, where A > 3.
1
43. Prove that a+3 1
+ b+3 1
+ c+3 + d 1+3 ≤ 2327
4
+ 27 · abcd
1
, where a > 0, b > 0,
c > 0, d > 0 and a + b + c + d  4.
Hint. We have that
 
1 1 1 1 3a + 1 3b + 1
abcd + + + ≤ · bcd + · acd
a+3 b+3 c+3 d +3 16 16
3c + 1 3d + 1 3 4 11 4 23
+ · abd + · abc ≤ abcd + + abcd  + abcd
16 16 4 27 108 27 27
(see Problem 2.5).

Selected Inequalities

Problems
Prove the following inequalities.
       
1. (ab + bc + ca)1 − abc 1 
≤ ab − 1c  + bc − a1  + ca − 1b , where abc  0.
 

2. sin2 β sin α2 sin α + β2 > sin2 α sin β2 sin β + α2 , where 0 < α < β and
α + β < π.
 a+b ab
3. a−b  ≥ 1, where a  b.


11 b−c
11 c−a
11
4. a−ba+b
+ b+c + c+a < 1, where a > 0, b > 0, c > 0.
1
5. x0 + x0 −x 1
+ 1
x1 −x2
+ · · · + 1
xn−1 −xn
≥ xn + 2n, where n ∈ N and x0 > x1 > . . . > xn .
n
6. Sk + Sm − Skm ≤ 1, where k, m ∈ N and Sn  1
i
.


i1
7. n+11
1 + 13 + · · · + 2n−1
1
> 1n 21 + 14 + · · · + 2n
1
, where n ≥ 2, n ∈ N.
1 2 n
8. 2 2 · 2 22 · · · 2 2n < 4, where n ∈ N.
9.
(a1 − a2 )(a1 − a3 )(a1 − a4 )(a1 − a5 ) + · · ·
+ (a5 − a1 )(a5 − a2 )(a5 − a3 )(a5 − a4 ) ≥ 0.
m+1 n+1
10. am + an ≥ mm + nn , where m, n ∈ N and a  mmm +n +nn
.
11. 2 · n! ≤ (m−n)! ≤ (m + m) , where m, n ∈ N and m ≥ n.
n (m+n)! 2 n
√ 
 
12.  2 − pq  > 3q12 , where p ∈ Z, q ∈ N.
√ √
13. 7 − mn > mn 1
, where m, n ∈ N and 7 − mn > 0.
 √ 
14. (x + y + z)2 −
3(xy + yz + zx) ≤ max(x, y, z) − min(x, y, z) ≤ 26 (x + y + z)2 − 3(xy + yz + zx).
15. 2 ab + bc + ac ≥ a + b + c + a1 + 1b + 1c , where a > 0, b > 0, c > 0 and abc  1.
16. (n − 0, 5)(1 + a2n ) ≥ a2n−1 + a2n−2 + · · · + a2 + a, where n ∈ N.
226 14 Miscellaneous Inequalities

17. x14 + · · · + xn4 ≥ x13 + · · · + xn3 , where x1 + · · · + xn ≥ n.

18. (−a1 + a2 + a3 + a4 )(a1 − a2 + a3 + a4 )(a1 + a2 − a3 + a4 )(a1 + a2 + a3 − a4 )


≤ 8(a12 a42 + a22 a32 ).
19. sin α + · · · + sin2 αn ≤ 49 , where α1 > 0, . . . , αn > 0 and α1 + · · · + αn  π.
2
√ 1 √ √ √ √ √
20. a + b − c + b + c − a + c + a − b ≤ a + b + c, where a, b, c are
the sides of some triangle.
21. a3 + b3 + c3 ≥ a2 (2c − b) + b2 (2a − c) + c2 (2b − a), where a > 0, b > 0, c > 0.
22. 1
+ b3 +c31+abc + c3 +a13 +abc ≤ abc
1
, where a > 0, b > 0, c > 0.

a3 +b3 +abc
23. 12uvw + u + v + w ≤
2 2 2
 1, where u  ≥ 0, v ≥ 0, w ≥ 0 and u + v + w  1.
2
 
24. n(n−1)
2
· 1
ai ·aj
≥ 4 · 1
ai +aj
, where n ≥ 2, a1 > 0, . . . , an > 0.
1≤i<j≤n 1≤i<j≤n

 + qr + rp) ≤2 + 9pqr, where p >0, q > 0, r > 0√and p + q + r  1.


25. 7(pq
26. a2 + a22 + a32 + a22 + a32 + a42 + · · · + an2 + a12 + a22 ≥ 3(a1 + · · · + an ).
 1   √
27. a12 + 2a22 + 3 a22 + 2a32 + 7 2a12 + a22 ≥ (5a1 + 4a2 + 2a3 ) 3.
√ √ √ √
28.  a2 + ab + b2 + b2 + bc + c2 + c2 + ca + a2 ≥ 3 · |a + b + c|.
 3 −b3 b3 −c3 c3 −a3  (a−b)2 +(b−c)2 +(c−a)2
29.  aa+b + b+c + c+a  ≤ 6
, where a > 0, b > 0, c > 0.
30. 1
xk
+ y1k + z1k ≥ xk + yk + z k , where k ≥ 2, k ∈ N, x > 0, y > 0, z > 0 and
1
x
+ 1y + 1z ≥ x + y + z, xyz  1.
31. x + y ≤ 2, where x > 0, y > 0 and x2 + y3 ≥ x3 + y4 .
2 2

32. loga (loga b) + logb (logb c) + logc (logc a) > 0, where c > b > a > 1.
33. sin(nα) ≤ 0, where n ∈ N, 0 < α < π and sin α + . . . + sin(nα) ≤ 0.
34. sin((n + 1)α) ≥ 0, where n ∈ N, 0 < α < π and sin α + . . . + sin(nα) ≤ 0.
x13 x63
35. x5 +x5 +x5 +x5 +x5 +5
+ · · · + x5 +x5 +x5 +x 5
+x5 +5
≤ 35 , where 0 ≤ xi ≤ 1, i  1, . . . , 6.
2 3 4 5 6 1 2 3 4 5
36. (ab + ac + ad + bc + bd + cd )2 + 12 ≥ 6(abc + abd + acd + bcd ), where
a + b + c + d  0.
37. (y3 + x)(z 3 + y)(x3 + z) ≥ 125xyz, where x ≥ 2, y ≥ 2, z ≥ 2.
38. (a + 3b)(b + 4c)(c + 2a) ≥ 60abc, where c ≥ b ≥ a ≥ 0. √
39. (a + b + 3c)(a + 3b + c)(3a + b + c) + 8(ab + bc + ac + abc) ≥ 349 abc, where
√ √ √ 27
a > 0, b > 0, c > 0 and a + b + c  1.

k
l
40. kp · ql ≤ 1, where p, k, q, l ∈ N and p + q ≤ k + l.

41. n x1 · . . . · xn ≥ (n − 1)a, where n ≥ 2, a > 0, x1 > 0, . . . , xn > 0 and
1
+ · · · + a+x
1
≤ a1 .
a+x1 n

42. tan α1 · · · tan αn+1 ≥ nn+1 , where α1 , . . . , αn+1 ∈ 0, π2 and tan α1 − π4 +· · ·+


tan αn+1 − π4 ≥ n − 1.
1
43. n−1+x 1
+ · · · + n−1+x
1
n
≤ 1, where x1 > 0, . . . , xn > 0 and x1 · · · xn  1.
4
44. a4 + b4 + c4 − 2(a2 b2 + b2 c2 + c2 a2 ) + 3(abc) 3 ≥ 0, where a ≥ 0, b ≥ 0, c ≥ 0.
45. a4 + b4 + c4 − 2(a2 b2 + b2 c2 + c2 a2 ) + abc(a + b + c) ≥ 0, where a ≥ 0, b ≥
0, c ≥ 0.
2 2 2
46. ab+c+b
+ bc+a+c
+ ca+b
+a
≥ 2, where a > 0, b > 0, c > 0 and a + b + c  1.
Selected Inequalities 227
   
a2 +bc 2 +cd 2 +da 2
47. + bb+1
a+1
+ cc+1 + dd +ab
+1
≥ 4, where a > 0, b > 0, c > 0, d > 0
and
√ abcd  1.

48. 3(ab + bc + ca) 9(a + b + c)2 + ab + bc + ca ≤ 9 (a + b + c)3 + abc ,


where a > 0, b > 0, c > 0.

2 3
49. 4(x3 +y3 +z 3 +xyz)2 ≥ x2 + y2 + z 2 + x+y+z
3
, where x > 0, y > 0, z > 0.
50. 2, 25(x2 − x + 1)(y2 − y + 1)(z 2 − z + 1) ≥ (xyz)2 − xyz + 1.
51. ab(a2 − b2 ) + bc(b2 − c2 ) + ca(c2 − a2 ) ≤ 41 (a2 +b2 +c2 )2 , where a > 0, b >
√ c > 0. √
0, √ √
52. a − 1 + b − 1 + c − 1 ≤ abc + c, where a ≥ 1, b ≥ 1, c ≥ 1.
53. (p(xy))2 ≤ p(x2 )p(y2 ), where p(t)  a0 t n + a1 t n−1 + . . . + an and a0 ≥ 0, a1 ≥
0, . . . , an ≥ 0.
54. x12 + · · · + xn2 ≥ 2n+1 3
.(x1 + · · · + xn ), where x1 < · · · < xn and x1 , . . . , xn ∈ N.
1
55. (1+a) 2 + 1
(1+b)2
+ 1
(1+c)2
1
+ (1+d )2
≥ 1, where a > 0, b > 0, c > 0, d > 0 and
abcd  1.
 n
56. 1 < √ xi √
1+x +···+x · x +···+x
< π2 , where n ≥ 2, x0  1, x1 > 0, . . . , xn > 0
0 i−1 i n
i1
and x1 + · · · + xn√ 1. 
57. 0, 785n2 − n < n2 − 12 + · · · + n2 − (n − 1)2 < 0, 79n2 , where n ≥ 2, n ∈
N.
a2 b2 c2
58. (a+b)(a+c)
+ (b+c)(b+a) + (c+a)(c+b) ≥ 34 , where a > 0, b > 0, c > 0.
59. 1
1+ab
1
+ 1+bc 1
+ 1+ca ≥ 23 , where a > 0, b > 0, c > 0 and a2 + b2 + c2  3.
60. a
b+2c
+ c+2a + a+2b ≥ 1, where a > 0, b > 0, c > 0.
b c

 n
ai xi2
61. 2 xi xj ≤ n−2 n−1
+ 1−ai
, where n ≥ 2, a1 > 0, . . . , an > 0, x1 >
i<j i1
0, . . . , xn > 0 and a1 + · · · + an  x1 + · · · + xn  1.
62. a3 + b3 + c3 ≥ a + b + c, where a > 0, b > 0, c > 0 and ab + bc + ca ≤ 3abc.
63. a2 x + b2 y + c2 z > d 2 , where x > 0, y > 0, z > 0, 1x + 1y + 1z ≤ 1 and a, b, c, d
are the sides of√some quadrilateral.
√ √

64. x + y + z ≥ x − 1 + y − 1 + z − 1, where x ≥ 1, y ≥ 1, z ≥ 1 and
1
+ 1y + 1z  2.
x  
65. x1k−1 + · · · + xnk−1 ≥ (n − 1) x11 + . . . + x1n , where 1+x1
k + · · · + 1+x k  1,
1
k≥
1 n
2, k ∈ N and √
√ x1 ≥ 1, . . . , xn√≥ 1. √
66. a2 + ab + b2 + b2 + bc + c2 + c2 + ca + a2 ≤ 5a2 + 5b2 + 5c2 + 4ab + 4bc + 4ac,
  a > 0, b > 0, c > 0.
where
 √
2 3 4 4 . . . n n < 2, where n ≥ 2, n ∈ N.
3
67.
    √
68. 1 + 2 + 3 + · · · + n < 2, where n ∈ N.

n
69. n! ≤ 7n+9 , where n ∈ N.
√ 16 √
70. 2
· √1 <
· 43 · · · 2n−1
1
< 23 · √12n , where n ≥ 2, n ∈ N.
2 2n 2 2n
√ √
71. Maclaurin’s inequality: k bk ≥ k+1 bk+1 , k  2, . . . , n − 1, where
228 14 Miscellaneous Inequalities

1
bk  (a1 · · · ak−1 ak + a1 · · · ak−1 ak+1 + · · · + a1 · · · ak−1 an + · · · + an−k+1 · · · an−1 an ),
Cnk

where n ≥ 2, a1 , a2 , . . . , an > 0.
72. a1 + a22 + · · · + ann ≥ an , where n ≥ 2 and ai+j ≤ ai + aj , i, j ∈ N, i + j ≤ n.
73. a1 a24 + a2 a34 + · · · + an a14 ≥ a2 a14 + a3 a24 + · · · + a1 an4 , where n ≥ 3 and a1 <
a2 < · · · < an .
u2
74. un+2 + un ≥ 2 + un+1 n
, where n ∈ N and u1  1, u2  2, un+2  3un+1 −
un , n  1, 2, . . . .
75. ai ≥ 0, i  1, 2, . . . , n, where n ≥ 3 and ai−1 +ai+1 ≤ 2ai , i  2, . . . , n−1.
76. max(a1 , a2 , . . . , an ) ≥ 2, where n ≥ 4 and a1 +· · ·+an ≥ n, a12 +· · ·+an2 ≥ n2 .
77. a0 +···+a
n+1
n
· a1 +···+a
n−1
n−1
≥ a0 +···+a
n
n−1
· a1 +···+a
n
n
, where n ≥ 2 and ai > 0, i 
0, 1, . . . , n, ai−1 ai+1 ≤ ai , i  1, . . . , n − 1.
2
n+1 n √
78. 1−x
n+1
> 1−xn
· x, where n ∈ N, 0 < x < 1.
79. sin(x1 + x2 ) + . . . + sin(xn−1 + xn ) + sin(xn + x1 ) > 2, where n  3, 4, . . . and
x1 + · · · + xn  π2 .
80. xy + yx > 1, where x > 0, y > 0.
81. a12 + · · · + an2 − n ≥ n−√2nn−1 · (a1 + · · · + an − n), where n ≥ 2 and a1 · · · an  1.
82. x1 arctan x1 + . . . + xn arctan xn ≥ n ln 2, where n ≥ 2 and x1 > 0, . . . , xn >
0, x1 · · · xn  1. p

n

n p
ai
ai
83. bi
≥n 2−p
· i1
n , where ai > 0, bi > 0, i  1, . . . , n, p ≥ 2.
i1 bi
i1
x x 2 n
84. x2 + · · · + xn ≥ 2x + · · · + nx , where x ≥ n ≥ 2, n ∈ N.
1
85. 2−a 1
+ 2−b 1
+ 2−c ≥ 3, where a ≥ 0, b ≥ 0, c ≥ 0 and a2 + b2 + c2  3.
2 + 4+b2 + 4+c2 + 4+d 2 + 4+e2 ≤ 1, where a ≥ 0, b ≥ 0, c ≥ 0, d ≥ 0, e ≥ 0
a b c d e
86. 4+a
1 1 1 1
and 4+a + 4+b + 4+c + 4+d + 4+e 1
 1.
 a1 +...+ak
p
n  p  n
p p
87. Hardy’s inequality: k
≤ p−1 ak , where p > 1 and ai >
k1 k1
0, i  1, . . . , n. n 
 
88. Abel’s inequality:  ak bk  ≤ B(|a1 | + 2|an |), where |b1 + . . . + bk | ≤
i1
B, k  1, . . . , n,  a1 ≤ . . . ≤ an or a1 ≥ . . . ≥ an .
n 
89. Abel’s inequality:  ak bk  ≤ Ba1 , where |b1 + · · · + bk | ≤ B, k 
i1
1, . . . , n, a1 ≥ · · · ≥ an ≥ 0.
n

k √
90. ai2 > 41 1 + 21 + · · · + 1n , where a1 > 0, . . . , an > 0 and ai > k, k 
i1 i1
1, . . . , n.
91. x11 + · · · + x1n ≤ y11 + · · · + y1n , where n ≥ 2, xi > 0, yi > 0, x1 + · · · + xi ≥
y 1 + · · · + yi , i  1, . . . , n and x1 y1 < x2 y2 < · · · < xn yn .
n−1
 cos(n−k)x n−1  cos(n+k)x 
92.  k
− k
 < 6, where n ≥ 2, n ∈ N.

k1 k1
Selected Inequalities 229
n−1 
 cos(n+k)x 
  cos(n−k)x n−1 √
 − π
93.  k k  < 4 2, where n ≥ 2, n ∈ N and 0 ≤ x ≤ 2
.
k1 k1
1 1 1 1
94. a1k + · · · + ank ≤ b1k + · · · + bnk , where k ≥ 2, k ∈ N, ai > 0, bi >
0, b1 + · · · + bi ≥ a1 + · · · + ai , i  1, . . . , n and b1 ≤ · · · ≤ bn .
95. x11 +· · ·+ x1n ≤ u11 +· · ·+ u1n , where u1  2, u2  3, uk+1  u1 · · · uk +1, k 
2, 3, . . ., and x1 , . . . , xn ∈ N, x11 + · · · + x1n < 1.
96. A(b − c)2 + B(c − a)2 + C(a − b)2 ≥ 0, if at least one of the following conditions
holds:
(a)a ≥ b ≥ c and B ≥ 0, B + A ≥ 0, B + C ≥ 0,
(b)a ≥ b ≥ c and A ≥ 0, C ≥ 0, A + 2B ≥ 0, C + 2B ≥ 0,
(c)a ≥ b ≥ c > 0 and B ≥ 0, C ≥ 0, a2 B + b2 A ≥ 0,
(d)A + B + C ≥ 0, AB + BC + CA ≥ 0,
(e)a ≥ b ≥ c, A ≥ 0, B ≥ 0, b2 B + c2 C ≥ 0, and a, b, c are the sides of
some triangle.
 
1
97. (xy + yz + zx) (x+y) 1 1
2 + (y+z)2 + (z+x)2 ≥ 49 , where x > 0, y > 0, z > 0.
 n
98. 1−h
2
< x2i (x2i+1 − x2i−1 ) < 1+h2
, where 0  x1 < x2 < · · · < x2n+1  1 and
i1
− x 
xi+1 i ≤ h, i  1, 2, . ..
, 2n.   
99. ai − aj  + bi − bj  ≤ ai − bj .
1≤i<j≤n 1≤i,j≤n
 n 4  n  n 
  2  2 2
100. Carlson’s inequality: ai ≤ π 2
ai i ai .
i1 i1 i1

Hints
   
1. (ab + bc + ca)1 − abc 1 
≤ ab + bc + ca − 1c − a1 − 1b .
sin2 β sin α2 sin(α+ β2 )−sin2 α sin β2 sin(β+ α2 )
2. sin α2 sin β2
 2 sin β−α
2
cos α+β2
sin(α + β).
 a+b 

3. ab ·  a−b  − 1 ≥ 0.
4. Let max(a, b, c)  a, and then a−b a+b
< 1, b−cb+c
≤ a−c
c+a
.
5. x0 − xn  (x0 − x1 ) + · · · + (xn−1− xn ), n ≥ 2. 
1

6. Skm  Sk + k+1 + · · · + 2k1 +· · ·+ (m−1)k+1


1
+ · · · + mk
1
≥ Sk +k· 2k1 +· · ·+k· mk
1
.


  
7. n+1 1 + 3 + · · · + 2n−1 − n 2 + 4 + · · · + 2n  2n(n+1) 2n n+1 + · · · + 2n −
1 1 1 1 1 1 1 1 1 1
 
1 + · · · + 1n .

8. 21 + 222 + · · · + 2nn < 21 + 212 + · · · + 212 + 213 + · · · + · · ·  1 + 21 + 212 + · · ·  2.


9. Let a1 ≤ a2 ≤ . . . ≤ a5 , then (a3 − a1 )(a3 − a2 )(a3 − a4 )(a3 − a5 ) ≥ 0,
(a1 −a2 )(a1 −a3 )(a1 −a4 )(a1 −a5 ) ≥ (a1 −a2 )(a2 −a3 )(a2 −a4 )(a2 −a5 ), and
(a5 − a1 )(a5 − a2 )(a5 − a3 )(a5 − a4 ) ≥ (a4 − a1 )(a4 − a2 )(a4 − a3 )(a5 − a4 ).
230 14 Miscellaneous Inequalities

m m−1 m−1
10. Let m ≥ n, then m ≥ a ≥ n and a − n  mnn · (m − a) ≥ mnn−1 +···+a +···+a
n−1 · (m − a).

11. (m−n)!  (m − n + 1) · · · (m + n) ≥ (2n)! and k(2m + 1 − k) ≤ m2 + m, k 


(m+n)!

m − n + 1, . . . , m. √ 
  √ √
12. If p ≤ 0, then  2 − pq  ≥ 2 > 13 ≥ 3q12 , and if 0 < p < (3 − 2)q,
√ 
  |2q2 −p2 |
then  2 − pq   q(√2q+p) ≥ q(√2q+p) 1
> 3q12 . It is left to consider the case

p ≥ (3 − 2)q.
13. m2 ≤ 7n2 − 3.

14. (x + y + z)2 − 3(xy + yz + zx)  21 (x − y)2 + (y − z)2 + (z − x)2 .


15. xy + yz ≥ x + 1z , where x > 0, y > 0, z > 0 and xyz  1.
16. 1 + a2n ≥ |a|  3  + |a|
k 2n−k
, where k ∈ {1, 2, . . . , n}.
 
17. x − x ≥ x (|x| − 1) ≥ |x| − 1.
4 3

18. 8(a12 a42 + a22 a32 ) − (−a1 + a2 + a3 + a4 )(a1 − a2 + a3 + a4 )(a1 + a2 − a3 + a4 )(a1 +


a2 + a3 − a4 )  (a12 + a42 − a22 − a32 )2 + 4(a1 a4 − a2 a3 )2 .
19. sin2 α + sin2 β ≤ sin2 (α + β), where α > 0, β > 0 and α + β ≤ π2 .
√ √ 2 √ √
≤(
a+b−c) +( b+c−a)
2 2

20. a+b−c+ b+c−a


2 2
 b.

21. a3 + c2 a ≥ 2 a3 · c2 a  2a2 c.
3 +abc  (a+b)(a 2 −ab+b2 )+abc ≤ (a+b)ab+abc  a+b+c .
22. a3 +babc abc abc c

23. (uv + vw + wu) ≥ 3(uv · vw + vw · wu + wu · uv)  3uvw.


2
 2
24. ai1·aj ≥ 4 · ai +a 1
j
.
25. Let min(p, q, r)  r, then

7(pq + qr + rp) − 9pqr  (7 − 9r)pq + 7r(p + q) ≤


(p + q)2 1
≤ (7 − 9r) + 7r(p + q)  2 − (3r − 1)2 (r + 1).
4 4
 √
26. x2 + y2 + z 2 ≥ 33 (x + y + z).
 √
27. x2 + 2y2 ≥ 33 (x + 2y).


2  √ 2
28. x + xy + y 
2 2 x + 2y + 23 y .
 3 3 
 −b 3
−c3 3
−a3  |a−b|·|b−c|·|c−a|
29.  aa+b + bb+c + c c+a   (a+b)(b+c)(c+a) (ab + bc + ca).
|c−a|
Let a ≤ b ≤ c, then c+a
< 1, ab+bc+ca
(a+b)(b+c)
< 1 and
 
6|a − b| · |b − c|  2|a − b| · |b − c| + 4|a − b| · |b − c| ≤ (a − b)2 + (b − c)2

+ ((b − a) + (c − b))2 .

30. (1 − x)(1 − y)(1 − z) ≥ 0, and therefore (1 − xk )(1 − yk )(1 − z k ) ≥ 0.


31. x2 − x3 ≥ y4 − y3 ≥ y3 − y2 and (x + y)(x3 + y3 ) ≥ (x2 + y2 )2 ≥ (x3 + y3 )2 .
32. loga (loga b) > logb (loga b).
33. sin α + · · · + sin(nα)  sin2 nα
2
· cot α2 + 21 sin(nα).
Hints 231

34. sin α + · · · + sin(nα)  sin2 (n+1)α


2
· cot α2 − 21 sin((n + 1)α).
xi3 3
xi
35. x15 +x25 +x35 +x45 +x55 +x65 −xi5 +5
≤ x15 +x25 +x35 +x45 +x55 +x65 +4
and 3xi5 − 5xi3 + 2  (xi − 1)2 (3xi3 +
6xi2 + 4xi + 2).
36. Set a  x+α, b  y+α, c  z+α, d  t +α, X  xy+xz+xt +yz+yt +zt, Y 
xyz + xyt + xzt + yzt. In this case, the given inequality can be rewritten as

2

X − 6α 2 + 12 ≥ 6 Y + 2αX − 8α 3 , where x + y + z + t  −4α.


Let us choose the number α such that 36α 4 + 12  −48α 3 , for example
α  −1.
We need to prove that X 2 ≥ 6Y , where x + y + z + t  4.
If Y ≤ 0, then X 2 ≥ 0 ≥ 6Y . And if Y > 0, then X 2  2Y (x + y + z + t) +
(xy − zt)2 + (xz)2 + (xt)
2
+ (yz)2 + (yt)2 ≥ 8Y > 6Y . Therefore X 2 > 6Y .
37. y + x ≥ 4y + x ≥ 5 xy4√
3 5
. √ √ √ √ √
(a+3b)(b+4c)(c+2a) ≥ 4 ab3 ·5 bc4 ·3 ca2 ≥ 60 ab3 · bc4 · c3/ 5 b2/ 5 a2 .
4 5 3 4 5 3
38.
39. (a + b + 3c)(a + 3b + c)(3a + b + c) + 8(ab + bc + ac + abc) ≥ ( 31 + 2c)( 13 + 2b)( 13 +

2a) + 8(ab + bc + ac) + 8abc ≥ 19 + 28 (ab + bc + ac) + 16abc ≥ 19 + 28 ( ab ·
√ √ √ √ √ 3 √ 3 √
bc + bc · ca + ca · ab) + 16abc  19 + 28 3
abc + 16abc ≥ 34927
abc.
p p q q p p q q
k+l
1
40. k + l ≥ p + q  k + · · · + k + l + · · · + l ≥ (k + l) · k · · · k · l · · · l .
    n−1
1

41. xi ≥ a a+xa+xi
1
+ · · · + a+x
a+xi
n
− 1 ≥ (n − 1)a a+x a+xi
1
· · · a+x
a+xi
n
.
42. Let tan αi  xi , then 1
1+x1
+ ··· + 1
1+xn
≤ 1.
xi1−α xi1−α
43. xi
n−1+xi
 xi
(n−1)x1α ···xnα +xi
 x1α ···xnα ≥ x11−α +···+xn1−α
, where (n−1)α  1−α.
(n−1) xα +xi1−α
 4 3  4 3  4 3 4 4 4 8 4 4 8
i
8 4 4 8 8 4
44. a 3 + b 3 + c 3 + 3a 3 · b 3 · c 3 ≥ a 3 b 3 + a 3 b 3 + b 3 c 3 + b 3 c 3 + c 3 a 3 +
4 8
 8 4 4 8
 8 4 4 8
 8 4 4 8
c3 a3 ≥ 2 a3 b3 · a3 b3 + 2 b3 c3 · b3 c3 + 2 c3 a3 · c3 a3 .
4
45. abc(a + b + c) ≥ 3(abc) 3 .
a2 +b
46. b+c  b+c − a.
a+b

(a+1)3
47. a2 + bc ≥ 2a(d +1)
.
48. Let a + b + c  p, ab + bc + ca  p −q
2 2
, where q ≥ 0. Then if q < p ≤ 2q,
3
2 p−2q
then min(abc)  0, and if p > 2q, then min(abc)  p+q 3
· 3 .
49. Let x + y + z  3u, xy + yz + zx  3v2 , xyz  w3 . Then one needs to prove
that 4(27u3 − 27uv2 + 4w3 )2 ≥ (9u2 − 6v2 + u2 )3 .
50. 1.5(x2 − x + 1)(y2 − y + 1) ≥ ((xy)2 − xy + 1).
51. Let a ≥ b ≥ c. If a > c, then without loss of generality one can assume  that

(a − c)2 + (b − c)2  1. Therefore a  c + cos α, b  c + sin α, α ∈ 0, π4 .
√ √ √
52. x · 1 + 1 · y ≤ (x + 1)(1 + y), where x ≥ 0, y ≥ 0.

2 √
2 √
2
53. p(x2 )  a0 · xn + a1 · xn−1 + · · · + an .
54. Let yi  xi − i, then 0 ≤ y1 ≤ · · · ≤ yn .
232 14 Miscellaneous Inequalities

55. Let 1
(1+a)2
+ 1
(1+b)2
+ 1
(1+c)2
+ 1
(1+d )2
 R2 , then
1 1
 R cos α cos β,  R cos α sin β,
1+a 1+b
1 1  π
 R sin α cos γ ,  R sin α sin γ , α, β, γ ∈ 0, .
1+c 1+d 2
n
xi x2
56. √ √  x1 + 
i1
1 + x0 + · · · + xi−1 · xi + · · · + xn 1 − x2 1
xn
+ ··· +  > x1 + x2 + · · · + xn  1.
1 − (x1 + · · · + xn−1 )2 

Let x1 + · · · + xi  cos αi , αi ∈ 0, π2 , i  1, 2, . . . , n. Then


√ xi
< sin(αi−1 − αi ), i  2, 3, . . . , n.
1−(x1 +···+xi−1 )2
√ 
57. πn4 − n < n2 − 12 + · · · + n2 − (n − 1)2 < πn4 .
2 2

2
a2 b2 c2
58. (a+b)(a+c) + (b+c)(b+a) + (c+a)(c+b) ≥ a2 +b2 +c(a+b+c)
2 +3ab+3bc+3ca .

32
59. 1
1+ab
+ 1+bc 1
+ 1+ca1
≥ 3+ab+bc+ca .
2
60. a
b+2c
 ab+2ac .a

 n
ai xi2  n
xi2 n
61. 1−ai
 1−ai
− xi2 .
i1 i1 i1
2 2 2 (a+b+c)2
62. a3 + b3 + c3  1a a + 1b b + 1c c ≥ 1 a+1 .
/ 2 / 2 / 2 / / b+1/ c
63. a x + b y + c z  1 x + 1 y + 1 z.
2 2 2 a b c
√ √
/ 2 / √ /2
( x−1) ( y−1)
+( z ) .
2
z−1
64. 1 +
x y  
≤ 1(+x1 k−1 +···+ n1 +x) k−1 , as
x1k−1 ·1 xnk−1 ·1 xk−1 +···+xk−1 ·n xik−1 xjk−1
65. n − 1  1+ ··· +
k−1 1 k−1 1 k−1 − 1
+xjk−1
·
x +x1 xn +xn x +xi
  x1 1 xn n xj
1 i

k−1 − 1 ≤ 0.
1 1
1 k−1
+x
xi i +x xj j
a2 +ab+b2 b2 +bc+c2 2 2 2 2 2
66. + + c +ca+a ≤ 5a +5b +5c +4ab+4bc+4ac
.
 a+b

b+c c+a 2(a+b+c)

m m+1 (m + 1) . . . n n < 2, where 2 ≤ m ≤ n.
m
67.
   √
68. m + m + 1 + · · · + n < m + 1, where 1 ≤ m ≤ n.
69. If n ≥ 5, then
     n
7n + 16 n+1 7n + 9 n 7n + 16 7
:  · 1+
16 16 16 7n + 9
 
7n + 16 7n n(n − 1) 49
≥ · 1+ + ·
16 7n + 9 2 (7n + 9)2
 
1 49n(n − 1)
> · 7n + 16 + 7n + > n + 1.
16 14n + 18

n
70. 2n+1
2n+2
> √
n+1
and 1
2
· 43 · · · 2n−1
2n
≤ √1 .
3n+1
Hints 233

√ √ √ √ √  √k  k+2
k
k+1 k+1
· k+2 bk k+1
bk+1 · k+2 bk
71. If 2 ≤ k ≤ n − 2, then k+2√bbk+1  k+2b√k+1 ≥ √2  b
√ k .
k+2 bk+2 ·bk k+2
bk+1
k+1
bk+1
    
72. (k + 1) a + a2 + · · · + ak+1  (a + a ) + a + a2 + a
1 1 k 1 k−1 + · · ·
 2 k + 1  2
a2 ak
+ a1 + + ··· + + a1 + ak+1
2 k
≥ (a1 + ak ) + (a2 + ak−1 ) + · · · + (ak + a1 ) + ak+1 ≥ (k + 1)ak+1 .
73. (a2 + a1 )(f (a2 ) − f (a1 )) + · · · + (an + an−1 )(f (an ) − f (an−1 )) ≥ (an + a1 )(f (an ) −
f (a1 )), where
f (x)  x4 is convex in R.
74. 2un ≤ un+1 < 3un , n  1, 2, . . . .
75. Let A  {i|ai  min(a1 , . . . , an )}. Then if 1 ∈ / A and n ∈ / A, then from k ∈ A
it follows that k − 1 ∈ A.
76. Let A  {i|ai ≥ 0} and |A|  m. If max(a1 , a2 , . . . , an ) < 2, then
4m + (2m − n)2 > a12 + · · · + an2 ≥ n2 , and therefore m  n.
77. Let aai+1i  bi , i  0, 1, . . . , n − 1, S  a1 + · · · + an−1 . Then b0 ≤ b1 ≤
. . . ≤ bn−1 , and therefore,
a0 an−i
 b0 · · · bi−1 ≤ bn−i · · · bn−1  , 1 ≤ i ≤ n, S ≥ (n − 1)2 a0 an .
ai an
n+1 n−1 n
2
78. 1−x
n+1
· 1−x
n−1
> 1−x
n
, n  2, 3, . . .
79. sin x > π2 · x, where 0 < x < π2 .
80. Let 0 < x < 1, 0 < y < 1. Then xy  1
y >
1
1+y( 1x −1)
> x+y
x
.
(1+ 1x −1)
81. c2 > c3 > · · · > cn > · · · and cn ≥ n−√n−1 , n  2, 3, . . . , where cn 
2n
2 n−2 2 n−3 2
min (t+1) t(t((tn−2 +2t
) +2(t ) +···+n−1)
n−3 +···+n−1) .
(0,1]
82. Let f (x)  x arctan x. Then f  (x)  (1+x2 2 )2 , and therefore, x1 arctan x1 + · · · +
xn arctan xn ≥ n · x1 +···+xn
arctan x1 +···+xn
.
83.    n
  n
2
an p  n  p
n
i
· bi ≥ ai2
i1
b i i1 i1
⎛ ⎛ ⎞ 2p ⎞2
n
a  n p
⎜ n
⎜ i1 i ⎟ ⎟ 
⎜ ⎟
≥⎜ n⎜ ⎟ ⎟  n2−p ai .
⎝ i1 ⎝ n ⎠ ⎠ i1

84. Let f (x)  x ln k + ln(ln x) − k ln x − ln(ln k). If x ≥ k ≥ 3, then f  (x) 


ln k + ln1x · 1x − kx > ln k − 1 > 0, and if x ≥ k  2, then
234 14 Miscellaneous Inequalities
 
 1 1 2 1 1
f (x)  ln 2 + · −  x ln 2 + −2
ln x x x x ln x
  
1 1 1 √ 
≥ 2 x ln 2 · −2 > 2 2 ln 2 − 2 > 0.
x ln x x

x k
Hence if x ≥ k ≥ 2, k ∈ N, then f (x) ≥ f (k), and therefore xk ≥ k x .
85. Let a2  x and f (x)  2−1√x . If 0 ≤ x ≤ 3, then f (x) ≥ f (1) + f  (1)(x − 1).
1

+ f  15 ·
2
86. 1
Let 4+a  x and f (x)  17xx−4x
2 −8x+1 . If 0 < x ≤ 4 , then f (x) ≤ f 5
1

x − 15 .
87. See
 n Problem 11.25(c).
88.  
 
 ak bk   |(a1 − a2 )b1 + (a2 − a3 )(b1 + b2 )
 
i1
+ · · · + (an−1 − an )(b1 + · · · + bn−1 ) + an (b1 + · · · + bn )|
≤ |a1 − a2 |B + |a2 − a3 |B + · · · + |an−1 − an |B + |an |B
 n   |a1 − an |B + |an |B ≤ B(|a1 | + 2|an |).
89.  
 
 ak bk   |(a1 − a2 )b1 + (a2 − a3 )(b1 + b2 )
 
i1
+ . . . + (an−1 − an )(b1 + · · · + bn−1 ) + an (b1 + · · · + bn )|
≤ |a1 − a2 |B + |a2 − a3 |B + · · · + |an−1 − an |B + |an |B  Ba1 .
90. Without loss of generality one can assume that a1 ≥ a2 ≥ · · · ≥ an .


n
ai2  (a1 − a2 )a1 + (a2 − a3 )(a1 + a2 )
i1
+ · · · + (an−1 − an )(a1 + · · · + an−1 ) + an (a1 + · · · + an )
√ √ √ √
> (a1 − a2 ) 1 + (a2 − a3 ) 2 + · · · + (an−1 − an ) n − 1 + an n
       
1 1 1 1 1 2 1 2
> a1 · √ + · · · + an · √ > · √ + ··· + √ ,
2 1 n 4 1 n

91. since 2 k > 1 + · · · + √1 ,
k
k  1, . . . , n.

1 1
· (x1 − y1 ) + · · · + · (xn − yn )
x1 y1 xn yn
   
1 1 1 1
 − · (x1 − y1 ) + · · · + −
x1 y1 x2 y2 xn−1 yn−1 xn yn
1
· (x1 − y1 + · · · + xn−1 − yn−1 ) + · (x1 − y1 + · · · + xn − yn ) ≥ 0.
xn yn
Hints 235

 n−1   n−1 
92.  cos(n − k)x  cos(n + k)x 
n−1  sin(kx) 
  
 −   2|sin(nx)| 
 k k   k 
k1 k1 k1
 n−1 
 sin(kx) 
 
≤ 2  < 6.
 k 
n−1  k1
  cos(n−k)x n−1  cos(n+k)x  √
93. If x  0, then  k
− k
  0 < 4 2. If 0 < x ≤ π , then
 2
k1 k1  x
there exists a positive integer m such that √2(m+1) < sin 2  ≤ √2m .
1 1
n−1  n−1 
  cos(n−k)x n−1  cos(n+k)x    sin(kx) 
For n ≤ m + 1 we have   −    ≤
k k  ≤ 2 k 
√ k1 k1 k1
2m|sin x| < 2 2.   √
Prove that if n ≥ m + 2, then  sin(m+1)x m+1
+ . . . + sin((n−1)x)
n
< 1
(m+1)|sin 2x |
< 2.
94. Without loss of generality one can prove that a1 ≤ a2 ≤ · · · ≤ an . Let Ai 
 1 k−1  1 k−2 1  1 k−1
bik + bik · aik + · · · + aik . Then Ai ≤ Ai+1 , i  1, . . . , n − 1,
and
1 1  1 1
b1k + · · · + bnk − a1k + . . . + ank
 
b1 − a1 bn − an 1 1
 + ··· +  (b1 − a1 ) −
A1 An A1 A2
 
1 1
+ · · · + (b1 − a1 + · · · + bn−1 − an−1 ) −
An−1 An
1
+ (b1 − a1 + · · · + bn − an ) ≥ 0.
An

95. Let x1 ≤ · · · ≤ xn . Then one can prove that x11 ≤ u11 , . . . , x11 + · · · + 1
xn−1

1
u1
+ · · · + un−1
1
.
If x1 + . . . + xn > u11 + . . . + u1n , then
1 1

   
1 1 1 1 1
≤1− + ... + <1− + ... +
x1 · . . . · xn x1 xn u1 un
1 1
  .
un+1 − 1 u1 · . . . · un

96. (a) A(b − c)2 + B(c − a)2 + C(a − b)2 ≥ A(b − c)2 + B(a − b)2 + B(b − c)2 +
C(a − b)2 ≥ (B + A)(b − c)2 + (B + C)(a − b)2 ≥ 0.

(b) A(b − c)2 + B(c − a)2 + C(a − b)2 ≥ min(A, C) (b − c)2 + (a − b)2 +
2
B(c − a)2 ≥ min(A, C) · (a−c)
2
+ B(c − a)2  21 (min(A, C) + 2B)(a − c)2 ≥ 0.
(c) A(b − c) +B(c − a) + C(a − b)
2 2
 ≥ A(b − c) + B(c − a)  (b −
2 2 2
2 2
c)2 A + (c−a)
(b−c)2
B ≥ (b − c)2 A + ab2 B ≥ 0.
236 14 Miscellaneous Inequalities

(d) Let A ≥ B ≥ C. Then A + B ≥ 23 (A + B + C) ≥ 0. If A + B > 0, then

A(b − c)2 + B(c − a)2 + C(a − b)2


 (A + B)c2 − 2c(bA + aB) + (A + C)b2 + (B + C)a2 − 2Cab ≥ 0,

since D  −(AB + BC + CA)(a − b)2 ≤ 0.


If A + B  0, then AB + BC + CA  AB ≥ 0, and therefore, A  B  C  0.
(e) A(b − c)2 + B(c − a)2 + C(a − b)2 ≥ B(c − a)2 + C(a − b)2  (a −
(c−a)2 2
b)2 B (a−b) 2 + C ≥ (a − b)2 B bc2 + C ≥ 0.
97. Let x + y  a, y + z  b, z + x  c and a ≥ b ≥ c. Then one needs to
prove that A(b − c)2 + B(c − a)2 + C(a − b)2 ≥ 0, where a, b, c are the sides
of some triangle and A  bc 2
− a12 , B  ac 2
− b12 , C  ab2
− c12 . We have
A  bc2
− a12 ≥ a22 − a12 > 0, B  ac 2
− b12 ≥ ac
2
− bc1
> 0,

2(b3 + c3 ) − 2abc
Bb2 + Cc2 
abc
2(b + c)bc − 2abc
≥ > 0.
abc
 n 
98.  1 
 x2i (x2i+1 − x2i−1 ) − 
 2
i1
 n 
 n 
 x2i+1 + x2i−1 
 x2i (x2i+1 − x2i−1 ) − (x2i+1 − x2i−1 )
 2 
i1 i1
 n 
 x 
 2i+1 + x2i−1 − 2x2i 
 (x2i+1 − x2i−1 )
 2 
i1
 
 n
 x2i+1 + x2i−1 − 2x2i  n
h h
≤  (x2i+1 − x2i−1 ) < (x2i+1 − x2i−1 )  .
 2  2 2
i1 i1
99. Without loss of generality one can assume that ai , bi ∈ [0, 1], i  1, . . . , n.
Let fi (x)  1 if x ∈ [0, ai ] and fi (x)  0 if x ∈ (ai , 1], i  1, . . . , n, gi (x)  1
if x ∈ [0, bi ] and gi (x)  0 if x ∈ (bi , 1], i  1, . . . , n. Then
 
min(ai , aj ) + min(bi , bj )
1≤i,j≤n 1≤i,j≤n

#1 #1
 (f1 (x) + · · · + fn (x)) dx +
2
(g1 (x) + · · · + gn (x))2 dx
0 0
#1 
≥ 2(f1 (x) + · · · + fn (x))(g1 (x) + · · · + gn (x))dx  2 min(ai , bj ).
0 1≤i,j≤n
Hints 237

n2
100. Let ci2  t + t
, i  1, . . . , n, t > 0. Then
⎛ ⎞2 ⎛ ⎞ 
n  n
1
⎝ ai ⎠ ≤ ⎝ (ai ci ) ⎠ 2
2
i1 i1
c i
⎛  ⎞
⎛ ⎞ n
2 a2 ⎛ ⎞
⎜ n i i ⎟
⎜  2 ⎟  n
t ⎠
⎜ ⎝ a ⎠t + i1 ⎟·⎝
⎜ i t ⎟ t 2 + i2
⎝ i1 ⎠ i1

⎛  ⎞
⎛ ⎞ 
n ⎛ ⎞
⎜  i2 ai2 ⎟ #i
⎜ n
i1 ⎟ ⎜ n
1 ⎟
≤⎜

⎝ ai2 ⎠t + ⎟·⎝
⎟ t dx⎠
⎝ i1 t ⎠ t 2 + x2
i1 i−1

⎛  ⎞
⎛ ⎞ 
n
2 a2 ⎛ n ⎞
⎜  i i ⎟ #
⎜ n 2 i1 ⎟ 1

≤⎜⎝ ⎠ ⎟ ⎝ dx⎠
ai t +
t ⎟· t t 2 + x2
⎝ i1 ⎠
0

⎛  ⎞
⎛ ⎞ 
n
⎜ n i2 ai2 ⎟
⎜  2 ⎟
⎜⎝ ai ⎠t +
i1 ⎟ · arctan n
⎜ t ⎟ t
⎝ i1 ⎠

⎛  ⎞
⎛ ⎞ 
n $⎛ ⎞⎛ ⎞
⎜  i2 ai2 ⎟ %
n % n 
n
π ⎜ ⎜ ⎝ i1 ⎟
⎟π %⎝
≤ · ai ⎠t +
2 ·& 2
ai ⎠⎝ i ai ⎠,
2 2
2 ⎜ ⎝ i1 t ⎟
⎠ i1 i1

$
% n 
% 2 2
% i1 i ai
if t  & n .
ai2
i1
Appendix—Power Sums Triangle

Let n and k be positive integers. Sums of the forms 1k þ 2k þ    þ nk often arise in


mathematics, particularly in algebra. These sums have been studied for hundreds of
years. They have wide applications in mathematics and can also be used for proving
algebraic Inequalities.
In 1631, the Prussian mathematician Johann Faulhaber (1580–1635) published
(Academiae Algebrae, 1631) the general formula for the sums of powers of the first
n positive integers. In 1713, Nicolaus Bernoulli (also spelled Niklaus), the nephew
of the Swiss mathematician Jacob Bernoulli, 7 years after Jacob Bernoulli’s death
in 1705 published (the manuscript of Jacob Bernoulli called Summae Potestatum in
the book Ars Conjectandi [18]) the general formula for the sums of kth powers of
the first n integers as a (k + 1)th-degree polynomial in the variable n with coeffi-
cients involving numbers Bj, which are now known as Bernoulli numbers. Besides
Bernoulli’s work there is a wide literature in which the interested reader can find
this general formula (and its proof), which has the following form:

X k  
n
1 X kþ1
i ¼
k
Bj nk þ 1j ;
i¼1
k þ 1 j¼0 j

where Bj = 0 if j is odd (j > 1) and

1 1 1 1 1 5
B0 ¼ 0; B1 ¼ ; B2 ¼ ; B4 ¼  ; B6 ¼ ; B8 ¼  ; B10 ¼ ; . . . :
2 6 30 42 30 66

A more detailed list of Bernoulli numbers can be found in the literature. A full
consideration and proof of this formula is outside the scope of this book.
In studying the works of Jacob Bernoulli, Johann Faulhaber, and other authors,
Hayk Sedrakyan noticed that for a high-school student to remember and apply this
formula can be really challenging. So his idea was to use this result to create a
simple and self-constructive Pascal-type triangle for sums of powers.

© Springer International Publishing AG, part of Springer Nature 2018 239


H. Sedrakyan and N. Sedrakyan, Algebraic Inequalities, Problem Books
in Mathematics, https://doi.org/10.1007/978-3-319-77836-5
240 Appendix—Power Sums Triangle

The “triangle.” Let us consider the following “triangle,” where k stands for the
power of the considered sum and n; n2 ; n3 ; n4 ; n5 ;    ; nk þ 1 stand for the terms of
the (k+ 1)th-degree polynomial in the variable n that the considered sum is equal to,
and the number written at the intersection of each row and each column stands for
the coefficient of the corresponding term of this polynomial.

Self-construction principle. Considering this “triangle” diagonally, we observe


that the coefficients on the dashed diagonals remain unchanged, the denominator of
each coefficient on the solid line increases by one for each next row (in other words,
this coefficient is equal to k þ1 1), and the arrows indicate how each row can be
recurrently constructed using the previous row (each time one needs to multiply the
coefficient of the previous row by a fraction whose numerator is equal to the value
of k written in that row and denominator is equal to the power of n written in that
column). Note that Bk = 0 if k is odd (k > 1), so every odd row (starting from the
third) starts with 0, and every even row k ¼ 2n; n 2 N; starts with B2n.
Example (application). Calculate the sum 15 þ 25 þ    þ n5 .

Using this triangle we obtain

5 3 4 5 1 1
15 þ 25 þ    þ n5 ¼ B4   n2 þ B2     n4 þ  n5 þ  n6 :
2 2 3 4 2 6

Thus, it follows that

1 5n2 1 5n4 n5 n6
15 þ 25 þ    þ n5 ¼   þ  þ þ :
30 2 6 2 2 6
Appendix—Power Sums Triangle 241

Therefore, we deduce that

n2 5n4 n5 n6
15 þ 25 þ    þ n5 ¼  þ þ þ :
12 12 2 6

Remark Note that neither this formula nor the general formula is easy to memorize
for a high-school student. Using the “triangle,” however, one easily derives the
formula even if it has been forgotten.
Bibliography

1. Andreescu, T., Enescu, B.: Mathematical Olympiad Treasures, Birkhäuser, Boston


Mathematical Notations, Dover Publications, New York (1993)
2. Andreescu, T. et al.: Old and New Inequalities, GIL Publications, Bucharest (2004)
3. Cajori, F.: A History of Mathematical Notations, Dover Publications, New York (1993)
4. Engel, A.: Problem-Solving Strategies, Springer, New York (1998)
5. Galperin, G., Tolpygo, A.: Moscow Mathematical Olympiads, Moscow Education, Moscow
(1986)
6. Hardy, G. et al.: Inequalities, Cambridge University Press, Cambridge (1948)
7. Kurschak, J. et al.: Hungarian Problem Book, Budapest, Random House/L.W. Singer
Company (1965)
8. Marshall, A., Olkin, I.: Inequalities: Theory of Majorization and Its Applications. Springer
Publishing, New York (1979)
9. Popov, S. et al.: Mathematical Exercises, Moscow center for continuous mathematical
education, Moscow (2013)
10. Rajesh, T.: 50 Greatest Mathematicians of the World, Ocean Books, New Delhi (2013)
11. Sedrakyan, H., Sedrakyan, N.: The Stair-Step Approach in Mathematics, Springer, New
York (2018). 10.1007/978-3-319-70632-0
12. Sedrakyan, H., Sedrakyan, N.: Geometric Inequalities: Methods of Proving, Springer, New
York (2017). 10.1007/978-3-319-55080-0
13. Sedrakyan, N., Sedrakyan, H.: Inequalities: Methods of Proving 1. Kyowoo Publishing,
Seoul (2015)
14. Sedrakyan, N., Sedrakyan, H.: Inequalities: Methods of Proving 2. Kyowoo Publishing,
Seoul (2015)
15. Sedrakyan, N., Avoyan, A.: Inequalities: Methods of Proving. Fizmatlit Publishing, Moscow
(2002)
16. Sedrakyan, N.: Inequalities: Methods of Proving. Edit Print Publishing, Yerevan (1998)
17. Sedrakyan, N.: Kvant, J., 97(2), 42–44, Moscow (1997). http://kvant.mccme.ru/au/
sedrakyan_n.htm

© Springer International Publishing AG, part of Springer Nature 2018 243


H. Sedrakyan and N. Sedrakyan, Algebraic Inequalities, Problem Books
in Mathematics, https://doi.org/10.1007/978-3-319-77836-5

You might also like